26.12.2013 Views

31151029036484.pdf

31151029036484.pdf

31151029036484.pdf

SHOW MORE
SHOW LESS

Create successful ePaper yourself

Turn your PDF publications into a flip-book with our unique Google optimized e-Paper software.

a A «<br />

LIBRARY<br />

JC^HNS HOPKINS UNIVERSITY


A<br />

TEEATISE<br />

ON<br />

PLANE AND SPHERICAL<br />

TRIGONOMETRY.<br />

BY<br />

WILLIAM GHAUVENET,<br />

PBOPESSOa OP MiTHEKATICS AND A8TK0N0MT IN •WASHINGTON UNIVERSIir,<br />

SAINT LOUIS.<br />

iinti ^Yiiin,<br />

PHILADELPHIA:<br />

J. B. LIPPINOOTT & 00.<br />

1878.


Q./\ 6-^1<br />

C f<br />

Entered according to Act of Congress, in tcs year 1860, by WILLIAM CHACTBHET, in the<br />

Clerk's Office of the District Court of the Eastern District of Pennsylvania.


PEEFACE.<br />

I HAVE in this treatise endeavored to arrange a course of<br />

trigonometrical study sufficiently extensive to enable the<br />

student to comprehend readily any applications of trigonometry<br />

he may meet with in the works of the best modem<br />

mathematicians. With this object, some topics have been<br />

introduced which are not usually found in works devoted<br />

specially to this subject.<br />

Among those topics, the most important is the solution<br />

of the general spherical triangle, or the triangle whose sides<br />

and angles are not limited, according to the usual practice,<br />

to values less than 180° The advantage of introducing<br />

such triangles into astronomical investigations is sufficiently<br />

shown in the applications made of them in the works of<br />

BESSEL and other German mathematicians; and especially<br />

in the Tliearia Motus Corporum Oodestiwn of GAUSS, who<br />

was the first to suggest their employment.<br />

The subject of Finite Differences of triangles, plane and<br />

spherical, occupies a large space in CagnoH's treatise, but<br />

has not been admitted into more recent works. It here<br />

occupies only a few pages, but no important result of


4 PKEFACE.<br />

Cagnoli's Table has been omitted, while a number of tHe<br />

formulae are much simpler than the corresponding ones<br />

given by him.<br />

Although my plan embraces a much more extensive<br />

course than is contained in the text-books commonly used,<br />

I have studiously kept in view the wants of academic and<br />

collegiate classes; and have so arranged the work that a<br />

selection of subjects of immediate importance may be readily<br />

made. The more elementary portions are printed in a larger<br />

type, and are intended to form, independently of the matter<br />

in the smaller type, a connected treatise which may be<br />

studied as though it were in a separate volume.<br />

Those who may afterwards wish to extend their knowledge<br />

will appreciate the advantage of having the higher<br />

departments of the subject treated in connection with those<br />

fundamental ones to which they are most intimately re<br />

lated. W. C.<br />

XT. S. NAVAL AOADEMT,<br />

Annapolis, Md., May 1, 1860<br />

NOTE TO THE FOURTH EDITION.<br />

IN this edition, besides a number of minor changes, and the correction of somt<br />

typographical errors, a very important modification has been made in the solution of<br />

the equation tan a; = i^ tan y by series (p. 145), which was given in former editions<br />

in the usual form as stated by all writers on trigonometry. This form was discovered<br />

to lack generality, and consequently to fail in certain applications, in consequence<br />

of the omission of the arbitrary term njr now introduced. Several subsequent<br />

investigations, depending on this, have in like manner been rectified.<br />

TJ. S. NATAL AOADBMT, April 1, 1864.


CONTENTS.<br />

PAET I.<br />

PLANE TEIGONOMETRT.<br />

CHAPTEE L<br />

FAOB<br />

MEASUEBS OF ANGLES AKD ABOS 9<br />

CHAPTEB, XL<br />

SiNBS, TANGENTS, AND SECANTS. FUNDAMENTAL FOEMUL^ 14<br />

CHAPTER XXI.<br />

TEIQONOMETKIO FUNCTIONS OE ANGULAE MAGNITUDE IN GENBKAI 22<br />

Sine and Tangent of a Small Angle or Arc<br />

SO<br />

CHAPTER XV.<br />

OENEEAL FOEMUXiE 31<br />

Formulae for Multiple Angles 36<br />

Relations of Three Angles 38<br />

Inverse Trigonometric Functions 41<br />

CHAPTER V.<br />

TEXGOSOMBTEIO TABLES 43<br />

Elementary Method of Constructing the Trigonometric Table 47<br />

CHAPTER VX.<br />

HoiTJTioN OS PLANE RIGHT TEIANGIES 51<br />

Additional Formulse for Right Triangles 54<br />

CHAPTER VIX.<br />

FoEMULiE FOE THE SOLUTION Or PLANE OBLIQUE TEIANGLBS 57<br />

A2 6


6 CONTENTS.<br />

CHAPTER VIXX.<br />

Pict<br />

SOLUTION OP PLANE OBLIQUE TEIANQLES<br />

Area of a Plane Triangle 74<br />

G4<br />

CHAPTER IX.<br />

MISCELLANEOUS PEOBLEMS RELATING TO PLANE TEIANQLES 75<br />

CHAPTER X.<br />

SOLUTION or CEBTAIN TEIOONOMETEIO EQUATIONS AND OE NUMEEIOAL EQUA­<br />

TIONS or THE SECOND AND THIRD DEGEEES 85<br />

CHAPTER SI.<br />

DUEEEENCES AND DiFEEEESTIALS OF THE TEIGONOMETEIC FUNCTIONS 101<br />

CHAPTER XII.<br />

DOTEEENCES AND DiEEEEENTIALS OFPLANE TEIANGLES 105<br />

CHAPTER XIII.<br />

TKIGONOMETEIC SEEIES. DEVELOPMENTS OE THE FUNCTIONS OF AN ANGLE IN<br />

TEEMS OE THE AEO, AND EECIPEOOALLY 115<br />

Computation of Natural Sines and Cosines by Series 116<br />

Computation of the Ratio of the Circumference of a Circle to its Diameter 120<br />

Computation of Logarithmic Sines and Cosines<br />

V22<br />

CHAPTEB XXV.<br />

EXPONENTIAL POEMUL^. TEINOMIAL OE QUADEATIO FACTOBS 127<br />

CHAPTER XV.<br />

1'BIGONOMETEIO SERIES CONTINUED. MULTIPLE ANGLES 135<br />

Development of the Sine and Cosine of the Multiple Angle in a Series of<br />

Ascending Powers of the Cosine of the Simple Angle 137<br />

Development of the Sine and Cosine of the Multiple Angle in a Series of<br />

Ascending Powers of the Sine of the Simple Angle 139<br />

Development of the Sine and Cosine of the Multiple Angle in a Serie's of<br />

Ascending Powers of the Tangent of the Simple Angle 140<br />

Development of any power of the Cosine of the Simple Angle in a Series<br />

of Sines or Cosines of the Multiple Angles, the Cosine of the Simple<br />

Angle being positive 141<br />

Development of any power of the Cosine of the Simple Angle in a Series<br />

of Sines or Cosines of the Multiple Angles, the Cosine of the Simple Angle<br />

being negative 142<br />

Development of any power of the Sine of the Simple Angle in a Series of<br />

Sines or Cosines of the Multiple Angles 144<br />

Certain Equations developed in Series of Multiple Angles 145


CONTENTS. 7<br />

PAET II.<br />

SPHEEICAL TRIGONOMETRY.<br />

CHAPTER L<br />

P^^^<br />

GENEEAL FoEMULiE 149<br />

Gauss's Theorem 161<br />

Additional Formulse 162<br />

Deduction of the Formulse of Plane Triangles from those of Spherical<br />

Triangles 160<br />

CHAPTER II.<br />

SOLUTION OE SPHEEIOAL RIGHT TEIANGLES 167<br />

Additional Formulae for the Solution of Spherical Eight Triangles....- 176<br />

Qaadrantal and Isosceles Triangles 177<br />

CHAPTER III.<br />

SOLUTION OE SPHEEIOAL OBLIQUE TEIANQLES 178<br />

Solution of Spherical Oblique Triangles by means of a Perpendicular 206<br />

Computation of Spherical Formulse by the Gaussian Table 211<br />

CHAPTER XV.<br />

SOLUTION OE THE GENEEAI SPHEEIOAL TEIASQLE 214<br />

Note upon Gauss's Equations 227<br />

CHAPTER V.<br />

AEEA OE A SPHEEIOAL TEIANQLE 229<br />

CHAPTER VL<br />

DlEEEEENOES AND DiFFEEENTIALS OP SPHEEICAL TEIANQLES 232<br />

CHAPTER VIL<br />

APPEOXIMATB SOLUTION OP SPHEEIOAL TEIANQLES IN CEBTAIN CASES 241<br />

Legendre's Theorem 244<br />

CHAPTER VIIL<br />

MISCELLANEOUS PEOBLEMS OE SPHEEIOAL TEIGONOMETET 240


PAET I.<br />

PLANE TEIGONOMETET.<br />

CHAPTER L<br />

MEASURES OF ANGLES AND AECS.<br />

1. TRIGONOMETRY is that branch of Mathematics which treats<br />

of methods of subjecting angles and triangles to numerical computation.<br />

2. PLANE TRIGONOMETRY treats of methods of computing plane<br />

angles and triangles.<br />

It embraces the investigation of the relations of angles in general,<br />

a branch of the science not necessarily connected with the<br />

elementary solution of triangles, and which has been distinguished<br />

as the Angular Analysig.<br />

3. By the solution of a triangle, in trigonometry, is meant the<br />

computation of unknown parts of the triangle from given ones.<br />

The triangle has six parts; three angles and three sides. It is<br />

shown in geometry, that when any three of these parts are given,<br />

provided one of them is a side, the triangle may be constructed,<br />

and the unknown parts found by mechanical measurement.<br />

In the same cases, by trigonometry, we compute the unknown<br />

parts from the three given ones, without resorting to construction<br />

and measurement: a method of inferior accuracy, on account of the<br />

unavoidable imperfections of the instruments employed, and the<br />

difficulty of distinguishing with the eye the smallest subdivisions of<br />

lines and angles.<br />

But here also the case is excluded in which the three angles are<br />

given without a side, because there may be an indefinite number o*<br />

plane triangles, whose angles are equal to the same three given ones, as<br />

2 0


10 PLANE TRIGONOMETRY.<br />

in Fig. 1. the triangles ABC, A'B' C\<br />

&e.. In this case, all these triangles<br />

are similar, and their sides are proportional<br />

; or the ratio oi AB to A C<br />

is equal to the ratio of A'B'to A! 0',<br />

ko.; so that the ratios of the sides to<br />

each other are fixed or determinate, although the absolute lengths<br />

of these sides are indeterminate.<br />

4. Now, in order to subject a triangle to computation, we must<br />

first express the sides and angles by numbers. For this purpose<br />

proper units of measure must be adopted.<br />

The unit of measure for the sides of plane triangles is a straight<br />

line, as an inch, a foot, a mile, &c.; and the number expressing a<br />

side is the number of units of the adopted kind that the side contains.<br />

5. The units by which angles are expressed are, the degree,<br />

minute, and second; distinguished by the characters " ' "<br />

A degree is an angle equal to -g'^ of a right angle; or a degree is<br />

jjj; of the whole angular space about a point, or ^J^j of four right<br />

Tig. 2.<br />

A'<br />

angles. Thus, Fig. 2, if the angular space about<br />

0 is divided into 360 equal parts, of which A OB<br />

is one, then A OB is one degree. The right angle<br />

will be expressed by 90°; two right angles by<br />

180°, and the whole angular space about a point<br />

by 360°.<br />

A minute is an angle equal to g'^ of a degree.<br />

Therefore, 1° = 60'; and a right angle =90 X 60'= .5400'.<br />

A second is an angle equal to g'j of a minute. Therefore, 1' =<br />

60": 1° = 60x60" = 3600"; and a right angle = 90 X 60 x 60"<br />

= 324000"<br />

Angles less than seconds are sometimes expressed by thirds,<br />

fourths, fifths, &c., marked '" '^ '', &c.; a third being g';; of a second;<br />

a fourth, g'j of a third; &c. But the more convenient method is to<br />

express them as decimal parts of a second; thus ^ of a right angle<br />

will be either<br />

or more conveniently<br />

12° 61' 25" 42'" SI"', &c.<br />

12° 51' 25".714, &c.<br />

6. The above division of angles is called sexagesimal, from the divisor 60 esnployed<br />

m the subdivision of the degree. The centesimal division, however, would be preferable<br />

in all cases, but cannot now be generally introduced without, at the same time,<br />

changing the arrangement of all our tables, the graduation of astronomical and


MEASURES OF AECS. 11<br />

other instruments, charts, &o. Nevertheless, the attempt has been made in France,<br />

and several standard works exist in the French language, in which it is employed<br />

throughout.<br />

In the centesimal or French division, the right angle is divided into 100 degrees;<br />

the degree into 100 minutes ; the minute into 100 seconds, &c. The reduction of<br />

these denominations from one to the other requires only a change in the position<br />

of the decimal, point; thus, in this system 00° 75' 84"-8 is the same as 607584"'b<br />

or 60°'75848 or 0'-C075848, the symbol q denoting a quadrant or right angle.<br />

To convert centesimal into sexagesimal degrees, since 100° dec. = 90° sex. deduct one<br />

tenth from the number of centesimal degrees.<br />

EXAMPLE. Eequired the number of sex. degrees in 85° 47' 43" dec.<br />

85°-4743 cent.<br />

Deduct JV = 8 -54743<br />

76°-92687 sex. degrees and dec. parts.<br />

55'-6122<br />

36"-782<br />

or<br />

76° 55' 36"-732 sexagesimal.<br />

To convert sexagesimal into centesimal degrees, since we must take y of the WK.,<br />

divide by 9 and move the decimal point one place to the right.<br />

EXAMPLE. Eequired the number of centesimal degrees in 76° 55' 36"-732 sex.<br />

Eeducing the minutes and seconds to the decimal of a degree, we have<br />

76°-92687 ses.<br />

y> of which is 85°-4743 cent,<br />

or<br />

85° 47' 43" centesimal.<br />

To distinguish the degrees of the centesimal from those of the sexagesimal division,<br />

the former are frequently called grades, and are denoted by the character »<br />

instead of °; thus the preceding angle would be 85* 47' 43".<br />

MEASURES OF ARCS.<br />

7. Since the angles at the center of a circle are proportional to<br />

the arcs of the circumference intercepted between their sides, these<br />

arcs may be taken as the measures of the angles, and we may express<br />

both the arc and the angle by the number of units of arc intercepted<br />

on the circumference.<br />

The units of, arc are also the degree, minute, and second. They<br />

are the arcs which subtend angles of a degree, a minute, and a<br />

second, respectively, at the Center. A'degree of arc is thus always<br />

alt) of the circumference, whatever the radius of the circle may be;<br />

and we o'btain the same numerical expression of<br />

an angle, whether we refer it directly to the angu-"<br />

lar unit, or to the corresponding unit of arc. The<br />

right angle A OA', Fig. 3, and its measure, the<br />

quadrant A A', are therefore both expressed by<br />

90°; the semicircumference by 180°, and the<br />

whole circumference by 360°.<br />

"'~~^^<br />

8. The radius of the circle employed in measuring anirles is then


12 PLANE TRXGONOMETBT.<br />

arbitrary, and we may assume for it such a value as will most simplify<br />

our calculations. This value is unity ; that is, the linear unit<br />

employed in expressing the sides of our triangles, or other lines<br />

considered. This value wUl be generally used throughout this<br />

treatise.<br />

9. To find the length of an arc of a given number of degrees,<br />

minutes, &c.<br />

The semi-circumference of a circle whose radius is unity is known<br />

to be 3-14159265; or, the radius being B, the semi-circumference<br />

IS 3-14159265 i2. Hence<br />

When.B = l<br />

Arc 180° = 3-14159265 72 =3-14159265<br />

u 1° = 0-017453293 i? =0-017453293<br />

a v =0-0002908882i? =0-0002908882<br />

u I" = 0-000004848137 i? =0-000004848137<br />

An arc x therefore, in the circle whose radius is unity, being expressed<br />

in degrees, or minutes, or seconds, we find its length by the<br />

formulse<br />

Arc a; = 0-017453293 2!°<br />

= 0-0002908882 a;'<br />

= 0-000004848137 a;".<br />

As these factors for finding the length of an arc are often used,<br />

it is convenient to have their logarithms prepared.* Thus<br />

Arc X = [8-2418774] x°<br />

= [6-4637261] a;'<br />

= [4-6855749] a;"<br />

m which the rectangular brackets are used to express that the logarithm<br />

of the factor is given instead of the factor itself.<br />

EXAMPLE. What is the length of the arc a; = 38° 17'48", the<br />

radius being = 1.<br />

.38° 17'48" =137868" log. 5-1394635<br />

Log. factor for seconds 4-6855749<br />

a; = 0-6684031 log. x 9-8250384<br />

10. To find the number of degrees, ^c. in an arc equal to the<br />

radius, '<br />

We have, from the preceding article,<br />

* The logarithms in the examples of this work wiU be taken from Stanley's Tables,<br />

(published in New Haven, by Durrie and Peck,) the best tables of seven-figure<br />

logarithms yet published in this country.


MEASURES OF ARCS. 13<br />

180°<br />

^ = 3-44169265 =^^°-2^^^^^S<br />

= 3437'-74677 = 206264' -806<br />

11. The angle at the center measured by an arc equal to the radius, is often<br />

taken as the unit of angular measure, as this angle will be of an invariable magnitude,<br />

whatever is the length of the radius. If x is the number of such units in a<br />

given angle, the number of degrees, &o., in it will be found by multiplying by the<br />

value of the radius in degrees, &o., found in the preceding article. Thus,<br />

x° = xR° = 57°-2957795a; = [1-7581226] x<br />

x' =xB' = 3437'-74677 a; = [3-5362739] a:<br />

x" = x U" = 206264"-806 x = [5-3144251] x<br />

Beciprocally, the angle being given in degrees, &o., we reduce it to the unit radius,<br />

by dividing by -8°, E, or R", thus,<br />

_ x" _x; _x!'<br />

which is evidently the same as multiplying by the faotSrs of Art. 9.<br />

It appears, then,.that an angle is expressed in the unit of this article by the<br />

length of the arc which measures the angle in the circle whose radius is unity<br />

Hence, an angle thus expressed is said to be given in arc. If we put (as is usual)<br />

ir = 3-14159265 • - •<br />

K is the circular measure of two right angles, or it is the expression of two right<br />

angles in arc. In trigonometry it is therefore common to employ rr to denote an<br />

7r<br />

angular magnitude of 180°; — a right angle ; 2 v four right angles, &o.<br />

12. The complement of an angle or arc is the remainder obtained<br />

by subtracting'the angle or arc from 90°.<br />

The supplement of an angle or arc is the remainder obtained by<br />

subtracting the angle or arc from 180°.<br />

Thus the complement of 30° is 60°; the supplement of 30° is<br />

150°.<br />

Two angles or .arcs are complements of each other when their<br />

sum is 90°. They are supplements of each other when their sum is<br />

180°<br />

13. According to these definitions, the complement of an arc<br />

that exceeds 90° is negative. Thus the complement of 120° is<br />

90° - -120° = — 30°. In like manner the supplement of 200° la<br />

ISO"^ -200° = -20°.


14 PLANE TRIGONOMETRY.<br />

CHAPTER n.<br />

SINES, TANGENTS, AND SECANTS.<br />

FUNDAMENTAL FORMULA.<br />

14. HAVING expressed the sides and angles of triangles by numbers,<br />

we are next to find such relations between them as shall enable<br />

us to combine these two diiferent species of quantity in computation.<br />

As every oblique triangle may be resolved into two right triangles<br />

by dropping a perpendicular from one of the angles upon the opposite<br />

side, the solution of all triangles is readily made to depend upon<br />

Fig. 4. that of right triangles. Let us therefore<br />

consider a series of right triangles, ABQ,<br />

AB'Q', AB"Q", &c., Fig. 4, which have<br />

a common angle A. The angles at B,<br />

B', B", being also equal, the triangles are<br />

0' a" similar; and by geometry<br />

BO:AB = B'O': AB' = B"0": AB"<br />

or by the definitions of ratio and proportion,<br />

BO _ B^'_ B"G"<br />

AB~ AB'~ AB"<br />

In like manner it follows that<br />

and<br />

BQ B'O' B"0"<br />

AQ A 0' AO"<br />

AB _ AB' _ AB"<br />

AQ" AQ' AQ"<br />

Hence it appears that the ratios of the sides to each other are the<br />

same in all right triangles having the' same acute angle; and,<br />

therefore, if these ratios are known in any one of these triangles,<br />

they will be known in all of them.<br />

These ratios, then, depending on the value of the angle alone,<br />

without regard to the absolute lengths of the sides, may be considered<br />

as indices of the angle, and have received special names, as follows:<br />

15. The SINE of the angle is the quotient of the opposite side<br />

divided by the hypotenuse.


SINES, TANGENTS, AND SECANTS. 1,5<br />

Thus, in the right triangle ABQ, Fig. 5, "«•»•<br />

1 we designate the sides by the small letters<br />

a, b, c, we shall have, (whatever the absolute<br />

length of the sides)<br />

, a • T> b<br />

sm A = —, sm B = —<br />

c 0<br />

16. The TANGENT of the angle is the quotient of the opposite side<br />

divided by the adjacent side.<br />

Thus tan A=-, tan 5 = -<br />

0 a<br />

17. The SECANT of the angle is the quotient of the hypotenuse<br />

divided by the adjacent side.<br />

Thus secA = -^, sec^ = -<br />

0 a<br />

18. The COSINE, COTANGENT, and COSECANT of an angle, are respeotively<br />

the SINE, TANGENT, and SECANT of the complement of the<br />

angle.<br />

Since the sum of the two acute angles of a right triangle is one<br />

right angle, or 90°, they are, by Art. 12, complements of each other;<br />

therefore, according to the preceding definitions, we shall have<br />

sia A = cos B =<br />

a<br />

c<br />

tan A = cot B =<br />

a<br />

1<br />

sec A = cosec B =<br />

e<br />

1<br />

cos J. = sin .B = —<br />

c<br />

cot A = tan B = —<br />

a<br />

cosec >4 = sec ^ = — a<br />

19. Since—is the reciprocal of —, it follows from the first and<br />

a<br />

c<br />

last of these equations, that the sine and cosecant of the same angle<br />

are reciprocals; and from the other equations, also, that the cosine<br />

and secant, the tangent and cotangent are reciprocals. That is,<br />

sin A = r cosec A = -<br />

cosec A<br />

sin A<br />

COS A = r<br />

sec A<br />

sec A •<br />

COS A<br />

(2)<br />

1 . ^ 1<br />

tan A = cot A r cot A- tan A<br />

4T more briefly,<br />

sin A cosec J. = cos J. sec J. = tan JL cot J. = 1 (3)<br />

(1)


iQ<br />

PLANE TRXGONOMETBT.<br />

SINES, &C. OF ARCS.<br />

"r<br />

D<br />

0<br />

Fig. 6.<br />

\ B'<br />

2"<br />

20. The sine, tangent, and secant of<br />

an arc are respectively the sine, tangent,<br />

and secant of the angle at the center<br />

measured by that are. Thus, Fig. 6,<br />

BQ_<br />

sin AJS = sin^O.B =<br />

OB<br />

The sine of an arc, therefore, does not depend upon the absolute<br />

length of the arc, but upon the ratio of the arc to the whole circumference,<br />

(Art. 7.) It follows that the relations (2) and (3) are also<br />

applicable when A expresses an arc.<br />

21. If the radius = 1, all the trigonometric functions above defined<br />

may be represented in or about the circle by straight lines.<br />

Representing the arc AB, or angle AOB, by x, we have, when OA<br />

= 0B = 1,<br />

sma;<br />

^^^?fi^BO<br />

= OB<br />

AT AT .^<br />

tan x = YTJ = ^f~ =-a.jt<br />

OT<br />

sec X = OA =<br />

OT<br />

-^=0T<br />

and from the arc A'B = 90° — a; we find in the same way<br />

coax = BI>=00<br />

cot X = A'T'<br />

cosec a; = OT'<br />

Therefore, in the circle whose radius is unity, the sine of an aro,<br />

or of the angle at the center measured by that arc, is the perpendicular<br />

let fall from one extremity of the arc upon the diameter<br />

passing through the other extremity.<br />

The trigonometric tangent is that part of the tangent drawn at one<br />

extremity of the arc, which is intercepted between that extremity and<br />

the diameter (produced) passing through the other extremity.<br />

The secant is that part of the produced diameter which is inter'<br />

cepted between the center and the tangent.<br />

The cosine is the distance from the center to the foot of the sine.<br />

In a circle of any other radius than unity, the trigonometric


FUNDAMENTAL FORMULAE. [7<br />

functions of an arc will be equal to the lines drawn as above, divided<br />

by that radius.<br />

The properties here stated have heretofore been used by most<br />

writers upon trigonometry as definitions, but without limiting the radius<br />

to unity; and it is evidently from this mode of viewing these<br />

functions that they have derived their names.<br />

22. Besides the functions already defined, others have been occasionally employed<br />

to facilitate particular calculations, as the versed sine, ^Mhich. in the circle is the<br />

portion of the diameter intercepted between the extremity of the ai-c and the foot<br />

of the sine; thus. Fig. 6, the versed sine of A £ is A O, or the radius being = 1,<br />

versin a; = 1 — cos x (4)<br />

by means of which formula we may always substitute versed sines for cosines, and<br />

reciprocally.<br />

The coversfd sinb (covers.) is the versed sine of the complement, and suversed sine<br />

(suvers.) is the versed sine of the supplement.<br />

The chords of arcs have also been used, and may be substituted for sines by the<br />

formula<br />

ch 2: = 2 sin J 3: (5)<br />

which is evident from Fig. 6, where if the arc B B' ^=x, we have chord B B' =..<br />

2BG=2sinAB.<br />

23. From what has now been stated, the student will perceive that<br />

angles are to be subjected to computation by means of the quantities<br />

sine, cosine, &c., commonly designated by the comprehensive<br />

term trigonometric functions.* It becomes necessary, therefore, for<br />

the computer to know the values of these functions for any given<br />

value of the angle. The trigonometric tables, contain these values<br />

for every minute, and sometimes for every second, from 0° to 90° ;<br />

and with these tables all the numerical computations of trigonometry<br />

are carried on. In practice, then, we are not required to compute<br />

the functions themselves, and we shall therefore defer the methods<br />

for that purpose to a subsequent part of this work, and proceed<br />

at once with the investigation of the formulse and methods by which<br />

these tables are rendered available.<br />

FUNDAMENTAL FORMULA.<br />

24. Q-iven the sine of an angle, to find the cosine.<br />

From the right triangle ABQ, Fig. 7, we **^have<br />

by geometry a^ + 5^ = 0^<br />

Dividing by c^, this equation becomes<br />

a^ b^ ,<br />

c^ e^<br />

* Also trigonometric lines, from the properties explained in Art. 21<br />

R<br />

B2


18 PLANE TRIGONOMETRY.<br />

or, by the definitions of sine and cosine (1),<br />

sin^ J. 4-cosM = 1 (6j<br />

in which the notation sin^ A signifies " the square of the sine of A."<br />

From this formula, if the sine is given, we find<br />

cos^ A = l — sin^ A = {1 + sin A) (1 — sin A)<br />

coaA = ^{l- sin^ A) = s/ [(1 -f sin A) (1 - sin AJ] (7)<br />

and if the cosine is given, we find<br />

sin JL = N/ (1 - cos'^ ^) = N/ [(1 + cos A) (1 - cos A)} (S)<br />

25. (riven the sine and cosine of an angle, to find the tangent.<br />

By (1) we have<br />

also<br />

therefore<br />

tan^ = T<br />

0<br />

sinu4<br />

cos J.<br />

a _ b<br />

c ' c<br />

sin^<br />

a<br />

"b<br />

tan J. =<br />

cos.<br />

And since the cotangent is the reciprocal of the tangent,<br />

. cos J.<br />

cot A = —; f<br />

sm^<br />

26. Q-iven the tangent of an angle, to find the secant.<br />

The right triangle ABQ, Fig. 7, gives<br />

e^ = J^ -f a^<br />

Dividing by 5% this becomes<br />

= l-fc'<br />

¥~^ • b^<br />

or, by the definitions of secant and tangent (1),<br />

secM = l -f tan^J.<br />

This formula applied to the complement of A gives<br />

Kg- 8.<br />

(9)<br />

(10)<br />

(11;<br />

cosec^ J. = 1 + cot^ A<br />

(12)<br />

27. The preceding formulse are also<br />

directly obtained from Fig. 8. If the<br />

angle AOB, or the arc AB, be denoted<br />

by X, the right triangle OBQ, gives<br />

BQ^ + OQ^=OB^<br />

or remembering that the radius is unity,<br />

by Art. 21,<br />

sin'' X + cos^ a; = 1 (13)


FUNDAMENTAL POEMULjE. 19<br />

The triangle O^C gives by the definition. Art. 16,<br />

BQ<br />

tan A OB = -^<br />

sin a;<br />

or tana; = (14)<br />

cos X<br />

'<br />

Since the angle BOD is the complement of BOQ, ta.nBOB =<br />

cot a;, and the triangle BOB gives<br />

„^^ BB OQ<br />

tan502>=^ = -^<br />

cos X , ,<br />

01 cot X = —. (15;<br />

sm a;<br />

^ '<br />

In a similar manner the triangles AOT, A! OT' give<br />

sec^ 2; = 1 + tan^ x (16)<br />

cosec* a; = 1 -f cot^ x (17)<br />

28. The following equations are easily demonstrated by combining (13), (14),<br />

(15), (16), (17), and employing the property of the reciprocals (2). They .are of<br />

frequent use.<br />

1 . tan X cos x<br />

3in3;= == tan a; cos a; =—— :^ n81<br />

cosec X sec x cot x ^ •<br />

1 , . cot X oin -y<br />

^ cot X sm X =<br />

sec X • cosec x tan x ^ '<br />

sina; = v'(l — cos" a;), cos2: = ,^/(l — sin'^) (20)<br />

seca; = Y/(1-1-tan'a;), cosec i = ^ (1-(-cot'2:) (21)<br />

tan a: = .^ (sec'a; — 1), cot a: = ^ (cosec'a;—1) (22)<br />

tan X 1<br />

sma;: V(l+.tan'2:) v'(l +cot'a:)<br />

cot 2; 1<br />

(23)<br />

''°^^~,/(l + cotf'2:)~"v'(l + tan'a:)<br />

sin 2; -1/(1 — cos'a:)<br />

(24)<br />

tana; = ^—^ = ^-^ ^<br />

Y/ (1 — sm' X) cos X<br />

(25)<br />

cos a; »/(l — sin'2;)<br />

cota;=^ ^-^— - ^ (26)<br />

,y{l — cos'a:) sma: '^ '<br />

29. To find the sine, ^c. of 30° and 60°.<br />

In Fig. 8, let the arc AB = 30°, and BB' = 2AB = 60° By<br />

Art. 21, sin JL^ = BQ, and by geometry the chord of 60°, or of onesixth<br />

of the circumference, is equal to the radius = 1; therefore<br />

2sin30° = 2 5C = ^.B' = l<br />

whence<br />

sm 30° = 1 = cos 60°<br />

'27)'


20<br />

PLANE TEXGONOMETRY.<br />

dhd by (7)<br />

cos 30° = x/[(l + J) (1-i)] = v/(i X i)<br />

whence cos 30° = J x/ 3 = sin 60°<br />

Then, by (9) and (2),<br />

(28)<br />

sin 30° i 1<br />

tan 30° =<br />

= cot 60°<br />

cos 30° iv^3 ,/3<br />

(29)<br />

1 _<br />

cot 30° =<br />

tan 30° ^<br />

1<br />

sec 30° =<br />

cos 30°<br />

v" 3 = tan 60°<br />

:6Q°<br />

(30)<br />

(31)<br />

1<br />

cosec 30° = = 2 = sec 60°<br />

•.sin30° ~ " ~"^^""<br />

^^^^<br />

30. To find the sine, ^c. of 45° Since 45° is the complement of<br />

45°, we have<br />

sin 45° = cos 45°<br />

whence by (13), putting x = 45°,<br />

sin2 45° -f 008^45° = 2 sin^" 45° = 2 cos^ 45° = 1<br />

sin^ 45° = cos^ 45° = J<br />

sin 45° -•: cos 45° = >/ i = i v/ 2<br />

(33)<br />

sin 45°<br />

tan45°^ : cot45° = 1<br />

(34)<br />

cos 45°<br />

1<br />

sec 45° = cosec 45° •<br />

sin 45 -o = ^/2 (35)<br />

These values are readily verified in the circle.<br />

Fig. 9, where OA TA' is a square described upon<br />

the radius. The diagonal OT bisects the right<br />

angle, whence J-OT = 45°, and tan 45° == AT<br />

= OJ. = 1; cot 45° = ^' T = 1; sin 45° = BQ<br />

= Oa = cos45°, &c.<br />

31. The sines and cosines of two angles being given, to find<br />

the sine and cosine of the sum, and the sine and cosine of the difference<br />

of those angles.<br />

Kg 10<br />

Fig. 11 Let the two angles he AOB<br />

cB<br />

y and BOQ, Figs. 10 and 11.<br />

B<br />

/<br />

f-iE ^<br />

\y<br />

At any point B in the line<br />

B<br />

/"; f<br />

\^ 0 B draw B G perp. to O'B.<br />

Draw BA and C B perp. to<br />

/I- DA L_<br />

'<br />

" o^— AD OA, and B E perp. to QB.


FUNDAMENTAL FOEMUL^. 21<br />

Then the triangles B QB and BOA are mutually equiangular,<br />

the three sides of the one being perp. to the three sides of the other<br />

respectively; therefore the angle B QB= AOB.<br />

Let<br />

Then, Fig, 10,<br />

Fig. 11,<br />

and in<br />

Fig. 10, sin {x + y)<br />

Fig. 11, sin [x — y)-<br />

and in both figures<br />

Again in<br />

Fig. 10,<br />

Fig. 11,<br />

x = AOB=BQB<br />

y=^BOQ<br />

x+y = QOB<br />

x-y= QOB<br />

QB BA+QE BA<br />

~ Q.0~ QO ~,'Q0'^<br />

QB BA - QE BA<br />

QO QO QO<br />

QE<br />

QO<br />

QE<br />

QO<br />

BA BA ' M) .<br />

^ ^ X-^ = sm a; cos y<br />

QO ~<br />

QB QE QB<br />

QO ~<br />

"QB x ^ =cosa; smy<br />

which being substituted in the above expressions of sin {x + y) and<br />

sin (x — y) give<br />

sin [x + y) =sin x cos y + cos x sin y<br />

(36)<br />

sin {x — y) == sin x cos y — cos x sin y (37)<br />

and in both figures.<br />

therefore<br />

, , ^ OB OA-EB OA<br />

cos {x + y)~ Q^- Q^ - Q^<br />

OB OA + EB OA<br />

cos{x-y)-^^- 0(7 ~0Q'^<br />

OA OA OB<br />

OQ = -OB''-OQ =<br />

EB EB BQ .<br />

OQ^W'OQ^''''''''''^<br />

'"'''''''y<br />

EB<br />

' OQ<br />

EB<br />

00<br />

cos (x -i- y) = cos a; cos a/ — sin x sin y (38)<br />

cos {x — y) = cos a; cos ^ + sin a; sin^ (39)<br />

and (36), (37), (38), and (39) are the required formula.<br />

These maybe considered as the fundamental formulse of the trigonometric<br />

analysis, and will form the basis of our subsequent investigations.<br />

They are equally applicable to arcs represented by x and<br />

y (Art. 20).


•i'i<br />

PLANE TEIGONOMETEY<br />

CHAPTER III.<br />

TRIGONOMETRIC FUNCTIONS OF ANGULAR MAGNITUDE IN GENERAL.<br />

32. THE definitions of sine, &c. given in the preceding chaptei<br />

apply only to acute angles, since the angle is there assumed to be one<br />

of the oblique angles of a right triangle. But we shall now take a<br />

more general view of angular magnitude and of the functions by<br />

means of which it is subjected to computation.<br />

If, Fig. 12, we suppose the line OA to revolve<br />

from the position OA to OA' in the direction of<br />

the arc AA' (or from right to left), it will describe<br />

\B an angular magnitude of 90°; when it ai-rives at<br />

OA" it will have described an angular magnitude<br />

of 180°; at OA'", 270°; and at OA again, 360°.<br />

If it now continue its revolution, when it arrives<br />

at OA' again, it will have described an angular magnitude of<br />

360° + 90°, or 450° ; and thus Ave may readily conceive of an angular<br />

magnitude of any number of degrees. In like manner we may have<br />

arcs equal to or greater than one, two, or more circumferences.<br />

To obtain trigonometric functions for angles and arcs thus generally<br />

considered, we shall avail ourselves of the fundamental formulae<br />

established in the preceding chapter; first deducing their values<br />

analytically, and then explaining their geometrical signification.<br />

33. To find the sine, ^c. of 0° and 90°. In (37) and (39)<br />

let X — y; the first members become sin [x — x) = sin 0°, and<br />

cos (x —' x) = cos 0° ; and by (13) they are reduced to<br />

sin 0° = sin x cos x — cos a; sin a; = 0<br />

cos 0° = cos^ X -f sin^ x =1<br />

and smce 0° and 90° are complements of each other. Art. 12,<br />

from which by (9) and (2)<br />

sin 0° = cos 90° = 0 (40)<br />

cos 0° = sin 90° = 1 (41)<br />

sin 0° 0<br />

tan 0° = cot 90° = ^ = r = 0 (42i<br />

cos 0° 1 ^ •


FUNCTIONS OF ANGULAR MAGNITUDE. 23<br />

cotO°= tan90°=J^, =!-=«, ^43)<br />

sec 0° = cosec 90° = —^ = ^ = 1 (44)<br />

cos 0° 1 ^ '<br />

cosec 0° = sec 90° = ~ = 1 _ •„ UK^<br />

sin 0° 0 ~ °° (*^)<br />

34. To find the sine, ^c. of 180°. In (36) and (38) let<br />

z=.-y = 90° ; these equations become by means of the preceding<br />

values<br />

whence by (9) and (2)<br />

sin 180° =1x0 + 0x1 = 0 (46)<br />

cos 180° = 0 x 0 - l x l = - l (47)<br />

0 1<br />

tan 180° = —^ = 0 cot 180° = ^ = oo (48)<br />

sec 180° = —^ = - 1 cosec 180° = ^ = oo (49)<br />

35. To find the sine, ^c. of 270°. In (36) and (38) let<br />

x =180°, y = 90°, then<br />

sin270°=0x0+(-l)xl = - l (50)<br />

cos 270° = (-l)x0-0xl = 0 (51)<br />

tan 270° = ^ = 00 cot 270° = — = Q (52)<br />

sec 270° = - = 00 cosec 270° = J- = — 1 (53)<br />

36. To find the sine, ^c. of 360°. In (36) and (38) let<br />

x=y = 180°; then<br />

sin360° = 0 x(-l) + (-l) x0 = 0 (54)<br />

cos360°=(-l)x(-l)-0 xO = l (55)<br />

the same values as for 0°, whence it follows that all the trig, functions<br />

of 360° are the same as those of 0°.<br />

The same process continued will give for 450° (= 360" + 90°),<br />

the same trig, functions as those of 90°; for 540° the same functions<br />

as for 180°, &c.


24 PLANE TEIGONOMETEr.<br />

37. The preceding values now furnish us at once with the values<br />

of the functions for all possible values of the angle. In (36) and<br />

(38) let X = 0°, they are reduced to<br />

sin y = sin 0° cos y -f cos 0° sin y — siny<br />

cosy = cos 0° cos«/ — sin0° sin y = cosy<br />

which are simply identical equations, and reveal no new property.<br />

But if in (37) and (39) we put x = 0°, we have, after substituting<br />

the functions of 0°,<br />

whence by (9) and (2)<br />

sin (—«/) = — sin y cos (— «/) = cosy (56)<br />

, . sin (— w) — sin y .^„^<br />

tan (— w) = —7—H = ^ = — tan«/ (57)<br />

"^ ^^ cos{ — y) cosy<br />

. COS(— W) COSW . ,;.QV<br />

cot(—«) = -^—^= .^- = -coty (58)<br />

^ ^' sm{ — y) —smy "<br />

sec (— y) = 7 N = = sec y (59)<br />

^ ^^ cos('^.?/) cosy " ^ '<br />

cosec (— ,y) = -^—, c = =— = — cosec y (60)<br />

^ ''' sm (— 2/) — sm y ^ ^ '<br />

or, the sin., tan., cot., and cosec. of the negative of an angle are the<br />

negative of the sin., tan., cot., and cosec. of the angle itself; and the<br />

cos. and sec. are the same as those of the angle itself.<br />

38. In (37) and (39) let a; = 90° ; we find after reduction<br />

sin (90° —y) = cos y<br />

cos (90° — «/) = sin y<br />

which agree with the definition of cosine, but give no new relations.<br />

But in (36) and (38) let x = 90°, we find<br />

sin (90° + 2/) = cos y, cos (90° -f «/) = — sin ?/ (61)<br />

whence by (9) and (2),<br />

tan (90° + 2/) = — cot 2/ cot (90° +«/) = — tan y (62)<br />

sec (90°+ «/) = —cosec y cosec (90°+«/) = sec y (63)<br />

or, the sin. and cosec. of an angle are equal to the cos. and sec. of the<br />

excess of the angle above 90° ; and the cos., tan., cot., and ges. are<br />

equal to the negatives of the sin., cot, tan., and cosec. of the excess<br />

of the angle alove 90°.


FUNCTIONS OF ANGULAR MAGNITUDE. 25<br />

69. In (37j and-(39) let x = 180° ; we find<br />

sin (180° - ?/) = sin y cos (180° —y) = -cosy (64;<br />

tan (180° — y) = — tan y cot (180° — y) = — coty (65)<br />

sec(180° — y) = — secy cosec (180° — y) = cosecy (66)<br />

or, the sin. and cosec. of the supplement of an angle are the same as<br />

those of the angle itself; and the cos., tan., cot., and sec. are the<br />

negative of those of the angle itself.<br />

40. If y is acute (that is, less than 90°), all its trig, functions are<br />

positive ; and since its supplement 180° — yis obtuse (that is, greater<br />

than 90°), it follows from the preceding article, that the sin. and<br />

cosec. of an obtuse angle are positive, while its cos., tan., cot., and<br />

sec. are negative.<br />

41. In (36) and (38) let x = 180° ; we find<br />

sin (180° +y) = — siuy cos (180° + t/) = - cos j/ - (67)<br />

tan (180° +y) = tan y cot (180° + y) = cot y (68)<br />

sec (180° -h y) = —secy cosec (180° + y) — — cosee«/ (69)<br />

by means of which, if y is acute, we obtain the values of the sines,<br />

&c. of angles between 180° and 270°.<br />

42. In (37) and (39) let x = 270° ; we find<br />

sin (270° -y) = -cosy cos (270° —y) = -smy (70)<br />

'tan(270°-?/) = cot?/ cot (270° — ?/) = tan t/ (71)<br />

sec (270°—«/) =—cosec?/ cosec (270°— «/)=—sec?/ (72)<br />

43. In (36) and (38) let x = 270° ; we find<br />

sin (270° +y) = -cosy cos (270° + y) = sin y (73)<br />

tan (270° +y) = -coty cot (270° + ?/) = - tan y (74)<br />

sec (270° +y) = cosec y cosec (270° + ?/) = - sec.y (75)<br />

44. In (37) and (39) let x = 360° ; we find<br />

sin (360° -y) = -siny cos (360° - y) = cos y (76)<br />

tan (360°-?/) = -tan y cot (360°-?/) = -cot j/ y11)<br />

sec (360° —y) = secy cosec (360° -«/)=- cosecy (78;<br />

or the functions of 360° — y are the same as those of — ?/ (Art. 37).<br />

45. In (36) and (38) let x = 360°; we find<br />

sin(360°+ ?/)=siny cos (360° + ?/) = cos?/ (79i<br />

9r, the functions of an angle which exceeds 360° are the same as<br />

those of the excess above 360°


ye<br />

PLANE TF.XGONOMETRT.<br />

It follows that the functions of 720° + y are the same as those<br />

of 360° + y, and therefore the same as those of y; and in like<br />

manner for an angle which exceeds any multiple of 360°.<br />

46. Since y — 90° is the negative of 90° — y, we obtain from<br />

Art. 37,<br />

sin (?/ — 90°) = — sin (90° — ?/)-= — cos ?/<br />

cos {y — 90°) = cos (90° —y) = sin y<br />

(80)<br />

whence also tan., &c.; and in the same manner we may find the functions<br />

of ?/ - 180°, y - 270°, y - 360°, &o.<br />

47. We shall now give the geometrical interpretation of the preceding<br />

results.<br />

In Fig. 13, let the radius revolve from the<br />

position OA to OA!, OA!', kc, as in Art. 32,<br />

thus describing a continuously increasing angular<br />

magnitude; or, which is equivalent, let<br />

the arc commencing at A increase continuously<br />

to AB, AA!, AB', kc. Then the changes<br />

in the values of the several trigonometric lines<br />

may be traced as follows.<br />

1st. TAesme being, by Art. 21, the perpendicular from one extremity<br />

of the arc upon the diameter drawn through the other extremity,<br />

we shall have sin AB = BQ, sin AB' = B' Q', sin A A!' B" = B" 0',<br />

sin AA"B'" = B'"Q, and if we make<br />

we have<br />

AB = A"B' = A"B" = AB'" = y<br />

siny = BQ<br />

sin (180° -y) = B' Q'<br />

sin (180° +y)= B" Q'<br />

sin (360° - ?/) = B'" Q<br />

The lines BQ, B' Q', B" Q', B'" Q, however, represent only the<br />

numerical values of the sines, and are here equal. But the results<br />

above obtained from our formulae enable us to distinguish between<br />

them by means of their algebraic signs. Thus, by (64), (67), (76),<br />

sin (180° — y) = sin y<br />

sin (180* +y) = — ainy<br />

sin (360° — y) = — Biay<br />

so that the sines from 0° to 180° are positive, while those from 180°<br />

to 360° are negative; or the sines which are above the diameter<br />

A A" are positive, while those which are below this diameter are


FUNCTIONS OF ANGULAR MAGNITUDE. 27<br />

negative; or still more generally, the sines that have opposite directions,<br />

with reference to the fixed diameter from which they are<br />

measured, have opposite signs.<br />

2d. The cosine being, by Art. 21, the distance from the center to<br />

the foot of the sine, we have<br />

but by (64), (67), (76),<br />

cos y •= 0 Q<br />

cos (180°-?/) = OC"<br />

cos (180° -\-y)=<br />

cos (360° ~y)=<br />

OQ'<br />

OQ<br />

cos (180° — ?/) = — cosy<br />

cos (180° + y)= — cosy<br />

cos (360° — ?/) = cos y<br />

so that the cosines on the right of the diameter A' A'" are positive,<br />

while those on the left of this diameter are negative; or rather the<br />

cosines that have opposite directions, with reference to the diameter<br />

from which they are measured, have opposite signs.<br />

We have here only exhibited a well-known principle in the application<br />

of analysis to geometry, viz.: that all lines measured in opposite<br />

directions from a fixed line have opposite signs.<br />

To interpret the results (56), it is only necessary to observe that<br />

a negative arc will be one reckoned from A towards B'", or in the<br />

opposite direction to that of the positive arc, so that<br />

sin A B'" = sin (- ?/) = B'"<br />

cos A B " = COS ( — y) = OQ — cos y<br />

Q=-BQ=-siny<br />

as in (56).<br />

The same principle applies to the tangents, but it will be simpler<br />

in practice to obtain their signs (as also those of the secants), analytically,<br />

from those of the sine and cosine, as has been already shown.<br />

It will be sufiicient to bear in mind the following table, which is alsc<br />

expressed by Fig. 13.<br />

1st QUAD.<br />

2d QUAD.<br />

3d QUAD.<br />

4th QUAD.<br />

SINE<br />

+<br />

+<br />

—<br />

CCSINB<br />

+<br />

-f


•IS<br />

PLANE TRIGONOMETRY.<br />

48. The particular values of the sine and<br />

cosine at A, A', A", kc, or sin. and cos. of<br />

0°, 90°, 180°, &c., may also be found by<br />

Fig. 13, upon the same principles; but this<br />

we leave to the student.<br />

49. GENERAL REMARK.—In the demon,<br />

strati m of the fundamental formulse for<br />

sin [x ± y), and cos [x ± y), Art. 31, the<br />

A"'<br />

angles x, y and x^y were all taken less than 90° and positive.<br />

In this chapter these formulae have been applied to angles of any<br />

magnitude, and the resulting functions have been shown to take<br />

opposite signs when the lines representing them take opposite directions.<br />

It follows that, in deducing trigonometric formulse from<br />

geometrical figures, we need not embarrass our demonstrations with<br />

the consideration of the various cases of the problem, or of the<br />

various values of the angles of the figure. The formula deduced<br />

from any supposed position of the lines of the figure will be of<br />

general application, provided in the practical application of this formula<br />

to the particular cases, we observe those values and signs of the<br />

trigonometric functions which have now been determined.<br />

50. The results of this chapter may be expressed hy a few general forniulfB.<br />

From (70) it appears that all the trigonometric functions return to the same values<br />

after one or more complete revolutions of 300°. If we represent the semi-circumference,<br />

or two right angles, hy a-(Art. 11), and let n = any whole number or zero,<br />

we shall have<br />

sin 4 n — := 0<br />

cos 4 re — = 1<br />

(81)<br />

sin (4 re + 1) -n- = 1<br />

cos(4re4-l).l- = 0<br />

(82)<br />

sin(4«+2) j = 0<br />

cos(4re+2).^ = —1<br />

(83)<br />

whence<br />

sin (4 n + 3) .^ :<br />

cos (4 re + 3) — = 0<br />

(84)<br />

tan 4 n — = 0 tan (4 n -f- 1) — = 00<br />

tan (4 n -f 2) • tan (4 re-f- 3) — = oo<br />

or the tan. of the even multiples ^,f-;-- = 0, and of the odd multiples =oo , so that<br />

ive may write more simplytan<br />

2 re ^ = 0 tan(2n-f 1) — = co (85)


FUNCTIONS OF ANGULAR MAGNITUDE. 29<br />

In these formulaB we have only to give n one of the values 0, 1, 2, 3, 4, &c., to<br />

obtain the functions of any given multiple of the right angle. Thus, we lind<br />

Bin 450° = sin 5 -1 == sin ^4 -{- 1) ^ = 1 by making n = 1, in (82).<br />

Since the subtraction of 8 re— from the arc will not change tUo functions, lii«<br />

above formulse are also true when re is a negative whole number.<br />

51. In a similar manner we obtain<br />

sin 4 re — + y = siny cos [4 re .^ + y | = cos t/. (86;<br />

sin 1^(4 re + 1) ^ + yj = cos y cos [|(4 re -f- 1) -^ -f- j/l = — sin y (87)<br />

sin |^(4re + 2)|- + 2/j = — siny cos r(4re + 2)|l + yj = —cosj/ (88)<br />

sin {i n-\- S) ^-\-y = — cos y cos (4 re -)- 3) — + y == sin y (8&)<br />

tan [^2 re |- -f 2/] = tan j; tan [^(2 re + 1) ^ + j-J = — cot y (90)<br />

in which n may be any whole number, positive oi negative, and y any angle, positive<br />

or negative.<br />

52. A still more concise form may be given to the formulse of the two preceding<br />

articles, as follows : re being, as before, any whole number, positive or negative.<br />

sin 2 re-|^ = 0 cos 2 re ^ = (— 1)" (91)<br />

sin (2 re -f 1) ^=(—1)» cos (2 re + 1) ^ = 0 (92)<br />

sin 12 re y-f-y | = (—l)"siny cos 12 re y + 2/ = (—1)" cos?/ (93)<br />

B'n[(2n-1 l)|. + y]=(—1)»C0S2/ cos[(2re+l) | + ;/] = —J-lj" sinj (91)<br />

and from these (85) and (90) may be directly deduced.<br />

53. We have seen that an angle being given, there is but one corresponding sine.<br />

On the other hand, a sine being given, there is an indefini'te number of angles corresponding<br />

; for if a denote the given sine, and y any corresponding angle, then a is<br />

also the sine of all the angles<br />

TT — y, 2^+2/, Srr — y, &.O.<br />

— TT — y, —lTT-\-y, — 3 !r — y, &c.<br />

or in general<br />

o2<br />

a z= sin y = sin [ rea- + ( — 1)" 2/] (95)


80 PLANE TRIGONOMETRY.<br />

In like manner if a is a given cosine, and y any corresponding angle,<br />

a = cos y = cos (2 rejr ± y) (96)<br />

and if « is a given tangent corresponding to the angle y,<br />

a = tan y = tan [mr -\- y)<br />

(97_'<br />

SINE AND TANGENT OF A SMALL ANGLE OR ARC.<br />

54. When the angle AOB=x, Fig. 14, -s<br />

very small, the sine and tangent are very nearly<br />

equal to the arc AB, which measures the angle,<br />

\ii the radius being unity ; and the cosine and secant<br />

are nearly equal to 0A=1 (Art. 21). Therefore,<br />

to find the sine or tangent of a very small<br />

angle approximately, we have only to find the<br />

ength of the arc by Art. 9 ; thus<br />

,sin 1" = arc 1" = 0-000004848137<br />

log. sin 1" = 4-6855749<br />

and X being a small angle, or arc, expressed in seconds,<br />

If X is expressed in minutes,<br />

sin X = tan a; = a; sin 1" (98)<br />

sin X = tan a; = a: sin 1' (99)<br />

If X expresses the length of the arc, the radius being unity,<br />

sin X = tan x = x (100)<br />

The employment of these approximate values must be governed by<br />

the degree of accuracy required in a particular application. It is<br />

found, for example, that they are sufiiciently accurate when the<br />

nearest second only is required in our results, provided the angle<br />

does not much exceed 1°.<br />

55. If X and y are any two small angles, it follows from the preceding<br />

article that<br />

sin x-.siny = x sin 1" •.ys\aV' = x:y<br />

that is, tlie sines (or tangents) of small angles are proportional to. the<br />

angles themselves. The application of this theorem, however liko<br />

that of the preceding, must depend upon the accuracy required in<br />

the problem in which it is employed.*<br />

•* For a full discussion of tlie limits under which this theorem may be employed,<br />

.see a pnper, by the author of this work, in the Astronomical .lournal, fCambridee'<br />

Mass.1 Vol i p. ?1.<br />

^ '


GENERAL FORMULA 81<br />

CHAPTER rV.<br />

GENERAL FORMULiB.<br />

56. WE have already obtained four fundamental equations, (36),<br />

(37), (38), and (39), involving two angles, x and y. From these we<br />

shall now deduce a number of formulse, either required in the subsequent<br />

parts of this work, or of general utility in the applications<br />

of trigonometry.<br />

57. The sum and difierence of the equations (36) and (37) are<br />

sin (a; -f" 2/) + ^^"^ (^ — 2/) ^^ 2 sin x cos y<br />

(1^1)<br />

sin {x -\- y) — sin (x — ?/) =; 2 cos x sin y (102)<br />

and the sum and difi'erence of (38) and (39) are<br />

68. If we put<br />

cos (a; + ?/) + cos (x — «/) = 2 cos x cos y<br />

cos {x -\- y\ — cos (a; — ?/) = — 2 sin a; sin y<br />

X -\- y =^ x'<br />

«^—!y = y'<br />

whence 2 a; = a;' -f ?/', x=\{x' -\-y')<br />

equation (101) will become<br />

2?/ = a;'-?/', y^W~y')<br />

sin x' + sin ?/' = 2 sin \ {x' + y') cos J {x' — y')<br />

(1^3)<br />

(10^)<br />

and (102), (103), and (104) admit of a similar transformation. But<br />

since x' and ?/' admit of all varieties of value, we may omit the<br />

accents and apply the formulas to any two angles x and y; we have<br />

thus<br />

-• sin a; + sin ?/ = 2 sm 1 (a; + ^ cos J fe — ^ (105)<br />

sin a; — sin ?/ = 2 cos i (a; + y) sin i (a; — ?/) (1061<br />

cos a; -f cos ?/ = 2 cos J (a; + y) cos Hx — y) _, (107)<br />

cos a; - cos ?/ = - 2 sin J (x + y) sin \ {x - y) (108)<br />

Each of these equations may be enunciated as a theorem; thus


32 PLANE TRIGONOMETRY.<br />

(105) expresses that "the sum of the sines of any two angles la<br />

equal to twice the sine of half the suln of the angles multiplied by<br />

the cosine of half their difi'erence."<br />

These formulse are of frequent use (especially in computations<br />

performed by logarithms), in transforming a sum or difierence into<br />

a product.<br />

69. Dividing (105) by (106), we have by (14) and (15)<br />

or by (2)<br />

sin a; -+- sin ?/ , _ , , , , , , ,<br />

-. r-^ = tan *(x + y) cot i (x — y)<br />

smx— smy ^\ ' ai 2 v J)<br />

sin X -f sin y _ tan \{x +y)<br />

sin X — •m\y tan \{x — y)<br />

and from (107) and (108) we find in the same manner<br />

We find also<br />

a09)<br />

cos a; — cosy<br />

cos a; cosy<br />

= — tani {x + y) tan|(a; — ?/) (HO)<br />

sin a;<br />

cos a;<br />

+<br />

+<br />

+<br />

+<br />

+<br />

sin X — siny<br />

sin?/<br />

= tani(a; + ?/) (Ill)<br />

cosy<br />

sin a; — siny<br />

= tan^(a;-?/) (112)<br />

cos a; cosy<br />

sin X siny<br />

= — cot |(a; — ?/) 1113)<br />

cos a; — cosy<br />

cos a; — cosy<br />

-cot|(a;-fy) (111)<br />

60. Divide the equations (36), (37_), (38) and (39) by cos a cosy;<br />

then by (14) we have<br />

sin (x + y)<br />

-^ -' = tan X + tan y (li 6)<br />

cos X cosy ^ V "y<br />

sin Ix — y)<br />

-^^j^t.nx-Uny (116)<br />

cos (x -\- y)<br />

^ ^ = 1 — tan a; tan y (117)<br />

cos a; cos?/ ^ V-^-^V<br />

cos {x.— y)


GENERAL FORMUL.*;.<br />

61. Divide (36), (37), (38) and (39) by sin a; siny; and by sin x<br />

cos y; then<br />

sin (x ± ?/)<br />

. ^ . -^^ = cot y ± cot X (119)<br />

sm X smy "^ ^ '<br />

cos (x ± ?/)<br />

-V ^ . -^^ = cot a; cot w qp 1 (120)<br />

sm X smy ^ ^^ ^ •'<br />

sin (a; ± ?/)<br />

"^'^ r?7<br />

• 1 ± cot a; tan y (121)<br />

sm a; cos" ^ ^ •'<br />

cos (a; ± y)<br />

^-^ ^ = cot a; q= tan y (122;<br />

sm X cos y ^ '<br />

62. Divide (115) by (117), and (116) by (118); then by (14)<br />

SJi<br />

/ s tana;—^ tan?/ rt^ix<br />

tan (a; — ?A = -— i-—^ (124)<br />

^ '^' 14- tan .-Han?/ ^ '<br />

by which, when the tangents of two angles are given, we may compute<br />

the tangent of their sum or difierence. To find the cotangent<br />

of the sum or difi'erence when the cotangents of the angles are given,<br />

divide (120) by (119),<br />

^, , , cot ?/cot a; =p 1 ,-ioc\<br />

cot (x±:y) = —^—;—^^^- (125)<br />

^ "^^ cot?/±cota; '<br />

63. Dividing (115) by (116), and (117) by (118j, (or from the<br />

equations of Art. 61), we have<br />

sin (a; + ?/) tan a;-j-tan ?/ cot?/-+cota; /loei<br />

sin (x — y) tan x — tan y cot y — cot x<br />

cos(x+y) 1 —tana;tan?/ cota;cot?/—1<br />

cos {x—y)~l-jr tan x tan t/ ~ cot a; cot ?/ -fl<br />

(127)<br />

04. Formulas for secants are obtained from those for cosines by means of (2) j<br />

thus we find<br />

sec (xdcy) = - cos s cos y ::p sm a; sm 2/<br />

and multiplying numerator and denominator by sec x sec y,<br />

I ,' \ sees sec 2/ -mo.<br />

sec (a: ± J/) = -—— -f- (128)<br />

1 q:; tan x tan y


34 PLANE TRIGONOMETRY.<br />

Also since<br />

sec X rfc sec y -<br />

1 , 1 cos y ± cos X<br />

cos a: ~^ cos 2/ cos a; cosy<br />

we find by (107) and (108)<br />

sec X -f- sec y<br />

2 cos J (x 4- y) cos J [x — y)<br />

cos a; cos y<br />

(129)<br />

sec X — sec y<br />

In the same manner from (105) and (106)<br />

2 sin H* + y) sin i(x—y)<br />

cos X cos 2/<br />

(ISO,<br />

, 2 sin J (a; -|- y) cos i (x — 2^)<br />

cosec X 4- cosec y = ^ ^ T ' . ^-^ — (131)<br />

sm X sm y ^ '<br />

2 cos i (a; -|- y) sin i (x — y)<br />

cosec X — cosec y = ^-^—-. —. ^-^ =^ (132)<br />

" - sin X sm y ^ '<br />

These formulse, although generally omitted in treatises on trigonometry, will be<br />

found useful in a subsequent part of this work.<br />

66. The product of (36) and (37), and of (38) and (39), are<br />

sin {x -+- y) sin {x —«/) = sin^ x cos^y — cos'^ x ain^y<br />

cos [x + y) cos {x — y) = cos^a; coa^ y — sin^ x ain^y<br />

By (13) we have cos^ a; = 1 — sin'- x and cos^ y = 1 — sin^ y,<br />

which substituted in the preceding equations, give<br />

sin (x + y) sin [x — y) = sin^ x •— sin^y = coa^y — cos^ x (133)<br />

cos {x + y) cos (x — y) = cos^ x — sin^ ?/ = cos^y — sin^ a; (134)<br />

66. In (36), (38) and (123), let ?/ = a;, we find<br />

-i sin 2 a; = 2 sin a: cos x (l^^j<br />

cos 2 a; = cos^ x — sin^ x (136)<br />

_<br />

2 tan X<br />

by which the functions of the double angle may be found from those<br />

of the simple angle.<br />

67. To find the functions of the half angle from those of the<br />

whole angle, we have, from (13) and (136),<br />

the sum and difi'erence of which are<br />

cos^ X + sin^ a; = 1<br />

cos^ X -^ sin^ X =.cos 2 x<br />

2 cos^ a; =• 1 -f cos 2 a;<br />

2 sin^ a; = 1 — cos 2 x


GENERAL FOEMULIE. 35<br />

As these express the relations of an angle 2 x and its half x, their<br />

meaning will not be changed by writing x and J x instead of 2 a;<br />

and X; whence<br />

2 cos^ J a; = 1-f cos a; (138)<br />

2sin^ia; = l-cosa; (139)<br />

the quotient of which is<br />

1 —cosa;<br />

68. The faUowing may be proposed as exercises.<br />

_ 2 tan J a; 2<br />

1 4" tan' J X cot \x-\- tan J x<br />

(141)<br />

2 tan \x 1<br />

tan a; = = . ^ , = —-j ^, (142)<br />

1 — tan' \x cot J a: — tan \x ^ '<br />

tan' J a; 4- 2 cot a; tan J a; — 1 = 0 (143)<br />

tan' \x — 2 cosec x tan J a; 4- 1 = 0<br />

1 — tan' \ X<br />

1 4- tan" \ X<br />

(•'•^^)<br />

(145)<br />

1 i 1 — cosa; ,, ,„,<br />

tan \ X = cosec a; — cot x = ; (146)<br />

'• sm a; ^ •'<br />

cot i a; = cosec a: 4- cot x = —V (147)<br />

'' ' sm a; ^ •"<br />

. , 1-4-sin a: — cosa; /IAOS<br />

tan lx = —-J (148)<br />

1 -f- sm a; 4- cos a; ,<br />

69 Several useful formulse result from the preceding, by introducing<br />

45° or 30°. If a; = 45° in (36), (37), (38), and (39), we<br />

have, by (33),<br />

cos ?/ d= sin y<br />

sin (45° ± y) = cos (45° =F y) = — .^ (^"^^^<br />

whence<br />

cos ?/ ± sin ?/<br />

tan (45° ± v) = cot (45° =F ?/) = ^ (150)<br />

^ ^'^ ^ •'^ cosy =p smy ^ ^<br />

in which either the upper signs must be taken throughout, or the<br />

lower signs throughout.<br />

If we divide the numerator and denominator of (150) by cos y or<br />

sin?/,<br />

1 rb tan y cot ?/ d- 1


d&<br />

PLANE TEIGONOMETEY.<br />

From this, by (67),<br />

tan?/ — 1<br />

tan (y - 45°) = ^ '' . ^ 152)<br />

^•^ ' tan y + 1<br />

70. Again, let a; = 90° ± ?/ in (138), (139) (140), and^(146),<br />

sin (45° ± i'y) = cos (45° ^ i y) = J (l-^^^ (153)<br />

tan(45°=t=j2.)=J(^fS|)<br />

(^^^^<br />

From the last we find<br />

tan (45° ± * y) = L^^l^ = -^f- (155><br />

^ - •'•' cos y 1 ::^ sm y ^<br />

tan (45° + i y) + tan (45° _ J j') = ^ = 2 sec i/ (150)<br />

tan (45° 4- J j/) _ tan (45° _ J j,) = IfifLZ = 2 tan y (157)<br />

the quotient of which is<br />

tan (45° 4-} 2^)-tan (45°-^ 2,) _<br />

tan (45° + i y) + tan (45° - i y) "'" ^ ^'°°''<br />

71. In (101), (102), (103) and (104), let a; = 30° ; then by (27)<br />

and (28)<br />

sin(30° 4-?/)4-sin(30°-?/) = cos?/ (159)<br />

sin (30° 4- y)- sin (30° - ?/) = sin?/ x/ 3 (160)<br />

cos (30° 4- ?/) 4- cos (30° - ?/) = cos ?/ >/ 3 (161)<br />

cos(30°4-?/)-cos,(30° —?/) = -siny (162)<br />

and in a similar manner we may introduce 60° ; but it is unnecessary<br />

to extend these substitutions, as they involve no difiiculty, and<br />

can be made as occasion demands.<br />

FoKMUL^ FOK<br />

MULTIPLE ANGLES.<br />

72/ From (101) and (102) we have<br />

sin (y -{- x) = 2 sin y cos a; — sin (y — x)<br />

sin {y -[- x) = 2 cos y einx -]- sin (y — x)<br />

in which let j^ = (m — 1) a;; then<br />

sin mx = 2 sin (rei — 1) a; cos x — sin (m — 2) a; (163)<br />

sin mx = 2 cos (m — 1) a: sin x -{- sin (m — 2) a; '164)<br />

which are the general formulse for computing the sine of any multiple mx, from the<br />

lower multiples (m — 1) a: and (m — 2) x, and the simple angle x.


FOEMULiE FOR MULTIPLE ANGLES. 37<br />

If we make m successively 1, 2, 3, 4, &c., these formulge give<br />

sin X == sin x = sin x<br />

sin 2 a; = 2 sin x cos x =2 cos x sin x<br />

sin 3 a; = 2 sin 2 a; cos x — sin a; =2 cos 2 a:, sin a; 4- sin a;<br />

sin 4 a; ^ 2 sin'3 x cos a: — sin 2 a: = 2 cos 3 x sin x -{• sin 2 a:<br />

&c.<br />

&c.<br />

73. From (103) and (104)<br />

cos (2/ 4" ^) = 2 cos y cos a; — cos (y — x)<br />

cos (2/ 4" 3;) = — 2 sin y sin x -\- cos (y — x)<br />

which, if we put y = (m — 1) 2:, become<br />

cos mx 1= 2 cos (m — 1) a: cos x — cos (m — 2) a; {^^^)<br />

cos mx = — 2 sin (m — 1) a; sin x -{• cos (m — 2) a; (166)<br />

If m is taken successively equal to 1, 2, 3, 4, &o.<br />

cos 2: = cos X = cos X<br />

cos 2 2; = 2 cos X cos x — 1 = — 2 sin x sin x -{• X<br />

cos 3 a: = 2 cos 2 a; cos x — cos x = — 2 sin 2 x sin a; -)- cos x<br />

cos 4 a: = 2 cos 3 x cos a; — cos 2 a; = — 2 sin 3 a; sin a; 4- cos 2 x<br />

&c.<br />

&c.<br />

74. In (123) let 2^ = (m — 1) a;; then<br />

tan (m — 1) a; -4- tan x<br />

,-ir.r,s<br />

tan mx = ^ -,—'-^ (167)<br />

whence<br />

1 — tan (m — 1) a; tan x<br />

„ tan 2 X •\- tan x<br />

tan X = tan 2; tan 3 a::— 1 — tan 2 X tan x<br />

2 tan 2; , . tan 3 2; -i- tan x<br />

tan 2 a; = ;—-— tan 4 a; ^ = fc—7 •<br />

1 — tan' X 1 — tan 3 x tan x<br />

&e.<br />

75. If in the expression for sin 3 x, Art. 72, we substitutt, the value of sin 2 x,<br />

we find<br />

sin 3 a: = 4 sin x cos' x — sin x<br />

by which we find the sine of the multiple directly from the functions of the simple<br />

angle. If this be substituted in the expression for sin 4 x, the latter will also be<br />

expressed in terms of the simple angle. By these successive substitutions we easily<br />

nbcain the following tables:<br />

sin X = sin x<br />

sin 2 a; = 2 sin x cos x<br />

sin 3 a; = 4 sin x cos' x — sin x<br />

sin 4 a; =: 8 sin x cos" x — 4 sin a; cos x<br />

&o.<br />

76. cos X = cos X<br />

cos 2 a; = 2 cos' x — 1<br />

cos 3 2: = 4 cos' X — 3 cos x<br />

cos 4 a; = 8 cos* x — 8 cos' a; 4" 1<br />

&c


38 PLANE TRIGONOMETRY.<br />

77. If in these equations we substitute for cos' x = 1 — sin' z they become<br />

sin X = sin a:<br />

sin 2 a; ^ 2 sin 2: ^ (1 — sin' a;)<br />

sin 3 a; = 3 sin x — 4 sin' x<br />

sin 4 a; = (4 sin a; — 8 sin' x) ,y (1'— sin' x\<br />

&o.<br />

78. cos a; =: ^ (1 — sin' x)<br />

cos 2 a: = 1 — 2 sin' x<br />

cos 3 a; = (1—4 sin' a:) .^ (1 — sin' a;)<br />

COS 4 a; ^ 1 — 8 sin' x-\- ^ sin* x<br />

&c.<br />

From the preceding tables it appears that the cosine of the multiple angle may<br />

always be expressed rationally in terms of the cosine of the simple angle ; but that<br />

the sine of only the odd multiples and the cosine of only the even multiples can be<br />

expressed rationally in terms of the sine of the simple angle.<br />

79. By successive substitutions we find from the formulse of Art. 74.<br />

tan<br />

X =: tan x<br />

„ 2 tan X<br />

tan 2 2; = - tan' X<br />

tan 8 X :<br />

tan 4 a; =:<br />

3 tan X — tan' x<br />

1—3 tan' X<br />

4 tan X — 4 tan" x<br />

1 — 6 tan' X 4- tan' x<br />

80. The preceding results are but particular applications of general formulse to<br />

be given hereafter, (Chapter XV.) They are introduced here for the convenience<br />

of reference in elementary applications. The powers of the sine or cosine of the<br />

simple angle may also be expressed in the multiples of the angle: but they are most<br />

readily obtained from the general formulas of Chapter XV.<br />

RELATIONS OF THHEE AUQLES.<br />

81. Let X, y, and e be any three angles ; we have, by (36) and (38),<br />

sin (a; 4- y + ^) = sin (x •\- y) cos 2 4- cos (a; -j- J/) sin z<br />

^ sin X cos y cos z -\- cos x sin y cos z<br />

-[- COS X cos y sin z — sin a; sin 2/ sin z (168)<br />

cos {x-{- y -\- z) = cos {x 4- y) cos z — sin {x -f- y) sin z<br />

= COS X cos y cos z — sin a; sin y cos z<br />

— sin X cos y sin z — cos x sin y sin z (169)<br />

and in the same way we may develop the sines and cosines oi x -\- y — z, 2; —y-\-Z;<br />

&c. ; but we may find these directly from (168) and (169) by changing the sign of z,<br />

y, &c., and observing (56).


RELATIONS OF THREE ANGLES. 39<br />

The quotient of (168) divided by (169) gives, after dividing the numerator ana<br />

denominator by cos x cos y cos z,<br />

tan (x + y + z) = ^an a; 4- tan 2/ 4- tan z — tan x tan y tan z ^^<br />

1 — tan X tan y — tan x tan z — tan y tan z '<br />

82. Let X, y and z be any three angles, and from the equations<br />

let cos z be eliminated; we find<br />

sin (x — z) = sin x cos z — cos x sin z<br />

sin (2/ — z) = sin 2/ cos z — cos 2/ sin z<br />

sin y sin (x — z) — sin x sin (y — z) = sin z (sin a; cos y — cos x sin y)<br />

If sin z is elicunated, we find<br />

:= sin z sin (a; —'• y)<br />

cos y sin (2; — z) — cos x sin (2/ — z) = cos z sin (a; — y)<br />

These equations may be more elegantly expressed, as follows:<br />

sin X sin (y — z) 4- sin y sin (z — a:) 4- sin z sin (a; — y) = 0 (171)<br />

cos X sin (2/ — 2) 4- cos 1/ sin (z — a;) 4- cos z sin (a: —• 2/) = 0 (172)<br />

A number of similar relations may be deduced from these by substituting 90° ± z,<br />

&c., for X, &c.<br />

83. Let<br />

v = i (x-^y + z)<br />

we have by (104)<br />

2 sin V sin (u — a;) = cos x — cos (2 v — x) = cos x — cos (y -[- z)<br />

2 sin (« — 2/) sin (v — z) = cos (y — z) — cos (2 v — y — z)<br />

the product of which is<br />

= cos (y — z) — cos X<br />

4 sin V sin (« — x) sin (« — y) sin (v — z) = cos x [00s (y — 2) 4- cos (y -\- z)]<br />

Reducing the second member by (103) and (134) ;<br />

— cos' X — cos (y 4- 2) cos {y — z)<br />

4 sin V sin (v — x) sin (» — ?/) sin (v — z) = 2 cos a; cos y cos z — cos' x<br />

In the same manner we find<br />

— cos' y — cos' z 4- 1 (173)<br />

4 cos V cos (» — a:) cos (« — y) cos (» — z) = 2 cos a; cos 2/ cos z -j- cos' a;<br />

4- cos' y + cos' z — 1 (174)<br />

84. The following may be proposed as exercises.<br />

sm a; 4-sin 2/4-sin z — sin (x-^-y-\-z) ^ 4 sin J (a; 4-2/) sin J (x-\-z') sin ^ (y-^z) (175)<br />

eoB X •[• cosy-}-cos z-{-aoa(x-\-y-\-z) =4cos J(a;4-2/)oos-J(a;4-z) cos J (y 4-2) (176)<br />

sin (2; 4- y -t- z)<br />

tan x 4- tan y 4- tan z — tan a: tan y tan z = '^ (177)<br />

' " ' cos a; cos y cos z ^ '<br />

cos (2; 4- '/ 4~ ^)<br />

cot a; 4- cot y 4- cot z — cot x cot 2/ cot z = ^ ;—'•—r—- CHSl<br />

^^ ' T^ " sm a: sm y sm z


40 PLANE TRIGONOMETRY.<br />

4 [si n (a;-f 3/4-z) -,-2 sin x sin y sin z]' = 4 [sin [x-\-y) cos z -f- cos (a;—y) sin zj'<br />

= [1 — cos (2 X 4- 2 y)-\ (14- cos 2 z) 4- [1 4- cos (2 a; — 2 t/)] (1 — cos 2z)<br />

4- 2 (sin 2 z -\- sin 2 J/) sin 2 z<br />

(179)<br />

= 2 (14-siu 2 a; sin 2 2/4-sin 2 a: sin 2 z 4" sin 2 y sin 2 z — cos 2 a; cos 2 y cos 2 z) _<br />

85. Let the sum of three angles x, y and z be K-, or a multiple of TT, that is, an even<br />

multiple of — a condition which is expressed by the equation<br />

2:4-2,4-z = 2n.^ (180)<br />

then, tan (x-{- y .-\- z) = 0, and the first member of (170) being thus reduced to<br />

zero, the numerator of the second number must be zero, or<br />

tan X -\- tan y -{- tan z = tan z tan y tan z (181)<br />

an equation, it must be remembered, that is true only under the condition (180).<br />

Since x, y and z may be selected in an infinite variety of ways so as to satisfy (180),<br />

it follows from (181) that there is an infinite number of solutions of the problem,<br />

" to find three numbers whose sum is equal to their product."<br />

Let the sum of three angles z, y and z be — or an odd multiple of -^ ; that is, let<br />

a;4-2/ + z=(2«4-l).| (182)<br />

then, tan (z -\- y -\- z) = cx, and the denominator of (170) must be zero, or<br />

tan X tan y -{- tan x tan z -f- tan y tan z = 1<br />

which, divided by tan x tan y tan z, gives<br />

cot X 4" cot y 4" cot z = cot x cot y cot z (183)<br />

a relation that holds only under the condition (182).<br />

86. Let<br />

x-\-y-\-z = n7r = 2n-^ (184)<br />

We have by (93) and (91)<br />

cos {x -\- y — z) = cos (n ir — 2 z) = (— 1)" cos 2 z<br />

ops {x — y-\- z) = COS (n 9r — 2y) = (— 1)" cos 2 y<br />

cos (y -\- z — z) = cos (nrr — 2 a:) = (— 1)" cos 2 x<br />

cos (y -\- z -{- x) = cos njr = (— 1)»<br />

the sums of the first two and of the second two are by (103)<br />

2 cos X cos (2/ — z) = (— 1)» (cos 2 z 4- cos 2 y)<br />

2 cos X cos (y -f z) = (— 1)" (cos 2 a; -f 1)<br />

and the sum and differende of these equations are<br />

4 cos X cos y cos z = _(— 1)" (cos 2 z 4- cos 2 y 4. cos 2 a; -{- 1)<br />

4 cos X sin y sin z = (— 1)" (cos 2 z 4- cos 2 y — cos 2 a; — 1)<br />

or ±4 cos X cos y cos z = cos 2 a: -^r cos 2 y 4- cos 2 z 4- 1 (185)<br />

± 4 cos a; sin y sin z = — cos 2 2: -|- cos 2 y 4- cos 2 z — 1 (186)<br />

'he upper sign being taken when n in (184) is even, the lower when n is odd.


INVERSE TRIGONOMETRIC FUNCTIONS. 41<br />

In the same manner we obtain<br />

rp 4 sin a; sin 2/ sin z = sin 2 a: 4- sin 2 y 4- sin 2 z (187)<br />

=p 4 sin a; cos y cos z = — sin 2 2; 4- sin 2 y 4- sin 2 z (188)<br />

the signs being taken as .above.<br />

Again, let<br />

2;4.y4-z= (2«4-l)^ (189;<br />

we shall find by the same process<br />

=t 4 sin a: sin y sin z ^ cos 2 a: 4- cos 2y -\- cos 2 z — 1 (190)<br />

± 4 sin 2: cos y cos z = — cos 2 2: 4- cos 2 y -|- cos 2 z -|- 1 (191)<br />

± 4 cos X cosy cos z = sin 2 2: 4- sin 2 y 4- sin 2 z (192)<br />

rb 4 cos X sin y sin z = — sin 2 a: 4- sin 2 y -{- sin 2 z (193)<br />

-p or — according as n in (189) is even or odd.<br />

87. If<br />

INVERSE TRIGONOMETRIC FUNCTIONS.<br />

y = sva. X<br />

y is an explicit function of x, and, since x and y are mutually dependent,<br />

X is an implicit function of y; but to express x in the<br />

form of an explicit function of y, we write*<br />

X = sin ~' y<br />

which is read, "a; equal to the angle (or arc) whose sine is y," and<br />

X is called the inverse functioti of y, or of sine x.<br />

In like manner tan"''?/ is "the angle or arc whose tangent is<br />

y," kc.<br />

88. Many of the formulas already given may be conveniently expressed with the<br />

aid of this not.ation. Thus, by (16),<br />

or if we put y = tan x<br />

X = sec ~' Y^ (1 4- tan' x)<br />

tan-' y = sec-' ^/ (1 4- y')<br />

* This notation was suggested by the use of the negative exponents in algebra.<br />

If we have y = nx, we also have x =z n~^ y, where y is a function of x, and x is<br />

the corresponding inverse function of y. The latter equation might be read " x is<br />

a quantity which multiplied by n gives y." It may be necessary to caution the beginner<br />

against the error of supposing that sin"' y is equivalent to —;——.<br />

For a general view it the nature of inverse functions, see Peirce's Difi". Calo.<br />

Arts. 13, et seq.<br />

6 B2


i2<br />

PLANE TRIGONOMETRY.<br />

And in the same way tho formulse of Art. 28 give<br />

sin —' y = cosec —' — =: cos —'


TRIGONOMETRIC TABLES. 43<br />

CHAPTER V.<br />

TRIGONOMETRIC TABLES.<br />

90. BEFORE prot :'eding to the numerical computation of triangles<br />

and to other applications of the preceding formulse, the student should<br />

make himself acquainted with the arrangement of, and the mode of<br />

consulting, the trigonometric tables. We shall here speak of those<br />

points only that are common to all tables, but it will be necessary<br />

to consult also the explanations that are always prefixed to a table<br />

in order to understand any peculiarity that may attach to it. We<br />

suppose also that he is acquainted with the nature and use of the<br />

common tables of logarithms of numbers.<br />

There are two principal trigonometric tables;* the first, called the<br />

Table of Natural Sines, ^c, contains simply the numerical values<br />

of the sines, tangents, &c. for each given value of the angle; the<br />

* The most convenient seven-figure tables yet published in this country are Sta?iley's,<br />

already mentioned, p. 12. Attached to these are also five-figure tables, and a<br />

table of anti-logarithms.<br />

Computers, engaged in extensive and varied calculations, generally provide themselves<br />

not only with tables of seven figures, but also with those of six, of five, and<br />

even of four figures—the selection and use of a particular table in any case being<br />

determined by the degree of precision sought for in the results. We might, indeed,<br />

employ seven-figure, or even ten-figure tables in all cases, and reject the final figures<br />

of our results, when a lower degree of approximation is thought sufficient; but it is<br />

clearly a loss of time and labor to employ other figures besides those which are ne<br />

cessary in arriving at the proposed degree of precision.<br />

The best six-figure tables are to be found in Bremiker's N'ova Tabula Berolinensis,<br />

(Berlin. 1852,) which are distinguished for simplicity of arrangement, as well as accuracy.<br />

Bowditch's five-figure tables, in his Epitome of Navigation, are valuable on account<br />

of their undoubted accuracy.<br />

Four-figure tables are to be found in various collections, as for instance, in Schumacher's<br />

Il'dlfstafeln, (edited by Warnstorff.)<br />

Of the German seven-figure tables we may cite those of Vega, of which Bremiker's<br />

edition is the best: of the English, Taylor's, Hutton's, Babbage's, Shortrede's; and<br />

if the French, Callet's, Bagay's, Borda's. T.aylor's, Shortrede's, and Bagay's give<br />

the log. functions to every second of the quadrant; Borda's give the functions corresponding<br />

to the centesimal division of angles, (Art. 6.)<br />

For computations requiring more than seven figui'es recourse must be had to the<br />

ten-figure tables of Vlacq, Thesaurus Logarithmorum Completus, edited by Vega,<br />

'Leipzig, 1794.)


44 PLANE TRIGONOMETRY.<br />

second, called the Table of Bogarithmic Sines, ^c, contains the lo><br />

garithras of the numbers in the first table. As the greater part of<br />

the computations of trigonometry are carried on by logarithms, the<br />

latter table is by far the most useful.<br />

TABLE OF NATURAL SINES,<br />

91. The arrangement of this table will be understood from a simple<br />

inspection. It contains the sines, &c. of angles between zero<br />

and 90°, generally for every minute, and the functions of angles<br />

consisting of a number of degrees, riinutes, and seconds, have to be<br />

found by interpolations similar in their nature to those that are required<br />

in using tables of logarithms of numbers. This interpolation<br />

is based upon the supposition that the diiferences of the sines, &c.,<br />

are proportional to the differences of the angles; and this proportion,<br />

though theoretically inexact, gives, in general, a sufiicient approximation,<br />

provided the diflerences of the angles of the table are<br />

sufficiently small. When the greatest accuracy is desired, the tables<br />

should give the angles to every second, or at least to every 10", and<br />

the sines, &c., should be given to at least seven decimal places.<br />

92. As every angle between 45° and 90° is the complement of<br />

another between 45° and 0°, every sine of an angle less than 45°<br />

is the cosine of another greater than 45° ; every tangent is a cotangent,<br />

&c.; hence the angles at the top of the tables generally extend<br />

only to 45°, and the same functions answer for the remaining 45°,<br />

by giving them at the bottom of the table the names of the complemental<br />

functions.<br />

93. As the sines, &c., pass through all their possible numerical<br />

values, while the angle varies from 0° to 90°, the tables are not extended<br />

beyond 90°; but we easily deduce the functions of all other<br />

angles by the principles of Chap. III.<br />

For the functiohs'of an angle between 90° and 180°, we may taks<br />

the same functions of its supplement, observing to prefix the proper<br />

algebraic sign. Art. 39. Thus, from Hutton's Tables we fini<br />

sin 140°, 16' = sin 39° 44' = 0-6392153<br />

cos 140° 16' = - cos 39° 44' = - 0-7690278<br />

tan 140° 16' = - tan 39° 44' = - 0-8-311992<br />

cot 140° 16' = - cot 39° 44' = - 1-2030810<br />

sec 140° 16' = - sec 39° 44 = - 1-3003431<br />

cosec 140° 16' ==- cosec 39° 44 = 1-5644181


TABLE OF TRIGONOMETRIC SINES. 45<br />

remembering that in the 2d quadrant all the functions are negative<br />

except the sine, and its reciprocal, the cosecant.<br />

Or, we may (Ai't. 38) deduct 90° from the given angle and take<br />

from the table the complemental functions of the remainder, prefixing<br />

the signs as before; thus<br />

sin 140° 16' =<br />

cos 50°a6' = &c.<br />

cos 140° 16' = - sin 50° 16' = &c.<br />

which is the better practical method, as the subtraction of 90° may<br />

be performed mentally.<br />

94. For angles between 180° and 270°, we deduct 180° and take<br />

the same functions of the remainder, prefixing the signs that belong<br />

to the 3d quadrant. Art. 41; thus<br />

sin 220° 26' = - sin 40° 26'<br />

cos 220° 26' = - cos 40° 26'<br />

tan 220° 26' = 4- tan_40° 26' &c.<br />

95. For angles between 270° and 360°, we may deduct 270° and<br />

take the complemental functions of the remainder, prefixing the sio-ns<br />

that belong to the 4th quadrant, Art. 43; thus<br />

sin 331° 27' = — cos 61° 27'<br />

cos 331° 27' = 4- sin 61° 27'<br />

tan 331° 27' = -<br />

cot 61° 27' &c.<br />

Or we may take the same functions of the difference between the<br />

angle and 360°, Art. 44, observing the signs.<br />

96. Above 360° we deduct 360°, and take the same functions<br />

with their signs, Art. 45; and if the angle exceeds 720°, 1080°, &c.,<br />

we deduct 720°, 1080°, &c. ; thus<br />

cos 840° 45' = cos 120° 45° = — sin 30° 45'<br />

tan 1372° 13' = tan 292° 13' = - cot 22° 13'<br />

TABLE OF LOGARITHMIC SINES, &;C.<br />

97. In this table we find the logarithms of the numbers in the<br />

Table of Natural Sines arranged in precisely the same manner; it<br />

will therefore require but little additional explanation.<br />

As the sines and cosines are all less than unity (being by their<br />

definitions proper fractions), their logarithms properly have negative<br />

indices; but these are,avoided in the usual manner by increasing the


4b<br />

PLANE TEIGONOMETEY.<br />

index by 10, so that we find the index 9 instead of — 1, 8 instQp,d<br />

of — 2, &c. The tangents under 45° being also less than unity,<br />

the indices of their logs, are also increased by 10.<br />

In some tables, to preserve uniformity, the indices of the logs, of<br />

all the functions are increased by 10, so that the log. secants and<br />

cosecants are always greater than 10. In using these tables, we ha^ e<br />

the general rule that for each log. function added in forming a sum<br />

we must deduct 10 from that sum.<br />

98. Since<br />

we have<br />

sec A<br />

1<br />

cos A<br />

log sec -4. = — log cos A<br />

or since the tabular log. cos. is increased by 10,<br />

log sec A = 10 — log cos A<br />

that is, the log. secant is the arithmetical complement of the tabular<br />

log. cosine. For a like reason log. cosec. is the ar. co. of the log.<br />

sm.; and log. cot. is the ar. co. of the log. tan.<br />

Also since<br />

sin J.<br />

tan A = J<br />

cos^<br />

log ten A = log sin A — log cos A<br />

by which property, together with the preceding, we may obtain by<br />

subtraction only, the log. tan. cot. sec. and cosec. from a table containing<br />

only the log. sin. and cos.<br />

99. When the natural sines, &c. are negative, we shall in this<br />

work indicate it by prefixing the negative sign also to their logarithms<br />

;* thus we shall write<br />

cos 140° 16' = - 0-7690278<br />

and log cos 140° 16' = - 9-8859420<br />

* strictly speaking, negative numbers have no logarithms, but in practice, the<br />

multiplication, division, &c.-of numbers is performed without reference to their signs,<br />

1. e. as if they were all positive, and the sign of the result is then deduced from the<br />

signs of the factors according to the rules of algebra. We employ logarithms simply<br />

to effect the first of these operations, i. a. the multiplication, division, &c. of<br />

the numbers considered as positive; and to facilitate the second operadon, or the<br />

determination of the sign, we prefix to the logs, the signs which are prefixed to the<br />

numbers to which they belong.


CONSTEUCTION OF TEIGONOMETEIC TABLES. 47<br />

A^the logs, of all the trig, functions are positive (being rendered<br />

BO by the addition of 10 where necessary), it will easily be remembered<br />

that the sign in the latter case is not that of the logarithm,<br />

but of the number to which it belongs.<br />

ELEMENTARY METHOD OF CONSTRUCTING THE TRIGONOMETRIC<br />

TABLE.<br />

100. By dividing TT = 3-1415926 by the number of seconds in<br />

180°, we found (Art. 9) the length of the arc 1", and (Art. 54), the<br />

sine of 1", which is sensibly equal to the arc. In the same manner<br />

we find, by dividing by 10800,<br />

and by (7)<br />

sin 1' = 0-0002908882<br />

cos 1' = v^ (1 - sin^ 1') = ^/ [(1 4- sin 1') (1 - sin 1')]<br />

= V (1-0002908882 x-9997091118)<br />

or performing the arithmetical operations<br />

cos 1' = 0-9999999577<br />

Then by (101) and (103)<br />

sin (a; 4- ?/) = 2 sin x cos y — sin [x — y)<br />

cos (a; -f ?/) = 2 cos x cos y — cos {x — y)<br />

in which we can suppose y to be constantly equal to 1' and x to become<br />

successively 1', 2', 3', &c. Thus, first substituting y = V,<br />

sin (a; 4- 1') = 2 sin x cos 1' — sin {x — 1')<br />

cos (a; -f 1') = 2 cos x cos 1' — cos {x — 1')<br />

themif x = 1', 2', 3', &c., we find for the sines<br />

sin 2' = 2 cos 1' sin 1' - sin 0' = 0-0005817764<br />

sin 3' = 2 cos 1' sin 2' - sin 1' = 0-0008726646<br />

sin 4' = 2 cos 1' sin 3' - sin 2' = 0-0011635526<br />

sin 5' = 2 cos V sin 4' - sin 3' = 0-0014544407<br />

&c.<br />

&c.<br />

BT^ for the cosines<br />

cos 2' = 2 cos 1' cos r - cos 0' = 0-9999998308<br />

cos 3' = 2 cos 1' cos 2' - cos 1' = 0-9999996193<br />

cos 4' = 2 cos 1' cos 3' - cos 2' = 0-9999993232<br />

cos 5' = 2 cos 1' cos 4' - cos 3' = 0-9999989425<br />

&c.<br />

kc.


48 PLANE TRIGONOMETRY.<br />

The whole difBculty in this operation consists in the multiplication<br />

of each successive sine or cosine by 2 cos 1' = 1.9999999154; but<br />

.this multiplication is much shortened by observing that<br />

so that if we put<br />

2 cos r = 1-9999999154 = 2 - -0000000846<br />

m = -0000000846<br />

we have 2 cos 1' = 2 — wi and therefore<br />

sin 2' = 2 sin 1' — sin 0' — m sin 1'<br />

sin S' = 2 sin 2' — sin V — m sin 2'<br />

sin 4' = 2 sin 3' — sin 2' — 7n sin 3'<br />

&c.<br />

cos 2' = 2 cos 1' — cos 0' — m cos 1'<br />

cos 3' = 2 cos 2' — cos V — m cos 2'<br />

cos 4' = 2 cos 3' — cos 2' — m cos 3'<br />

&c.<br />

which are computed with great facility.<br />

101. It is not necessary, however, to continue this process beyond<br />

30° ; for by (159) and (162) we have<br />

sin (30° + y) = cosy — sin (30° — y)<br />

cos (30° 4- ?/) = cos (30° — y) — sin y<br />

so that the table is continued above 30° by the simple subtraction<br />

of the sines and cosines under 30° previously found. Thus, making<br />

y successively 1', 2', 3', &c.<br />

sin 30° r = cos 1' - sin 29° 59'<br />

sin 30° 2' = cos 2' - sin 29° 58'<br />

sin 30° 3' = cos 3' - sin 29° 57'<br />

&c.<br />

cos 30° r = cos 29° 59' - sin 1'<br />

J<br />

cos 30° 2' = cos 29° 58' - sin 2<br />

cos 30° 3' = cos 29° 57' - sin 3'<br />

&c.<br />

This last process requires to be continued only to 45° since the<br />

sines and cosines of the angles above 45° will be respectively the<br />

cosines and sines of their complements below 45°.


CONSTRUCTION OF TRIGONOMETRIC TABLES. 49<br />

102. The tangents and cotangents may be found from the sin&s<br />

and cosines by the formulsB<br />

sin X<br />

cos X<br />

tan X = cot X = -:<br />

cos X<br />

sm X<br />

and the secants and cosecants by the formulse<br />

sec X =<br />

1 1<br />

cos X<br />

cosec x<br />

sm X<br />

103. In so extended a computation as the construction of the entire<br />

table, it is necessary to verify the accuracy of the work from<br />

time to time, by separate and independent calculations. By means<br />

of (138) and (139) we can find from the cosine of an angle the sine<br />

and cosine of its half; hence from the cos. 45° = >/ J we can find<br />

sin. and cos. of 22° 30', and from these the sin. and cos. of 11° 15<br />

by the same formulse; and from cos. 30° = J \/ 3 we can find sin.<br />

and COS. of 15°, 7° 30', and 3° 45'. If these agree with those found<br />

by the first process, the whole work may be considered as correct.<br />

104. There are various other angles whose functions can be expressed under finite<br />

forms more or less Smple, and may therefore be employed for the purpose of verification.<br />

Let x = 18°; then 3 a; 4- 2 a; ^ 90° and cos 3 a; = sin 2 a;, whence, by Art. 70,<br />

4 cos' X — 3 COS X = cos 3 2; = 2 sin x cos x<br />

4 cos' X — 3 = 2 sin a;<br />

4 (1 — sin' a;) — 3 = 2 sin 2;<br />

sin' a: 4- J sin a;<br />

= J<br />

which equation of the 2d degree being resolved, gives sin x =<br />

sin 18° = cos 72° =<br />

^ ^ " ^<br />

whence<br />

cos 18° = sin 72° = N/aO + ^v"^)<br />

4<br />

From these by (138) and (139), we find the sine and cosine of 9° and 36°; then<br />

by (37) and (39) those of 36° — 30° = 6°, whence those of 3°; after which it<br />

will be easy to form a table of the exact values of the sines and cosines for every<br />

3° of the quadrant.* These expressions, however, are not of much use, directly,<br />

in tho construction of tables, as we have much better methods; but they lead to a<br />

formula of verification which is of some importance. We findeos36°<br />

= ^^Aii<br />

* A table of this kind is given by Cagnoli in his Trigonometrie.<br />

7 E


60 PLANE TRIGONOMETRY.<br />

And by (102)<br />

sin (72° -f y) — sin (72° — y) = 2 cos 72° sin y = ^ ~ sin j,<br />

sin (36° -f y) — sin (36° — y) = 2 cos 86° sin y = "^ "^ sin y<br />

the difference of these equations gives<br />

sin (36° -f y) — sin (36° — y) = sin (72° 4- y) — sin (72° — y) 4" sin y<br />

which is Buler'a formula of verification. By giving y any value at pleasure, the correctness<br />

of five sines of the tables is examined. By substituting 90° — y for y in<br />

this formula it is easily reduced to the following<br />

sin (90° — y) 4- sin (18° 4- y) 4- sin (18° —y) = sin (54°-}- y) 4- sin (54° — y;<br />

which is known as Legendre's formula, though not essentially different from Euler's.<br />

105. The method that has here been given for computing the trigonometric table,<br />

though simple in principle is nevertheless sufficiently operose. The method by infinite<br />

series, to be given hereafter, will bo found to be much more rapid and simple<br />

in practice.


SOLUTION OF PLANE RIGHT TEIANGLES. 51<br />

CHAPTER VI.<br />

SOLUTION OF PLANE RIGHT TRIANGLES.<br />

106. IN order to solve a plane right triangle, two parts in addition<br />

to the right angle must be given, one of which must be a side,<br />

The solution is effected directly by means of our Mg. 15.<br />

definitions of sine, &c., which are expressed by<br />

the equations (1). As three of the six functions<br />

are only the reciprocals of the other three, we<br />

shall base the solutions upon the foUowino- three ;<br />

(Fig. 15):<br />

, a , b , a<br />

svn A = — cos A = — tan A = -r<br />

0 c 0<br />

Since each of these equations expresses a relation between three<br />

parts-^-an angle and two sides—it follows that in order to apply them,<br />

or in order to solve the triangle trigonometrically, there must be<br />

given two of these parts ; and that of the three parts considered,<br />

one must be an angle while the other two are sides. Thus, if an<br />

angle and side are given, the third part sought must be a side; but<br />

if two sides are given, the third part sought must be an angle.<br />

In every instance the choice of the proper equation will be determined<br />

by the precept,—employ that trigonometric function of the<br />

angle which is equal to the ratio of the two sides considered.<br />

107. CASE. I. G-iven the hypotenuse and one angle, or c and A.<br />

To find a. We consider a, c and A; and since the ratio of a and<br />

c is given by the sine, we have<br />

sin A = — whence a = c sin A (195)<br />

0<br />

To find b. Considering b, c and A, we have the ratio of b and c<br />

expressed by the cosine, or<br />

b<br />

cos A = — whence b = c cos A (19^)<br />

e<br />

To find B. We have 5 = 90° - A.


52 PLANE TRIGONOMETRY.<br />

Tho required quantities a and b being equal to the product of two<br />

factors, the computation is conveniently performed by the addition<br />

of the logarithms of these factors.<br />

EXAMPLES.<br />

1. Given c = 672-3412, A = 35° 16' 25"; to find the other parts.<br />

By (195) By (196)<br />

e = 672-3412 log 2-8276897 log 2-8275897<br />

A = 35° 16' 25" log sin 9-7615382 log cos 9-9119049<br />

a = 388-2647 log* 2-5891279 5 = 548-9018 log* 2-7394946<br />

Ans. a = 388-2647<br />

b = 548-9018<br />

B = 54° 43' 35"<br />

2. Given a = 42567-2, B = 87° 49' 10"; find the other parts.<br />

Ans. a = 1619-626<br />

b = 42536-37<br />

A= 2° 10' 50"<br />

108. CASE II. Given the hypotenuse and one side, or c and b.<br />

To solve this case trigonometrically, we must first find an angle.<br />

To find A. We have<br />

cos A = ^ (197)<br />

To find a.<br />

We have, by the preceding case,<br />

sin J. = — a = c sin J. (198)<br />

c<br />

But a may be found by geometry from the equation<br />

a^ -j-52 = c2 whence a^ = c^ — h^<br />

a = ^ {c^- b') = v' [(e 4- b) {c - 6)] (199)<br />

EXAMPLES.<br />

1. Given c = 672-3412, b = 548-9018; find A and a.<br />

By (197) By (198)<br />

b = 548-9018 log 2-7394946<br />

c = 672-3412 log 2-8275897 log 2-8275897<br />

A = 35° 16'25" log cos 9-9119049 log sin 9-7615382<br />

a = 388-2647 log 2-6891279<br />

* Ten is rejected from each of these indices because the logarithms of the sine<br />

• and cosine in the table are ten too great. Art. 97


SOLUTION OF PLANE RIGHT TRLANGLES. 53<br />

By (199)<br />

c= 672-3412<br />

b = 548-9018<br />

c + b = 1221-2430 log 3-0868021<br />

c - b= 123-4394 log 2-09145-38<br />

2)5-1782559<br />

a = 388-2647 log 2-5891279<br />

Ans. A =.36°16'25"<br />

B = 64°43'35"<br />

a = 388-2647<br />

2. Given c = -092357, b = -058018; find a.<br />

Ans. a = -071859<br />

109. CASE III. Given an angle and its adjacent side, or A and b.<br />

To find a. We have<br />

tan A = -r whence a = b tan A. (200^<br />

To find e.<br />

We have<br />

cos A = — whence, by (2), e = -r = b sec A (201)<br />

or directly from the secant<br />

see A — -J- whence c = b sec A<br />

b<br />

EXAMPLES.<br />

1. Given A = 88° 59', 5 = 2-234875 : find the other parts.<br />

Ans. a = 126-9365<br />

c = 125-9563<br />

5 = 1° r<br />

2. Given B = 60°, a = 10; find e. (See Art. 29).<br />

Ans. c = 20.<br />

110. CASE IV. Given an angle and its opposite side, or A and a.<br />

To find e. We have<br />

sin JL = — c = -.—T = a cosec A (202)<br />

c sm J. ^ '<br />

To find b. We have<br />

tan ^ = 4 5 =^-^ = a cot JL (203^<br />

6 tan A


64 PLANE TEIGONOMETEY.<br />

EXAMPLES.<br />

1. Given A = 25° 18'48", a = -085623; find 5.<br />

Ans. b = -1810278<br />

2. Given B = 39° 17'5", 6 = -01; find c.<br />

Ans. c = -0157934<br />

111. CASE V. Given the two sides, or a and b.<br />

To find A and B. We have<br />

To find c. We have<br />

tan A = cotB = j (204)<br />

sin ^ = — c = --. 7 = a cosec A (205)<br />

0 sm A ^ '<br />

We may also find c directly by geometry, from<br />

e^ = a^ 4- 6^ whence e = -y [a^ + b^)<br />

but this is not readily computed by logarithms.<br />

EXAMPLES.<br />

1. Given a = 30, 5 = 40; find c. Ans. c = 60<br />

2. Given a = 8-678912, b = 2-463878; find A and c.<br />

Ans. J[= 74°9'4"-l<br />

c = 9-021875<br />

ADDITIONAL FoEMCLiE FOE. EIGHT<br />

TEIANQLES.<br />

112. By inspecting the tables it will be seen that when the angles are very small,<br />

the cosines differ very little from each other ; consequently a small angle cannot be<br />

found with very great accuracy from its cosine. For a similar reason an angle that<br />

is nearly 90° cannot be accurately computed from its sine. It is therefore desirable,<br />

when a reqiiired angle is small, to find it by its sine, and when near 90° by its cosine,<br />

or in either case by its tangent or cotangent; and for this purpose special formula!<br />

.are sometimes necessary. We shall deduce several such formulse, from which<br />

one adapted to a particular case may be selected.<br />

113. From (197) we find, by (139^<br />

., . n . « . . ^ ^ c — b<br />

1 — cos yl = 2 sm' A ^ = 1 =<br />

c c<br />

= Jte)<br />

(206)<br />

which may be used instead of (197), when A is small, that is when b is nearly equal<br />

to c. It glTes also<br />

c-'b = 2c sin' J A (207)<br />

by which c — b may be accurately found when A is small


ADDITIONAL FORMULA FOE EIGHT TRIANGLES, 55<br />

Also from (197), by (140)<br />

-M = jG-^-^)=JC^)="-^ (20S)<br />

114 From the equation<br />

we deduce by (153) and (154)<br />

Bin A = —<br />

c<br />

Bin(4.5°±J^)=J(^'') (209)<br />

a<br />

and from tan J4 = -7- we find by (151),<br />

cos (45° ± M) = J ( ^ ) (210)<br />

tan(45°±J^) = J(i0) (211)<br />

115. By (136) we have<br />

tan (45° dt= A) = l^!^ (212)<br />

o zp. a<br />

cos 2 A ^ cos' A — sin' A = 4'—a'<br />

(vhich, since 2.4=.4-f-90° — .8 = 90° — (B — A), gives<br />

By (135)<br />

Bin(^-^)=i^-^/-'') (213)<br />

cos (.B — ^) = sin 2 ^ = 2 sin ^ cos .4 = - ^ (214)<br />

and from (213) and (214)<br />

tan(^-^)=i^ + |)i^) (215)<br />

by which B — ^ is found with great accuracy when b and a are nearly equal.<br />

EXAMPLE. Given


66 PLANE TRIGONOMETRY.<br />

rapid change of these diJerences. We avoid the use of these large difl'erences, and<br />

gain somewhat in accuracy, by employing the approximate value of sin. J A given<br />

by (98), whence<br />

sin M = J ^ Bin r, iA = ?^^Ai<br />

sin 1"<br />

Thus we have found above log sin J J4 = 7-9160547<br />

Art. 54, log sin 1" = 4-6855749<br />

J ^ = 1700"-12 = 28' 20"-12<br />

log J A ='3'-2304798<br />

But to obtain J A with the utmost precision, recourse must be had to the following<br />

process, which is constantly employed in observatories, and wherever sma; 1 angles<br />

are to be computed with extreme accuracy. Special tables are prepared containing<br />

for every minute from 0° to 2° the logarithms of<br />

sin X<br />

tan x<br />

which do not vary rapidly, and may therefore be taken with accuracy from the<br />

tables. Then we have<br />

sin a; = a;, sin a:<br />

tan a: = a;.<br />

X<br />

tana:<br />

z<br />

Bin X<br />

tana;<br />

ANjtable of this Hnd will be found on page 156 of Stanley's Tables, where the<br />

notation used is<br />

q r= log sin x,<br />

n = log x<br />

and therefore in the column marked j—n we find the log . Thus in the above<br />

example we have found log sin J ^ = j = 7-9160547<br />

and from the table - q—n = 4-6855700<br />

J .4 = 1700"-14 = 28' 20"-14 log J ^ = „ = 3-2304847<br />

which is the true value ot ^ A within 0"-01.<br />

Stanley's Table contains also the values of<br />

, tan X<br />

log = q — « (? = log tiin ^> n = log 2:1<br />

'"S ilTS = ? 4- « (? = log cosec X, n = log x)<br />

^°^ te^ = ? 4- » (? = log cot X, n = log x)<br />

the use of which may easily be inferred from the example just given.


FOEMULiE FOE OBLIQUE TRIANGLES. 57<br />

CHAPTER VII.<br />

FORMULA FOR THE SOLUTION OF PLANE OBLIQUE TRIANGLES<br />

116. As every oblique triangle may be resolved into two right<br />

triangles by a perpendicular from one of the angles upon the<br />

opposite side, we are enabled to deduce all the formulse for their solution<br />

from those of the preceding chapter.<br />

117. The sides of a plane triangle are proportional to the sines<br />

of their opposite angles.<br />

Denote the angles of the triangle ABQ,<br />

Pig. 16.<br />

Fig. 16, by A, B and Q, and the sides opposite<br />

these angles respectively by a, b and a.<br />

From G draw GP perp. to AB and put<br />

QB = p. Then in the right triangles A G.P, B QB, we have,<br />

by (195)<br />

whence<br />

p = b sm. A,<br />

5 sin ^ = a sin B<br />

which, converted into a proportion, gives<br />

p = a ain B<br />

a : b = sin A : sin B<br />

and in the same way we may prove that<br />

a : c = ain A : sin Q<br />

b : c — ain B : sin G<br />

and these three proportions may be written as one, thus :<br />

or thus,<br />

a -.b : c = sin A : sin B : sin. G<br />

a l e<br />

sin A sin B sin G<br />

When the perpendicular falls without the<br />

triangle. Fig. 17, the angle GBP is the supplerafent<br />

of B, but by Art. 39, it has the same<br />

8in«, so that the triangle GBP gives A<br />

^ = a sin QB P = a sin J5<br />

Fig. 17.<br />

(216)<br />

(217)<br />

(218)


fiS<br />

PLANE TRIGONOMETRF.<br />

the same as was found from Fig. 16. The proposition i^ therefore<br />

general in its application.*<br />

118. The sum of any two sides of a plane triangle is to their difference<br />

as the tangent of half the sum of the opposite angles is to<br />

the tangent of half their difference.<br />

For, by the preceding article,<br />

a : b = sin A : sin B<br />

whence, by composition and division,<br />

a -h b : a — b = ain A -{- sin B : ain A — sin B<br />

But from (109) ii x = A, y = B yfe obtain the proportion<br />

sin J. 4- sin 5: sin A — sinB = tan J (J. 4- JB) : tan ^{A — B)<br />

which, compared with the above, gives<br />

a 4- 6 : a - 6 = tan i (J. 4- 5) : tan J (^ - ^) (219)<br />

This may also be written<br />

a 4- S _ tan i{A+B)<br />

b tan ^{A-B)<br />

(220)<br />

and we may infer the same relation between b, c, B, Q and a, c,<br />

A, Q.<br />

119. The square of any side of a triangle is equal to the sum of<br />

the squares of the other two sides diminished by twice the rectangle<br />

of these sides multiplied by the cosine of their included angle.<br />

In the triangle ABQ, Figs. 16 and 17,*<br />

we have either<br />

^ BP = c-AP or BP=AP-o<br />

but in both cases<br />

But the triangle A GP gives by (196)<br />

BP^ = AP^ + c^ — 2cx<br />

AP<br />

Adding QP^ to both members, we find<br />

AP<br />

a2 = 52 + c^-2c<br />

=b cos A<br />

xAP<br />

* The consideration of Fig. 17 was not strictly necessary according to the principle<br />

stated in Art. 49. It may, however, be useful for the student to verify thai<br />

principle when convenient.


FORMULAE FOR OBLIQUE TRIANGLES. 59<br />

which substituted in the preceding equation gives<br />

as was to be proved.<br />

a^ = 53 -f c'- - 2bc cos A (221)<br />

We have in the same way<br />

53 = a2 4- c2 _ 2 -xc cos B (222)<br />

c^ = a" -{•b^-2ab cos (?> (223)<br />

120. The same result is obtained from the following equations (which are evident<br />

from Fig. 16, where c = AP-\- BB}<br />

From the first of these<br />

whence<br />

a =z b cos G -{- c cos B "I<br />

i = e cos yl 4- fflcos C I (224)<br />

c = a cos B -\- b cos A<br />

c cos B ^ a — b cos 0<br />


60 PLANE TRIGONOMETRY.<br />

This may be simplified by representing the half sum of the three<br />

sides by s, or by putting<br />

a-f5-fc = 2s<br />

whence<br />

a-b + c = a-i-b-i-c-2b = 2s-2b = 2{s-b)<br />

a-^b — c=a + b-i-c-2c = 2s-2c = 2{s — c)<br />

which substituted in (226) give<br />

sin-.^^(-^H^--) (227)<br />

In the same manner we should find from (222) and (223)<br />

si,. X 5 = (i^^liHf), 3in^i(7=^^^=4r^- (228)<br />

122. Add both members of (226) to unity; then<br />

But by (138)<br />

, 2bc-hh^ + c^-a^ (b-hcY-a^<br />

l + cosA= 26^ = 2bc<br />

1 4-cos J. = 2cos^i^<br />

Also, (5 4- e)' - a' = (5 4- e 4- a) (5 4- (? — a)<br />

therefore<br />

crf-A<br />

cos 2 ^ -<br />

(5+c4-a)(54-.-a)<br />

4 j^<br />

Substituting g in the numerator as in the preceding article<br />

and therefore also<br />

cos^i^=lii_Z^ (229)<br />

cos^B = t(l^, eosHa=ii^) (230)<br />

123. Dividing (227) by (229), we have, by (14)<br />

and in the same manner<br />

tanHg = (^-;H«-^),<br />

,,,.^a^is-a){s-h)<br />

s{s — b^ ^ 8{s — c)<br />

(232)


FORMULA FOR OBLIQUE TRIANGLES. 61<br />

124. The preceding formulse are sufficient for the resolution of all the usual cases<br />

of plane triangles ; but there are others that are occasionally useful. From (218)<br />

we find, by (105), (106) and (135),<br />

a-Jf- b _ sin 4 4- sin B _ sin j (A -\- B) cos ^{A — B)<br />

c sin C sin J C cos | 0<br />

a — 4 sin .4 —• sin B cos^J (4 4"'.^) sin J (4 — B)<br />

c sin C sin J C cos J Q<br />

But since .4 4- J? 4- C = 180°,<br />

.4 4- i? = 180° — C, J {A -f .B) = 90° — J C<br />

sin J (.4 4- 5) = cos J 0,<br />

by means of which we find<br />

cos J {A -f ^) = sin J C7<br />

a-\-b _ cos i {A — B) _ cos } (A ~ B)<br />

c sin J (7 cos \ [A -\- B)<br />

a~b _ sin J (.4 — ^) _ sin J [A — B)<br />

c cos J C sin J (^ 4- -^)<br />

(233)<br />

(234)<br />

The quotient of (233) divided by (234) gives (220).,<br />

125. Adding unity to both members of (233), or subtracting it, we have, oy<br />

(103) and (104)<br />

a-\- b -{. c _ cos I [A-{- B) -\- cos j- [A — B) _ 2 cos J ^ cos J i?<br />

c cos I (4 4- JB) sin J 0<br />

a -\- b — c cos^ [A — B) — cos ^ [A-\- B) 2 sin |^ ^ sin j- B<br />

c cos J [A -\- B) sin J C<br />

Similarly from (234) we find<br />

c-f a —4 _ sin i (4 4- ^) 4- sin j (A — B) _ 2 sin J ^ cos J 5<br />

c sin J (4. -\- B) cos | 0<br />

_c —^4-_4 _ sin ^ (4 4- -B) — sin | (4 — B) _ 2 cos ^ A sin \ B<br />

c sin J (.4 •}- B) cos J G<br />

Substituting s =: J (a 4- 5 -j- ")> these equations become<br />

'<br />

c<br />

cos J A cos J B<br />

sin J C<br />

(2S5)<br />

a — c sin ^ A sin J i?<br />

c sin J C<br />

(236)<br />

3<br />

t<br />

— 4<br />

C<br />

—<br />

c<br />

a<br />

sin<br />

M cos J<br />

cos iG<br />

B<br />

cos J 4 sin J B<br />

cos ^ G<br />

F<br />

(237)<br />

C2S«\


62 PLANE TEIGONOMETEY.<br />

From these equations we can deduce immediately (227) &o. ; for example exchanging<br />

c for 4 in (236), we have<br />

« — 4 ^_ sin ^ A sin ^ (7<br />

4 sin J 5<br />

which, multipUed by (236) gives (227).<br />

126. Four times the product of (227) and (229) is by (135)<br />

whence<br />

Bin' ^ = ^j^ . » (J — a){s — 4) (» — c)<br />

BinA = -^,/ls{s — a)(s-b){3 — c)] (239)<br />

Exchanging A for B and C successively, this gives also<br />

sin B = —^/la (« — a) {a — b) (« — c)] (240)<br />

In these equations put*<br />

sin


FOEMULiE FOE OBLIQUE TELINGLES. 63<br />

128 The following equations are added as exercises.<br />

tan i A s — 4 ,^ ,„ s — e<br />

?--55 = , tan A A tan i B = •<br />

ta.ni B s — a ^ ^ s<br />

sin [A — B) (g 4- 4) (a — 4)<br />

sin {A-\- B)~ c'<br />

tan J A tan J B cot J C =<br />

cot J ^ 4- cot } i? 4- cot J C = -^<br />

{s-cy<br />

A'<br />

sin i ^ sin i .B sin i C = —;—-<br />

abca<br />

Ks<br />

cos J J1 COS i JS cos i C = —;—<br />

^ ^ ^ abc<br />

tan J .4 tan J i? tan J C = —<br />

£^ z= « (« — a) (i— 4) (s — c)<br />

= i [44'(T- - (a' - 4' -


tt4<br />

PLANE TRIGONOMETRY.<br />

Kg. 18.<br />

CHAPTER VIIL<br />

SOLUTION OF PLANE OBLIQUE TRIANGLES.<br />

129. CASE I. Given two angles and one<br />

side, or A, B and a. Fig. 18.<br />

To find the third angle. We have<br />

C=180°-(J.4-.B)<br />

To find b and c. We apply the theorem of Art. 117, and state<br />

the proportions thus: the sine of the angle opposite the given side ia<br />

to the sine of the angle opposite the required side, as thi' given side<br />

is to the required side. Thus we have<br />

whence<br />

and<br />

whence<br />

ainA:sinB<br />

= a:b<br />

asmB<br />

0 = —-.—T- = aamB cosec :A (244)<br />

smA<br />

sin A: sin Q = a: c<br />

a sin (7 . ~<br />

c = —f—T— = asmG cosec : A (245)<br />

am. A<br />

EXAMPLES.<br />

1. Given A = 50° 38' 62", B = 60° 7' 25"<br />

to find G, b and c.<br />

A = 50° 38' 52"<br />

B = 60° 7' 25"<br />

G = 69° 13' 43"<br />

a = 412-6708<br />

A + B = 110° 46' 17"<br />

G = 69° 13' 43"<br />

By (244).<br />

log cosec 0-1116730<br />

log sin 9-9380702<br />

log 2-6166037<br />

log b 2-6653469<br />

h = 462-7505<br />

and a = 412-6708,<br />

By (245).<br />

log cosec 0-1116730<br />

log sin 9-9708129<br />

log 2-6156037<br />

log c 2-6980896<br />

c = 498-9875


SOLUTION OP OBLIQUE TRIANGLES. 65<br />

2. Given A = 100° 16' 35", B = 25° 16' 13", and b = 29-167<br />

find a and c.<br />

Ans. a = 67-22857<br />

c = 55-59178<br />

130. CASE I. Given A, B and a. Second solution. We find C = 180° — (4 -f B) ;<br />

then, by (233) and (234)<br />

, , coal (B — G) COS cos J i{B~G) (.B —. G) „, ,<br />

4 4- c = a . 1\„. J = a . sin ^-^T—i—'- J J4 (24B<br />

' COsiiB-\-G) sin i/( y '<br />

sin J (B — G)<br />

which give the sum and difi'erence of the required sides ; adding half the difference<br />

to half the sum, we find the greater side, and subtracting half the difference from<br />

half the sum, we find the less side.<br />

131. CASE I. Given A, B and a. Third Solution. When A and B are nearly equal,<br />

and great accuracy is desired, we may compute the difference between a and 4 ; for<br />

we have, from (244),<br />

a — 4 = a<br />

a sin B sin A — sin B<br />

sin A ' ain A<br />

^ _ J ^ 2 g cos i (^ 4- B; sinijA- B)<br />

sin A ^"<br />

EXAMPLE. Given A = 35° 40' 12"-3, B = 35° 37' 48"-6, and a = 26246-948.<br />

A = 35° 40' 12"-3<br />

B = 3-5° 37' 48"-6<br />

i [A -{• B)=^ 35° 39' 0"-5<br />

i{A — B)= 0° 1' ll"-9<br />

ffl = 26246-948<br />

a — b= 25-499<br />

4 = 26221-449<br />

log cosec 0-2342442<br />

log 2 0-3010300<br />

log cos 9-9098720<br />

log sin 6-5423038<br />

log 4-4190788<br />

log 1-4065288<br />

0-23424<br />

0-30103<br />

9-90987<br />

6-54230<br />

4-41908<br />

1-40652<br />

One of the advantages of this process is, that a —-4 may be found with sufficient<br />

accuracy with five-figure tables, as in the second column of logarithms above. If a<br />

had been given to ten figures instead of eight, we should still have been able with<br />

the seven-figure logs, to find a — 4 to seven figures, and therefore 4 to ten figures,<br />

which could not be done by the ordinary methods without ten-figure tables.<br />

132. CASE II. Given two sides and an angle opposite one of<br />

them, or a, b and A.<br />

To find B. To find the angle opposite the other given side, we<br />

apply Art. 117, and state the proportion thus : the side opposite the<br />

given angle is to the side opposite the required angle as the sine of<br />

the given angle is to the sine of the required angle. Thus, with the<br />

present data, we have<br />

, . .1 . T. , . _ 5 sin ^ ^,<br />

a: 0 = sin ^ : sm B whence sm B = (249)<br />

9 F2


66 PLANE TRIGONOMETRY.<br />

To find G. •\5Pe have. C= 180° -{A + B)<br />

To find c,.. Having found Q, we now have the data of Case I,<br />

therefore,, by (245)<br />

c = 0! sin C cosec A (250)<br />

Fig. 19.<br />

133. It is shown in geometry that when two<br />

sides and an angle opposite one of them are<br />

given, there may be constructed two triangles,<br />

as in Fig. 19, whenever the given angle is ^.<br />

acute and the given side opposite to it is less<br />

• than the other given side. In one of them, the required angle B is<br />

acute, and in the other it is obtuse, and the two values are supplements<br />

of each other ; for<br />

B = BB'Q^ 180°-AB' G<br />

These two values of B are given in the trigonometric solution by<br />

the consideration that sin B found by (249) is at once the sine of an<br />

acute angle, and the sine of its supplement. Art. 39.<br />

In general, when an angle is determined<br />

only by its sine,it admits of two values, supplements<br />

of each other, unless the conditions of<br />

the problem' are such as to exclude one of these<br />

values. In the present case, the obtuse value<br />

of B is excluded when a is greater than b, and<br />

there is but one triangle whether A is acute or<br />

obtuse, as in Fig. 20.<br />

134. If the given parts were such that<br />

a = b ain A<br />

a would be equal to the perpendicular from Q upon the side e, and we<br />

should have but one solution, namely, a right triangle, B and its<br />

supplement both being 90°.<br />

135. If the given parts were such that<br />

a 90°.<br />

136. When there, are two solutions, represent the two values of .B<br />

by B' and B", then the two values of G will be<br />

G' ^ 180° -{A + B') = 180° -B' -A = B"-A (251)<br />

G" = 180° - {A 4- B") = 1%0°-B"-A = B' -A (252)


and the two values of e will be<br />

SOLUTION OF OBLIQUE TRIANGLES. 67<br />

c' = a sin Q' cosec A c'' = a sin Q" cosec A (253)<br />

EXAMPLES.<br />

1. Given a = 31-23879, b = 49-00117 and A = 32° 18'; find B, Q<br />

and G.<br />

a = 31-23879<br />

b = 49-00117<br />

A= 32° 18'<br />

B'=<br />

66°66'56".3<br />

5" = 123° 3' 8".7<br />

G'= 90° 46' 3"-7<br />

C"=<br />

24°38'56".3<br />

ar. CO. log 8-5063068<br />

log 1-6902064<br />

log sin 9-7278277<br />

log sin 9-9233399<br />

log sin 9-9999627<br />

log cosec A 0-2721723<br />

log a 1-4946942<br />

log sin 9-6201962<br />

0-2721723<br />

1-4946942<br />

log c' 1-7668292 log c" 1-3870627<br />

c' = 58-46601 c" = 24-38163<br />

Ans. B = 56° 56'56"-3 ^ C-B^ 123° 3' 3"-7<br />

(7= 90° 45' 3"-7 lori C= 24°88'56"-3<br />

c = 58-45601 J t c = 24-38163<br />

2. Given a = -061234, h = -042356, ^ = 55° ; find B, G and c.<br />

Ans. B = 42° 37' 32"-7<br />

Q= 82° 22'27"-3<br />

c= -06199202<br />

3. Given a = -042356, b = -051234, ^ = 55° ; find B, G and c<br />

Ans. B = 82°<br />

WS5"-1<br />

(7 = 42° 46'24"-3 )-or <<br />

c = -03610331<br />

f 5 = 97°45'24"-3<br />

Q = 27° 14' 35"-7<br />

c = -02366993<br />

4. Given a = 40, 5 = 50, A = 63° 7'48"-4; find B.<br />

Ans. B = 90°.<br />

5. Given a = 40, 6 = 60, J. = 60° ; solve the triangle.<br />

Ans. Impossible.<br />

6. Given b = 40, c = 50, i? = 100° ; solve the triangle.<br />

Ans. Impossible,


68 PLANE TRIGONOMETRY-.<br />

fig 21. _ 137. CASE II. Given a, 4 arid ^. Second Solution. W«<br />

may solve, separately, the two right triangles AJPG, BPG,<br />

Fig. 21, which is a convenient method when there are<br />

two solutions. We first find B by (249) ; then we have<br />

^P = 4cos4, .BP= a cos JB<br />

and<br />


SOLUTION OF OBLIQUE TEIANGLES. 69<br />

2. Given a = 31-0005, b = 15-1101, Q = 10° 15'; find A and B.<br />

Ans. A = 160°17'13".7<br />

B = 9° 27'W.3<br />

8. Given a = 2-4*878, b = 9-021876 and Q = 74° 9'4"-2; find<br />

A and B.<br />

Ans. A = 15°50'55"-8<br />

B = 90° 0' 0"<br />

4. Given b = 15-1101, c= 31-0005, A = 10° 15'; find 5 and Q.<br />

Ans. B = 9°27'46"-3<br />

G = 160° 17' 13"-7<br />

139. Having found A and B as above, the most convenient mode of finding c is, bj<br />

(!i33) or (234), which give<br />

.= (g+4)i^!H£±4) (2.56)<br />

' cos J (4. •—• B)<br />

^ '<br />

,, sin l(A-\- B)<br />

c = (a — b) -.~iy.~^~-J (257)<br />

^ ' Bin J (^ — B) ^ ^<br />

for we have, from the process of finding A and B, the log. of g 4" ^> or of a — 4,<br />

and the values of J (^ -f- B) and J (.4 — P), so that we have only two new logs, to<br />

find, which are taken out at the same opening of the tables with the tangents of<br />

i{A-JfB)s.ndi(A—B).<br />

140. CASE III. Given a, b and Q. Second Solution. When a<br />

and b are given by their logarithms, which occurs when they are<br />

deduced by a logarithmic process from other data (as, for example,<br />

in the computation of a series of triangles in a survey), we proceed<br />

as follows. Let x be an auxiliary angle, such that<br />

tan a: = -I" (258)<br />

an assumption always admissible, since a tangent may have any<br />

value from 0 to oo .<br />

We deduce<br />

tan X — 1 a — b<br />

tan a; -f 1 a -f 6<br />

or by (152)<br />

tan (a; - 46°) = ^ ^<br />

ivhich substituted in (254) gives<br />

tan 1 (.1 - .B) == tan (a; - 45°) tan ^{A-^B) (259)


70 PLANE TRIGONOMETRY.<br />

We find X from (258) and employ its value in (259). As this<br />

method does not require the preparation of a -f J and a — 5, it is<br />

quite a? short in practice as (254).<br />

EXAMPLE.<br />

Given log a = 8-7950941, log b = 8-3706056, and (7= 110° 32'.<br />

(Same as Ex. 1. Art. 138.)<br />

log a = 8-7950941<br />

log 5 = 8-3706056<br />

X = 69° 22' 46"-8 log tan 0-4244885<br />

X - 45° = 24° 22' 46"-8 log tan 9-6562825<br />

J (JL 4- B) = 34° 44' log tan 9-8409174<br />

^{A-B)-= 17° 26' 32"-9 log tan 9-4971999<br />

141. CASE III. Given a, 4, and G. Third Solution. To express .4 or .B directly<br />

in terms of the data, we have, from (218) and (224)<br />

the quotient of which is<br />

and in the same manner<br />

c sin A =zz a sin 0<br />

c cos .4=4 — a cos G<br />

. a sin C /„»„,<br />

tan A = J 7, (260)<br />

4 — a cos G ^ ''<br />

tan 5 = - ^ - ^ (261)<br />

a — 4 COB O<br />

'• ^<br />

142. CASE III. Given a, 4 and G. Fourth Solution. To find c directly from the<br />

data, we have, by (223)<br />

o" = a» 4- 4' — 2 a4 cos C<br />

which, however, is not adapted for logarithmic computation.<br />

follows. Substitute by (139)<br />

then<br />

cos (7 = 1 — 2 sin' J G<br />

c* = o' 4- 4' — 2 a4 -f 4 ai sin' J C<br />

= (a — by+4: ab sin' | G<br />

It may be adapted as<br />

, IS / /i I 4 a4 sin' i G\<br />

(a-4)'<br />

(262)<br />

Let X be an auxiliary angle, such that<br />

tan'I :<br />

4 04 sin' } G<br />

(a-4)'<br />

2 sin i G ,—r ,„,.„,<br />

tan X = _J ^ab (263)


SOLUTION OF OBLIQUE TRIANGLES. 7]<br />

then the radical in the above equation becomes ^/(l 4- tan' x) = sec 2;; tierefore,<br />

c = (a — 4) sec x (264)<br />

143. Wo may also adapt (223) for logarithmic computation by means of (138)<br />

which gives<br />

whence<br />

Let<br />

cos C = — 1 4- 2 cos' J O<br />

c* = a« 4- 4' 4- 2 a4 — 4 a4 cos' J G<br />

1<br />

, , , , / / . , 4 a4 cos' i C\<br />

(265)<br />

2 cos i (7 ^—J- _„..,<br />

sin a; ^ -A—. .^ ab (266)<br />

(2 -{- 4<br />

then the radical becomes v'' (1 — sin' z) = cos a:; therefore,<br />

c = (a 4- *) cos X (267)<br />

144. It is to be observed, that the supposition (263) is always possille, since a<br />

tangent may have any value between 0 and co , and therefore an angle z may always<br />

be found having any given number as its tangent. As the greatest value of a<br />

sine is unity, it is not so obvious that the supposition (266) is always possible; but<br />

whatever the values of a and 4<br />

therefore<br />

(a—4)'^0<br />

{a 4- 4)'^ 4 a4<br />

4^


72 PLANE TRIGONOMETRY.<br />

145. CASE IV. Given the three sides, or a, b and c.<br />

To find A, B and G. We have from (227) and (228),<br />

[s—b){s—c)<br />

sin i J. - J (^F^) be<br />

si.ii>-j(fc^4^')<br />

sin<br />

j


SOLUTION OF OBLIQUE TRIANGLES. 73<br />

EXAMPLES.<br />

1. Given a = 10, 5 = 12, c = 14; find the angles.<br />

By (268).<br />

a = 10 arCO 1 9-0000000 ar co 1 9- 0000000<br />

5 = 12 ar col 8-9208188 ar col 8-9208188<br />

c = 14 arCO 1 8-8538720 ar co 1 8-8538720<br />

2s=36<br />

s = 18<br />

8 —a= 8<br />

8-6= 6<br />

s— c= 4<br />

log sines<br />

i J. = 22° 12'27"-6<br />

log 0-7781513<br />

log 0-6020600<br />

log 0-9030900<br />

log 0-6020600<br />

2)9-1549021 2)9-3590220<br />

^A 9-5774510<br />

log 0-9030900<br />

log 0-7781518<br />

2)9-6020601<br />

rB 9-6795110 iG 9-801030G<br />

i.B = 28°33'39"-0 J C=39°13'63"-6<br />

J. = 44°24'55"-2 5 = 57° 7'18"-0 (7= 78° 27'47"-0<br />

Verification. A-\-B-h G = 180°<br />

2. Given a = -8706, b = -0916, c = -7902; find the angles.<br />

Ans. A = 149° 49' 0".4<br />

B= 3° 1'66".2<br />

C= 27° 9' 3".4<br />

3. Given a = .6123864, b = -3538971, c = -3090507 ; find Q.<br />

Ans. C = 36°18'10"-2<br />

146. The computation by (270), when all the angles are required, will be much<br />

facilitated by the introduction of an auxiliary quantity*<br />

(s — a) [a — 4) (« — e)<br />

•=JC<br />

(271)<br />

from which we find by (270)<br />

tan J .4 = ,<br />

-' a — a<br />

tan J B :<br />

T<br />

tan i (7 =<br />

(272ti<br />

•* This quantity T is the radius of the inscribed circle. See (289)<br />

'0 G


74 PLANE TRIGONOMLTRY.<br />

ExAMPLi. Given a = 6053, 4 = 4082, c = 7068. We find<br />

a = 8601-5<br />

ar. CO . log 6-0654258<br />

» — a = 2548-5<br />

log 3-4062846<br />

« — 4 = 4519-5<br />

log 3-6550904<br />

« — c = 1533-5<br />

log 3-1856838<br />

J J = 29° 20'54"-47<br />

^ .B = 17° 35' 31"-70<br />

J C7 = 43° 3' 33"-83<br />

rerifieation. 90° 0' 0"-00<br />

Hg. 22.<br />

the difference of which is<br />

log?-<br />

r<br />

log tan i A •= log —<br />

r<br />

log tan J P — log ^ J<br />

log tan J C = log<br />

T<br />

2)6-3124846<br />

= 3.1562423<br />

= 9-7499577<br />

= 9-5011519<br />

= 9-9705585<br />

147. The case where the three sides are given Is sometimes<br />

solved as follows. From G, Fig. 22, draw C?<br />

perp. to c. Then<br />

_ ^C" = ^P'4- CP', PC" = PP'4-CP'<br />

AG' — BG'^ AP' — BP'<br />

or (.4C4-.BC7) (AG—BG) = {AP-\-BP) (AP—BP)<br />

and if .4P — BP = d, this equation gives<br />

^^(5+a)J4-a)<br />

^2^3^<br />

Then, since ^P-f PP = c, and ^P —PP =


PROBLEMS RELATING TO PLANE TRIANGLES,<br />

CHAPTER IX.<br />

raSCELLANEOUS<br />

PROBLEMS RELATING TO PLANE TEIANGLES.<br />

149. In a given plane triangle, to find the perpendicular from one of the angles upoi.<br />

the opposite side.<br />

\,eip be the perpendicular from G upon c. We have<br />

or by (239) and (278),<br />

^ == 4 sin ^ (279)<br />

P = ^-yi^{^-a){a-b){s-c)-]=^-^ (280)<br />

the expression for^ in terms of the three sides, where a z= }^ {a-\- b -\- e) and A'is<br />

the area of the triangle.<br />

If we substitute in (279) sin J1 = — sin C, it becomes<br />

or, if we substitute the value


76 PLANE TEIGONOMETEY.<br />

161 To find the radius of the circle circumscribed about a plane triangle.<br />

fig.^<br />

The center 0 of the circle. Fig. 23, lies in the perpen<br />

dicular erected from the middle point of one of the sides,<br />

as A B. Let the radius = P. We have, by geometry,<br />

and in the triangle<br />

AOB = iAOB = C<br />

AOB,<br />

AB c<br />

sin ADD =z sin G = AO 2R<br />

whence<br />

R = 2 sin G<br />

(284)<br />

Substituting the value of sin G from (241),<br />

B<br />

a4c<br />

abc<br />

• '4:,y\a{s — a){a — b){s — c)-] ~~ iK<br />

(285)<br />

From (229) and (230), we easily find<br />

Ks<br />

cos i- A cos i B cos i G = -5—<br />

a4c<br />

irhioh combined with (285) gives<br />

^<br />

f<br />

4 cos J A cos J P cos J G<br />

(286)<br />

152. To find the radius of the circle inscribed in a plane triangle.<br />

Kg. 24.<br />

The required center 0, Fig. 24, is in the intersection<br />

of the three lines bisecting the angles,<br />

E/^ \ /V^<br />

and each of the perpendiculars 0 D, 0 E, OF, is<br />

equal to the required radius := r. The value of<br />

0 P in terms of ^ P = c, OAB = \A, - d<br />

^^ OB A = J Bis by (282)<br />

sin J .4 sin J P. sin J A sin \ B<br />

^ — ^- sin \(A + B) ~ " ' cos J C<br />

(287)<br />

This is reduced by means of (236) to<br />

Substituting the value of tan J G,<br />

J- = (a — c) tan J C<br />

r = J/ {s-a)(s-b) (s-c)\ ^ X<br />

(288)<br />

(289)<br />

This is reduced by means of (231) and (232) to<br />

7 = s tan J A tan J P tan J C<br />

(290;


PROBLEMS RELATING TO PLANE TRIANGLES. 77<br />

153. Besides the inscribed circle, strictly so called, there are three other circles<br />

that touch the three sides, (or sides produced), and are exterior to the triangle, iis<br />

in Fig. 25 These have been named escribed circles. Their centers are frund<br />

Tig. 25.<br />

geometrically, by bisecting the exterior angles PCC", GBB', &o. Designate the<br />

centers of the circles lying within the angles A, B, and G respectively, by 0', 0".<br />

and 0'", and their radii by /," r", r'". We find the perpendiculars from 0', &« ,<br />

upon B 0, &c by (282) to be<br />

r"= 4<br />

cos J B cos J G<br />

' sin J (P4- G)<br />

cos J A cos J 0<br />

Bin i{A+0)<br />

= 4.<br />

cos J A cos J B<br />

sinJ(^4-P) -<br />

By means of (235) we reduce those values to<br />

T' =1 a tan J 4, r" = a tan J P,<br />

Substituting the values of tan J ^, &o.<br />

cos i B cos J O<br />

cos J A<br />

cos J .4 cos J (7<br />

cos J P<br />

cos i ^ cos J P<br />

cos J (7<br />

r'"<br />

JT<br />

VV «— a / s — a<br />

][s — a) (s — e) \ __ K<br />

s — / s — 4<br />

-a) (s —4) \ K<br />

/ » — c<br />

a tan J (7<br />

(291)<br />

(292)<br />

(293)


78 PLANE TEIGONOMETEY.<br />

Also, by means of (236) applied successively to a, 4 and c, we may reduce (291)<br />

to the following:<br />

r* z= (« — a) tan J A cot J B cot J G<br />

t-" = (s — 4) cot J ^ tan J P cot J C • (294)<br />

T"' = (s — c) cot J ^ cot J P tan J C<br />

154. Relations between the radii of the circumscribed, inscribed, and three escribed circles<br />

of the preceding article, and the three perpendiculars from the angles upon the opposite<br />

sides.<br />

The four equations of (289) and (293) give<br />

•r' r"<br />

K*<br />

s(s — a) (s — 4) (<br />

Dividing this successively by r', r", &c.<br />

/ /' jj'i r r" r"<br />

- = :^ = ^•<br />

c) A^'<br />

= {s-af<br />

rr'r'"<br />

r" = (»-*)' ——- = {a- ef<br />

Again, we have, (Art. 127),<br />

tan J A tan J P 4- tan J A tan i G + tan J P tan J C = 1<br />

and substituting in this the value of the tangents from (292)<br />

r' r" +r' r"' + r" r"' = s' = '''""''".'<br />

(295)<br />

r296)<br />

From (292) we find<br />

tan IA<br />

tan J P<br />

14-1-1-1 = 1<br />

r' ^ r" ^ r'" r<br />

r" tan J ^ z= r' tan J P<br />

(297)<br />

from which it follows that in Fig. 25, the distances A D and P D', are equal (P, D'<br />

being the points of contact of the circles 0', 0" with .4 P produced), and therefore<br />

BD^^Ajy. Other curious geometrical properties may be traced with the aid of<br />

our equations.<br />

From (284),<br />

a 4 c<br />

R = . 4 sin J ^ cos J -4 4 sin J P cos J P 4 sin J C cos J C<br />

which combined with (287) and (291) give, by Art. 127,<br />

r<br />

-g- = 4 sin J .4 sin J P sin J C7 = cos .4 -j- cos ..> -f cos (7 — 1<br />

^ = 4 sin J .4 cos J P COB J (7 = — cos .4 -f- cos P 4- cos (7 4- 1<br />

r"<br />

^ = 4 cos J ^ sin J P cos J C = cos .4 — cos B+ coa C+l<br />

r'"<br />

^ = 4 cos J ^ cos J B. sin J C = cos ^ -f cos P — cos C -f- 1<br />

(298)


PROBLEMS EELATING TO PLANE TEIANGLES.<br />

Changing the signs of the first of these equations, the sum of the four is<br />

r' + T" + r"'—r<br />

= 4, R = i{r' + r" + r'" — r)<br />

B<br />

79<br />

(299)<br />

Finally, if p', p", p'" denote the three perpendiculars from the angles upon the<br />

Sides a, 4, c respectively we have by (283), (289) and (297) the following relation :<br />

1+L+L=L+L+L<br />

f<br />

r"'<br />

(299*)<br />

155.. To find the distance between the centers of the circumscribed and inscribed circles.*<br />

Let P, Fig. 26, be the center of the circumscribed, Kg. 26.<br />

and 0, that of the inscribed circle. Put P 0 ^ D.<br />

By Arts 151 and 152,<br />

whence<br />

and by (221)<br />

P^P = 90° —C, OAB = iA<br />

FAO = 90° — G—lA = i{B—G)<br />

POf = PA^+ 0A^ — 2PA . OA cos PAO<br />

Z)' = if 4- ^<br />

sin' J A<br />

2g?-cos^(P— G)<br />

sin J A<br />

By (298)<br />

4 Rr sin J P sin J (7<br />

sin' I A sin J 4<br />

therefore<br />

D<br />

; if-<br />

2Prcos^ {B+ G)<br />

sin J A<br />

P'<br />

if —2P?- (300)<br />

156. Let P 0' =^ If, Fig. 26, 0' being the center of the escribed circle lying<br />

within the angle A. If r* = radius of this circle, we have, as in the preceding<br />

wkicle<br />

2 Er' cos 1{B— G)<br />

Zl" = if 4- -.•<br />

sin' I A sin J .4<br />

4 P/ cos J P cos J C<br />

sin' J A "~ sin J ^<br />

7)" = if -f 2 P/<br />

i)'" = if 4- 2 Rr"<br />

(801<br />

D"" = P" 4- 2 Rr"'<br />

Hymers' Trig. Appendix, Art. 58.


80 PLANE TEIGONOMETEY.<br />

the expressions for the distances of the centers of the three escribed circles from<br />

that of the circumscribed circle.<br />

The sum of (300) and (301) gives by (299)<br />

p» + B" + P"' + JD"" = 12 R'^ (302)<br />

157. Given two sides of a plane triangle and the difference of their opposite angles, (or<br />

a, 4, and A — P), to solve the triangle.<br />

We have i (A + B) directly from (220), which also solves the case where two<br />

angles and the sum or difference of two sides are given.<br />

158. Given the angles and the sum of the sides, (or -4, P, G, ar.d a -j- 4 -j- c = 2 s).<br />

By (235)<br />

_ sin i G<br />

' cos J A cos J P<br />

ind a and 4 are found by similar formulae.<br />

159. Given one angle, the opposite side, and th' aum of the squares of the other two<br />

ndes, (or C, e, and a' -|- 4' = e').<br />

In the identical equations<br />

(a 4- 4)' = e' 4- 2 a4, (a — 4)' = e" — 2 a4<br />

substitute the value of 2 a4 given by (223), namely,<br />

.,» ..'<br />

2a4 = cos 0<br />

we find (a4-6)' = e'4 -, (a_4)' = «' —! 1.<br />

' cos C ^ ' cos C<br />

which determine a -j- 4, and a — 4, and therefore a and 4.<br />

To compute these equations by logarithms, let<br />

e' — c' (e 4- c) (e — e) ,„„„,<br />

S = ?r = Vr (303<br />

' cos G cos O ^ '<br />

then (a 4-4)'= e'-f-^', {a — by = e'— g'<br />

that is a -|- 4 is the hypotenuse of a right triangle whose sides are e and g; and<br />

a — 4 is one side of a right triangle whose other side is g, and whose hypotenuse is t.<br />

Let the angle opposite g be denoted by x in the first triangle and by x' in the second,<br />

then by the formulaj of right triangles<br />

tan X = —<br />

a 4" * i<br />

sinx'=^ a —• b = e cos j/<br />

(304J<br />

so that the problem is solved by logarithms by finding log g from (303) and employing<br />

its value in (304).<br />

The above may serve as an example of ° geometrical method of introducing the<br />

auxiliary quantities, which is occasionally useful. The analytical process in the<br />

present instance is similar to that of Art. 143; thus<br />

«'+* = ^JO + f)<br />

a-4 = .J(l-f)


PEOBLEMS EELATING TO PLANE TEIANGLES. 81<br />

therefore if tan a; = — we have /(l-|-^)=secs<br />

and if sin x' = — we have /(l — — j=:Cosx'<br />

whence the same formula as before.<br />

160. Given an angle, its opposite side, and the difference of the squares of the other two<br />

sides, (or G, c, and a' — 4' r^ /').<br />

We have by multiplying (233) by (234)<br />

sin {A — B) _ a'— 4' _/'<br />

Bin G ~ ? ~ 1<br />

sin {A — B)== 1- sin 0<br />

whence A — P, and since A + B = 180° — G, the angles are determined There<br />

will be two solutions given by sin [A — P) except where the obtuse value of ^ — 7i<br />

is greater than A + B.<br />

161. Given the three perpendiculars from the three angles upon the opposite sides.<br />

Denote the perps. upon a, 4 and c respectively by a', b' and c', and let<br />

«" = 4, ^" = ^, - = i<br />

a 0 c<br />

If i = 2 area of the triangle<br />

and therefore<br />

aa' = 44' = cc' T^TC<br />

a ^ a" h, 4 r^ V'lc, c = c"k<br />

Substituting these values of a, 4 and c in (225), (227,) &c.<br />

J"'4.c"'_a"' .,, . (s"_4")(«" —c")<br />

cos A = — \ ,„ „ , sm' J 4 = '- ^, i, &c.<br />

2 4 a 4 c<br />

in which 2 s" = a" 4- 4" 4- c".<br />

162. Given the radii of the circumscribed and inscribed circles, and the perpendicular<br />

from one of the angles upon the opposite side, to solve the triangle.<br />

Let c be the side to which the perpendicidar (p) is drawn. We have found forP, /<br />

and^ the expressions.<br />

P = 2 sin (7 ~ 2 sin (.4 4- P) — 4 sin i (_A + B) cos J (4 -f P)<br />

'•="•<br />

sin I A sin I B<br />

sinj(^4-P)<br />

11<br />

sin A sin P<br />

•P = " • Sn(^4-P)


82<br />

PLANE TIUGONOMF.TET.<br />

Eliminating c, we have<br />

—^ =•= sin A sin B<br />

2R<br />

(m)<br />

r<br />

4 cos J (^ 4- P) sin J ^ sin JP<br />

(n)<br />

from which two equations A and P are to be found.<br />

(n) becomes<br />

Developing cos i {A + B),<br />

— = 4 sin J ^ cos J ^ sin J P cos J P — 4 sin' J ^ sin' J B<br />

P<br />

= sin j4 sin P — 4 sin' J ^ sin' J P<br />

which subtracted from (m) gives<br />

^~f r = 4 sin' J ^ sin' J P<br />

(»)<br />

•Dividing the square of [m) by (o), we find<br />

2P(^ —2r)<br />

= 4 cos' J ul cos' J P<br />

whence<br />

sinMsinJ^ = J f ^ ' ' ) =<br />

JO —2r<br />

2^\2B{p — 2r)1<br />

cos J^ COS ^B = P<br />

2,/\2R{p-2r)-\<br />

Tlie difference and sum of these two equations give<br />

COB \{A + B)<br />

\/[2R(p-^r)}<br />

COS l(A — B) = v/ [2 P (i> - 2 r)-]<br />

(305)<br />

which determine J (-4 -f- P) and i {A — P) and therefore A and P.<br />

are then found by the formula<br />

c = 2 P sin C<br />

The sides<br />

Fig. 27. 163. In a given plane triangle ABO, Fig. 27, to find a<br />

point P ^Hch that the three lines dr


PROBLEMS RELATING TO PLANE TRIANGLES.<br />

In the triangles APG,BPG,y^c<br />

from which<br />

BG:<br />

4 sin z a sin y<br />

sin ^ sin t<br />

have<br />

sm z a sin /S<br />

sin y 4 ' sin a.<br />

sin x+ sin y tan ^(z + y) m+1<br />

sin X — sin y tan J (x — y) m — 1<br />

tan I {z — y)<br />

m —-1<br />

m4- 1<br />

To compute this equation by logarithms, let<br />

tan •<br />

a sin ,8<br />

4 sin a<br />

tan i {x + y)<br />

then by (152), tan ^{x — y) = tan {y — 45°) tan i{x + y)<br />

eo that the angles x and y are found by (306) and (307).<br />

164. The following problems are proposed as exercises.<br />

In a plane triangle AB G —<br />

1. Given c, the perp. upon c = p and a -f- 4 = m.<br />

Bin' X :<br />

4^'<br />

[m + c) [m — c)<br />

tan J C =<br />

2pc<br />

{m + c) (in — c)<br />

2. Given c, the perp. upon c ^ p, and a — 4 = m.<br />

tan' X<br />

4j>'<br />

(c -j- «) (c — n)<br />

tan J C =<br />

• 4 = c cos X<br />

a 4" 6 = " sec s<br />

(c 4" ") (" — '*)<br />

• 2^0<br />

83<br />

(307)<br />

8. Given C, e, and a4 = g'.<br />

tan z •= — cos J C<br />

a 4" ^ = " sec 3<br />

2 (7<br />

sin z' = — sin J C<br />

a — 4 = c cos a;'<br />

4. Given C, the perp. from G =: p, and a -f 6 = m.<br />

tan 2 == — tan 1 C c = m tan i a;<br />

5. Given G, the perp. from C = p, and a — b = n.<br />

tan X = — cot J C c = ncotlx<br />

P<br />

6. Given c, C, and a 4- * = '"-<br />

cos J (,4 — P) = — sm J C a — 4 = c<br />

sm h (4 — P)<br />

cos } C


84 PLANE TRIGONOMETRY.<br />

7. Given c. A, and a+ b = m.<br />

tan i B ^ —,— cot i A<br />

^ m+ c ^<br />

8. Given a 4- ^ = '"> the perp. upon c = p, and the difterenoe of the segments<br />

of c = (f.<br />

tan H^ 4- ^) = J [_-.,,._ ^,) .„,, _ ^, _ ^^. J<br />

2 »(?<br />

tanH^-^)=^;;^<br />

or with an auxiliary angle<br />

4i,'<br />

(m 4"


SOLUTION OF TRIGONOMETRIC EQUATIONS. 85<br />

CHAPTER X.<br />

SOLUTION OF CERTAIN TRIGONOJIETEIC EQUATIONS AND OF NU­<br />

MERICAL EQUATIONS OP THE SECOND AND THIRD DEGREES.<br />

166. THE solution of a prohlem in -which the unknown quantity<br />

IS an angle, often depends upon that of one or more equations, involving<br />

different functions of the angle, which cannot he reduced by<br />

merely algebraic transformations. We shall select a few simple examples<br />

of such equations from among those that most frequently<br />

occur in astronomy.<br />

167. To find zfrom the equation<br />

sin (ct -f 2) — m sin z (309")<br />

in which a, and m are given. "We have, by (119),<br />

sin ( ci -t- 2) = sin a sin z (cot z 4- cot oi)<br />

which becomes identical with (309) by taking<br />

whence the required solution<br />

If the proposed equation were<br />

we should find<br />

sin «, (cot z 4- cot oi) =m<br />

cotz = -. cot 01, (310)<br />

sm «.<br />

sin [ot, — z) = m sin z (311)<br />

cot 2 = —; 4-cot« (312)<br />

sin«<br />

Unless z is limited by the nature of the problem in which these<br />

equations are employed, there -will be an indefinite number of solutions<br />

; for all the angles a, 2 4- 180°, z 4- 360°, z + 540°, &c., in<br />

general all the angles s 4- w TT have the same cotangent. [See<br />

(68), (79).J In most cases, however, we consider only the first two<br />

of these solutions, taking the values of z always less than 360°<br />

H


g6<br />

PLANE TRIGONOMETRY.<br />

Similar remarks apply in all cases where an angle is determined<br />

by a single trigonometric function; but if the prohlem is such as to<br />

give the values of two functions of the required angle, as the sine and<br />

cosine, the solution is entirely determinate under 360°, since there<br />

cannot be two difi'erent angles less than 360° that have the same<br />

sine and cosine.<br />

168. The solution of the preceding article requires the use of a<br />

table of natural cotangents ; to obtain a formula adapted for logarithmic<br />

computation entirely,^ we deduce from (309) the following<br />

sin (a 4- 2) 4- sin 2 _ m 4- 1<br />

sin (* 4- 2) — sin 2 m — 1<br />

But by (109), if a; =» 4- 2, 2/ = 2, we have<br />

which substituted above, gives<br />

sin {ot, -\- 2) -f sin 2 _ tan (24-2"*)<br />

sin (a 4- 2) — sin 2 tan J a,<br />

, / , 1 X m-\- 1<br />

tan iz + *a,) = =- tan i «<br />

^ ^ ' TO — 1 ^<br />

which determines 2 -f J a, whence 2 is found by deducting f a,.<br />

The computation of this equation is facilitated in most cases by<br />

introducing an auxiliary angle, such that<br />

tan (p = m<br />

an assumption always admissible, since while the angle varies from<br />

0 to 90° the tangent varies from 0 to oo, so that an angle cp may<br />

always be found having any given number as its tangent.<br />

We have then by (152),<br />

TO -f 1 tan (p -f 1<br />

T = . ! 1 = cot (


SOLUTION OP TRIGONOMETRIC EQUATIONS.<br />

hj<br />

169. To find z from the equation<br />

We deduce<br />

tan (o4 -f 2) = TO tan 2 (316)<br />

tan («. -f 2) -f tan 2 TO 4- 1<br />

tan (a 4- 2) — tan 2 TO — 1<br />

80 that by (126) and (152) the solution is<br />

tan (p = TO, sin (» 4- 2 2) = cot (cp — 46°) sin


88 PLANE TRIGONOMETRY.<br />

172. The preceding examples wiU sufiice to indicate the method to be followed<br />

with all the equations of the following table. The solutions of the equations involving<br />

cosines may be obtained from those involving sines, by exchanging z for 90° ± z,<br />

or a for 90° rt a..<br />

Logarithmic solutions of the first four will be obtained by imitating the process of<br />

Art. 171.<br />

EQUATIONS.<br />

SOLUTIONS.<br />

1.<br />

2.<br />

3.<br />

4.<br />

5.<br />

6.<br />

7.<br />

8.<br />

9.<br />

10.<br />

sin {a. ± z) sin z = nt<br />

cos (a. rb z) cos z = m<br />

sin (a. ± z) cos z ==. m<br />

cos (a ± z) sin z = m<br />

sin (i ±z) = m sin z<br />

cos {dL rbz) = m cos z<br />

sin {a.:±iz) •= m cos z<br />

cos (a ± z) :^ OT sin z<br />

tan (« ± z) tanz = m<br />

tan (rt rtz) = m tanz<br />

cos (a ± 2 z) = cos :t =p 2 m<br />

cos (a ± 2 z) = 2 m — cos a.<br />

sin (a ± 2 z) = 2 m •— sin a<br />

sin (a ±2z) = sina±2 7?s<br />

tan tp =m, tan (z ± J a) = cot ((6 q= 45°) tan J a<br />

tan0 = m, tan (^^a±z) = tan(45° —


SOLUTION OF TRIGONOMETRIC EQUATIONS. 89<br />

l78. To find z from the equation<br />

sin (a -|- z) = m sin (/S + z).<br />

Put z' = ,S -j- z, a' = a — jj, then this equation becomes<br />

sin (a' + z') = m sin z'<br />

which is of the form (309) and may be solved by (309*) or (311) ; then z = z' — /3.<br />

In the same manner equations of this form, involving cosines or tangents, may be<br />

reduced to those of the preceding table.<br />

174. To find k and z from the equations<br />

We have, by division.<br />

& sin 2 = TO<br />

h cos z = n<br />

m<br />

tan 2 = —<br />

n<br />

y (320,<br />

I<br />

which gives two values of z, one less, the other greater than 180°;<br />

whence, also, two values of h from either of the equations<br />

TO<br />

sm 2<br />

The solution becomes entirely determinate (2 not exceeding 360°)<br />

as follows :<br />

1st. When the sign of h is given. Eor if k is positive, sin 2 has<br />

the sign of TO, and cos 2 the sign of n, and 2 must be taken in the<br />

quadrant denoted by these signs. If k is negative, the signs of sin 2<br />

and cos 2 are the opposite to those of TO and n, and 2 must be taken<br />

accordingly.<br />

2d. • When z is restricted by either the condition 2 < 180°, or<br />

2 > 180°. For under either of these conditions the tangent gives<br />

but one solution. If 2 < 180°, k has the sign of TO ; and if 2 > 180°<br />

k has the opposite sign to that of TO.<br />

3d. When 2 is restricted to acute values, positive or negative. For<br />

under this condition a positive tangent will give 2 between 0° and<br />

-r 90° ; and a negative tangent, between 0° and — 90° ; and k will<br />

always have the sign of n.<br />

It follows that TO and n being any given numbers whatever, we<br />

may always satisfy the conditions expressed by (320), 1st, by a positive<br />

number k and an angle 2 between 0° and 360° ;' 2d, by a number<br />

Ic (unrestricted as to sign) and an angle 2 < 180° ; 3d, by a<br />

number k (unrestricted as to sign) and an angle 2 > 180°; and 4th,<br />

by a number k, and an.angle 2 in the 1st or 4th quadrant.<br />

12 H2


90 PLANE TRIGONOMETRY.<br />

EXAMPLE.<br />

To find k and 2 from (320), {k being a positive number), whn<br />

,„ = _ 0-3076268, n =-{• 0-4278735.<br />

A; sin 2 - 0-3076258<br />

A; cos 2 4- 0-4278736<br />

{a) log & sin 2 - 9-4880228<br />

{b) log A cos 2 4- 9-6313147<br />

(a) - (6) 'log tan 2 - 9-8567081<br />

z 324° 17' 6"-6<br />

(c) log sin 2 - 9-7662280<br />

(a) - (e) log k 4- 9-7217948<br />

k 4- 0-5269808<br />

Upon this problem and the deductions we have made from it, rests<br />

the method of introducing the auxiliary angles required in solving<br />

many of the formulse of spherical trigonometry. It is applicable<br />

to any equation that can be reduced to the form of that solved in<br />

the following article.<br />

175. To solve the equation<br />

m cos z + n ainz = q<br />

{2>2V<br />

TO, n and q being given.<br />

The fij-st member will be reduced to the form h sin (cp 4- 2") by as<br />

suming k and cp such that<br />

whence<br />

k sin cp = TO, /c cos (p = w (322)<br />

k sin


SOLUTION OF TRIGONOMETRIC EQUATIONS.<br />

If we restrict cp to values less than 180°, (as we may do according<br />

to the last article), we may find it by the equation<br />

yi<br />

and then<br />

TO<br />

tan (J) = —<br />

n<br />

sin (cp 4- 2) = — sin $ = — cos


92 PLANE TRIGONOMETRY.<br />

176. To sohe the equation<br />

a sin (a 4- z) 4- 4 sin (/S -j- z) 4- c sin (>. 4- z) 4- &c. = q (825)<br />

Developing by (36) and putting<br />

this becomes<br />

a sin a 4" * ^i'^ Z' 4" '^ ^'"^ > "t" ^''- ^^ "'<br />

a cos a 4" * "OS /? 4- " ''°^ y "^ ^"^ = "<br />

m cos z 4- »» sin z = J<br />

which is solved in the preceding article. The same process applies if any or all of<br />

the terms contain cosines.<br />

177. To find k and zfrom the equations<br />

k sin (a 4- z) = m<br />

4 sin [0+ z) :^ n<br />

The sum and difference of these equations are, by (105) and (106),<br />

whence<br />

2 k sin [J (a 4- ^) 4- ^] cos J (a — /?) = m + n<br />

2 k cos [J (a 4- ;8) 4- z] sin f (a — 13) = m — n<br />

m -j- n<br />

2 k sin [J (o 4- /?) 4- z] =<br />

cos J (o — ^)<br />

2 k cos [J (a 4- /J) 4- z] =<br />

m — n<br />

Bin J (a — ;S)<br />

(326)<br />

(327)<br />

from which 2 i and J (a -f- /3) -j- « are determined by Art. 174. The logs, of the<br />

second members of these equations should be computed separately, for the purpose<br />

of readily discovering the signs of the sine and cosine in the first members. The<br />

Bolution is determinate (according to Art. 174) when the sign of k is given.<br />

From (327) we find, by division.<br />

tan [J (. 4- /?) 4- z] = ^-±^ tan i{a-fs) (328)<br />

which requires a less number of logs, than the separate computation of (327), but<br />

we are obliged to refer to (327) to determine (by an inspection of the second members)<br />

the signs of the sine and cosine.<br />

If we assume<br />

tan * =<br />

we may compute (328) by tho formula<br />

tan [J (a 4- ;3) 4- z] = tan (45° 4- 0) tan J (a — /?)<br />

- (329)


SOLUTION OF TRIGONOMETRIC EQUATIONS.<br />

EXAMPLE.<br />

In (326) given a = 200°, ^ = 140°, m = — 0-42345 and « = — 0-20123, to<br />

find z and k, k being positive.<br />

(")<br />

(a) - (4)<br />

(a) -<br />

(c)<br />

By (35!7).<br />

m + n<br />

m — »<br />

i (" 4-,?)<br />

J (" - ^)<br />

log (m 4- ")<br />

log cos J (ii — /3)<br />

log 2 A sin [J (a 4- /3) 4- z]<br />

log (m — n)<br />

log sin J (a — ^)<br />

log 2 i cos [J (a + 0) + z]<br />

log tan [J (a + /l) + z]<br />

H-4-/2 4-2)<br />

^<br />

log sin [J (a -f /3) 4- z]<br />

log 2 k<br />

2 k<br />

k<br />

— 0-62468<br />

— 0-22222<br />

170°<br />

30°<br />

— 9-7956576<br />

4- 9-9375306<br />

— 9-8581270<br />

— 9-3467831<br />

4- 9-6989700<br />

— 9-6478131<br />

4- 0-2103140<br />

238°21'38"-6<br />

68°21'38"-6<br />

— 9-9301171<br />

4- 9-9280099<br />

0-8472467<br />

0-4236234<br />

178. A more general solution of (326) is the following.* Let j. be any angle as<br />

sumed at pleasure, and in (171) let<br />

a; = o4-z, y = /3-}-z, z' = 3-4-z<br />

(distinguishing the z of (171) by an accent); then we shall find<br />

sin (a —/S) sin (j- 4- z) = sin (a — y) sin (^ 4- z) — sin (^ — y) sin (a -|- z)<br />

In this let y (whose Talue is arbitrary) be exchanged for j. 4- 90° ; then<br />

sin (a— 0) cos (y + z) = — cos (a — y) sin (/S + z) + cos {0 — y) sin (a 4- z)<br />

Multiplying these equations by k and substituting m and n from (326)<br />

k sin (a — 0) sin (j. -f z) = m sin [y — 0) — n sin {y — a)<br />

k sin [a — 0) cos (y + z) =i m cos {y — B) — n cos {y — o)<br />

which (y being assumed at pleasure), determine k .ind y -\- z.<br />

If we take j. = 0, we find<br />

•— OT sin 5 4- ?! sin u<br />

tan z = -—-J<br />

m cos 0 — n cos "<br />

If } = a,<br />

k sin (a -f- z) ^ TO 1<br />

, , , > mco3(a—/?)-B I (381)<br />

h cos (a -f z) = r^-^ '-^—. j ^ -'<br />

^ ' ' sin (a — B)<br />

li y = 0, we have a similar result.<br />

If J- ^ J (a 4- (3) we obtain the solution of the preceding article.<br />

If A is required, without first finding z, we have, by adding the squares of (330)<br />

k sin (o — /?) = .^ [m' 4- n' — 2 m m cos (? — 0)] (332),<br />

-* GAUSS. Theoria Motus Corporum Ccclestium, Art. To.<br />

(330)


94 PLANE TRIGONOMETRY.<br />

J 79. To find k and zfrom the equations<br />

k coa [a+ z) = m )<br />

kcoa{0+z) = n J ^ '<br />

These are reduced to the form (320) by substituting 90° + % and 90° -f /? for a and /?.<br />

We find, however, by a process similar to that of Art. 177,<br />

2kBi.l_U^ + 0) +<br />

'-\=^f^'0)<br />

2.cos[J(a4-/J)4-^] = - ^ P y )<br />

tan A =<br />

cot [J {a + 0) + z] = tan (45° 4- 0) tan ^ (


EQUATIONS OF THE SECOND AND THIRD DEGREES. 95<br />

182 In like manner, if the proposed equation is<br />

ive assume<br />

whence<br />

n cos (d -j- z) = m cos (/? + z)<br />

k cos (a 4- z) = TO<br />

k cos [0 + z) = n<br />

and k and z are found by Arts. 179, 180. As the sign of k (in this and the preceding<br />

article) may be arbitrarily assumed, there will be two solutions.<br />

NuMEEicAL EQUATIONS OF THE SECOND AND THIRD<br />

DEGREES.<br />

183. To solve the equation<br />

x'' + .px+ q = 0 (338)<br />

when q ia easenfially positive, and p either positive or negative.<br />

We have from (144), exchanging x for ip,<br />

tan' } * — 2 cosec * tan J * -f 1 = 0 (339)<br />

and (338) may be reduced to this form by substituting<br />

X = z .yq<br />

In which we may take the radical only with the positive sign, since we may assume<br />

X and z to have the same sign. We thus reduce (338) to<br />

which compared with (339) gives<br />

P<br />

— 2 cosec 0 = -^—, z = tan J ^<br />

V?<br />

or sin 0 = ^^, x = y/qt&n J ^ (340)<br />

p<br />

which gives two values of (p less than 360° and consequently two values of x. If<br />

e be the least of these two values of


96 PLANE TRIGONOMETRY.<br />

184. To solve the equation<br />

x' + px — q^Q (842)<br />

when — q is essentially negative, p being either positive or negative.<br />

We have, by (143)<br />

tan' \'p + 2 cot


EQUATIONS OF THE SECOND AND THIRD DEGREES. 97<br />

BO tliat, by the theory of equations, «, and t^ are the two roots of an equation of the<br />

a^<br />

second degree in which the absolute term is — -H= and the coefiioient of the second<br />

tern is 4; that is, they are the roots of the equation<br />

e+bt — "^ = 0 (m\<br />

If then we find the two roots «, and 4 of (m) by the preceding methods we shau<br />

have<br />

x = y + z = ^i, + ^t^<br />

(n)<br />

It will be necessary to consider the sign of a in the equation (OT).<br />

1st. When a is positive, (m) comes under the form (342) and the solution bv (344)<br />

gives<br />

and by (n)<br />

x = J~[^ta.ni 9 — ^001^9^<br />

and if we assume<br />

this becomes, by (142)<br />

tan ^ Ip = ^ tan J 9<br />

a; = - 2 j - | cot^i<br />

Collecting these results we have, for the solution of (345), ivhen a is positive,<br />

2 /"a^<br />

tan 9 = -T- v/ 97' ^^^ J 'P = -^ tan J 9<br />

2 / -^ cot (<br />

• (346)<br />

in which the radicals<br />

/ —- and / — are to be considered positive, and 9 is to be<br />

taken < 90° with the sign of the tangent. But two of the three values of ^ tan J 9<br />

being imaginary, the given equation has but one real root.*-<br />

•^ If »• represent the real value of ^ tan J 9, and a„ a^ the two imaginary root* of<br />

unity, the real value of x ia<br />

and the imaginary values are<br />

'' =J Y (.'"^ -^) "' = JT (- - k)<br />

or since a, a, = 1<br />

13 I


98 PLANE TRIGONOMETRY.<br />

2d. When a is negative and — 4 a' < 27 4'. Equation (m) becomes<br />

and is of the form (338); its roots are therefore found by (341) which gives<br />

2 I a» / 4a»<br />

4'<br />

8ine=-^J-2= = -J-274<br />

/ «"" / ^<br />

cot J 9<br />

ana by (M) a: = / ^ f .^ tan J 9-}-.^ cot J 9 j<br />

or if we put, as before, tan J 0 = -^ tan J 9, the solution of (345), when a is negative, ia<br />

2 a"<br />

sin 9 = —-- / —27 tan J0 = i^tanj I<br />

(347)<br />

cosec 5*<br />

which gives one real root, (the other two being imaginary, as above), when sin 9 ia<br />

possible, i. e. when — 4 a" < 27 4'.-*<br />

Substituting the values of a, and a,<br />

-14-V/-3 - 1 - ^ - 3<br />

a. = ft, _ 2<br />

and also r = tan J ^ — = cot J $<br />

J<br />

r<br />

a<br />

-^ (cot 0 4- cosec ^ y/ — 3)<br />

;~a~<br />

a:,. = I -^ (cot 0 — cosec ip .^ — 3)<br />

flr finally, x, being the real root, the imaginary roots are<br />

^, = — -2 Y~ sec ^ v' — 1<br />

a:. = — — 4<br />

Y~ ^^"'P'/ — ^<br />

• * The two imaginary roots will be found, by a process similar to that employed<br />

in the preceding note, to be<br />

X^ = 2 ^ COB^ v/ —1<br />

a:, a:, ^ 3<br />

a;, = — -g- 4- —|— cos 0 v' — 1<br />

in whioh a;, is the ro^l root found ty (347).


EQUATIONS OF THE SECOND AND THIRD DEGREES. 99<br />

3d. When a is negative and — 4 a' > 27 4'. In this case sin 9, in (347), is impossible<br />

and the preceding solution fails. This is the irreducible case of Cardan's rule, the<br />

roots appearing under imaginary forms, although it is known that they are all three<br />

real. It is, however, readily solved trigonometrically.<br />

In Art. 77, putting ^ for x, we have<br />

sin'


100 PLANE TRIGONOMETRY.<br />

so that tnere are but three different values of sin


DIFFERENCES AND DIFFERENTIALS. 101<br />

CHAPTER XI.<br />

DIFFERENCES AND DIFFERENTIALS OF THE TRIGONOMETRIC<br />

FUNCTIONS.<br />

186. IN the applications of trigonometry, it is often required to<br />

compute a function of one angle from that of an angle which differs<br />

from the first by a small quantity. In such cases it is generally<br />

most convenient to compute the difference of the two functions,<br />

which may be applied to either to obtain the other.<br />

187. To find the increment of the sine or cosine of an angle, corresponding<br />

to a given increment of the angle.<br />

Let the angle x be increased by A x, (this notation signifying difference,<br />

or increment of as), and let the corresponding difference or<br />

increment of the sine be expressed by A sin x and of the cosine by<br />

A cos a;; we have, by this notation,<br />

and by (106) and (108)<br />

A sin a; = sin (a; -f A a;) — sin x<br />

A cos X = cos (a; 4- A a;) — cos x<br />

A sin a; = 2 cos (a; 4- J A x) sin J A a; (350)<br />

A cos a; = — 2 sin (a; -1- J A a;) sin |- A a; (351)<br />

which are the required formulse.<br />

We here consider the difference always as an increment, i. e. an<br />

increase, and give it the positive (algebraic) sign; its essential sign<br />

may, however, be negative, and it will then be in fact a decrement.<br />

Thus, in (351) the second member will be negative so long as<br />

a; 90°, and < 270°<br />

188. To find the increment of the tangent and cotangent. We have<br />

and by (116) and (119)<br />

A tan X = tan (a; 4- A a;) — tan x<br />

A cot a; = cot (a; -f A a;) — cot x<br />

sin A a; / , . \ • ^ /orroN<br />

A tan a; = T—r^—\ = sec (a; 4-A a;) sec a; sm A a; (651)<br />

cos (a; 4- A a;) cos a; ^ '<br />

A cot a; = s—^— = — cosec ix 4- A a;) cosec a; sin A a; (363)<br />

sin (a; 4-A a;) sma;<br />

'<br />

i2


102 PLANE TRIGONOMETRY.<br />

189, Tb find the increment of the secant and cosecant. We have<br />

or by (130) and (132)<br />

h sec X =<br />

sec [x + ixx) —• sec x<br />

A cosec z = cosec (a: 4- ^ ^) — cosec x<br />

2 sin (a: 4- j- A x) sin } A a;<br />

A sec a; = ^—,—;——4 — (354)<br />

cos (x + t^x) cos z ^ '<br />

— 2 COS (a: 4- J A a:) sin J A. a;<br />

A cosec z = ;—^7—-7-^—r-A = (855^<br />

sm (a; -f. A a:) sm x<br />

190. To find the increments of the squares of the trigonometric functions corresponding<br />

to a given increment of the angles<br />

We have<br />

whence by (133)<br />

A sin' X = sin' (a: 4- A a:) — sin' x<br />

= cos' X — cos' (x + AX)<br />

A sin' X = — A cos' a; = sin (2 a; 4- A a;) sin A a! (356)<br />

From (115), (116), and (119) we deduce<br />

, . , . sin (x +y) sin (z — y)<br />

tan' X — tan' y = ^ ^^' \ ^^<br />

cos' X COS y<br />

whence<br />

cot' :. - cot' y =<br />

^^^(?^+l)^^±=l)<br />

sin' X sin' y<br />

, „ sin (2 a; -4- A a:) sin A a; ,„,_,<br />

A tan' z = \ , I ( , (357)<br />

cos' (a; 4- A a;) cos' z ^ '<br />

— sin (2 a; 4-A a;) sin A a;<br />

A cot' X = —. , ) / / . , (358)<br />

sm' (a; 4- A a;) sin' x ^ '<br />

From (16) we have<br />

the difference of which gives<br />

and in the same manner, from (17),<br />

sec' (a; 4- A a;) = tan' (a; 4- A a;) 4- 1<br />

sec' z = tan' x+\<br />

A sec' a: = A tan* x (359)<br />

A cosec' a; = A cot' x (360)<br />

and the values of A tan' x, A cot' x, may be substituted in (359) and (360).<br />

191. When the increment of an angle, or arc, is infinitely small,<br />

it is called the differential of the angle, or arc; and the corresponding<br />

increments of the trigonometric functions are the differentials of<br />

these functions.<br />

The differential of x is denoted by dx; of sin a; by d! sin x, kc.


DIFFERENCES AND DIFFERENTIALS. 103<br />

192. To find the differentials of the trigonometric functions from<br />

the differential of the angle.<br />

Let the angle x and its increment Aa; be expressed in the unit of<br />

Art. 11; or, which is equivalent, let x and Aa; be the arcs which<br />

measure the angle and its increment in the circle whose radius = 1.<br />

It is evident that the less the arc, the more nearly does it coincide<br />

with its sine or tangent; therefore, when Aa; is infinitely small, or<br />

becomes dx,<br />

sin dx = dx sin ^dx = ^ dx<br />

This may be demonstrated more rigorously thus.<br />

infinitely small, we have cos dx = 1, whence<br />

sindx , - ,<br />

5— = cos dx= 1<br />

tan dx<br />

sin dx= tan dx<br />

When dx is<br />

but the arc cannot be less than the sine, nor greater than the tangent,<br />

and therefore<br />

dx = sin dx = tan dx<br />

Again, when Aa; is infinitely small, or becomes dx, we must, according<br />

to the principles of the differential calculus, reject it when<br />

connected with finite quantities by the signs -f or —; thus we must<br />

substitute a; for a; 4- dx, or for a; 4- J dx.<br />

Upon these principles we find the differentials directly from the<br />

finite differences (350), (351), (352), (353), (354) and (355) as follows:<br />

d sinx = cos x dx (361)<br />

cZ cos a; = — sin x dx (362)<br />

di&n X = sec^ a; c?a; = (1 -f tan^ x) dx (363)<br />

dcotx= — cosec^ xdx = — {!-{• cot^ a;) dx (364)<br />

d sec X = tan x sec x dx (365)<br />

d cosec X = — cot x cosec x dx (366)<br />

193. In the same manner the equations (356), (357), (358), (359) and (360) give<br />

d sin' X = — d cos' x = sinix dx (367)<br />

d tan' a; = i sec' a: = ?^^4^ dx (368)<br />

cos z<br />

=<br />

2 sin a; 2 tan a; .<br />

=— dx = —-^— aa; (369)<br />

COB x cos'X<br />

— sin 2 a; , .....<br />

dcot' x= d cosec' x = —-, dx (61O)<br />

sin z<br />

=.-=l^dx=^^l^dx (371)


104 PLANE TRIGONOMETRY.<br />

194. Although the equations (361), (362), (363), (364), (365) and<br />

(366), are rigorously true only when dx is infinitely small, they may<br />

be used when dx is a finite difference, instead of the equations, (350),<br />

(351), (352), (353), (364) and (365), provided dx is sufficiently small<br />

to be considered equal to its sine without sensible error, and is also<br />

very small in comparison with x. This is very frequently the case in<br />

practice, and the differential equations are then preferred on account<br />

of their simplicity. It is only necessary to observe that dx must<br />

be expressed in arc, i. e. in terms of the unit radius ; if it is given<br />

in seconds, it may be reduced to arc by Art. 9.<br />

195. To find the differential of an angle from the differentials of<br />

its famotions.<br />

Fiom (361) we have<br />

dx = ^^^ (372)<br />

cos a; ^ '<br />

but it is convenient in this case to employ the notation of inverse<br />

functions. Art. 87. Thus, iS y = sin x, x = sin ~^y, and the preceding<br />

equation becomes<br />

dain-^y^^^f^_^^^ (378)<br />

In the same manner from (362), &c., we find<br />

~'y<br />

_<br />

—^y<br />

^/(l-<br />

f)<br />

(374)<br />

dtan' -ly:<br />

dy<br />

1+y^<br />

(375)<br />

dcot~ 'y-<br />

— dy<br />

1 + y'<br />

(376)<br />

'y =<br />

dy<br />

y'^ {y^-<br />

•1)<br />

(377)<br />

d cosec"" 'y =<br />

— dy<br />

y^{y^-<br />

1)<br />

(378)


DIFFERENCES AND DIFFERENTIALS OF PLANE TRIANGLES. 105<br />

CHAPTER XIL<br />

DIFFERENCES AND DIFFERENTIALS OF PLANE TRIANGLES.<br />

196. IN trigonometrical investigations it is Mg.29.<br />

often necessary to determine the effect of a<br />

small change in one of the data, upon the computed<br />

parts. Thus, Fig. 29, ii A, AB and<br />

A G, of the plane triangle ABQ, are the data,<br />

and AQ is subject to an error of QQ', the required parts will be<br />

subject to errors which are respectively, the differences between<br />

A QB and AG'B, AB Gand AB Q', B Qand B G' In the same<br />

figure, the data may be supposed to be A, A B and AB G, and the<br />

angle ABQ may be regarded as subject to the error QB G' which<br />

produces the corresponding errors in the remaining parts. In the same<br />

manner, the data may be A, A B, and A QB, A GB being variable,<br />

or. A, A B, and B G,B G being variable. In all these instances, A<br />

and A B are constant, while the remaining four parts are variable, and<br />

may be considered as receiving, simultaneously, certain increments<br />

which are related to each other. We propose, then, to solve the<br />

general problem :<br />

Bfi a plane triangle, any two parts being constant, and the rest<br />

variable, to determine the relations between the increments of the<br />

variable parts.<br />

It is evident that the solution of this problem resolves itself into<br />

an investigation of the differences of two triangles which have two<br />

parts in common. We shall consider the several cases successively ;<br />

distinguishing the triangle formed from the given one by the application<br />

of the increments, as the derived triangle.<br />

197. CASE I. A andc constant. The six parts of the given triangle,<br />

ABQ, Fig. 29, being A, B, Q, a, b, e, those of the derived triangle<br />

formed by varying all but A and c, are A, B -h AB, C 4- A (7,<br />

a -f Aa, b -f Ab, and c. In these two triangles we have<br />

A-i-B-i-Q= 180°<br />

A + B + AB+Q-^AG= 180°<br />

whence AB+AQ==0, AB = - AO (379)<br />

14


106 PLANE TRIGONOMETRY.<br />

Also in the two triangles we have<br />

a = c sin A cosec G<br />

a -{• Aa = c sin A cosec {G 4- AC)<br />

the half difference of which by (355) is<br />

[m)<br />

(w)<br />

- _ e sin A cos {G-h i AQ) ain ^ AG , .<br />

i^''-- sinCsin(C+AC) ^^'<br />

jAa _ j-Aa _ acos((7-f|AC7)<br />

sin IAB sin I AG sin(C4- AG)<br />

(380)<br />

The half sum of (m) and («) by (131) is<br />

'^+ ^^"'<br />

_ csin J.sin((74-i-AC)cos JAC<br />

sinCsin((74-A(7)<br />

which combined with {p) gives<br />

jAa _ ^ Aa _ a-f jAa<br />

tan J A5 "" tan i AC "~ tan ((7 4- |A6')<br />

(381)<br />

From (260) we have<br />

c sin ^<br />

tan C = b— ccosJ.<br />

whence<br />

J — ccos J. = e sin J. cot G<br />

b -^ Ab — c coa A = c ain J cot (C 4- AQ)<br />

the difference of which by (353) is<br />

, _ csin.4 sin A(7<br />

sinCsin(C4-A0)<br />

therefore<br />

Ab Ab a<br />

sin AB sin AC sin (C 4- AC)<br />

(382)


DIFFERENCES OF PLANE TRUNQLES. 107<br />

This equation gives by (135)<br />

_ J A5 _ a<br />

sin 1 AC cos 1 AC " siir(CTXC)<br />

and dividing (380) by this<br />

Aa _ cos(C4-iAC)<br />

A6 cos i AC<br />

(383)<br />

It is to be observed that the increments (or half increments) of<br />

the angles must be deduced from their sines or tangents, since it is<br />

only by these functions that a small angle can be accurately determined.<br />

Moreover, a small arc being nearly equal to its sine or tangent,<br />

the equations (380), (381) and (382) express very nearly the<br />

ratios of the increments of the sides to the increments of the angles,<br />

or rather to those increments reduced to arc by Art. 9, or Art. 54.<br />

198. CASE II. A and a constant. We have as in the preceding<br />

case AB = — A C; and in the two triangles •<br />

b ain A = a sin B<br />

(5 4- *A b) sin A = a sin {B 4- AB)<br />

the difference and sum of which give<br />

I Ab ain A = a cos (-S 4- J A.B) sin J AB<br />

(5 4- i Ab) sin A = a sin {B -\- I AB) cos J AB<br />

whence by di-vision<br />

lAb ^ _ i-A5 ^ b-^^Ab<br />

tan 1 AB tan J AC ~ tan (-B 4- i AB)<br />

{p)<br />

(384)<br />

In the same way<br />

hAe I'-C _ e4-iAc<br />

taniAC tanjA^ tan (C 4-i AC)<br />

(385)<br />

From the equations<br />

c sin A = a sin C<br />

{c 4- Ac) sinA =• a sin (C 4- AC)<br />

we find I Ac sin A = a cos (C 4- J AC) sin | AC {q)


log<br />

PLANE TRIGONOMETRY.<br />

which combined with the equation [p) gives, since sin JAC=— sinjA5,<br />

From [p) we also have<br />

A6 cos {B-{-l AB) ,Qgg.<br />

Ac ~ cos (C 4- i AC) ^ ><br />

jAb ^ _^A5 5 cos (.g 4-i A.g)<br />

sin J A.B ~ sin J AC sin 5<br />

(387)<br />

which, when A 5 is to be found from AB, is more convenient than<br />

(384). In the same way from [q)<br />

I Ac _ _ I Ac ^ ccoa{Q+ I AG) .^<br />

sin i AC ~" sin A C^B sin C ^<br />

2<br />

199. CASE III. 5 and c constant. We have<br />

c sin B — b sin C<br />

c sin (^ 4- A£) = & sin (C 4- AC)<br />

the sum and difference of which give<br />

c sin (^ 4- J AB) COS J A5 = J sin (C 4-. J AC) COS J AC {p)<br />

ccos {B + l A-B) sin J A-S = 6 cos (C 4- J AC) sin i AC (?)<br />

the quotient of these gives<br />

By (224) we have<br />

tan I AJ5 ^ tan {B + ^ nB) . „.<br />

tan J AC tan (C 4-J AC) ^ '<br />

a = b cos C 4- c cos 5<br />

a 4- Aa = 6 cos (C4- AC) 4- c cos [B +<br />

the sum and difference of which give<br />

AB)<br />

a4-iAa = Jcos(C4-JAC)COS JAC4-CCOS(5 -\-^AB) cos IAB<br />

— jAa= Jsin(C4-i-AC)sinjAC4- csin{B -\- ^AB) sin^AS<br />

These expressions are reduced by [p) and {q) to<br />

a-f jAa=ccos(.B4-jA.B)cosi-A.ScotiAC(tan|A.B 4-tan|AC) (r)<br />

— J Aa=c sin (.B 4- J A.B) cos J A.B (tan J A.B 4- tan J AC) (»)<br />

and by division<br />

I Aa _ a -f J A«<br />

cot (-B f J A5)<br />

(390)


DIFFERENCES OP PLANE TRIANGLES 109<br />

In the same way we have<br />

iAa<br />

tan JA5<br />

a-hiAa<br />

cot (C 4- i AC)<br />

(391)<br />

Since the sum of the three angles is constant,<br />

AJ. 4- A-B 4- AC = 0<br />

i(A5 4- AC) = -iAA<br />

therefore by (116)<br />

tan<br />

1 * r. ! 4. T ^rt sinl(A.B4-AC)<br />

J A5 4- tan J AC = -——^ -—-^<br />

cos i AB cos i AC<br />

sin J<br />

AA<br />

cos J A5COS J AC<br />

(9<br />

which substituted in (s) gives<br />

lAa _ esin{B-hi AB)<br />

sin J AA cos J AC<br />

(892)<br />

and in the same manner<br />

_i_Aa<br />

sin J AA<br />

_ 5sin(C4-jAC)<br />

COS J A.B<br />

(393)<br />

Substituting [t) in (r) we find<br />

sinjAC_<br />

sin J AA<br />

ecos(.B4-JA.B)<br />

a -f J A a<br />

(394)<br />

siniA.B 5cos(C-f-i AC)<br />

whence also -.—z -r = , , ,<br />

sin J AA<br />

a-\- i Aa<br />

By differencing the equation<br />

(395)<br />

we find instead of (392) and (393)<br />

a^ = J2 4- e^ — 2 5c cos J.<br />

I Afl _bc sin (J. 4- J AA)<br />

sin J A J ~ a-h^Aa<br />

K<br />

(396)


no<br />

PLANE TRIGONOMETRY.<br />

200. CASE IV. A and B constant. We have<br />

smB<br />

b = -;—T a<br />

sin .4<br />

5 4- A6 = -.—T (a + Aa)<br />

sin.A<br />

sin 5<br />

whence Ab = ^^^ Aa<br />

In this case the third angle is also constant and there are but<br />

three variables related by the equation<br />

_.^« = ^ = _ ^ (397)<br />

sin A sm.B sm U<br />

This case is not strictly included in the general problem as stated<br />

in Art. 196, since the two triangles have not two parts in common.<br />

201. The second members of the equations (380), (381), (382),<br />

(383), (384), (385), (386), (387), (388), (389), (390), (391), (392),<br />

(393), (394), (395), (396), involve the increments themselves, -which<br />

are the quantities sought. It is therefore necessary, in many cases,<br />

to solve these equations by successive approximations.<br />

For a first approximation we consider the increments in the second<br />

member to be = 0, employing ^ for ^ 4- ^AB, kc, and taking<br />

cos ^ AB = 1, &c. This will evidently produce but a slight error<br />

so long as the increments are small as compared with the entire<br />

parts of the triangle. We then obtain a. second approximation, by<br />

recomputing the equation in its complete form, employing in the<br />

second members the approximate values of the incremeijts. With<br />

these second values we may, in the same way, obtain a third approximation,<br />

&c. Theoretically, it requires an infinite number of sucli<br />

approximations to arrive at a perfect result; but in practice, the<br />

tenths or hundredths of seconds being the limits of accuracy, it is<br />

rare that more than a second approximation is necessary.<br />

It is also to be observed that in computing the values of small<br />

quantities such as the increments in question, we may employ logarithms<br />

of only four or five decimal places and take the angles to the<br />

nearest minute. This is in fact one of the chief advantages of computing<br />

by differential formulse, rather than by the direct formulae<br />

applied to each of the two triangles successively.


DIFFERENTIAL VARIATIONS OF PLANE TRIANGLES.<br />

Ill<br />

EXAMPLE.<br />

In a plane triangle whose parts are<br />

A = 68° 41' 48"-9 B = 35° 11' 3".4 C = 86° 7' 7 '-7<br />

a = 6053 b = 4082 e = 7068<br />

let A and a be constant while b is diminished by 50-5; to find the<br />

change in the angle B.<br />

We have in this case Ab = — 50-5; and by (387)<br />

i Ab sin B<br />

sin i AB = , / T, , , „,-<br />

^ 5 cos (.B 4- i A.B)<br />

1ST APPROX.<br />

2D APPROX.<br />

iA5<br />

5<br />

B<br />

lAB<br />

B+IAB<br />

log 1 AS<br />

ar. CO. log. b<br />

log sin B<br />

ar. CO. 1. COS {B -\- ^ AB)<br />

log. sin J A^<br />

lAB<br />

- 26-25<br />

4082<br />

36° 11'<br />

0<br />

36° 11'<br />

- 1-4028<br />

6-3891<br />

9-7606<br />

0-0876<br />

- 7-6396<br />

-15'0"<br />

35° 11'<br />

-15'<br />

34° 56'<br />

1<br />

\ - 7-6520<br />

i<br />

0-0863<br />

- 7-6383<br />

-14' 56"-8<br />

It is evident that changing the angle -B 4- i- AB by only three seconds<br />

would not affect the fourth place of its cosine; a third approximation<br />

is therefore unnecessary, and we have finally A.B = — 29'53"-6.<br />

As the log. sines of small angles do not vary proportionally with the<br />

angles, it will conduce to accuracy to employ the methods explained<br />

in Art. 115.<br />

DIFFERENTIAL VARIATIONS OF PLANE TRIANGLES.<br />

202. The equations (380), (381), (382), (383), (384), (385), (386),<br />

(387), (388), (389), (390), (391), (392), (393), (394), (39.5), (396) and<br />

(397) become differential by making the increments infinitely small,<br />

that is, by omitting the increments -svhen connected with finite quanti-


112 PLANE TRIGONOMETRY.<br />

ties by the signs 4- or —, and substituting the increment itself for its<br />

sine or tangent, and unity for its cosine, (Art. 192.) The character<br />

d must also be substituted for A. These changes being made, we<br />

easily deduce the following differential relations.<br />

CASE I. A and c constant.<br />

dB = - dQ<br />

da da<br />

a cot C<br />

JB^ ~ 'dQ ^<br />

db<br />

dB~<br />

db<br />

dQ<br />

da<br />

db^<br />

a<br />

sin C<br />

cos C<br />

> (398)<br />

CASE II.<br />

A and a constant.<br />

db<br />

dB<br />

dB =<br />

db<br />

' dQ<br />

- dO<br />

bcoiB<br />

dc<br />

iQ<br />

dc<br />

'dB~<br />

e cot C<br />

(399)<br />

db<br />

dc ~<br />

COS.B<br />

cos C<br />

CASE<br />

III.<br />

b and c constant.<br />

dA-irdB-\-dQ = = 0<br />

dB tan 5<br />

tanC<br />

da<br />

—T7I ^^ — '^ tan B.<br />

da<br />

dA'<br />

cainB = bsin C<br />

da<br />

= — a tan C<br />

dB<br />

> (400)<br />

dC c .^ dB<br />

-5-r = -cos B, -Tj—r. =<br />

dA a ' dA<br />

cos C


DIFFERENTIAL VARIATIONS OF PLANE TRIANGLES. 113<br />

CASE IV. The angles. A, B, G, constant,<br />

da db dc<br />

sin A sin B sin C<br />

(401)<br />

203. These differential relations are often employed when the increments<br />

are very small, instead of the equations of finite differences.<br />

We have already seen that the equation of differences often requires<br />

to be solved by successive approximations, the first approximation<br />

being in fact obtained by employing the corresponding differential<br />

equation. In all cases therefore where a second approximation in the<br />

use of finite differences could not alter the result of the first, it is<br />

plain that the differential equation is sufficiently accurate.<br />

The increments of the angles must generally be expressed in arc.<br />

Thus if dB is given in seconds we must divide it by B" = 206264".8,<br />

or substitute dB sin 1" for dB.<br />

dA<br />

But in such fractions as —j^, this substitution is evidently unnodB<br />

cessary provided the two increments are always expressed in the<br />

same unit, as minutes, seconds, &c.<br />

EXAMPLE.<br />

In a plane triangle whose parts are<br />

J. = 58°41'48"-9 5 = 36°ir3"-4 C = 86° 7' 7"-7<br />

a = 6053 b = 4082 c = 7068<br />

suppose b and c to be constant and the angle A to receive the increment<br />

dA = 20"-6 ; find da and dQ.<br />

From (400) we have<br />

da — dA sin 1" c sin B<br />

^ — dA ccosB<br />

aU =<br />

a<br />

log dA 1-3139 log (- dA) - 1-3139<br />

log sin 1" 4-6856 log c 3-8493<br />

lege 3-8493 log cos.B 9-9124<br />

log sin B 9-7606 ar. co. log a 6-2180<br />

log da 9-6094 log cZC - 1-2936<br />

da 0.407 dQ - 19".7<br />

16 B:2


114 PLANE TRIGONOMETRY.<br />

204. The error of employing the differentials in any case may be determined approximately<br />

by developing the equation of finite differences and comparing it with<br />

the corresponding differential equation. We shall select a simple example.<br />

We have from (387) and its corresponding differential equation in (399)<br />

, , b cos (B + i^B) . . _<br />

i A6 = ^.^—W^ sm J AB<br />

•^<br />

sin B<br />

Ab = b cot BAB<br />

the first of -which when developed gives<br />

sinl"<br />

. , „ 2bsin (B+iAB) . , _ . , „<br />

i Ab — b cot Bain iAB \ „ '- sin J AB sm J AB<br />

2 -^ sm .o<br />

or substituting sin J AB = J A5 sin 1", sin J AB = J AJS sin 1", and also B for<br />

B + i AB inthe second term, which will affect so small a term but slightly,<br />

Ab = bcotBAB<br />

sin 1" — — (A5 sin 1")'<br />

Comparing this -with the differential equation above, the error of employing the<br />

latter is approximately<br />

-|-(ABBinl")'<br />

which for AB = 1° is — -000015 b.<br />

It appears from this example that the error is expressed by a term involving the<br />

square of the increment; and if we develop aU the equations of finite differences we<br />

shall find that they differ from the corresponding differential equations by terms involving<br />

the squares and higher powers of the increment. Hence, employing the differentials<br />

instead of the finite differences amounts to neglecting the terms involving the squares<br />

and higher powers of the increments.<br />

205. The differential relations above obtained could have been deduced more directly<br />

from the formulse of plane triangles by differentiation, employing the values<br />

of the differentials given in Art. 192. Thus in CASE I, A and « being constant, if<br />

we differentiate the equation<br />

a = c sin ^ cosec O<br />

we have da ^ c sin A rf cosec (7<br />

= — c sin A cot G cosec G dG<br />

= — a cot GdG<br />

%a in (398)<br />

The studsnt may exercise himself by deducing the other relations of (398), (399)<br />

and (400) in. a similar manner.


TRIGONOMETRIC SERIES.<br />

^15<br />

CHAPTER XIII.<br />

TRIGONOMETRIC SERIES. DEVELOPMENTS OF THE FUNCTIONS OF iiN<br />

ARC IN TERMS OF THE ARC, AND RECIPROCALLY.*<br />

206. THE investigation of trigonometric series is most readily<br />

carried on with the aid of a few elementary principles of the Differential<br />

Calculus. All that will be required here will be no more than<br />

is generally given in the first chapter of a treatise on that subject,<br />

namely, the differentiation of simple algebraic functions, and Taylor's<br />

Theorem. We shall employ the following expression of this theorem :<br />

fiy+h)=fy^^.]L+'^Jl.tt.+^lJy..Jt_+^^, (402)<br />

i\!}-r ) ''"^ dy 1 dy^ 1-2 ^ dy^ 1-2-3 ^ ^ '<br />

in which fy denotes what f {y + K) becomes when A = 0 and<br />

d "ft/ cZ^ "Fa<br />

—^—, ' „ , &c., are the successive differential coefficients, or dedy<br />

dy^ '<br />

rivatives of fy.<br />

207. To develop sin x and cos x in terms of x.<br />

We shall first develop sin [y -\- x) and cos [y 4- x) by (402). By<br />

(361j and (362), if<br />

fy = siny<br />

, d.fy d sin y<br />

we have -5-^ = —^—^ = cos y<br />

dy dy<br />

d^.fy _ d cos y<br />

dy^ dy sin y<br />

d\fy dsvny ^<br />

dy^ dy "<br />

d*.fy<br />

dy*<br />

d cos y<br />

dy = am.y<br />

* The leading results of this Chapter being of very general utility and const<br />

application are printed in the larger type, but as they are not referred to in the ?<br />

sequent large print of this work, and moreover require a limited acquaintance w<br />

the Differential Calculus, the student can omit them at the first perusal, .and p<br />

directly to Part II


119 PLANE TRIGONOMETRY.<br />

SO that the values of the coefficients of the series (402) recur in the<br />

order -J- siny, -f cosy, — siny, — cosy, and therefore/(?/ -f- a;) =<br />

sin(«/4-a;) = sin«/4- cosy-j sin?/—-^ cosi/j^^Tg 4-&c. (40;?)<br />

If we commence with<br />

fy = cosy<br />

the coefficients will recur in the order -{- cos y, — sin y, — cos y,<br />

-f siny, and (402) will give<br />

3? OJ 3"<br />

cos (?/ 4- a;) = coay — siny -^ cosy-y^ 4- sinyj-^ +kc. (404)<br />

If now we put y = 0 in (403) and (404), siny = 0, coay = 1, thtalternate<br />

terms of the series vanish, and we have<br />

or Q' 2/ Cu<br />

sin a; = y - ^ ^ + 1.2.3.4.5 " 1.2-3-4-6-6-7 + ^°-<br />

(^*^^)<br />

cos a; = 1 - -j:2~ + rMl " 1-2-3-4-5-6 + ^"^<br />

(4°^)<br />

It may be observed that (406) can be deduced from (405) by differentiation.<br />

208. The series (405) and (406) are directly available for the construction<br />

of the trigonometric table. For this purpose x in the series<br />

must be expressed in arc, since (361) and (362), upon which the pro<br />

ceding demonstration rests, require x to be in arc. Art. 9.<br />

Find COS. 10°.<br />

EXAMPLE.<br />

Reducing 10° to arc, by Art. 9, we have<br />

a; = 10 X -01746329 = -1746329<br />

and computing separately the positive and negative terms of (406),<br />

1 = 1- - - ^ = - -01523086<br />

=<br />

1-2-3-4<br />

-00003866<br />

""""""""<br />

- T ^ =- - -00000004<br />

l__g<br />

1-00003866 - -01623090<br />

- -01523090<br />

cos 10°= .98480776<br />

agreeing with the tables, which give -9848078. The student may,<br />

for practice, verify any other sine or cosine of his table.


TRIGONOMETRIC SERIES. 117<br />

209. To develop tan x in terms of x.<br />

Representing the coefficients in the series (405) and (406) by letters, we have<br />

X — a,z' + a, z' — a, a;' -^- &c.<br />

tana; =<br />

1 •— a^z' + a,x* — a, x' + &c.<br />

(407><br />

1 1<br />

In which<br />

1-2 a. = &o.<br />

1-2-3<br />

If we perform the division of the numerator by the denominator, we perceive that<br />

the result will be a series containing only odd powers of a;, and commencing with the<br />

term x. But as the law for the successive formation of the coefficients is not easily<br />

shown in this way, we shall resort to the following process. Assume the series to be<br />

tan z = c,x + c,x' + a, x' + &c.<br />

and differentiate it; we find, by (363), after dividing by dx,<br />

1 4- tan' a: = c, -f 3 c, a;' 4- 5 o, a:* 4- &o.<br />

. or, since from the division of (407) we know that c, = 1,<br />

tan' X z= Sc,z''+ 5c,z' +7 c,z' + 9c,2^ + &c.<br />

The square of (m) is<br />

tan' X = Ci Ct z^ + c, c,<br />

which compared with (n) gives<br />

+ c, c.<br />

C, -^ c, c.<br />

z' + c, c,<br />

+ c, C-,<br />

(c, c, + c, c.)<br />

A' 4- c, f,<br />

+ c, c,<br />

+ c, e,<br />

+ c,c^<br />

x' + &c.<br />

4-&C.<br />

4- &c.<br />

4- &c.<br />

(m)<br />

(n)<br />

c, = Y (c, c, + c,c,+ c. e,)<br />

c, = -g- (c, c, + c,c^+ c, c, + c, e,)<br />

&c. &c.<br />

where the law of derivation is obvious. We have preserved the factor c„ althongh<br />

it is equal to unity, in order to render this law more apparent.<br />

Since the first and last terms of these expressions are equal, as also the terras<br />

equally distant from them, we may write them as follows :<br />

c. = 1<br />

Ci = Y (c. "•)<br />

«. = -r (2 «a ".)<br />

c, = y (2 c, e, 4- c, c.)<br />

Vs = -- (2 c, c, -f 2 c, c,)<br />

c„ = jy (2 c, c. 4- 2 c, c, 4- c, c.)<br />

&c.<br />

&c.


118<br />

PLANE TRIGONOMETRY.<br />

in which form any coefficient c,„ + , when n is even, is expressed by y terms all ol<br />

ffl 4- 1<br />

whose coefficients are = 2 ; and when n is odd, by —^—terms all of whose coefficients<br />

are 2 except the last, which is 1.-*<br />

If we now substitute the value of c, = 1, and deduce the numerical values of the<br />

coefficients successively, we shall find<br />

, x- , 2 a;' 17 a;' 62 x' 1382 a:"<br />

,tan x-^z + -^ + - ^ + -gSTgy 4" -3i75:7;:9- + W^^T^ll + *"*• ^^^^><br />

210. To develop cot z in terms of z.<br />

If we invert (407) we have<br />

1 — a,a;' 4- a,a;' — &o. .....<br />

cot X = ^ .T . J— (409)<br />

X — a, x' 4- "•i^ — ""'-<br />

and the first term of the actual division is —, the second term — (a, — a,) z, and<br />

the succeeding terms evidently involve only the odd powers of x. Therefore let<br />

cot X = d.x — d,x' — d.x' — &o. (o)<br />

X<br />

The coefficients cannot be determined by the method of the preceding article in<br />

consequence of the negative exponent in the first term; but they are directly deducible<br />

from those of the series for tan x. We have by (142)<br />

tan a; := cot X — 2 cot 2 x<br />

(p)<br />

Now the series (o) being true for any value of x wUl give cot 2 x by substituting 2 z<br />

for X, whence<br />

2 cot 2 X = — — 2' rf, X — 2' rf,x» — 2' d,x' —- &o.<br />

X<br />

Subtracting this from (o) we have by (p)<br />

tan X = (2' — 1) (7, X 4- (2* — 1) d,x' + {2' — 1) d, z' + &c.<br />

Designating the coefficients of (408) by c„ c„ c„ &c. we have also<br />

tan X = c^j} + c^x' + c,-, x' + &o<br />


Substituting the values from (408)<br />

TRIGONOMETRIC SEE,tES. 119<br />

.-. = 1 c. = - c--. = -3-&o.<br />

and reducing the coefficients to their simplest forms, we find the series (o) to be<br />

1 a; x= 2z' x' 2x"<br />

cot X = 1- . A,c C410)<br />

X 3 3'-5 3'-5-7 3»-5'-7 3'-5-7-9-ll ^<br />

211. By a process similar to that of Art. 209, but which we leave to the student,<br />

we find<br />

x' 5 X* 61 x^ 277 x"<br />

secx = i4--4-—4-2j:g^4-2^T35:f 4-&0. (4ii)<br />

And from (408) and (410) by means of the formula<br />

cosec X = J (cot J a; 4- t^^i J ^)<br />

•we find<br />

1 z 7 x' 31 x' 127 x'<br />

cosecx = -4- —4-25:3^4-2^^31:5:^-^ 2^:51:5^:7+ ^'=-<br />

212. yo develop sin~^ ^ m ierms c/«/. (See Art. 87).<br />

Let x = sin~^ y (or sin x = y); then by (373)<br />

(^^^^<br />

d^-<br />

V{l-y')~^^~y'<br />

Developing the second member by the Binomial Theorem,<br />

| = l + i^^ + 2-3^^ + g|/-^&c. {m)<br />

As this contains only even powers of y, the series from which it<br />

would be obtained by differentiation must contain only odd powers<br />

of y; therefore, let<br />

X = a^ 4- a^^ -f a^^ + a^y'' 4- kc.<br />

{n)<br />

There will be no term independent of y if we limit x to values between<br />

0 and db 90°, for then when «/ = 0 we must also have x — 0.*<br />

Differentiating, we have<br />

J<br />

= «! 4- 3 a3?/^ 4- 5 a,y' + 7 ay 4- kc.<br />

which compared with (m) gives<br />

1 1*3 1"3'6<br />

ai=l 3 33 = 2 5fl!j=2q '''ay = 2:^&c.<br />

•*• The series (413) obtained under this limitation expresses but one of the values<br />

of sin-^y, but if we denote the series by s, we shall have by (95) the following expression,<br />

including all the values,<br />

sin~' y = n ?r.+ (—1)"3<br />

n being an integer, positive or negative, or zero.


120 PLANE TRIGONOMETRY,<br />

therefore (n) becomes<br />

a; = sin-2, = y 4 - | . 4 4 - g |-4-1|| • f + &c. (413)<br />

It is unnecessary to develop cos~'^ since we have<br />

cos '«/ = -o<br />

Sin '«/<br />

213. To develop ta.n~^y. Let x = tan^^y, then by (376)<br />

'^ = {l + y^)-^ = l-y^+y*-tf+kc.<br />

ay<br />

from which we infer, as in the preceding problem, that the required<br />

series contains only odd powers of y; therefore let<br />

then<br />

x = a^y-h a^y^ 4- a^ + ay 4- &c.<br />

-T- = «! 4- 3 ay 4- 5 ay 4- 7 ay 4- &c.<br />

ay<br />

which, compared with (m), gives<br />

ai=l 3a3 = — 1 5a5 = l 7a, = — l&c.<br />

so that the series is<br />

x = ta,n-^y = y — iy^-{-iy^—^y''-\-kc. (414)<br />

214. To compute the ratio (= TT) of the circumference of a circle<br />

to its diameter.<br />

We have heretofore assumed this ratio to be known from geometry,<br />

where it is found by means of circumscribed and inscribed polygons<br />

which are made to differ from the circle by as small a quantity as we<br />

please; but (414) enables us to express its value in a series. We<br />

have tan -2" = 1) therefore if we make ^ = 1 in (414) we have<br />

{m)<br />

(n)<br />

But this series converges too slowly to be of any use. To obtain a<br />

rapidly converging series y must be a small fraction. We might em-<br />

TT 1 . . .<br />

ploy tan •— = —^-Q- (Art. 29), but in consequence of the radical, it i?


TRIGONOMETRIC SERIES.<br />

12I<br />

simpler to resolve -^ into two or more arcs whose tangents are known,<br />

and to compute the value of each of these arcs by the series.<br />

effect this let<br />

then by (123)<br />

To<br />

-^ = tan ^ ^ 4- tan~'i{' (416 ><br />

, ^ ., t-i-t'<br />

tan -p = 1 =<br />

4 1-tt'<br />

whence *'=T+1 (417)<br />

from which, assuming any value of t at pleasure, the corresponding<br />

yalue of t' is found.<br />

If we take i = 1, -we find if' = J; therefore by (416) and (414)<br />

-— = tan ^ 1 -f tan"" 'J<br />

A few terms of these series give<br />

-J = -4636476 4- -3217506 = -7853982<br />

TT = 3-14169<br />

more accurately vr = 3-14159 26536 89793<br />

1 3<br />

If we take t =-^, we find *' = "r. but the above supposition is<br />

evidently the best adapted for rendering both series sufficiently convergsnt.*<br />

215. To resolve sin z and cos x into factors.<br />

The series (405) shows that x is a factor of sin x, and gives<br />

sinx = x(l-.^4-j-^_&c.)<br />

(p)<br />

16<br />

* See NOTE at the end of th.s chapter, p. 124.


122 PLANE TRIGONOMETRY.<br />

and the factors of the series within the parenthesis must evidently be of the form<br />

x^<br />

1-J (?)<br />

A being a constant, but having a different value in each factor. The required factors<br />

must be such as to reduce the second member of [p) to zero whenever the first<br />

member is zero. Now sin x is zero for the value x = 0, whence x is a factor as already<br />

seen, and also for x = rtnjr, « being any integer; therefore the general<br />

valuj of (j) is<br />

whence<br />

A<br />

.4 = n'sj-'<br />

which, substituted in [q), gives as the general factor<br />

l--i^<br />

n'jr'<br />

Making n successively = 1, 2, 3, &c., the equation {p) becomes therefore<br />

^^^=^(^-l4)(^-2-^'-)0-3S^)---<br />

^''')<br />

The factors of cos x in (406) must also be of the form (j); but cos x is zero for<br />

x=':ii^2n+\)<br />

-^, n being any integer or zero, and the general value of (q) is<br />

(27»4-l)V_Q<br />

.4.2' ""<br />

whence<br />

A = -—"X^<br />

which, substituted in (j), gives the general factor<br />

2' x'<br />

•'~(2«4-l)'5r'<br />

Making n successively = 0, 1, 2, 3, &c., we have<br />

216. Logarithmic ainea and cosines. By means of (419) and (420) the logarithmic<br />

sines and cosines of the tables are readily computed.<br />

^ 7r ,<br />

Put X = m —, then<br />

and taking the logarithms<br />

TT mv 1. m'\ /. m'\ /, m'\<br />

«nm-= — (1-2^) (^1-J,) (^1-gj) .:.<br />

COS.-^=(l--)(l-g,)(l--)...<br />

log sin . ^ = log |1 4- log m + log ^1 _ ^^ 4- log ^1 — ^^ 4. ..<br />

logCOS -^ = log (1 - y,) 4- log (1 - ^,) 4- log (1 - ^,) 4- -


Developing these logs, by the known formula<br />

TRIGONOMETRIC SERIES. 123<br />

log(l — n)= — M(n+in''+in'+<br />

&o.)<br />

(inwliioh M = modulus of common logs.) and arranging according to the po-nersol<br />

m, we have<br />

log sin<br />

= 108-0-4- log '»<br />

•'"•T(2-'+T'+T' + ^''-)<br />

.vf(i, 4-14-14-.0.)<br />

•&o.<br />

.EfL + ^+^ 6,4-^0.)<br />

3 V2'<br />

log cos - ^ = - m'. -(^.j, + - 4- _ + &c. )<br />

—•I(|.-^|.+i+-)<br />

-"^ •¥(-?+ 3-'+5-.+ *


124 PLANE TRIGONOMETRY,<br />

Compute log sin 9°. We have<br />

and therefore by (421)<br />

EXAMPIE.<br />

m X 90° = 9° m = — logm = — I<br />

log sin 9° = 10-19611 98770 — 1-<br />

— 0-00178 5%45<br />

— 0-00000 14689<br />

— 0-00000 00023<br />

= 10-19611 98770 — 1-00178 74357<br />

log sin 9°= 9-19433 24413<br />

217. If in (419) we put x = -^, we have<br />

«^'^f = ^=YO-i)0-i)0-i)-<br />

_ fl- /2' — 1\ /4' — 1 \ /6' —1\<br />

"~ 2 \ 2' / \ 4' / \ 6' / •"<br />

_ ^ (2-l)(2 4-l)(4-l)(4-hl)(6-l)(64-l)...<br />

" ~ 2 2 2 4 4 C 6 . . .<br />

i j - 2 2 4 4 6 6<br />

whence .^ = - . - . - . - . - . - . . . (423<br />

which is Wallia'a expression of jr.<br />

NOTE to page 121. Computation of ir. Many other series besides those of Art.<br />

214, may be given for computing rr. One method of obtaining them is to resolve<br />

tin"' t and tan~' t into two others, and thus make J A- to depend upon three or more<br />

arcs. From (194) we easily deduce<br />

1 1 n<br />

tan-' — = tan-' —; 1- tan-' -5-j p- (a)<br />

m m+ n rrir+ mn + \<br />

1 1 • n<br />

tan—' — = tan-' tan—' —^ J-T- • (V\<br />

m m — n m —mn + \ ^<br />

in which m being given, « may be assumed at pleasure. The numerators of the<br />

ft-actions in the last terms will reduce to unity when m' -|- 1 is divisible by ra; ii<br />

therefore we assume n and p so as to satisfy the condition<br />

we shall have<br />

np =. rt^ +\<br />

tan—' — = tan—' ;— -f- tan—' —; 1 ol<br />

m m + n m+ p<br />

tan—' — =: tan—' tan—' • lel<br />

m m — n p — m '<br />

(c)


TRIGONOlVfETRIC SERIES. 125<br />

For example, let m = B; then m' 4- 1 = 10 = 1 x 10 = 2 x 5, so that we may<br />

take n = 1, J) = 10; or » = 2, ^ = 5, whence by (d) and (e)<br />

tan—' — = tan-' — + tan"' —<br />

O 4 Id<br />

= tan—' -^ — tan-' —<br />

Substituting in (418)<br />

= tan~' — 4- tan—' -5-<br />

~ = tan-' -^ + tan-' j + tan"' y^<br />

= 2 tan"' — tan—' —<br />

= 2 tan- i 4- tan-' i (/)<br />

= tan-' y 4- tan-' — 4- tan— -^ (g)<br />

The equation (/) was employed by CLAUSEN of Germany, in computing it- to 200<br />

decimal places, and (g) was employed by DASB, also of Germany, in computing TT to<br />

the same number of figures. These computations were carried on independently of<br />

each other, and the results when communicated to SCHUMACHER, (who gives them in<br />

the Astronomische Nachrichten, No. 589), were fotfnd to agree to the last figure<br />

They prove the value previously found by Mr. Rutherford to be erroneous beyond<br />

the 150th figure.<br />

By means of the formulse (a), (6), (c), (d) and (c) we may again subdivide the<br />

arcs as often as we please. Thus, it is easy to deduce<br />

|- = 2 tan— -- 4- tan— -=-4-2 tan"' —<br />

= 2 tan~' —- 4- tan—' —- -I- tan"' -— + tan-' -j7--r-<br />

5 ' 4 ' 7 268<br />

= 4 tan- i - tan- 1 4- tan"' i 4- tan- ^<br />

= ^ *''""' T - *^'^~" W + '"""' 2l8<br />

= 4 t a n - i - t a n - ^<br />

which last is known as Machin'a formula. In deducing it we have reduced the difference<br />

of two arcs to a single aro by means of formula (a).<br />

Another method is, to find by trial, or otherwise, an arc a multiple of which is<br />

nearly equal to -7-, and whose cotangent is a whole number; and then deduce the


i26<br />

PLANE TRIGONOMETRY.<br />

difference between this multiple and -j-.<br />

Thus it is known (from the trigonometho<br />

tables) that cot 11° 15' = 5 nearly; therefore by the last formula of Art. 79, putting<br />

1<br />

tan X = -—,<br />

and by (194)<br />

therefore<br />

as was foimd above.<br />

1 120<br />

4 tan— -=- = tan— -—^<br />

o 119<br />

, 120 a- ,120 , , 1<br />

*""" W - T = *"" IT^ - *''•'"' 1 = "^'^ 239<br />

!!- 1<br />

-r- = 4 tan—'<br />

4 5<br />

tan~<br />

If'we resolve tan-' -—^ by means of (c), (d) and (e), we have m = 239,<br />

m* -|- 1 = 57122 = 2-13' = np, which offers several suppositions for n and p; if<br />

we take n = 13' = 169 and^ = 2-13" = 338, we find by (e)<br />

^ =<br />

4tan-l-tan-^4-tan-l<br />

whicii was employed by Rutherford.<br />

If we take n = 1, p = 57122, we find by {d)<br />

^ =<br />

4tau-i_tan-^-tan--^


EXPONENTIAL FORMUL.^. 127<br />

CHAPTER XIV.<br />

EXPONENTIAL FORMULiE.<br />

TRINOMIAL OR QUADRATIC FACTORS.<br />

218. To demonstrate Buler's formulae<br />

cos a; = i(e='v'-i4-e-'^i^-') (424)<br />

sina; = ^-^-—J-(e^>'—— e-«v'-i) (425)<br />

in which e is the Naperian base of logarithms, or,<br />

It is shown in the theory of logarithms that<br />

wtcve for brevity we write<br />

^^ = ^ + (!) + |)+($+($+^-<br />

(1) = 1 (2) = 1-2 (3) = 1-2-3, kc.<br />

We have by (406) and (406), employing the above notation,<br />

^ a;^ ^ a;^ a;« .<br />

cos a;=l-p-^ 4-^-^4-&c.<br />

m<br />

^^'^^=(l)-(3) + (5)-(7) + ^°the<br />

terms of which are the same as those of (426), but with alternate<br />

signs. If the signs in these two series were all positive, the sum of<br />

the two would be equal to (426); and it is evident that we shall make<br />

them positive by substituting<br />

x^= —z^ or a; = a v' — 1<br />

which gives<br />

cosa; = l4-p^4-p^4-^4-&c,<br />

. z^ s*" z'<br />

Bin a; = s %/ — 1 (1 4- TgT -F Tgs -F 7=v 4- &c. ^


128 PLANE TRIGONOMETRY.<br />

whence<br />

But<br />

z , ^ , z" z'<br />

sm a; = 7^ 4- TQS 4- 7F\ 4- 7=v 4- &c.<br />

^_1 — (1) ' (3)-(5)-(7)<br />

,osx-^-^-^ainx^l-\-^^ + y^+^^ + kc.=^<br />

therefore<br />

cosa; —i/—1 sina; =e-'^''-i (427)<br />

If in this equation we substitute — x for x, we have, by (56),<br />

cosa; 4-^/ —1 sina; = e='''-i (428;<br />

The sum and difference of these equations are<br />

2cosa; = e'"»'-i4-e^"'»'~' (429)<br />

2-^/-lsina; = e'"t'-i —e-*/-' (430)<br />

whence (424) and (425).<br />

219. The quotient of (430) divided by (429) is<br />

220. If we put<br />

y = e''^-'' = cos a; 4- %/ — 1 sin a; (432)<br />

we have y~^ — e~=° i'"' = cos a; — i/ — 1 sin a; (433)<br />

and (429) and (430) become<br />

2 cos a; = y -f «/-' (434)<br />

2V — Isinx =-y —y-^ (435)<br />

If mx be substituted for x in these formulse, we have<br />

ym = ^r^xv-•L = COS mx-\- >/ — 1 siu mx (436)<br />

y-^ = e-m=^ V-i= COS ma; — v^ — 1 sin mx (437)<br />

2 cos Twa; = t/ 4- ^z- (438)<br />

2 s/ — i sm ???a: = J/ — y- (439)


EXPONENTIAL FORMULA. 129<br />

221. Moivre's Formula. The value of y from (432), compared<br />

with (436) gives<br />

(cos X -\- \/ — 1 sin a;) = cos mx 4- >/ — 1 sin mx (440)<br />

which is Moivre's Formula. It shows that the involution of the expression<br />

cos a: -f N/ — 1 sin a; is effected by the multiplication of<br />

the angle.<br />

Again, if we multiply (432) by<br />

we have<br />

cos a:' 4- -v/ — lsina;'= e^'»'-^i<br />

(cos a; -f v' — 1 sin a;) (cos a;' 4- -v/ — 1 sinx') = ei'^'^'^V- i<br />

= coa ix-{• x')-\- \/ — 1 sin [x -f x')<br />

which shows that factors of this form are multiplied by the addition<br />

of the angles.<br />

We have also<br />

[cosx-'t\/—lsina;)(eosa;—\/—lsina;)=cos^a;4-sin^a;=e°= 1 (441)<br />

222. General form of Moivre's Formula. As long as m is an integer, both members<br />

p<br />

of (440) can have but one value ; but if ??» = —• the first member becomes<br />

v_<br />

(cos X -f- y/ — 1 sin x)« = -v^ (cos z + ,y — 1 sin z)'<br />

which has q different values* in consequence of the radical of the degree q, while<br />

the second member<br />

P<br />

P<br />

cos •=— x+ .y — Isin — X ( i)<br />

S 3<br />

has but one value.<br />

In order that both members may have the same generality, as should be the case<br />

with every analytical expression, it is necessary to suppose that we take for the arc<br />

X not merely the arc less than the circumference which has the given sine and cosine,<br />

but also all the arcs which have the same sine and cosine; that is, c denoting the<br />

circumference, all the arcs<br />

X, z + e, z + 2c, X + Sc, &c.<br />

Now there is an infinite number of these arcs, but only q of them can give differentvalues<br />

to (a); for all the values of the arc in (a) will be<br />

Zx, ^z+^A' ^ x + ^ , .P.z+^J^=^,<br />

q ' g ^ q' g 5 ? g<br />

^x + l^, ^x+ ^1+^)P\ &c. &0.<br />

g g g 2-<br />

-* That is g values real and imaginary; thus it is shown in algebra that ^^u' = + a<br />

and —a; ;ya'= + a, a( ^ ) and a( ^ )><br />

ya'=<br />

4- a, — ff, 4- «^/ — 1, — a v^ — 1; &c.<br />

1-


130 PLANE TRIGONOMETRY.<br />

But —x+ .Ss— z= — X + pe has the same sign and cosine as — x;<br />

g g g i<br />

£- x+ '•" "*"—i-£-_ = (—x4-—)4-P


TRINOMIAL OR QUADRATIC FACTORS. 131<br />

TRINOMIAL OR QuAnitATio FACTORS.<br />

225. To find the quadratic {trinomial) factors of the expression z' — 2 z "• cos if -j- 1 ;<br />

m being integral.<br />

By (438) and (484) we have<br />

y"» — 2 y"* cos mx -f 1 =0<br />

y^ — 2y cos x 4-1 = 0<br />

Therefore if we ^nt y = z, mx = 2n v + •p, or x = ^^'^ + 'i> ^^<br />

m<br />

2' —2z'»cos^-|-1 = 0 (448)<br />

3' — 2z co8""'^+^-f-l = 0 (449)<br />

As these two equations exist at the same time, they have common roots, and the<br />

second is therefore a divisor or factor of the first; but this factor has m values in<br />

consequence of the m values of cos "'^ "^ ^ (Art. 222), found by making<br />

B = 0, 1, 2, 3 . . . OT — 1. Therefore the m quadratic factors of (448) are all ex<br />

pressed by (449), and we have<br />

sara_2z'" cos * -f 1 = (^ z' — 2z cos — -I- l)<br />

\ m l<br />

x(^'-2zcos ^^+^ -t 1)<br />

V m l<br />

x{z^-2zcos±^t<br />

+i)<br />

X<br />

/ . n 2 (m — 1) v+ ip , , \<br />

X (^z'-2zcos -^^ ^—H^ + lj (450)<br />

226. To obtain the simple factors of (448), we have only to find the two simple<br />

factors of each of the quadratic factors in (450), or to find the two factors of the<br />

general quadratic (449). Now, by the theory of equations, if z, and z^ are the two<br />

roots of (449), the first member is equal to<br />

but we have by (432)<br />

(Z — 0,) (Z — Zj)<br />

1 , 1 . 2n7r+


132 PLANE TRIGONOMETRY<br />

EXAMPLES.<br />

1. Find the quadratic and simple factors of<br />

z* —2z'4- 1<br />

Eere m = 2, 2 cos * = 2, cos ^ = 1, t = 0; and by (450),<br />

by (451)<br />

s« _ 2 z' -f 1 = (z' — 2 z cos 0 4- 1) (z' — 2 2 cos fl- 4- 1)<br />

= (z._2z4-l)(z'4-2z4-l)<br />

= [z — (cos 0 4- v' — 1 sin 0)]<br />

X [z — (cos 0 — v^ — 1 sin 0)]<br />

X [z — (cos fl- 4- N/ — Is '-)]<br />

X [z — (cos fl- —.,/ — 1 sin fl-)]<br />

= (z_l)(z_l)(z4-l)(z4-l)<br />

2. Find the factors of z* -f- 2 z' 4- 1- Here m = 2, 2 cos # = — 2, 0 = A-, and<br />

z'4-2z'4-l = (z'4-l)(z'4-l)<br />

3. Find the factors of z* — z' 4- 1.<br />

= (z-v/-l)(z4-^/-l)(^-v'-l)(^4-^/-l)<br />

z* — z' -t- 1 = (z' — 2 z cos 30° 4- 1) (z' 4- 2z cos 30° 4- 1)<br />

= (z' — z v^ 3 4- 1) (z' 4- z V 3 4- 1)<br />

= (z-i^S-i^-l)(z-.i^3 + i^-^<br />

X (z4-iV34-Jv/-1)<br />

4. Find the factors of z" — 2z° -f- 1.<br />

(^4-Jv'S-Jv'-i)<br />

2«_2z'4- 1 = (z'—2z4- 1) (z'-f z4- 1) (z'4--z4- 1)<br />

= (z-l)'(z4-J4-Jv'-3)'(z4-J-J^-3)'<br />

227. To find the quadratic factors of ^ — 1 when m is odd.<br />

In (450) let ^ = 0, it becomes<br />

(z" — 1)' = (z — 1)' X ( z' — 2 z cos ~ + l)<br />

X ( z' — 2 z cos -i^ 4- l)<br />

\ m l<br />

X<br />

x(z'-2zcos^^"'-^)"4-l) (452)<br />

Now m being odd, m — 1 is even, and the number of trinomial factors in (452),<br />

exclusive of (z — 1)', is even ; but<br />

2(m —l)fl- / 2n-\<br />

i —^ '-— = cos 12 B- ) = cos<br />

m \ m J<br />

2;<br />

so that the first and last of these factors are equal. In the same manner it is shown<br />

that any two of these factors equally distant from the first and last are equal,


TRINOMIAL OR QUADRATIC FACTORS.<br />

13ii<br />

Bierefore, imiting these equal factors and extracting the square root of both members,<br />

we have, ichen m is odd,<br />

z" — 1 = (z — 1) X ( z' — 2 z cos -i^ 4- 1)<br />

X (z^ — 2zaoa — + l)<br />

\ m /<br />

X<br />

X (z'-2zcosi!^^1^4-l) (453)<br />

228. To find the quadratic factors of i^ — 1, when m is even.<br />

When m is even, m — 1 is odd, the number of factors in (452), exclusive of (z — 1)',<br />

is odd, and the middle factor will not combine with any other. This factor is the<br />

( __ I and contains<br />

and is therefore equal to<br />

2(_<br />

cos = cos fl- = 1<br />

m<br />

z" 4- 2 z 4- 1 = (z 4- 1)»<br />

BO that uniting the remaining factors, and extracting the square root, we have,<br />

when m is even.<br />

. 1 = (z — 1) (z 4- 1) X ( z" — 2 z cos ^4-1)<br />

x(z'-2zcos^4-l)<br />

(^,._2zcos(^^4-l) (451)<br />

229. To find the factors of z^ + 1, when m is odd.<br />

In (450) let j) = A-, it gives<br />

(z 4- 1)' = ( z' — 2 z COB -^ 4- 1)<br />

X<br />

(^z'-2zcos-^4-l)<br />

/ „ . (2 m —l)fl- , A<br />

X ( z^ — 2 z cos i '- \- 1 )<br />

\ m /<br />

and it is easily shown, as in the preceding articles, that the factors equally distant<br />

from the first and last are equal, and that tlie middle term is z' -f- 2 z -j- 1 = (z -}- 1 )*'<br />

M


134 PLANE TRIGONOMETRY.<br />

Hence we find, when m is odd,<br />

z" -}- 1 = (z 4- 1) X ( ^^ — - ^ "^os Vi "^ •^)<br />

X ( z' — 2 z cos -^^ -1- 1 )<br />

\ m J<br />

230. To find the factors of if" + 1, when m is even.<br />

The same process gives<br />

(^,= _2zcos(^^^^4-l) (455)<br />

.4-l =<br />

(z'-2zcoB^4-l)<br />

X (z' —2rco8^4-l)<br />

X (,._2zcos(!!^^4-l) (456)<br />

231. The simple factors of (453) and (454) are obtained from (451) by putting<br />

^ = 0, and those of (455) and (456) by putting ip = JT. There will be found pairs<br />

of equal factors as in the preceding articles, but all the different simple factors will<br />

be found by taking only the positive sign of the radical ,y — 1.<br />

232. Any function of the form z"" —2p z"" 4- ? ay also be resolved into quadratic<br />

factors. It is only necessary to reduce it to one of the preceding forms. By<br />

i-esolving the equation<br />

we shall find from its two values of z"<br />

2«m _ 2j3 z" -)- J = 0 (457)<br />

z*"' —2i>z»4-j= (z- —(;>4-^^' —?)) X {^"'— (P — s/p'- g))<br />

and if we put the absolute term in one of these factors = ±01'" (according to its<br />

sign) it becomes<br />

= a'" ( ^ ± 1 ) = a"* (z ± 1)<br />

in which z = az', and the factors of this last expression may be found by one of<br />

the preceding articles.<br />

If, however, the values of z" in (457) are imaginary, i. e. if p" < q, this method<br />

fails to discover the real quadratic factors, and we must proceed as follows. Put<br />

q = o*", then the proposed function becomes<br />

„^/^_i^ !!: + i) = ,.(^_i^.z'»4-i)<br />

\ a"" a"' o"" ^ / \ a"' ' /<br />

P<br />

in wliich z = az'; and since in the present casep < a", -jj is a proper fractio:<br />

and we may put — = cos (p, which reduces the given function to the form (450).


MULTIPLE ANGLES.<br />

135<br />

CHAPTER XV.<br />

TRIGONOMETRIC SERIES CONTINUED.<br />

MULTIPLE ANGLES.<br />

233. THE true developments of sin mx and cos mz in series, when m is not restricted<br />

to integral values, were first obtained by Poinsot, and form the subject of °<br />

memoir read by him before the French Academy of Sciences, in 1823.* The following<br />

problem is the basis of these investigations.<br />

234. To develop (4 4- •v/'f — 1)"*, in a series of ascending powers of k. Let<br />

and assume<br />

z = (k+^lc" — !]<br />

z = A„ + A,k + A^k^ +<br />

Differentiating (o) and putting<br />

we find<br />

dz<br />

A,k'<br />

z' = m (A4-.y/i' —iV 'x(lthe<br />

v/(*'-l)/ i)-<br />

square of which gives<br />

m'z'—(4' —l)z" = 0<br />

Differentiating this and putting<br />

we find, after dividing by z',<br />

y. _ "^^<br />

~" IT<br />

m' z — kz! — (A' — l)z" = 0<br />

Again, differentiating (6) twice, we find,<br />

^ =A, + 2A,k+S A,7c'. + nA K'-' .<br />

.4--in A"<br />

v/(*'-l)<br />

z" = 1.2 A, + 2.3 A,k+ 3.4 A^ 4' + (n—l) n A„ J»-^<br />

Substituting in [d) the values of z, z', z", given by (6) and (e), we have<br />

D = m'A + m'A,<br />

J»<br />

— -4,<br />

4-1.2 A<br />

4-2.3^,<br />

A-f- m' ^<br />

— 2^,<br />

— 1.2 4,<br />

4-3.44.<br />

4-m'4„<br />

— ra 4„<br />

— (n —1)»4„<br />

4-(»4-i)(«4-2)4,.+,<br />

(a)<br />

(A.<br />

(«.'<br />

(rf)<br />

} (^<br />

m which each of the coefiioients of the ppwei-s of k must be zero. To discover the<br />

aw which governs these coefficients, it will suffice to examine that of the general<br />

Uirm, or the coefScient of k", which is<br />

(m' — «') A„+(n+ 1) [n + 2) 4„+, =0<br />

whence<br />

. m' —«' .<br />

^»+. — — (j, + mn+2) ^"<br />

•* See the published memoir, " Reeherches sur VAnalyse des Sections Angulaires,"<br />

?aris, 1825


I'dH<br />

PLANE TRIGONOMETRY.<br />

so that frcm the first coefficient, A„ we find by making n = 0, 2, i, 6, &o.,<br />

^'<br />

TT^<br />

_ m' —2' m' (m' — 2')<br />

^4 34— ^' — r2"^34 "^°<br />

_ '«° —4" ^ _ m'(m' —2') (m' —4')<br />

•^' - 5^6~ ^' 1-2-3-4-5-6 ^°<br />

and from the second coefBcient, 4„ we find by making « = 1, 3, 5, &o.<br />

m" —1' ,<br />

Therefore, if we put<br />

m'-3' (m'-l')(m'-3')<br />

^' f5~ '-• 2^34^5 ^'<br />

. _ m' —5' _ (m' — 1') (m' — 3') (m' — 5°)<br />

'~ 6-7 '~ 2-3-4-5-6-7<br />

&c.<br />

the equation (J) becomes<br />

z = 4o^4-4,.K-'<br />

and it only remains to find A„ and 4,. In (a), (6), (c) and (e), put & = 0; we find<br />

z = (^ — l)"' = A„ z'= m (v'— 1) "•-'= 4.<br />

Therefore we have, finally.<br />

z = (J4-.^A;» —1)"'= (.y—l)''K+ (^ — l)''—mK' (458)<br />

235. To develop (y/1 — h' + h .^/ — 1)"" in a series of ascending powers of h. We<br />

nave<br />

(^/l _ A' 4- A ^ - 1)- =(^ - !)•» (h + v'A' - 1)"<br />

therefore by (458), exchanging k for h,<br />

(.^ 1 _ A' 4- A ^ -1)-" = (v'-l )•» [(y - l) i? 4- (-v/ - l)"-' mS'-]<br />

in which 5" and fl"' are what £• and K' become when h is put for k.<br />

the imaginary factors in the second member, observing that<br />

(v/ -1)" X (^/ D" = (v' l)"" = (1)?<br />

Ooabining


MULTIPLE ANGLES. 137<br />

(which must not be put equal to unity, since m may be a fraction, and unity has<br />

imaginary roots,) and also that<br />

(^_l)mX (V—l)'»-' = ^—1(,/ —l)-' X(y—l)"'-' = v/ —1(1)~<br />

we have<br />

in which<br />

(v'l ^A"'4- h^—\)^ ={l)^ 11+ ,/ _ 1 (l)^m //' (459)<br />

„ 1 »«' ,. . '«" (m' —2') ,<br />

m—I<br />

^'= A - f!l^ A'+ i!^!^i3ij^ A. - .c.<br />

236. To develop the sine and cosine of the multiple angle in a series of ascending.powers<br />

of the cosine of the simple angle.<br />

When m is an integer, this problem requires us simply to develop sin mx and<br />

. . P<br />

cos mx in a series of powers of cos x ; but when m is a fraction = —, the Sngle mz<br />

has q values which have the same sine and cosine, (Art. 222), if we consider x to<br />

represent all the angles which have the same sine and cosine as the simple angle.<br />

We shall therefore employ Moivre's Formula in its general form (442), or<br />

(cos X 4- \/ — 1 sin x)"* =: cos m (2 »TT 4- ;;)<br />

Putting k = cos x we have by (458) and (446),<br />

(cosx4-^—Isinx) = (/c4-^A'—l)-"<br />

= {^ — l)r-^E+ {^—l)'"-mK'<br />

( m{in'+l)7r \ / m (in'+ 1) ,r\ _<br />

/(m —l)(4n'4-l)fl--\ ^, , ^ , . /(m —l)(4?i'4-l)fl--\ ^,<br />

-f cos (^^ '-^ !—'—].mK'+.^ — lsin[- ~ -~^ j.mK'<br />

Comparing the real and imaginary terms of these two values of (cos x -{-y/— 1 sin x)*",<br />

we have<br />

^„ . , /m (47i'4-l)fl-\ /(m —1) (4«' + l)fl-\<br />

cos OT (2 nfl--f-x) = cos( -^— '—'-— 1 . K+ cos I ^' '--^ •—'-—1 . m K'<br />

• .o I N - /'"»(4«'4-l)^\ „, - /(m —l)(4«'4-l)fl-\ „,<br />

Binm(2nfl--f-z) = sin( —i—g-!—'—] .K+ain[^


133 PLANE TRIGONOMETRY.<br />

thsrefore these two angles can only differ by some multiple of 2 fl-, or we must<br />

have<br />

m(4«4-l)fl-_^m(4«'4-l)fl- ^^<br />

2 2<br />

whence ni (n — «') = n"<br />

but m being a fraction —, and », n' numbers of the series 0, 1, 2,<br />

cannot have m (n — «') equal to an integer n", unless it is zero ;•* therefore<br />

n — «' = 0, n = n'<br />

and the above developments are<br />

. q — 1, we<br />

/m(4n4-l)fl-\ ,^ , / (m —l)(4n4-l)flr\<br />

cos m (2 «fl- 4- x)=cos (^ '' ^ ' ). S:+ cos (^ ^ ^g —^ )• ^^ (**^*')<br />

/m(4»-|-l)fl-\ „ , . / (m —l)(4n4-l)fl-\<br />

sin m (2 nfl-4- x)=sin (^ ^ ^ )- -Sr4- sin (^ ^ '-\ '—^ ). mK' (461)<br />

in which<br />

m' . , m' ()«' — 2') . .<br />

Z- = 1 — ^^ cos' X 4 \.2,.s.i =08* X — &o.<br />

m'—1' , , (m'— 1') (m'— 3') . ,<br />

K' = COB X g^g— COB' X4- ^ 2^^^ " ~<br />

It hence appears that, in general, it requires the combination of two series to express<br />

the cosine and sine of a multiple angle in powers of the cosine of the simple<br />

angle, when m is fractional.<br />

237. When m is an integer, one of the terms of (460) and (461) will always become<br />

zero, and we shall have but a single series to express the function of the multiple<br />

angle. The first members become in all cases<br />

cos (2 mn fl- -|- mx) = cos mx<br />

sin (2 mn w + mx) = sin mx<br />

and the second members vary according to the form of m. In (460), if<br />

m = 4 m', COB mx = K<br />

m = 4 m' -f- 1, cos mx = mK'<br />

m ^ i m' + 2, cos mx = — K<br />

m = i m' + S, cos mx ^ — mK'<br />

and since when OT is even, the series K terminates, and when m is odd, the series K'<br />

terminates, these four equations are all finite expressions, and will give the equation*<br />

of Art, 76, by making OT = 1, 2, 3, &o.<br />

In (461), if<br />

OT = 4 m',<br />

sin mx = — mK'<br />

OT = 4 m' 4- 1, sin mx = K<br />

m = i m' + 2, sin OTX = mK'<br />

OT = 4 m' -f- 3, sin OTX = — K<br />

- (462)<br />

P<br />

* Since — is supposed to be reduced to its lowest terms, p and q are prime to each<br />

other , therefore, if =--^ is not zero, q must divide n — n'; which is impossible,<br />

7<br />

since the greatest value of either re or re' is g- — 1.


MULTIPLE ANGLES. 139<br />

In these formulae, however, the series do not termitate, but by -differentiatin-g<br />

(162) we find for<br />

, = li ; = — m sin x (cos x •<br />

m' 1'<br />

OT = 4m'-f. 1, sin mx == sinx (1 ^-;j cos' x 4- &o.)<br />

1-2<br />

4 m' 4- 2, sin mx = m sin x (cos x -<br />

4 m' 4" 3, sin mx = — sin x (1 —<br />

2-3<br />

2-3<br />

• cos' X + &o.)<br />

• cos' X + kc.)<br />

-1'<br />

•cos' X + &c.)<br />

1-2<br />

(463)<br />

aU of which terminate and give the equations of Art. 75.<br />

238. To develop the sine and cosine of the multiple angle in a series of ascending powert<br />

•)/ the sine of the simple angle.<br />

We take as before<br />

(cos z + .^y — 1 sin x)"" = cos m (2 re3- 4- a:) 4- v^ — 1 sin rei (2 resr -j- x)<br />

Putting A = sin X, we have, by (459) and (444),<br />

(cos X 4- ^ — 1 sin x)-" = (v^ 1 — A' 4- A v* — 1)"<br />

fn<br />

ffl—1<br />

= (!)•' H+^—l{l)~mH'<br />

= cos m re' fl-. 11+ v^ — 1 sin m re' A- . S<br />

+ y/— Icos (m— 1) re'fl-. mH' — sin (m — l)n'v .mil<br />

Comparing the real and imaginary terms of these equations,<br />

cos OT (2 ra^ 4- x) = cos mrirr . H— sin (m— 1) re'fl- . m H'<br />

sin m (2 refl- -)- x) = sin mn'?r . 11+ cos (OT — 1) re'fl-. mil'<br />

and to find what values of re and re' correspond, let x = 0, then A= sin x = 0, iif = 1,<br />

ff' = 0, and we have<br />

cos 2mmr = cos mn!TT<br />

sin 2 OT refl- ^ sin mn'rr<br />

from which we infer that 2 mnfl- = OTB'A-, or 2 re = re', and hence<br />

cos m (2 ren- 4- 2;) = cos 2 OTre fl- . .ff — sin 2 (m — 1) re A- . mB'<br />

sin m (2n-!r + z) = ain 2 mmr . II-^ coa 2 (m — 1) rear. mS'<br />

in which m being a fraction = —, reis any number of the series 0,1, 2, 3,. .<br />

and<br />

M:<br />

H'<br />

1 — ^-^ sin' z 4-<br />

OT* — 1'<br />

2-3<br />

' (m' — 2') sin* X — &c.<br />

1-2-3-4<br />

sin* X +<br />

m' — 1') (m' — 3')<br />

2-3-4-5<br />

sin* X — &o<br />

(464)<br />

(465)<br />

— 1;<br />

239. When mis an integer, the first members of (464) and (^65) become cos mx<br />

and sin mx; and the ooefacients of the second members jontain only multiples of<br />

In-; therefore we have<br />

cos mx = H<br />

sin mx =: mil'<br />

But the series .&• terminates only when m is even, and the series M' only when m li


140 PLANE TRIGONOMETRY.<br />

odd, and we must also employ the derivatives of these equations to obtain finite expressions<br />

in all cases ; thus we have also<br />

sm mx = -<br />

dll<br />

mdz<br />

cos mx = • dW<br />

dx<br />

Therefore differentiating the series i?and //', we shall have, when<br />

2 m', cos mx : 1-2<br />

OT = 2OT'-(-1, cosmxz= cos x (1-<br />

sin' X 4" &o.<br />

- (466)<br />

. sin' X 4- &c.)<br />

1-2<br />

m = 2 m'.<br />

sin OTX =: OT COB X (sin z<br />

O / 1 1 • , • '<br />

m = 2m + 1, sm mx = m (sm x<br />

2-3<br />

2-3<br />

• sin' z -j- &c.)<br />

sin' X + &c.)<br />

(467)<br />

all of which terminate, and give the equations of Arts. 77 and 78.<br />

240. To develop the sine and cosine of the multiple angle in a aeries of ascending powert<br />

of the tangent of the simple angle.<br />

We have<br />

coa m {2 nrr + x) + .y — 1 sin m (2 rea--f- z) = (cos z -J- \/ — 1 sin x)"<br />

= cos"" x{\+^ — 1 tanx)"<br />

Expanding by the Binomial Theorem, and putting<br />

T= 1-<br />

.^i-^-^) ,^^,^_^'n(m-l)[m-2)im-Z) ^^^, ^ _<br />

1-2<br />

-&a.<br />

g" = OTtanz^'"^"'-^„H"'-^)<br />

1-2-3<br />

tan' X + &o.<br />

we have<br />

cos m (2 re fl-4-z) 4-y/—1 sin m (2 re B--f-z) = cos^z (?-}-^—1 T')<br />

But the imaginary and real quantities are not yet distinctly separated in the second<br />

member, for m being fractional cos" z has a number of imaginary values. If<br />

we designate its real value by cos"" x, all its values are included in the expression<br />

cos"" z(l)'" = cos'"x(cos2 OTre' T -|- v' — 1 sin 2 mn' it)<br />

(vhich, substituted above for cos"" x gives<br />

cosm(2refl-4-z) +.y—lsinm(2refl-4- x) rscos^z (cos 2 mre'fl-. T— sin 2 mre'r. T')<br />

+ v'— 1 coB"'x (sin2mre'fl-. T + cos 2 mn'sr. 7")<br />

Comparing the real and imaginary terms, we now have<br />

cos OT (2refl- 4- x) = cos"" x (cos 2 mre's- T— sin 2 mrt-rr . T')<br />

Bin OT (2 refl- 4- z) = cos"" x (sin 2 mn'v . T + coa 2 mn'-?r . T')<br />

ind it is shown as in the preceding problems that re = re', whence<br />

cos m (2refl- -|- x) = cos" z (cos 2mmr T— sin 2 OTBA- . 7") (468)<br />

sin m (2 refl- 4- z) = cos"" x (sin 2 mrefl- T+ cos 2 mrefl-. 7") (469)


MULTIPLE ANGLES. 1-11<br />

in which m being a fraction = .^, nis any number of the series, >, 1, 2, . . . ^ — I;<br />

andCO3 x denotes only the real value of {/(aos'x)''.<br />

241. By the division of (469) by (468)<br />

, , tan 2mrefl-. 7"4- 7" ,,„„,<br />

tan m (2refl-4-z) = ^5 5 ^ ^ (470)<br />

* ' T —• tan zmuTr.T' ^ '<br />

242. TFACT! m is an integer, both the series T and 7" terminate, and in all cases<br />

cos 2 re.n fl- = 1, sin 2 mre fl- = 0 ; and (408), (469) and (470) give<br />

cos mz = cos z. T (471)<br />

sin mz = cos*" z. 7" (472)<br />

7"<br />

tan mz ^ -— (473)<br />

which last expression embraces all the equations of Art. 79.*<br />

243. Before the memoir of Poinsot, developments were given for the multiple arcs<br />

m series of descending powers of the sine or cosine of the simple arc; but he has<br />

shown that these developments are impossible, except when m is integral, and in this<br />

case the series are the same as the preceding, with the terms written in inverse<br />

order.<br />

244. To develop any power of the cosine of the simple angle in a series of sines or cosines<br />

of the multiple angles, the cosine of the simple angle being positive.<br />

If y = cos z 4- -v/ — 1 sin z, we have, by (434) and the Binomial Theorem,<br />

(2 cos z)-" = (y + y-')"" — y"" + my"^-' + "!:i^ L j,-. _j_ &c.<br />

and by Moivre's Formula,<br />

y" = cos m (2 re fl- 4" z) 4- -v/ — 1 sin m (2 re A- 4- z)<br />

OT2/"^'=:mcos (m — 2) (2ren-4-z) 4- »" v''—1 sin (m — 2){2n-!r + z)<br />

' ^ t ^ y ^ = . ^iapn COS (re^-4) (2n,r+z)+'!L(^^_i si„ (m-4)(2re,r4-x)<br />

Therefore, if we put<br />

we have<br />

&c.<br />

Pj„„_jj. = cos m(2njr + z) + m cos (m — 2) (2 refl- 4- z) -|- &c.<br />

.P'anir^j = sin m (2 re fl- 4- z) 4- "* sin (OT — 2) (2n?r + x) + &c.<br />

&c.<br />

(2 cosz)" = P,„„^4-^ —1P',„,^ (a)<br />

- Now m being a fraction (2 cos z)"" has imaginary values, but ivhen cos x is positive,<br />

it will have at least one real positive value, and then (2 cos z) being understood to<br />

lenote only this real value, all the values are included in the formula<br />

(2 cos x)" X (1)*" = (2 cos x)'" (cos 2 mn'a- 4- \/ — 1 sin 2 mre'a-)<br />

-* Although the formulse for multiple angles require, in general, the combination ol<br />

two series when m is not an integer, yet there are certain cases, even when m is a<br />

fraction, in which one or the other of the series will disappear. iSee the memoir of<br />

I'omfot, cited at the beginning of this chapter.


142" PLANE TRIGONOMETRY.<br />

Therefore we have<br />

(2 cosx) (cos 2 mn' Tr+ ,/ — 1 sin 2 mn' ir) = i'dm + i -\- >/ — ^ 7*ann-+»<br />

Cr.mparinf the real and imaginary terms,<br />

(2 cos x)'" cos 2mn'ir = Pa„„+x<br />

(2 cos z)"' sin 2 mn'ir = P',„ „^i<br />

and to find the corresponding values of re and re', let z = 0, then (2 cos z)'" = 2"",<br />

and tie series become<br />

and in the same way<br />

Therefore our formulje become<br />

P,„, = cos 2 mre flr (1-|- OT 4- " »"" 4" ^°-)<br />

= cos 2 mrefl- (1 + 1)""<br />

= 2"" cos 2 mre ir<br />

P',„,r = 2»" sin 2 mre T<br />

2"" cos 2 mre' A- = 2*" cos 2 mre n-<br />

2" sin 2 OTre'fl- = 2 sin 2 mresand<br />

as in former cases, it is shown that re =: re', so that we have finally<br />

(2 cos x)-" = -^''"^-'-'' (474)<br />

cos 2 mre A-<br />

(2 cos z)-" = - ."^''''--^ - (475)<br />

^ ' sm 2 mre fl- ^ '<br />

From this it appears that the real and positive value of (2 cos z)"'may be expressed<br />

either by a series of cosines or by one of sines of the multiple angles, and by<br />

comparing (474) and (475), we have the following constant relation between these<br />

series.<br />

P',„„+j, sin 2 mre fl-<br />

-Pinir+x coa2mnir<br />

245. If re = 0, (474) gives<br />

(2 cos x)'"=Pj =; cos mx+m COS (m — 2) x -] ^—= — cos (OT — 4) x -j- &c. (476)<br />

which may be employed as the general development of the real value of (2 cos z)"",<br />

when X < -=-.<br />

Ji<br />

246. The same supposition of n = 0, gives sin 2 mre A- = 0, and (475) gives<br />

therefore,<br />

„, , m (m — 1) . ,<br />

0 = P'l = sin OTX 4- m sm (m — 2)x-\ ^-^ '- sin (m- 4) z -f &c. (477)<br />

R remarkable property of this series of sines of multiple arcs, which holds for all<br />

TT<br />

values of m, provided x < -^.<br />

247. To develop any power of the cosine of the simple angle in a aeries of ainea or coainet<br />

df the multiple angles, the cosine of the simple angle being negative.


MULTIPLE ANGLES 143<br />

If the denominator of re is even, there is no real value of (2 cos x)*" when cos x<br />

IS negative ; but we may put<br />

(2 cos x) = ( -2 cosz)"' (— l)m<br />

= (— 2 cos x)-" [cos m (2 re' -f 1) fl- 4- .^ — l"sin m (2 re' -j- 1) r]<br />

which, substituted in equation (a) of Art. 244, gives<br />

(— 2 cos x)" cos m (2 re' -f 1) fl- = P,„,+^<br />

(— 2 cos x)"- sin m (2 re' -f 1) fl- = P',„,+,<br />

Making z = 3-, cos z = — 1, (— 2 cos z)"" = 2"', and the series become, by the<br />

process shown in Art. 244,<br />

• and we have<br />

-Pd n+o , = 2"- cos m (2 re 4- 1) fl-<br />

-P'(o n+i)i = 2"' sin m (2 re 4- 1) fl-<br />

2"' cos m (2 re' 4- 1) fl- = 2-" cos m (2 re 4- 1) A-<br />

2-" sin m (2re' -f- 1)fl- = 2'" sin m{2n+l)ir<br />

whence, as before, n = re', and our formulse are<br />

(— 2 cos z)"- = ^'" -^Y ,. (478) -<br />

^ ' cos m (2 re-f- 1) A- ^ '<br />

{r- 2 cos z)"- = -. ^^"'^j-' . (479)<br />

sin m (2 re -|- 1) fl- ^ '<br />

by which it appears that the real value of (— 2 cos z)" is also expressed either by<br />

a series of cosines or of sines of multiple arcs, which series have the constant relation<br />

248. If re = 0, (478) and (479) give<br />

/"•in^r+x sin m (2n + l)ir<br />

F^yi^^x cos m (2 re 4-1) A-<br />

P 1<br />

(— 2 cos z)"" = — = (cos mx -I- OTcos (m— 2) z -J- &c.) (4801<br />

COSOTfl- cos OTA- ^ / ' / \ I<br />

P' 1<br />

(— 2 cos z) = -;—^ = -; • (sin OTX-4- m sin (m — i.) z 4- &c.) (481)<br />

sm m fl- sm OT A-<br />

In this case sin OTA- is not zero, unless m is an integer, so that the series i", does<br />

ir<br />

not become zero when x > -^, and both (480) and (481) maybe employed as the true<br />

developments of (— 2 cos z)"".<br />

249. When m is an integer, the series (476) and (480) always terminate at the<br />

(m + l)th term.; and, since in (480) cos rejA- = ± 1, according as OTIS even or odd,<br />

and (— 2 cos x)"* = ± (2 cos z)"" in the same cases, both (476) and (480) becoruo<br />

(2 cos z)" = cosmx4- mcos (m — 2) X4 ^—s "os (m —4)z4- &c. (482)<br />

But the series (481) becomes zero, so that (482) is the only series by which<br />

(2cosz)"* can be developed in functions of the multiple arcs, when m is integral.


144 PLANE TRIGONOMETRY.<br />

250. To develop any power of the sine of the simple angle, in a series of sines or cosines<br />

of the multiple angles.<br />

If y = cos z 4- y/ — 1 sin z, we have, by (435) and the Binomial Theorem,<br />

(.^ _ 1) (2 sin z)"- = [y —<br />

y~')'"<br />

, m (m — 1) „_. „<br />

= ym _ my^-^+ —^—^ .V"" — &o.<br />

in which y", y"-^, &c. have the same values as in Art. 244, but the signs of the<br />

coefEcients are alternately 4- and —, so that if we put<br />

9,„„^.^ = cosm(2refl-4-z) —mcos(m —2) (2re A-4-z) 4- &c.<br />

§',„„^.x = sinm (2n fl- 4- z) — OT sin (OT — 2) (2 re A- 4- z) 4- &c.<br />

we have<br />

(V* — l)" (2 s ^)'" = 9»»' + '+V<br />

— '^ Q'-iu^+x<br />

Substituting the value of (v* — 1)"" by (446), and comparing the real and imaginary<br />

terms, we find<br />

-~ . . m (in' + 1) TT ^<br />

(2 sm z)»> COB —^—Y— = V!...


MULTIPLE ANGLES. 145<br />

253. The series (485) and (486) become zero when m is an integer, as follows:<br />

If m = 2m', 0 = sin mx — m sin (m — 2) x -f- &c. (491)<br />

m = 2 m' 4- 1, 0 = cos mz — m cos (m — 2) z -)- &c. (492)<br />

The reason why these series are zero is obvious, since they terminate ai, the<br />

(m 4-l)th term, the terms equally distant from the first and last are equal with<br />

opposite signs, and the middle term of (491)_is zero.<br />

254. Given the equation<br />

tan z = p tan y (493)<br />

to express z ^ly ina series of multiples of y.<br />

Substituting the values of tan x and tan y given by (431)<br />

whence<br />

or rutting<br />

gixv~i — I g-ji/y-. 1<br />

•->+ 1 ^- e'»>'-' +i<br />

{p+l)e-yV-^—{p—l)<br />

p+1 — [p — 1) e'!'>'-'<br />

? = 7+1 (494)<br />

fixy-t -. ^ _ _ ,jy_, ( i_ tl \<br />

(..)<br />

gi (I-!,) y -. = ^ 7 ^<br />

1 —ye'!"'-'<br />

Taking the Naperian logarithms of both members,<br />

2 (2; —y)-^/—1 =log (1 —g-e-'si'-') —log (1_ je^yV-.)<br />

and developing the second member by the formula<br />

we have<br />

log (1 — re) = — re — J re' — J re' — &c.<br />

2 (z —y) v"-1 = — qe-'yy-' — ^ q^ e-'yV-' — ^ q' e-'vy— — &c.<br />

Substituting in the second member by (430),<br />

+ qfeyV-i + 1 g''g'yV-^ _|_ 1. jSgOjy-. + &<br />

(? (^*y) y—• = ?<br />

1 e—'y/—><br />

g<br />

from which, by taking the logarithms and substituting as before,<br />

(J)<br />

3; ,„ sin2y sin4y sin6y<br />

z + y - ^ 2 ^ 3j' '*"• ^"^<br />

In this investigation, we have, in effect, used Moivre's formula, in its limited or<br />

less general form; but the requisite generality may be given to our results, by observing,<br />

that (498) would hold if we were to substitute tan z = tan (re'B--j-z), tany<br />

-T^ tan (n"ir+y), and therefore we may substitute for the first member of (A),<br />

19 N


146 PLANE TRIGONOMETRY.<br />

n'lT +x — (re"fl- 4- y) = 2; — y — («" — re') fl- = z — y — mr, re being (like re and n") at<br />

arbitrary integer or zero. Hence, the required general development of x — y in<br />

series is<br />

X — y = nw+ q sin 2y-f- ^ j' sin 4y-|-J j' sin 6 y-j- &c. (495)<br />

In like manner, since tan x = tan (x — re'fl-), tan y = tan (y — re"fl-),we may substitute<br />

in the first member of (c), x — n'lr + y —ra"fl- = z -f- y — nir, and the general development<br />

otx + yi-a series is<br />

sin2« sin4y sin6y , „ ,,„„,<br />

x + y^nir - J ^ - - ^ - ^ 4 - & C . (496)<br />

In these formulse z and y are supposed to be expressed in arc, and to obtain<br />

z qr y in seconds, the terms of the series must be divided by sin 1".<br />

255. The preceding problem is particularly useful in finding z when p and y are<br />

given, and z is nearly equsjl to y; in which case p is nearly equal to unity, either<br />

y or — is a small fraction, and one of the series (495), (496) converges rapidly.<br />

EXAMPLKS.<br />

1. Given y = 50° and/> = 1-00065, to find z from (493).<br />

Taking only the first term of the series (495), and assuming re ^ 0,<br />

2y = 100° log sin 2y 9-99335<br />

-00065<br />

^ = -2^00065 ^"S^ ^-""^^<br />

ar 00 log sin 1" 5-31443<br />

x — y = 65"-995 log {x — y) 1 -81952<br />

z = 50° 1' 5"-995<br />

2. Given y = 50° and^ = — 1-00065, to find z from (493). In this case<br />

_ 2-00065<br />

^ ~ -00065<br />

and the computation by (496), if we assume re = 0, is<br />

2 y = 100° log sin 2 y 9-99335<br />

1 -00065<br />

q 2-00065 ,,(-i)- ^ \ q<br />

6-51174<br />

arCO log sin 1" 5-31443<br />

z 4- y = — 65"-995 log (z -4- y) — 1-81952<br />

z = —y — 65"-995 = — 50° 1' 5"-995<br />

or, ifre = l,z=]SO° —.50° 1'5".995 = 129° 58'54".005.<br />

In general, (493) is to be solved by (495) when p is positive, and by (496) when/)<br />

is negative.<br />

256. Given the equation<br />

sin (a 4- z) = m sin z (497)<br />


which is reduced to (493) by putting<br />

MULTIPLE ANGLES. 147<br />

1 1 1 m4-l<br />

z = z4-Ja y = } * ^ = ^ ^<br />

whence q = ?—,— = —<br />

^-f-1 m<br />

and (495) becomes<br />

sin* sin2a sin 3a<br />

z=inirA. ^ -=,—, 4- - ^ + kc. (498)<br />

' m ' 2 m' ' 3 m^ ' ' •'<br />

which is to be employed when m > 1; and (496) becomes<br />

z4- « = rea- — m sina — J m'sin 2 a — Jm'sin 3 a—&c. (499)<br />

which is to be employed when m


148 PLANE TRIGONOMETRY.<br />

259 In a plane triangle A B G, given a, b and G, to find A or B by a series of multiplet<br />

efC.<br />

By (260)<br />

a<br />

T ^'^<br />

tan A =<br />

a<br />

1 r- cos C<br />

Which, compared with (503), gives, by (505),<br />

b<br />

a . ^ , a' sin 2 (7 , ffi3 sin 3 C , . ,^„„^<br />

^ = -j-smC'4-y,.—^—4--p.—^-4-&c. (500)<br />

n being necessarily = 0 in this case. Bis found by the same series, interchanging<br />

a and b.<br />

260. In a plane triangle, A B G, given a, b and G, to find cby a series cf multiples of C.<br />

We have<br />

•<br />

c' == a' 4- 6' — 2 ab cos 0 (507)<br />

c' i' 2S •,<br />

— = -^ — cos C 4- 1<br />

a a' a<br />

by(451) = f-^ —(cosC4-^—IsinC)"]<br />

X ri._(oosC'—v' —lsinC)1<br />

f-= [l-.l(cos04-^-lsinC)]x[l-y<br />

(oosC-v/-lsin(7)'|<br />

Taking the common logarithms, employing in the second member the formula<br />

log (1 — re) = — Jf (re 4- J re' 4- J re' 4- &c.)<br />

and applying Moivre's Formula (440) in expressing the powers of cos C ± v^ — 1 sin 0,<br />

we have<br />

2 log c — 2 log b =<br />

— JWr-| (C03C4-V' —IsinC) 4- -|^ (


PAET II.<br />

SPHERICAL TEIGONOMETllY.<br />

CHAPTEE I.<br />

GENERAL FORMULAE.<br />

1. SPHERICAL TRIGONOMETRY treats of the methods of computing<br />

the unknown from the known parts of a spherical triangle.<br />

It is sho-wn in geometry,* that a spherical triangle may, in general,<br />

be constructed when any three of its six parts are given, (not<br />

excepting the case where the three angles are given). We are now<br />

to investigate the methods by which, in the same cases, the unknown<br />

parts may be computed.<br />

We shall at first confine our attention to such triangles only as<br />

are treated of in geometry, namely, those whose sides are each less<br />

than a semicircumference, and whose angles are each less than two<br />

right angles; that is, those in which every part is less than 180°.<br />

2. It is shown in geometry, that if a solid angle is formed at the<br />

center of a sphere by three planes, the three arcs in which these<br />

planes intersect the surface of the sphere form a spherical triangle.<br />

Now the real objects of investigation in spherical trigonometry are<br />

the mutual relations of the angles of inclination of the faces and<br />

edges of a solid angle ; but, for convenience, the spherical triangle<br />

which forms the base of the solid angle is substituted for it. The<br />

sides of the triangle being proportional to the angles of inclination<br />

of the edges of the solid angle, are taken to represent those angles ;<br />

And the angles which those sides form with each other are regarded<br />

* The student is here supposed to be acquainted with Spherical Geometry, at<br />

least so much of it as is to be found in Legendre's treatise, or in that of Prof. Peirce,<br />

of H arvard University.<br />

n2 149


150 SPHERICAL TRIGONOMETRY.<br />

as identical vfith the angles of inclination of the faces of the solid<br />

angle. But, since varying the radius of the sphere would not, in<br />

any respect, change the solid angle, or the values of the angles which<br />

enter into it, the mutual relations in question ought to be deduced<br />

without any reference to the magnitude of the radius of the sphere.<br />

In fact, we shall deduce our fundamental formulse from a direct consideration<br />

of the solid angle itself.<br />

3. In a spherical triangle, the sines of the sides are proportional<br />

to the sines of the opposite angles.<br />

Let AB 0, Fig. 1, be a spherical<br />

Fig. 1.<br />

triangle, 0 the center of the sphere.<br />

The angles of the triangle are the<br />

inclinations of the planes AOB,<br />

AO Q and BOG, to each other, and<br />

> a will be designated by A, B and Q;<br />

their opposite sides respectively will<br />

be designated by a, b and c, as in<br />

plane triangles. The trigonometric<br />

functions of these sides will be the same as those of the angles<br />

BOG, AOG, AOB, which they subtend at the center of the sphere.<br />

(PI. Trig. Art. 20.)<br />

From any point B' in 0 B, let fall B'F perpendicular to the plane<br />

AO G; and through B'P let the planes B'PA', B'PQ' be drawn<br />

perpendicular to 0^ and 0 G, intersecting the plane OAQ in the<br />

lines PAI, PG', and the planes .405, BOQ in the lines A!B', B'O'.<br />

The plane triangles A'P B', B'P G' are right angled at P ; aiid<br />

OA'B', 0 C'B' are right angled at A' and G' The angle B'A'P,<br />

being formed by two lines perpendicular to OA, is the measure of<br />

the inclination of the planes AOB, AO G, or .of the angle A; and<br />

B G'P is the measure of the angle G.<br />

We ha^e therefore, by PI. Trig. Art. 15,<br />

sm A = sin B'A'P ••<br />

sin G= sin B'O'P-<br />

B'P<br />

WA'<br />

B'P<br />

B'G'<br />

whence<br />

sin A<br />

sin G<br />

B'P<br />

B'A'<br />

B'G' _ B'G'<br />

B'P ~ B'A'<br />

(m)


GENERAL FORMULA.<br />

l&l<br />

Again,<br />

B'O'<br />

sin a = sin B'OG' = -r,,^<br />

B (J<br />

sine ^sinB'OA'<br />

B'A'<br />

— B'O<br />

Comparing (m) and [n).<br />

sin a B'G' B'O B'G'<br />

B'O ^ B'A' B'A'<br />

sma sin 4<br />

sin 0 sin G<br />

which in the form of a proportion is<br />

sin a : sin c = sin 4 : sin G<br />

which is the theorem that was to be proved.<br />

4. In Fig. 1, A, a, G and c, are each less than 90°, but the construction<br />

would not vary if any of these parts were greater than 90°,<br />

except that the points A' and G' might be found in the lines AO, GO,<br />

produced through(9; and one or more of the right triangles A'B'P,<br />

kc, would contain the supplements of A, a, G, or a instead of these<br />

quantities themselves. But the sine of an angle and of its supplement<br />

being the same, the preceding demonstration would still be<br />

valid, so that the theorem is applicable to any spherical triangle.<br />

Indeed, according to PI. Trig. Art. 49, this result follows from<br />

the nature of the trigonometric functions themselves, and the demonstration<br />

of the preceding theorem might therefore be considered as<br />

general, without requiring a special examination of the various positions<br />

of the lilies of the diagram.<br />

6. In a 'spherical triangle, the cosine of any side is equal to the<br />

product of the cosines of the other two sides, plus tJie continued product<br />

of the sines of those sides and the cosine of the included angle.<br />

Let the plane B'A'G', Fig. 2, be ^ig- 2.<br />

drawn perp. to OA, intersecting the<br />

planes J.O.B, BO CmdAOG, in the<br />

lines A'B', B'G' and A'G'. Then the<br />

angle B'A'G' = A, and B'OG' = a,<br />

and by PI. Trig. Art. 119, in the triangles<br />

J.'.S'C", OB'Q', we have<br />

B'G" = A'B'^ + A'G" - 2 A'B'.<br />

A'G' cos A<br />

B'G'^ ==0B'^+ 0G'^-20B' 0 Q' cos a<br />

(1.1


152 SPHERICAL TRIGONOMETRY.<br />

Subtracting the first of these equations from the second, and observing<br />

that in the right triangles OA'B', OA'G',<br />

we have<br />

OB'^-A'B"=0A'% OG'^-A' C"^ = 0 A""<br />

0 = 2 OA'^ 4- 2 A'B'. A'G' coa A - 2 0 B' 0 G' cos a<br />

OA'.OA' , A'B'.A'G'<br />

whence cos a = jy-gr-Qgr + Q j^, Qg, cos J.<br />

Substituting the trigonometric functions derived from the right triangles<br />

OA!B', OA'G',<br />

cos a = cos 5 cos e -f sin 5 sin c cos A (2)<br />

which is the theorem to be proved. It may be regarded as the fundamental<br />

theorem, for the preceding (1) can be deduced from it, but<br />

as the process is somewhat circuitous, we have preferred deducing<br />

the two theorems from independent constructions.<br />

6. In the construction of Fig. 2, both b and o are supposed less<br />

than 90°, while no restriction is placed upon A and a; but the equation<br />

(2) is no less applicable to all the other cases if the principle of<br />

PI. Trig. Art. 49 be granted. As that principle may not be suifi-<br />

Fig. 3. ciently evident to the student unacquainted with analytical<br />

geometry, we shall verify it in this case, as follows.*<br />

1st. In the triangle ABG, (Fig. 3), let b < 90° and<br />

c > 90°. Produce BA, BG to meet in B', forming the<br />

lune BB'; then A B'= 180° — c, a.ni b are both < 90°,<br />

and the preceding demonstration would apply to the<br />

triangle A B'G. Therefore, applying (2) to A B'G, we<br />

have<br />

cos(180°-a) = cos5co8(180°-c)4-sin5sin(180°—c)cos(180°-A)<br />

or by PI. Trig. (64),<br />

— cos a = — cos b cos c — sin b sin c cos A<br />

and changing all the signs<br />

' cos a = cos 5 cos c -f sin b sin c cos A<br />

the same result that would have been found by applying (2) directly<br />

to ABG.<br />

* Hymer's Spherical Trigonometry. Cambridge, 1841.


GENERAL FORMULA. 153<br />

2d. In the triangle ABO, Fig. 4, let b > 90°, c > 90° ;<br />

produce AB and AG to meet in A'; then A'B andA'Q<br />

being both less than 90°, the formula (2) is applicable to<br />

A'BG. Therefore<br />

cos a = cos (180° - b) cos (180° - c)<br />

4- sin (180° - b) sin (180° - c) cos A<br />

= (— cos b) (— cos c) 4- sin 6 sin c cos A<br />

= cos b cos c 4- sin 5 sin c cos A<br />

the same result as before.<br />

7. The theorems expressed by (1) and (2) being applied successively<br />

to the several parts of the triangle, give the two following<br />

groups:<br />

sin a ainB = sin 5 sin A<br />

sin b ain G = sin c sin 5 I (3)<br />

sin c sin 4 = sin a sin G<br />

cos a = cos b cos e 4- sin 5 sin c cos A<br />

cos b = cos c cos a 4- sin e sin a cos B^ I (4)<br />

cos c = cos a cos 5 -f sin a sin b cos G<br />

8. Let A'B'G', Fig. 5, be the polar triangle<br />

of ABG, and designate its angles and sides by<br />

A', B', G', a', b' and ci Then, by geometry,<br />

A' = 180° -a, a' = 180° - A<br />

B' = 180° - b, b' = 180° - B<br />

O' = 180° -c,<br />

c' = 180° - C<br />

and applying the first equation of (4) to A'B'G',<br />

or by PL Trig. (64),<br />

cos a' = cos b' cos c' -f sin 5' sin c' cos Jl<br />

— cos J. = (— cos B) (— cos C) 4- sin B sin C (— cos a)<br />

— cosA= cos B cos (7 — sin 5 sin G cos a<br />

Changing the signs of this, we have the first of the following group:<br />

cos A — — cos B cos 0 -{- sin B sin 0 cos a<br />

cos B = — cos C cos JL 4- sin (7 sin JL cos b > (5)<br />

cos (7 = — cos .4 cos .S 4- sin A sin .5 cos c<br />

20<br />

J<br />

')


154 SPHERICAL TRIGONOMETRY.<br />

It is thus that, by means of the polar triangle, any formula of a<br />

spherical triangle may be immediately transformed into another, in<br />

which angles take the place of sides, and sides of angles.<br />

9. Several other important fundamental groups of formulse are<br />

obtained from the preceding with the greatest ease.<br />

The first of (4) multiplied by cos c is<br />

cosa cose = cos b cos^ c -f sin 5 sine cos c cos A<br />

and the second of (4) is the same as<br />

the difierence of which is<br />

cos a cos G 4- sin a sin o cos B — cos b<br />

sin a sin e cos .B = (1 — cos^ o) cosb — sin 5 sin e cos c cos A<br />

Since 1 — cos^ c = sin^ c, this may be divided by sin o, and gives<br />

sin a cosB = sin e cos 5 — cos c sin 5 cos .4 "j<br />

whence sin b cos G = sina cos c — cos a sin c cos B V (6)<br />

sin G cos 4 = sin 5 cos a — cos b sin a cos G<br />

If we interchange B and G, and therefore also b and c, the group<br />

becomes<br />

sin a cos G = sinb cos c — cos b sin c cos A<br />

sin b cos A = sin c cos a — cos c sin a cos B<br />

sin c cQsB = sin a cos 5 — cos a sin b cos Q<br />

10. If (6) and (7) are applied to the polar triangle, they give,<br />

after changing the signs of all the terms,<br />

and<br />

sin Acoab — sin G cos B 4- cos Csin.B cos a<br />

sin B cos G =ainA cos (74- cos J. sin G cos b \ (8)<br />

sin (7 cos a = sin 5 cos J. 4-cos-B sin J. cos e<br />

sin.4.cosc = sinB cos C-f cosjBsin 0 cosa )<br />

sin Bcosa== sin (7 cos J. 4- cos Gain A cos 5 V (9)<br />

sin (7 cos 5 = sin J. cos ji5 4-cos J. dn.B cose<br />

11. Dividing the first of (6) by the following derived from (3),<br />

sin a sin 5 . ,<br />

-.—J— = sm 0<br />

smA<br />

^<br />

J<br />

J<br />

J<br />

(7)


we find the first of the following group<br />

GENERAL FORMULA. 155<br />

sin AcotB = sin c cot 6 — cos c cos J.<br />

sin-B cot (7 = sin a cot e — cosacosi? V (10)<br />

sin G cot J. = sin 5 cot a — cos b cos 0<br />

and in the same way from (7), or by interchanging the letters B and<br />

0, b and e in (10), we find<br />

sin J. cot C = sin 5 cot c — cos b cos A<br />

sin B cot 4 = sin e cot a — cos c cos B<br />

sin G cot B = sin a cot b — cos a cos G<br />

(11)<br />

If (10) are applied to the polar triangle, we find (11), so that no<br />

new relations are elicited.<br />

12. The preceding formulse are sufiicient to furnish a theoretical<br />

solution for every case of spherical triangles, but some transformations<br />

are required to facilitate their application in practice.<br />

In the first of (4) substitute, by PI. Trig. (139),<br />

we find, by PI. Trig. (39),<br />

cos J. = 1 — 2 sin^ 1A<br />

cos a = cos [b — e)—2 sin b sin c sin^ J A (12)<br />

and we have similar expressions for cos b and cos c.<br />

If we substitute in (4), by PI. Trig. (138),<br />

we find, by PI. Trig. (38),<br />

cosJ. = -l<br />

4-2cos2iJ.<br />

cos a — cos (6 4- e) 4- 2 sin b sin e cos^ J A (13)<br />

and, of course, similar expressions for cos & and cos e.<br />

13. Substituting in (5)<br />

cos a = 1 — 2 sin^ J a = — 1 4- 2 cos^ J a<br />

we find by the same process<br />

cos J. = — cos(.B -h G) — 2ainB sin Gsin^ J a (14)<br />

cos J. = — cos {B — G) + 2 sin B sin G cos^ J a (15)<br />

which might have been obtained by applying (12) and (13) to the<br />

polar triangle.


156 SPHERICAL TRIGONOMETRY.<br />

14. If ii) (12) we substitute cosa = 1 — 2 sin'J a, cos (b — o) = 1 —-2 sin'^(6 — c),<br />

we obtain the first of the following equations; and the others are obtained by a<br />

similar process from (12), (13), (14) and (15).<br />

sin' ^ a = sin' J (6 — c) -)- sin i sin c sin' ^ A (16)<br />

sin' i a = sin' i (b + c] — sin i sin c cos' J A (17)<br />

cos'i a = cos' J (6 — c) — sin 6 sin c sin' ^ A (18)<br />

cos'^ a = cos'J (J -j- c) -(- sin 6 sin 0 cos'J A (19)<br />

sin' J ^ = cos' J (.B 4- C) 4- sin B sin G sin' J a (20)<br />

sin' J ^ = cos' i (B—G) — sinB sin C cos' J a (21)<br />

cos' } ^ = sin' J (5 -f C) — sin .S sin C sin' J a (22)<br />

oos'^^ = sin'J (i?— G)+ sin .S sin Ccos'J a (23)<br />

15. By PI. Trig, we have<br />

1 = cos' iA + sin'iA<br />

cos A = cos' ^ A — sin' J A<br />

whence<br />

COB 6 COB c = COS b cos c cos' J ^ 4- (ios 5 cos o sin' J jl<br />

sin J sin c cos ^ ^ sin 6 sin c cos' ^ A — sin 5 sin c sin' J ^<br />

the sum of which is, by (4),<br />

cos a = cos (6 — c) cos' J .4 -f- cos (J -f- c) sin' J ^ (24)<br />

and substituting 1 — 2 sin' J a, kc, for cos a, &c.<br />

sin' J a = sin' J (i — c) cos' iA + sin' J (i -f c) sin' j ^ (25)<br />

cos' I a = cos' J (6 — c) cos' J ^ 4- cos' J (i -f- c) sin' J ^ (26)<br />

In the same manner we deduce from (5)<br />

cos J4 = — cos {B — G) sin' J a — cos (JS -f C) cos' J a (27)<br />

sin'J.4= cos'J(5—C)sin»Ja4-cos'J(.B-f- C)cos'Ja (28)<br />

COB'J^= Bin'J(J3—C)sin'Ja4-sin'i(.B4-C)cos'Ja (29)<br />

It is hardly necessary to add that each of the equations (12 to 29) gives a group<br />

of three, by applying it successively to the three sides or three angles of the triangle.<br />

16. From (12) we find<br />

. ,, . cos (6 — c) — cos a<br />

am^A = ^ . '<br />

2 sm 6 sm e<br />

If, in PI. Trig. (108), we put<br />

X =a, y = b — 0<br />

whence<br />

i{^ + y) = i{^+b-c), ^{x-y) = ^{a-b-i-c).<br />

we find<br />

cos (5 — c) — COS a = 2 sin J (a — 5 -f e) sin J (a 4- S — c)<br />

which, substituted in the above equation, gives<br />

si„a X A = -i^H^-i + o)sir.i{a-hb~^<br />

sin 0 sin e<br />

^


then<br />

GENERAL FORMULAE.<br />

Let s denote the half sum of the sides, that is, let<br />

a4-54-c = 2s,<br />

J(a4-54-c) = s<br />

a—b-\-c = a-hb-}-c-2b = 2s-2b=2{s-b)<br />

a4-5 — (j = a4-J4-e — 2c=2s — 2c=2(s — e)<br />

which substituted in (30) give<br />

whence also<br />

. sin (s — b) sin (s — c)<br />

sm- •^ sm 0 sm c<br />

sin^ J B<br />

17. From (13) we find<br />

sin (s — e) sin (s — a)<br />

sm e sm a<br />

21 n — sin (s — a) sin (s — b)<br />

. •9- 0 — ; : J<br />

sm- sm a sm b<br />

157<br />

> f^l)<br />

cos^J J- =<br />

cos a — cos (b 4- c)<br />

2 sin b sin c<br />

and from PI. Trig. (108), by making<br />

a;=54-e, y ^ d<br />

J (a; 4- 2/) = i (a 4- 5 4- c), J (a; — «/) = J (S 4- c — a)<br />

we find<br />

cos a — cos (5 -f c) = 2 sin J (a -f 5 -f e) sin J (6 4- c — a)<br />

which, substituted above, gives<br />

, , , 2 ^ ^ = '^^^''^^-^^^^^^^'-'''^ (32)<br />

^ sm 0 sm e ^ ''<br />

Introducing, as in the preceding article, s = J (a -f J 4- c).<br />

cos^ iA =<br />

003=" iB =<br />

cos^ i (7 =<br />

.{s-a)<br />

sin 6 sin c<br />

sin s sin (a — b)<br />

sin e sin a<br />

sin s sin (s — c)<br />

sin a sin J<br />

(33)


158 SPHERICAL TRIGONOMETRY.<br />

18. The quotient of (31) divided by (33) gives<br />

sin (s — b) sin [s — c)<br />

tan^ IA =<br />

sin s sin (s — a)<br />

tanH5 =<br />

tan= J G ••<br />

sin (s — c) sin (s — a)<br />

sin 8 sin (s — 5)<br />

sin (s — a) sin (s — 6)<br />

sin s sin (s — e)<br />

V (34)<br />

19. From (14) we find<br />

sin^ J a<br />

cos A 4- cos (.g 4- G)<br />

2 sin B sin G<br />

from which, by PI. Trig. (107), we deduce<br />

and if we put<br />

^ - cos i{A-i-B+ G) cos i (.5 4- G- A)<br />

sin" * a<br />

sin^BsinC<br />

2xh—<br />

sm-<br />

i{A-i-B-i-G)<br />

= S<br />

(85)<br />

sm-<br />

j^ _ — cos«ycos {S — A)<br />

^ sin B sin G<br />

sm-'f e •<br />

~COSAS'COS(*S'—5)<br />

sm G'sm.A<br />

— cos S cos {S— G)<br />

sin A sin B<br />

(36)<br />

The first member of each of these equations being a square, the<br />

second member must be essentially positive, although its algebraic<br />

sign is negative ; in fact, since by geometry 2*S^> 180°, >S^> 90°,<br />

cos S is negative, and — cos S is positive.<br />

20. From (15) we find<br />

cos^ J a<br />

cos A 4- cos {B — 0)<br />

2 sin .B sin G<br />

from which we deduce, by a process similar to the preceding,<br />

cos^ J a<br />

cos I (J.-^4-(7) cos J<br />

sin B sin G<br />

(A-^B-G)<br />

(37)


GENERAL FORMULA. 159<br />

.._oo^{S-B)coa{S-G)<br />

^<br />

sm B sm G<br />

^_cos(,S'- G)cos{S-A)<br />

cos 2 ^<br />

1 sin G sin A<br />

_ cos {S-A) cos (S-B)<br />

,2 1<br />

cos ^ " sin J. sin B<br />

21. Frcm (36) and (38)<br />

(38)<br />

21 _ — COS S COS (S — A)<br />

^^'^ ^ "^ ~ cos (^-5) cos (,S-(7)<br />

, ,, , — cos S cos (iS' — B)<br />

tan-" * 0 = — . „—vv,—5^^-:=—'—r.<br />

" cos (^S* — G) cos (^ - A)<br />

(39)<br />

2 ^ _ — COS *? cos (iS* — C)<br />

^^"^ ^ "^ ~ cos(^-^)cos(;S'-.B)<br />

We might have deduced (36), (38), (39), by applying (31), (33),<br />

(34) to the polar triangle.<br />

22. Napier's Analogies. Dividing the 1st of (34) by the 2d, we<br />

find<br />

tan J A sin (s — I)<br />

tan J B sin (s — a)<br />

Regarding this as a proportion, we have, by composition and division.<br />

tan ^ A -\- tan ^ B _ sin (s — 5) 4- sin (s — a)<br />

tan ^ A — tan J B sin (s — b) — sin (s — a)<br />

[m)<br />

In PI. Trig. (109), if we put x=s<br />

we have<br />

x-hy<br />

and by PI. Trig. (126),<br />

— 2s — a — b = c<br />

X —y =' a — b<br />

— b, y = s — a, whence<br />

sin (s — 5) -f sin {s — a) _ tan J e<br />

sin (s — 5) — sin (s-a) ~ tan J (a — 6)<br />

tan 1 A 4- tan|^ _ sin |( J. + ^)<br />

tan 1^ — tin | B ~ sin J (^ — B)


160 SPHERICAL TRIGONOMETRY.<br />

Therefore (m) becomes<br />

sin |(J. 4- -B) _, tan|c<br />

sin ^{A — B) ~ tan J {^b) \- (40)<br />

or sin J (J. -f i?): sin J (J. — i?) = tan J e : tan i (a — J)<br />

which is the first of Napier's Analogies.<br />

23. Again, the product of the 1st and 2d of (34) gives<br />

, . , T, sin (s — e)<br />

tan iAt&nlB = ^^ -^<br />

sm s<br />

or 1: tan J A tan ^ B — sin s : sin (s — c)<br />

whence, by composition and division,<br />

1—tan J J. tan J .B sin s — sin (s — e)<br />

1 -f tan J J. tan J 5 ~~ sin s 4- sin (s — c)<br />

{n)<br />

By PI. Trig. (109), if a; = s, «/ = s — e, we have<br />

and by PI. Trig. (127),<br />

Therefore (n) becomes<br />

sin s — sin (s — c) tan ^ c<br />

sin s 4- sin (s — e) tan J (a -f J)<br />

1 —tanJJ.tan|5 _ cos J (J. 4--8)<br />

1-F tanJJ.tanJjB ~ cos J (J. — i?)<br />

cos ^ (^ 4- ^) _ tan I e ]<br />

cosi(.4—^) ~ tan i (a 4- b) (41)<br />

or cosi(J.4--B):cosJ(J. —^)=tanie:tanj(a 4-J) j<br />

which is the second of Napier's Analogies.<br />

24. If (40) and (41) are applied to the polar triangle, we sha'l find<br />

or<br />

or<br />

sin 1 (a 4- S) _ cot | C<br />

sin I {a- b) ~ tan J (J.-.B) } (42)<br />

sin J (a 4-5): sin J {a —b) = cot J (7: tan J (J.—.B)<br />

cos I (a 4- 5) cot J G<br />

cosi{a-b) ^ Unl{A-\-B) } (43)<br />

cos J (a -f 6): cos J (a — 5) = cot J G: tan J (J. -f JB)<br />

which are the third and fourth of Napier's Analogies.


25. Gauss's Theorem. If<br />

GENERAL FORMULA.<br />

IGl<br />

p = coaicsini(A + B) P = cos J CcosJ (a — b)<br />

J = cos J c cos J (.4 + B) § = sin J C cos J (a 4- b)<br />

r = ain ic ain i (A — B) R = coa ^ G sin J (a — S)<br />

« = sin J c cos J (.4 — £) S = ain i G sin ^ (a + b)<br />

then the products P X g, P X r, p X ^, g X r, q X s, r X s, are respectively equal to the<br />

vroducta P X Q, J'X Ji, J'X S, Q X M, Q X S, R x S.<br />

First. From (3) we have<br />

sin c (sin A ± sin B) = sin C (sin a =fc sin b)<br />

which, by PI. Trig. (105), (106) and (135), are reduced to<br />

Bin J c cos J c sin J (^ -f B) cos J (^ — .B) = sin J C cos J C sin J (a 4- 6) cos J (a — i)<br />

sin J c cos J c cos J (.4 -f .S) sin J (^ —.B) = sin J C cos J C cos J (a 4- i) sin J (a — A^<br />

or<br />

Second. From (6) and (7)<br />

pa = PS and qr = §5<br />

sin c (cos 5 rt cos A) = (Izp. cos C) sia (a dt J)<br />

which, by PI. Trig, are reduced to<br />

sin J c cos J c COB ^(A + B) cos ^(A —.B) = sin J C sin J C sin J (a-}- b) cos § (a |. i)<br />

sin J c cos J c sin J (4 4- .8) sin J (^ — -S) = cos J C cos J (7 sin J (a — i) cos § (a — 4)<br />

or<br />

Third. From (8) and (9)<br />

ys = QS and pr = PR<br />

(1 ± cos c) sin (.4 ± 5) = sin G (cos 5 =t: cos a)<br />

which, by PI. Trig, are reduced to<br />

cos J ccos J c sin J (.44-.B) cos J (.4-1- B) = sin J Ccos J G cos ^ (a + b) cos J (a — i)<br />

sin J c sin J c sin J (A — B) cos i (A—B) =:sin^ Ccos J (7 sin J (a 4- S) sin J (a — A)<br />

or<br />

pq = P§ and rs = RS<br />

26. The notation of the preceding article being still employed, the quantitiea p'', q', r*, *•,<br />

are respectively equal to i", §', i?", (S"-<br />

We have<br />

and<br />

the quotient of which ia<br />

and in the same way<br />

pg X pr = PQ X PR<br />

j7-=<br />

s' = 6"<br />

21 o2<br />

QR<br />

pt _ pa -pfhence ^ = ± P<br />

j' = §' ? = ± «<br />

r'= R' r = ±R<br />

« = ± S


162 SPHERICAL TRIGONOMETRY.<br />

27. In these last equations, the positive sign must be used in all the second members, or<br />

the negative sign in all of them. For if we take<br />

^ = 4-P<br />

the equations<br />

pq = PQ, pr = PR, pa = PS<br />

being divided by this, give<br />

g = +Q, T = + R, s = + S<br />

and if we talce<br />

i> = - P<br />

the same equations, divided by this, give<br />

g=—Q, r = —R, a = — S<br />

We have therefore the following, which are generally cited as Gauss's Equations.<br />

cos ieaini (A + B) = cos J C cos J (a — b)<br />

cos ^ecosi (A + B) = sin J C cos J (a + b)<br />

sin ^c ain i (A — B) = cos i G aini (a — b)<br />

or<br />

sin i c coa i (A — B) = sin i C sin i [a + b)<br />

- "<br />

cos icsin^ (A + B) = —cosi G cos i (a — b)<br />

cos iccos^ (A + B) = — sin J C cos ^ (a + b)<br />

Bin i c sin i (A — B) = — cos J C sin J (a — b)<br />

sin J c cos J (4 — B) = — sin J (7 sin i {a+ b)<br />

If, however, we consider only those triangles whose parts are all less than 180°, tho<br />

first of these groups, (44), is alone applicable, for we must then have p = +P; since<br />

cos J c, sin i (A + B), cos ^ G, cos J (a — J) are then all positive quantities. The use<br />

of (45) will be seen in the chapter on the solution of the general spherical triangle.<br />

Napier's Analogies, (40), (41), (42) and (43) can be deduced directly from (44).<br />

ADMTIONAL FonwvLJE.<br />

28. We shall here add some formulas which, though not so frequently used as the<br />

preceding, are either remarkable for their elegance and symmetry, or of importance<br />

in certain inquiries of astronomy and geodesy.<br />

29. The product of (30) and (32) gives<br />

4 sin s sin (« — a) sin (s —<br />

Bin'.4 —<br />

*) sin<br />

sin' b sin' c<br />

Put<br />

n' = sin s sin (s — a) sin (s — *) sin<br />

then<br />

. 2re<br />

sin b sin c<br />

and in the same manner<br />

the quotient of which is<br />

- D 2re<br />

sin a sin c<br />

din A<br />

ein B<br />

sin a<br />

sin b<br />


30 We have also from (35) and (37)<br />

and if<br />

ADDITIONAL FORMULA. 16-3<br />

sin' a = — ^ g°s g cos (S — A) cos (S — B) cos (S — 0)<br />

sin' B sin' C<br />

From (48) and (51),<br />

(49)<br />

iV' = _ cos 5 cos (S — A) cos (5" — B) cos (-S—C) (50)<br />

2Ar<br />

sin B sin C<br />

re sin a sin 5<br />

N sin ^ sin B sin C<br />

81. If we develop (47) and (50) by PI. Trig. (173) and (174)<br />

(51)<br />

(52)<br />

4 re' =1 — cos' a — cos' b — cos' '^ 4- 2 cos a cos 5 cos c (53)<br />

4iV = 1 — cos'^ — cos'5 —cos'C—2cos^ cos J? cos G (54)<br />

32. The following simple results are easily deduced from the equations (31 to 38;<br />

cos J A cos J B<br />

sin J G<br />

sin s<br />

sin c<br />

cos J ^ sin J If sin (s — a)<br />

cos J G<br />

sin e<br />

sin J A cos J P sin (s — 6)<br />

cos J C<br />

sin c<br />

sin J J4 sin J P sin (s — c)<br />

sin J C<br />

sin e<br />

sin J a sin J 5 — cos 5"<br />

cos J c sin C<br />

sin J a cos J S cos (S — A)<br />

sin J c<br />

sin G<br />

cos J a sin J 5 cos (S — B)<br />

sin J c<br />

sin G<br />

cos J a cos J i cos (,S— G)<br />

cos J c<br />

sin G<br />

• (55)<br />

83. By means of (55) and (56) we can deduce expressions for the functions of<br />

t, a — a, &c., in terms of the angles, or of S, S — A, kc, in terms of the sides<br />

We have, from (51),<br />

. 2N N_^<br />

sin ^ sin P 2 sin J ^ cos ^A sin J P cos J P<br />

which, substituted in (55), gives<br />

N ,.-,<br />

2Bin J.4smJ5siii J(7<br />

^<br />

sin (. - 0) = ^.^^--^-^--J-^__^-^ (581<br />

whence, by interchanging the letters, we have also sin (s — a) and sin (s — b).


16-i SPHERICAL TRIGONOMETRY.<br />

Again, we have<br />

whence<br />

sin (s — c) = sin s cos c — COB « sin e<br />

sin s cos c — sin (a — c)<br />

COB a = :<br />

Bin c<br />

which, by (55), is reduced to<br />

and from the equation<br />

cos i J1 COS i P COS c — sin J .4 sin J P<br />

cos a =; ? ? -.—=--=<br />

sm J 0<br />

•we find, by substituting (55) and (59),<br />

COS (a — c) = cos s COB c + sin a sin e<br />

, , — sin i .4 sin i P cos c 4- cos i A cos J P .^^<br />

cos (s — c) = ^ i-T—r-?;—' —- I w)<br />

^ ' sm J C '<br />

To eliminate c from the second members of (59) and (60), we have, by (5),<br />

cos G + cos A cos P<br />

sin A sin P<br />

(69)<br />

whence<br />

, . , _ cos C + cos A cos P<br />

cos * .4 cos * P cos c = —-.—;—^—,—-.—-, _ •<br />

-* •* 4 sm J ^ sin J P<br />

. , , . , _ cos C? -f- cos .4 cos P<br />

sm * ^ sin * P cos c = —^ -^j-^ , „<br />

•* -* 4 cos J .4 cos J P<br />

which, substituted in (59) and (60), give<br />

cos A + cos B + cos G — 1 1 — sin' ^ A — sin' J P — sin' J ^ rn-i\<br />

°°^ ' ~ 4 sin ^ ^ sin J P sin J (T" ~ ' 2 sin J ^ sin } P sin J G *• '<br />

, cos ^ 4- cos P — COS G + 1 cos' ^ .4 -j- cos' ^ P — cos' ^ G ,„„.<br />

cos (a — c) _ 4 cos J ^ cos J P sin J (7 ~ 2 cos J ^ cos } P sin J (7 *• ^<br />

From the preceding we easily deduce<br />

, 2 N sin e ,„g,<br />

COB ^-|- cos P 4" cos C — 1 cos c — tan J ^ tan J P ^<br />

. 2 JV" sin c ..,.<br />

tan (s — c) -- ^^^ A+OOSB — COBG+1~ cot^AcosiB- cos e '' '<br />

84. The equations (57 to 64) applied to the polar triangle, giv3,<br />

2 cos i a cos i b cos } c ' -'<br />

(oos^-C7) = 2iS^^lI^jr53Fj7 (66)<br />

. „ sin J a sin J i cos G + cos J a cos J 5<br />

•'"^=—^^ cosiT ^^ ^ (67)


ADDITIONAL rORMUL.ai. 165<br />

sin /S = 1 -4- cos a -f cos a -f cos c _ cos' ^ a -f- cos' ib + cos' j e — 1 ,„„,<br />

4 cos J a cos J 6 cos J c 2 cos J o cos J 6 cos J c ^ -'<br />

sin (S—G) = "QS ^ a cos ^ & cos C+ sin ^ a sin ^ & .<br />

cos J c<br />

* '<br />

. ,^ fn 1 — cos a — cos i -t- cos c sin' ^ a -f- sin' J 6 — sin' J c .<br />

4 sin J a sin J 6 cos J c 2 sin ^ a sin J 6 cos J c *• '<br />

_ , ^ 2re sin C<br />

1 4- cos a -j- cos i 4- '"'s c cos £7 -{- cot J a cot J 6 ^ '<br />

cot (S-G)= -, ^" , , = „ , f°f , , . (72)<br />

"• ' 1 — cos a — cos 0 -j- cos c cos G + tan j a tan Jo ^ '<br />

85. From (73) we find<br />

, . „ 2 cos i a cos i 6 cos J c — cos' A a — cos' i i — cos' J c 4- 1 ,nn^<br />

1 — sm 5" = ^ ? ^ , VT -,—? 2_a_ (78)<br />

2 cos J a cos J 0 cos J c ^ '<br />

, , . „ 2 cos i a cos J 6 cos i c-4-cos'i a 4-cos'i J 4-cos'ic — 1 ,_,,<br />

1 -J- sm 5 = ^ ^ =^—^ VTT-—r-^—-^ (74)<br />

' 2 cos J a cos J 0 cos J c ^ ^<br />

the numerators of which may be reduced by PI. Trig. (173) and (174), by making<br />

z = Ja, y = J6, z=Jc, whence u = i(a+b<br />

&c.: therefore,<br />

+ c) = is, v — i = J (s — a),<br />

1 — Bin S = 2 sin J 8 sin J (s-a) sin ^ (s — b) ain j (s — c)<br />

cos J a cos J 6 cos J c<br />

1-1- M .5 — ^ °°^ iscoai(s — a) cos } (a — i) cos J (a— c) •<br />

""" cos J a cos J i cos J c ^ ^<br />

The product of these equations reproduces (65); their quotient is, by PI. Trig. (154),<br />

tan' (45° — J S) = tan J s tan J (s — a) tan J (s — b) tan J (» — c) (77)<br />

86. Gagnoli'a Equation.—Multiplying the first equation of (4) by cos A, we fii d<br />

*• ^<br />

COB a cos A =<br />

COB 6 cos c cos ^4 -J- sin b sin c — sin b sin c sin' A<br />

and from (5) in a similar manner,<br />

cos a cos J. ^ — cos P cos C cos a -J- sin P sin 0 — sin P sin C sin' .i<br />

Observing that by (3) we have sin 5 sin c sin' ^ = sin P sin C sin' a,<br />

these two equations give,<br />

sin i sin c 4- cos b cos c cos 4 = sin P sin G — cos P cos G cos a (78)<br />

a relation between the six parts of the triangle, first given by CAGNOLI. It is a<br />

property of this equation that either member is a function which has the same value in a<br />

given spherical triangle and its polar triangle. Thus, if we distinguish the sides and<br />

angles of the polar triangle by accents,.we have*<br />

sin b sin c + cos b cos c cos A = sin b^ sin c' + cos i' cos c' cos A' (79)<br />

* See Matbematioal Monthly, (Cambridge, Mass.,) vol. i. p. 282.


166 SPHERICAL TRIGONOMETRY.<br />

S7. To dettvJ". the formulae of plane triangles from those of spherical triangles.<br />

The analogy of many of the preceding formulas with those of plane triangles is<br />

Bufficieutly obvious. We can, in fact, deduce the plane formulfe from those of this<br />

chapter, by regarding the plane triangle as described upon a sphere whose radius is infinite,<br />

the triangle being an infinitely small portion of the sp/iere. The quantities a, b and<br />

c, must, in this case, express the absolute lengths of the sides ; and the angles which<br />

a b c<br />

they subtend at the center of the sphere, expressed in arc, will be —, —, —, r being<br />

the radius of the sphere. When r is very large, —, —, —, are very small; md we<br />

r r r<br />

may express the values of sin —, cos —, &c. approximately, by one or two terms of<br />

r r<br />

their expansions in series, PI. Trig. (405) and (406), and if their values be substituted<br />

in our spherical formulae, we shall obtain approximate relations between the<br />

sides and angles of the triangle. If we then make r infinite we shall obtain exact<br />

relations between the sides and angles of a plane triangle.<br />

Thus we have<br />

. a a a' . „ a' , „<br />

Bin^ ^"'7 ---211^ +^°- "-TITf+^osin<br />

P b b b'<br />

sin — 2.3 r, .. 7-' ., 4- ^ &c. b — 2.3 r'<br />

and making r infinite, we find the formula of PI. Trig.<br />

In the same manner<br />

sin^<br />

sin P<br />

a b e<br />

COB cos — cos —<br />

, r r r<br />

cos A = . i . c / b b<br />

sin — sm —<br />

a<br />

b<br />

^l-~ + kc.-{l-^-^ + ~-kc.)<br />

{~-2^+-'^-\~~-^+'^-)<br />

'<br />

Ve<br />

ibc-^-kc.<br />

2r'<br />

and making r infinite, we have the formula of PI. Trig.<br />

S« 4- c' — a'<br />

cos A =<br />

2 be<br />

Formulse that involve only the sines or tangents of the sides may be reduced im.<br />

mediately to the plane formulae by substituting a, b, kc, for sin a, tan a &o. Thus.<br />

(31 to 34) give the corresponding formulae of PI. Trig, by omitting the symbol sin.<br />

and (40), (41), by omitting the symbol tan. when these symbols are prefixed to sides


SOLUTION OF SPHERICAL RIGHT TRIANGLES. 167<br />

CHAPTER II.<br />

SOLUTION OF SPHERICAL RIGHT TRIANGLES.<br />

38. WHEN one of the angles of a spherical triangle is a right<br />

angle, the general formulae of the preceding chapter assume forms<br />

that are remarkably analogous to the relations established for tho<br />

solution of plane right triangles, and equally simple in their application.<br />

39. Let (7=90°, Fig. 6. From (3) we<br />

have<br />

. . sina .<br />

sin J1 = -. sm G<br />

sine<br />

but since Q= 90°, sin (7= 1; therefore,<br />

sin a<br />

and, in the same manner, sm A = —.—•<br />

(801<br />

. sm sin e 5<br />

smB = sine J<br />

that is, the sine of either oblique angle of a spherical right triangle<br />

is equal to the quotient of the sine of the opposite side divided by the<br />

sine of the hypotenuse. Compare PI. Trig. (1).<br />

40. From (11), we find<br />

sin5 cote—sinJLcotC<br />

cos A — TTTT"—•<br />

cos 0<br />

but if C= 90°, cot C= 0; therefore,<br />

or<br />

. ,sin 6 cot e ^ , ^<br />

cos A = i— = tan 6 cot e<br />

tan J cos b tanrt<br />

cos A = 7 cos B = 1—^ (ol)<br />

tan e tan c ^ '


16&-<br />

SPHERICAL TRIGONOMETRY.<br />

that is, the cosine of either angle is equal to the tangent of the adjacent,<br />

side, divided by the tangent of the hypotenuse. Compare<br />

PI. Trig. (1).<br />

41. From (10), we have,<br />

sin 6 cot a — cos b cos Q<br />

cot A =<br />

: 7y<br />

sm O<br />

which, when O = 90°, becomes<br />

1. A - I ^ sin 6<br />

cot A = sm 6 cot a =<br />

tana<br />

or, taking the reciprocals,<br />

tana .„ tan 6 ,_„,<br />

tanJ.=-=—5; tz.nB = -. (82)<br />

sm 0 sm a ^ '<br />

that is, the tangent of either angle is equal to the tangent of the opposite<br />

side, divided by the sine of the adjacent side. Compare<br />

PI. Trig. (1).<br />

42. From (5), we find,<br />

. . cos 54-cos (7 cos J.<br />

sin.A = J—;—7=<br />

cos b sin G<br />

and if (7= 90°,<br />

cos 5 . _ cos J. ,„„,<br />

sm A = r sm B = (83)<br />

cos b cosa ^ '<br />

that is, the cosine of either angle, divided by the cosine of its opposite<br />

side, is equal to the sine of the other angle. In PI. Trig, we have<br />

sin A — cos B.<br />

43. From (4), we have,<br />

cos e = cos a cos 6 4- sin a sin b cos G<br />

or, when (7=90°,<br />

cos e = COS a cos b (84)<br />

that is, the cosine of the hypotenuse is equal to the product of the cosines<br />

of the two sides. In Vl. Trig, e^ = a^ 4- b^.<br />

44. From (5),<br />

cos G-{- cosA cos B<br />

cos c = -.—1—-.—^<br />

am. A smB<br />

or, when (7 =90°,<br />

cos A cos B<br />

cos c = -.—1—;—„- = cot J. cot 5 (8.5)<br />

sm A sm B<br />

^ '


SOLUTION OF SPHERICAL RIGHT TRIANGLES. 169<br />

hat is, the cosine of the hypotenuse is equal to the product of the cotangents<br />

of the two angles. In PL Trig., 1 = cot JL cot B.<br />

45. No difficulty will be found in remembering the preceding formula<br />

for spherical right triangles, if they are associated with the<br />

corresponding ones for plane triangles: thus.<br />

In plane right triangles.<br />

sin J.=<br />

cos J. =<br />

tan^ =<br />

a<br />

G<br />

b<br />

G<br />

a<br />

T<br />

smA = coa B,<br />

sin-B<br />

cos-B<br />

tan 5<br />

c^ = a^ + §2<br />

e<br />

a<br />

c<br />

b_<br />

a<br />

sin.B = cos^<br />

1 = cot J.cot.B<br />

In spherical righ^ triangles.<br />

, sm a<br />

sin^ = -^—<br />

sin e<br />

cos JL = tan b<br />

tan c<br />

tana<br />

tan A = -:—r<br />

sm 6<br />

sin.A = cos-B<br />

cos h<br />

. -_ sm b<br />

sm B = ^—<br />

sm e<br />

cos-B =<br />

tana<br />

tan e<br />

tan.B =<br />

tan 6<br />

sin a<br />

cos J.<br />

sin 5 =<br />

cos a<br />

cos e = cos a cos b<br />

cos e = cot J. cot-B<br />

.46. Napier's Rules. By putting these ten equations under a difi'erent form, Napier<br />

uontrived to express them all in two rules, which, though artificial, are very generally<br />

employed as aids to the memory.<br />

In these rules, the complements of the hypotenuse and of the two oblique angles<br />

are employed instead of the hypotenuse and the angles themselves. The right angle<br />

not entering into the formulae, they express the relations of five parts, but in the<br />

rules the five parts considered are a, b, co. c, co. A and co. P. Any one of these<br />

parts being called a middle part, the two immediately adjacent may be called adjacent<br />

parts and the remaining two, opposite parts. The right angle not being considered,<br />

the two sides including it are regarded as adjacent parts. The rules are :<br />

I. The sine of the middle part is equal to the product of the tangents of the adjacent<br />

parts.<br />

II. The sine of the middle part is equal to the product of the cosines of the opposite parts.<br />

The correctness of these rules will be sho-wn by taking each of the five parts as<br />

middle part, and comparing the equations thus found with those already demonstrated.<br />

1st. Let CO. c be the middle part; then co. A and co. B are the adjacent parts,<br />

a and b the opposite parts, and the rules give<br />

sin (co. c) = tan (co. A) tan (co. P)<br />

sin (co. c) = coa a cos b<br />

cos c = cot J1 cotP<br />

cos c = COS a cosi<br />

which are (85) and (84).<br />

2d. Let CO. A be the middle part; then co. c and b are the adjacent parts, co. P<br />

»nd a the opposite parts, and the rules give<br />

an (co. A) = tan (oo. c) tan b or cos A = cot e tan b<br />

sin (co. A) = cos (co. P) cos a<br />

22 P<br />

cos .4 = sin P cos a


170 SPHERICAL TRIGONOMETRY.<br />

In tht same maimer, if co. B is taken as the middle part,<br />

sin (oo. P) = tan (co. c) tan a or cos P = cot c tan a<br />

sin (co. P) = cos (co. A) cos b<br />

cos P = sin .4 oo? b<br />

and these four equations are the same as (81) and (83).<br />

3d. Lei a be the middle part; then co. B and b are the adjacent parts, co. A<br />

and CO. c the opposite parts, and the rules give,<br />

sin a =: tan (oo. P) tan b or sin a = cot P tan b<br />

sin a = cos (co. A) cos (co. c)<br />

sin a = sin .4 sin e<br />

In the same manner, if b is taken as the middle part,<br />

sin b = tan (co. A) tan a or sin J = cot ^ tan a<br />

sin b = cos (co. P) cos (co. c)<br />

sin 6 = sin P sin c<br />

and these four equations are the same as (80) and (82).<br />

It appears, therefore, that these rules include all the ten equations previously<br />

proved; and they include no others, since we have taken each part successively as<br />

the middle part.<br />

In the application of these rules, it is unnecessary to use the notation co. A, co. P,<br />

CO. c, since we may write down at once sin A for cos (co. A), kc*<br />

47. In order to solve a spherical right triangle, two parts must<br />

be given, and from the equations of Art. 45, that equation must be<br />

selected which expresses the relation between these two parts and<br />

the required part.<br />

When Napier's Rules are employed, it is only necessary to determine which of tho<br />

three parts—the two given and the one required—is to be taken as the middle part.<br />

" These three parts are either all adjacent to each other, in which case the middle<br />

one is taken as the middle part, and the other two are adjacent parts; or one is<br />

separated from the other two, and then the part which stands by itself is the middle<br />

part, and the other two are opposite parts."f<br />

48. In order to distinguish the functions of parts less than 90°<br />

from those greater than 90°, it will be necessary carefully to observe<br />

their algebraic signs, according to PI. Trig. Art. 40. But when a<br />

required part is determined by its sine, since the sine of an angle<br />

and of its supplement are the same, there will be two angles, both<br />

of which may be regarded as solutions, except when this ambiguity<br />

is removed by either of the following principles.<br />

* If we employ as the five parts, the hypotenuse, the two angles, and the complements<br />

of the two sides including the right angle, these parts will be the complements<br />

of those used in Napier's Rules, and we shall have<br />

MAUDOTT'S RULES.—I. The cosine of the middle part is equal to the product of the cotangents<br />

of the adjacent parts.<br />

II. The cosine of the middle part is equal to the product of the sines of the opposite parts.<br />

With a little attention at the commencement, however, and by observing the analogy<br />

exhibited in Art. 45, the student will find that he will have little use for either<br />

of these artificial rules.<br />

f Peirce's Spherical Trigonometry.


SOLUTION OP SPHERICAL RIGHT TRIANGLES. 171<br />

49. In a right spherical triangle, an angle and its opposite side<br />

are always in the same quadrant, that is, either both less or both<br />

greater than 90°. For, by (83),<br />

cos B<br />

sin A = cos b<br />

m which, since sin A is always positive, {A < 180°), cos B and<br />

cos b must have the same sign; that is, B and b must be either both<br />

less or both greater than 90°.<br />

50. When the two sides including the right angle are in the same<br />

quadrant, the hypotenuse is less than 90°, and when the two sides<br />

are in different quadrants, the hypotenuse is greater than 90°.<br />

For, by (84),<br />

cos e = cos a cos b<br />

in which, if a and b are in the same quadrant, cos a and cos b have<br />

like signs, and cos e is positive, that is, c < 90° ; but if a and h<br />

are in difi'erent quadrants, cos a and cos b have difi'erent signs, and<br />

cos e is negative, that is, c > 90°<br />

We proceed now to the solution of the several cases.<br />

51. CASE I. G-iven the hypotenuse and one angle, or e and A,<br />

Fig. 6.<br />

To find a. The relation among the three<br />

Mg. 6.<br />

parts, e. A, and a, (as in PL Trig, with the<br />

same data), is given by the sine of A; and<br />

by Art. 45,<br />

from which we find*<br />

sm.A =<br />

sm c<br />

sin a = sin e sin A (86;<br />

There will be two values of a corresponding to the same sine, but,<br />

by Art. 49, the true value is that which is in the same quadrant<br />

as A.<br />

To find b. The relation among the three parts, e, A, and b, (as<br />

in PI. Trig, with the same data), is given by the cosine of-4,,or,<br />

from whichf<br />

cos-4 = tan 6<br />

tan c<br />

tan b = tan c cos A (87)<br />

* This equation would be found by Napier's Rules, taking a as the middle part,<br />

f We find the same result by Napier's Rules, taking co. A as the middle part


172 SPHERICAL TRIGONOMETRY.<br />

To find B, We have, by (85),*<br />

cos e = cot A cot B<br />

from which cot B = . = cos c tan A (88')<br />

cot A<br />

^ '<br />

The quadrants in which b and B are to be taken, will be determined<br />

by means of the signs of tan b and cot B, according to PI.<br />

Trig. Art. 40.<br />

Gheck. To guard against numerical errors, it is often expedient<br />

to compute the same quantity by two difi'erent and independent<br />

methods. In many cases, however, -sve may test the accuracy of<br />

several operations by a single formula, which may be called the<br />

check. In the present instance, when the three parts, a, b, and B,<br />

have been found, we should have, by (82), the relation<br />

sin a = tan b cot B<br />

so that if the work is correct, we shall find<br />

log sin a = log tan b 4- log cot B<br />

EXAMPLES.<br />

1. Given e = 110° 46' 20", A = 80° 10' 30", to solve the triangle.<br />

By (86). By (87). By (88).<br />

e, log sin 9-9708106 log tan — 0-4210061 log cos — 9-5498045<br />

A, log sin 9-9935833 log cos 4- 9-2320794 log tan 4- 0-7615038<br />

log sin a 9-9643939 log tan b — 9-6530855 log cot B — 0l3113083<br />

log tan b — 9-6530855<br />

Gheck. log sin a 4-9^9643938<br />

Ans. a = 67° 6'52"-7, b = 155°46'42"-7, B = 153° 58'24"-5<br />

2. Given c = 120°, A = 120° ; solve the triangle.<br />

Ans. a = 131° 24' 34"-7 b = 40° 53' 36"-2 B = 49° 6' 28 '-S<br />

52-. If A = 90°, we must also have, by (85), c = 90°, and then<br />

0 0<br />

tan 0 = — tan P = -jr-<br />

BO that b and P are both indeterminate; that is, there is an indefinite number of<br />

triangles which satisfy the given values of e and A; but since<br />

cos P = cos b sin A = cos b<br />

we always have B z=h; and since<br />

sin a = sin c sin ^ = 1<br />

we have a = 90°, and all the parts of the triangle are equal to 90°, except b and P<br />

H only c is .given = 90°, all the parts of the triangle are equal to 90°, except A<br />

and a ; and we have Ax= a.<br />

* Or by Napier's Rules, taking co. c as the middle part.


SOLUTION OF SPHERICAL RIGHT TRIANGLES. 173<br />

53. CASE II. Given the hypotenuse and a side, or e and a.<br />

To find A. We have by (80),<br />

To find B. By (81),<br />

• . sin a . _^<br />

sm A = -. = cosec e sm a (o9}<br />

sm c<br />

^ '<br />

^ tana ,.„,<br />

cos B = = cot e tan a (90)<br />

tan c<br />

^ '<br />

To find b. By (84),<br />

cos c = COS a cos b<br />

from which cos o = = cos c sec a (91)<br />

cos a<br />

^ '<br />

Check.<br />

We have between J., B, and 6, the relation<br />

cos -B = sin ^ cos b<br />

EXAMPLES.<br />

1. Given e = 140°, a = 20° ; solve the triangle.<br />

By (89). By (90). By (91).<br />

c,log cosec 0-1919325 log cot - 0-0761865 log cos - 9-8842540<br />

a, log sin 9-5340517 log tan 4- 9-5610659 log sec 4- 0-0270142<br />

log sin A 9-7259842 log cos B - 9-6372524 log cos b - 9-9112682<br />

log sin A 4- 9-7259842<br />

Gheck. log cos B - 9-6372524<br />

Ans. A= 32° 8'48"-l<br />

.B = 115°42'23"-8<br />

5 = 144°36'28"-4<br />

2. Given e = 101° 16' 16".7, b = 115° 42' 38"-5 ; find A.<br />

Ans. ^ = 65° 32'56".4<br />

64. When a = c and consequently both = 90°, sin ^ = 1, A ^ 90°, and<br />

cos P = -rr- cos J = -jT- COS P = COS J<br />

go that B = b, but both are indeterminate as in Art. 52.<br />

P2


174 SPHERICAL TRIGONOMETRY.<br />

55. CASE III. Given one angle and its opposite side, or A and a.<br />

We shall have<br />

sin J. • sm a whence sin e = cosec A sin a (92)<br />

tan .4 = tan a<br />

sin 6<br />

sin J = cot ^ tan a (93)<br />

sin.B =<br />

cos J.<br />

cosa<br />

sin J5 = cos J. sec a (94)<br />

Glieck. sin b = sin c sin B<br />

In this, case, there are always two solutions, all the required parts<br />

being determined by their sines, and the ambiguity not being removed<br />

by either Art. 49 or Art. 50. This also appears from Fig. 7.<br />

If AB and AG\>e produced to meet in A!, ABA' and<br />

AG A! are semicircumferences and A = A!; the triangles<br />

ABG and A'BG both contain the given parts A and a,<br />

but c', b' and B' are respectively the supplements of c,<br />

b and B. It must not be inferred that in every case all<br />

the required parts are less than 90° in one triangle, and<br />

greater than 90° in the other; but the proper values for<br />

each triangle must be selected by Arts. 49 and 50.<br />

EXAMPLES.<br />

1. Given A = 100°, a = 112°; solve the triangle.<br />

Ans. e= 70°18'10"-2 ) ( e = 109°41'49".8<br />

J = 154° 7'26"-5 V or < b= 25°52'33".5<br />

B = 152° 22,' l"-3 J I B= 27°36'58".7<br />

2. Given A = 80°, a = 68° ;<br />

Ans. e= 70°18'10".2<br />

6= 25°52'33".5<br />

B= 27°36'58"-7 .<br />

solve the triangle.<br />

r e = 109°41'49"-8<br />

>• or < 5=154° 7'26"-5<br />

I .B = 152°23' 1".3<br />

3. Given B = 150°, b = 160° ; solve the triangle.<br />

Ans. e = 136°50'2.3"-3 ] r c= 43° 9'36".7<br />

a= 39° 4'50"-7 > or < a = 140° 55' 9".3<br />

A= 67° 9'42"-7 i I .A = 112° 50'17"-3


SOLUTION OF SPHERICAL RIGHT TRIANGLES. 175<br />

56. CASE IV. Given one angle and its adjacent side, or A and b.<br />

We shall find the required parts by the equations<br />

Check,<br />

cos B = sin^ cos b (95)<br />

tan a = tan A sin b (96)<br />

cot e = cos A cot b (97)<br />

cos B = tan a cot e<br />

EXAMPLES.<br />

1. Oiven A = 80° 10' 30", b = 155° 46' 42".7; solve the triangle.<br />

Ans. B = 158° 58'24:"-5<br />

a= 67° 6'52"-6<br />

c = 110°46'20"-0<br />

2. Given B = 152° 23' 1".3, a = 112° 0' 0" ; solve the triangle.<br />

Ans. A = 100°<br />

5 = 154° 7'26"-5<br />

c= 70°18'10"-2<br />

57. CASE Y. Given the two sides, a and b.<br />

We find the required parts by the equations<br />

cos c = cos a cos 5 (98)<br />

cot A = cot a sin b (99)<br />

cotB = sina cot b<br />

Check, cos e = cot J. cot B<br />

0-^^)<br />

Given a = 116°,<br />

EXAMPLE.<br />

b = 16°; solve the triangle.<br />

Ans. e= 114° 55'20'-4<br />

A= 97°39'24"-4<br />

B= 17°4r39"-9<br />

58. CASE VI. Given the two angles, A and B,<br />

The required parts are found by the formulse<br />

cos e = cot J. cot B<br />

(1*^1)<br />

cos a = cos ^ cosec B (102)<br />

cos b = cosec A cos B<br />

Check, cos e = cos a cos h<br />

(}-^^)


176 SPHERICAL TRIGONOMETRY.<br />

EXAMPLE.<br />

Given J. = 60° 47' 24".3,^ = 57° 16' 20".2.: solve the triangle.<br />

Ans. G = 68° 56' 28"-9<br />

a = 54° 32' 32"-l<br />

b = 51° 43' 36".l<br />

ADDITIONAL FO-RMVLIE FOB THE SOLUTION OF SPAEEIOAL RIOHT TRIANGLES.<br />

59. As in plane trigonometry, cases occur in which particular solutions of greater<br />

accuracy than the ordinary ones are required. (PI. Trig. Art. 112.)<br />

BO. From (89) we find<br />

1 — sin A sin c — sin a<br />

1 -}- sin ^ sin c 4- sin a<br />

which by PI. Trig. (154) and (109) is reduced to<br />

tan«(45°-J^)=*-"^iii:^! (104)<br />

^ ' tan i (c + a)<br />

which will give a more accurate result than (89), when A is nearly 90°.<br />

61. From (91) we find<br />

1 — cos b cos a — cos c<br />

1 4- cos b cos a 4- cos c<br />

or tan' J J = tan ^ (e + a) tan J (c — a) (105)<br />

whi


QUADRANTAL AND ISOSCELES TRIANGLES. 177<br />

of which (110) may be used when c is small, and (111) when c is nearly 180°, instead<br />

of (101).<br />

64. The equations (92), (98), and (94), of CASE III. give<br />

tan' (45° - J c) = t^°H^-°) ^^^^^<br />

^ ^ ^ tan i (.4 4-a) ^ '<br />

tan' (45° - * 6) = ''",^'^7"^ (113)<br />

^ ' sm (A + a) ^ '<br />

tan' (45° — J P) = tan J (^ — a) tan J (.4 4- a) (114)<br />

The roots of these equations having the double sign, we may take the angles<br />

45° —• J e, etc. either with the positive or negative sign, whence the two solutions<br />

of the problem, as in Art. 55.<br />

65. Some of the solutions may be adapted for computation by the table of natural<br />

sines. Thus from (86), (95), and (98),<br />

sin a = .J [cos (c — A) — cos (c + A)J (115)<br />

cosP= J [sin (6 4-^) —sin (5 —.4)] (116)<br />

cos c = J [cos (a 4- *) 4- cos (a — 6)] (117)<br />

66. The following relations are occasionally useful:<br />

From (83) we have<br />

cos a sin .4 cos ^ sin 2 A<br />

cos b sin P cos P sin 2 B<br />

(118)<br />

From (80) and (83),<br />

sinP sin A cos A sin 2 ^<br />

sm c sin a cos a sin 2 a<br />

(119)<br />

From (80) and (84),<br />

sin A sin a cos a sin 2 a<br />

cos b sin c cos c sin 2 c<br />

(120)<br />

67. Yarious relations may be deduced from the general formulas of the preceding<br />

chapter by making G = 90°. The following are easily obtained :<br />

sin (e — a) = cos c tan b tan J P = cos a sin b tan J P<br />

sin (c 4- a) = cos c tan b cot J P = cos a sin b cot J P<br />

cos (c — a) = cos b + sin a sin b tan J B<br />

cos (c+ a) = cos b •— sin a sin b cot J P<br />

sin (a — b) = 2 sin c sin ^ (A + B) sin f (A —B)<br />

sin (a 4" *) = 2 sin c cos J (^ -f- -^) '^"^ i (-^ — -^)<br />

. „ cos i a cos i i „ sin i a sin i i<br />

sm S = . — cos S" = —-,—^—<br />

cos ^ C<br />

COS ^ c<br />

tan S = — cot J a cot J 5 (121)<br />

QUADBANTAL AND ISOSCELES TBIANOLES.<br />

68. The polar triangle of the right triangle is a quadrantal triangle, one side (the<br />

side opposite the angle G) being equal to 90°. The solution of such triangles is as<br />

simple as that of right triangles, the formuljB for the purpose being obtained from<br />

the preceding, by the process of Art. 8. It is unnecessary to produce them here, as<br />

quadrantal triangles are generally avoided in practice, and when unavoidable are<br />

"-eadily solved by means of the polar triangle.<br />

An isosceles triangle is easily solved by dividing it into two right triangles by a<br />

oerpendicular from the angle included by the equal sides.<br />

23


]7a SPHERICAL TRIGONOMETRY.<br />

CHAPTER IIL<br />

SOLUTION OF SPHERICAL OBLIQUE TRIANGLES.<br />

69. In the solution of spherical oblique triangles, a required part<br />

may sometimes be found by its sine, in which case there will be two<br />

values of that part, answering to the conditions, unless the proper<br />

value can be determined by other considerations. In certain cases,<br />

the true value can be selected by applying one or more of the folio-wing<br />

principles, some of which are demonstrated in geometry. We<br />

still consider only those triangles each of whose parts is less than 180°.<br />

I. The greater side is opposite the greater angle, and conversely.<br />

II. Bach side is less than the sum of the other two.<br />

III. The sum of the sides is less than 360°.<br />

IV. The sum of the angles is greater than 180°.<br />

V. Hach angle is greater than the difference between 180° and<br />

the sum of the other two angles.<br />

For, by IV., A + B+ G> 180°<br />

whence,<br />

A > 180° — {B-\-G)<br />

But if -B 4- G> 180°, we have, in the polar<br />

triangle, A'B'G', Fig. 8, by II.,<br />

a' < 6' 4- e'<br />

180°-J. < 180°-5 4-180°- 0<br />

-A< 180° - (^ 4- {B+ (7)-180°<br />

VI. A side which differs more from 90° than another side, is in<br />

the same qvadrdtnt as its opposite angle.<br />

For, by (4), we have<br />

. cos a — cos b cos e<br />

cos A = ;—,—;<br />

sm 6 sm e<br />

In which the denominator is always positive.<br />

If, then, a differs


SOLUTION OF SPHERICAL OBLIQUE TRIANGLES. 179<br />

more frcm 90° than b or than c, we have, (neglecting the signs for<br />

a moment),<br />

and still more<br />

cos a > cos 5 or >• cos e<br />

cos a > cos J cos e<br />

Hence cos a being numerically greater than cos b cos e, the sign of<br />

the whole numerator, and therefore the sign of cos A, is the same<br />

as that of cos a; that is, A and a are in the same quadrant.<br />

VII. An angle which differs more from 90° than another angle,<br />

is in the same quadrant as its opposite side. For, by (5),<br />

cos a-<br />

cos A -f cos B cos G<br />

sin B sin G<br />

in which, if A difiers more from 90° than B, or than G, cos A determines<br />

the sign of the -whole fraction, and therefore the sign of cos a.<br />

VIIL In every spherical triangle there are at least two sides which<br />

are in the same quadrants as their opposite angles respectively. This<br />

follows from VI. and VII.<br />

IX. The sum of two sides is greater than, equal to, or less than,<br />

180°, according as the sum of the two opposite angles is greater than,<br />

equal to, or less than, 180° In other words, the half sum of two<br />

sides is in the same quadrant as the half sum of the opposite angles.<br />

For, by (41),<br />

tan J (a 4- b) cos J (J. 4- B) = tz.nl c coal {A — B)<br />

the second member of which is always positive, so that tan J (a -"- b)<br />

and cos \[A -\- B) must have the same sign.<br />

70. CASE I. Given tivo sides and the ineluded<br />

angle, or b, c and A. (Fig. 9.)<br />

First Solution; when the third side and<br />

one of the remaining angles are required.<br />

To find a. The relation between the given ^^^^ ^-^ B<br />

parts b, 0, A and the required part a is expressed<br />

by the first equation of (4),<br />

cos a = cos c cos S 4- sin e sin b cos A<br />

by which a may be found by computing separately the two terms of<br />

the second member and adding |;heir values to form the natural cosine<br />

of a ; but we should thus be required to use, besides the table<br />

of log. sines, also the table of logarithms of numbers, and the table<br />

of natural sines and cosines. To adapt it for logarithmic computation<br />

by the table of log. sines exclusively, we employ the prir^ess of<br />

(M)


IgQ<br />

SPHERICAL TRIGONOMETRY.<br />

PL Trig., Arts. 174, 175. Thus, let ^ be a number and $ an auxiliary<br />

angle such that<br />

A sin (B = sin & cos .A )<br />

then (M) becomes<br />

J , r ^'"^<br />

^ cos


SOLUTION OF SPHERICAL OBLIQUE TRIANGLES. 181<br />

In the use of these formulse, as indeed of all that follow, tho<br />

sighs of all the functions must be carefully observed, according to<br />

PI. Trig. Arts. 37 and 40.<br />

We may take (p between 0 and 180°, less or greater than 90°,<br />

according as the sign of its tangent is positive or negative; or we<br />

may take it numerically less than 90° in all cases, but positive or<br />

negative according to the sign of its tangent, (PL Trig. Arts. 37<br />

and 174).<br />

Gheck. The quotient of (ra) divided by (m*) is<br />

cot B tan (c — $)<br />

cos a sin b sin A<br />

which multiplied by the following, from (3),<br />

sin a sin ^ = sin b sin A<br />

gives tan a cos B = tan (c —


182 SPHERICAL TRIGONOMETRY.<br />

EXAMPLES.<br />

1. Given 5 = 120° 30'30", c = 70°20'20", .4 = 60° 10'10";<br />

find a and B.<br />

By (122).<br />

b = 120° 30' 30" log tan 5 - 0-2297071<br />

A = .50° 10' 10" log cos A 4- 9-8065322<br />

(p = 132° 36' 44"-2* .log tan $ - 0-0362393<br />

c= 70°20'20"-0<br />

c_cp=_ 62°16'24".2<br />

By (122). By (123). By (124).<br />

logcos(c—?)-}- 9-6676893 log sin (c — j) —9-9470304 logtan (c — ?) —0-2793410<br />

w 00 log COST-? — 04693898 ar co log sin p 4-0-1331505 log tan a 4-0-4291648<br />

log cos 6 —9-7055761 log cot ^-f 9-9212038 log cosP—9-8501762<br />

log cos a 4:9-5426552 log cot P—0-0013847 Gheck. —0-2793410<br />

a = 69°34'55"-9<br />

P = 135°5'28"-8<br />

2. Given 5 = 120° 30'30", e = 70° 20'20", ^ = 50° 10'10";<br />

find a and G.<br />

Ans. a = 69° 34' 55"-9<br />

(7= 50° 30' 8"4<br />

3. Given 5 = 99° 40'48", c = 100° 49'30", ^ = 65° 33'10";<br />

find a and B,<br />

Ans. a = 64° 23' 15".0<br />

5 = 95° 38' 4".0<br />

4. Given b = 99° 40'48", e = 100° 49' 30", A = 65° 33' 10";<br />

find a and G.<br />

Ans. a = 64° 23' 15".0<br />

(7=97°26'29".l<br />

5. Given b = 98° 2' 20", e = 80° 35' 40", A = 10° 16' 30";<br />

find a and G.<br />

Ans. a = 20° 13' 30"-l<br />

(7=30°35'56"-7<br />

72. If B, (7 and a were all required, we might find a and (7 by<br />

(125), and then B by Art. 3, which gives<br />

sin a: sin t = sin A : sin B<br />

. „ sin b sin A<br />

or sm xj = :<br />

sm a<br />

*We may also take? = —47° 2.3' ]5"-8, whence c —j = 117° 43' 35- 8, -which<br />

will give the same values of a and B as found in the text.


SOLUTION OF SPHERICAL OBLIQUE TRIANGLES. 183<br />

Of the-two values of B less than 180° given by this formula, the<br />

proper one may generally be selected by the principles of Art. 69.<br />

There are cases, however, in which all the conditions there given are<br />

satisfied by both values of -B,* and on this account it is preferable,<br />

in general, to combine (123) and (125), or to employ the following<br />

solution, when the three unknown parts are all to be found.<br />

73. CASE I. Given b, e and A. Second Solution ; when the two<br />

remaining angles are required, or when the three unknown parts are<br />

all required.<br />

We have, by Napier's Analogies, (42) and (43),<br />

whence<br />

sin J (6 4- e) : sin |(6 — e) = cot J J.: tan ^{B— G)<br />

cos J (5 4- e): cos J (5 — c) = cot J J.: tan J (-B 4- (7)<br />

*^^H^-^)=:-i^ff^]cotjA (126)<br />

*^^H^+^) = ^^^f^jcotJ^ (127)<br />

which determine ^{B— G) and l{B-\- G); then the half difi'erence<br />

added to the half sum gives the greater angle, and the half difi'erence<br />

subtracted from the half sum gives the less angle.<br />

If c > &, we may write c — b, G— B,in the place oi b — c, B — G.<br />

We may now find a by either of Napier's Analogies, (40), (41),<br />

which givef<br />

sini(5 4-C)<br />

*^^ * "^ = sinl {B-G) *^° * (5 - c^) (128)<br />

cos IIB+G)<br />

^^'^ * "^ = cosl{B-G) *'''' ii^ + ") (129)<br />

* By Art. 69, "VL, if b difi-ers more from 90° than c, B is in the same quadrant<br />

aa b, and all ambiguity is removed. If e differs more from 90° than b, we may fina<br />

a and B by (122) and (123), and then G by the formula<br />

. _ sin c sin A<br />

sm G = : •<br />

sm a<br />

G being taken in the same quadrant as c:<br />

f We may also find a from any one of Gauss's Equations (44), which become, vr,.<br />

the present ease,<br />

cos J a sin'J [B + G) = cos J A cos J (i — c)<br />

cos J a cos J (P 4- G) = sin ^ A cos i (b + c)<br />

sin J a sin J (P —• G) = cos ^ A sin ^ (b — c)<br />

sin J a cos J (P — d) = sin ^ A ain ^ (b + c)


184 SPHERICAL TRIGONOMETRY.<br />

1. Given b =<br />

find B, 0 and a<br />

By (126).<br />

irCO log sin 1(6 4- c) 4- 0-0019487<br />

logsini(6-c)4-9-6273228<br />

log cot i-^ 4-0-3296529<br />

EXAMPLES.<br />

120° 30' 30", c = 70° 20' 20", A<br />

We have<br />

1 (J 4- c) = 95° 25' 25"<br />

1 (6 - c) = 25° 5' 5"<br />

ij. = 25° 5' 5"<br />

log tan \{B-G) 4- 9-9589244<br />

\(B-G)= 42° 17' 40"-2<br />

^=135° 5'28"-8<br />

50° 10' 10";<br />

By (127).<br />

ar cologcosi(5 + e) - 1-0244829<br />

logcosJ(6-e) 4-9-9569757<br />

log cot J J. 4-0-3296529<br />

log tan \{B^C) -173"llill5<br />

1 (^ 4- (7) = 92° 47' 48"-6<br />

(7= 50° 30' 8".4<br />

By (128).<br />

By (129).<br />

arCO log sini(.B- (7)4-0-1720227 ar colog cos \{B- (7)4-0-1309469<br />

log sin i(5-hC)4-9-9994824 log C0SK.B4- Q)- 8-6883709<br />

logtani(5-c)-F9-6703471 log tan i(J 4-c)-1-0225342<br />

log tan J a 4-9-8418522<br />

log tan la 4-9-8418520<br />

1 a = 84° 47' 28"-0<br />

2. Given b = 99° 40' 48", e<br />

find B, 0 and a.<br />

Ans. B = 135° 5' 28"-8<br />

0 = 60° 30' 8".4<br />

a = 69° 34' 56"-0<br />

100° 49' 30", A = 65° 33' 10";<br />

Ans. B = 95° 38' 4"-0<br />

G = 97° 26' 29"-l<br />

a = 64° 23' 15"-1<br />

74. It may be remarked with regard to (128) and (129) that, when b and e (and<br />

consequently P and G) are nearly equal, a small error in the previous determination<br />

of the small angle J (P — G) may produce alarge one in log sin \(B — 0), and<br />

consequently in log tan J a found by (128). In that case, therefore, (129) must be preferred.<br />

In like manner, if J(J -|- c), and consequently ^(B + G), are nearly equal to<br />

90°, (129) -will become inaccurate, and then (128) is to be preferred.<br />

Formula (128) would fail entirely if P = C, and formula (129) would fail if<br />

J (P 4- C) = 90°, since the second members in these cases would assume the indeterminate<br />

form —.<br />

75. CASE I. Given b, c and A. Third Solution. Wlien the<br />

third side is alone required, the computation by(122)is inmost cases<br />

as convenient as any other; but theie are various other methods


SOLUTION OF SPHERICAL OBLIQUE TRIANGLES. 185<br />

derived from the formulse of the preceding chapter, which hav» been<br />

employed with advantage in particular applications. Among the<br />

most convenient are the following, from (12) and (13):<br />

cos a = cos (6 — e) — 2 sin b sin e sin^ J A (130)<br />

cos a = cos (5 -f e) 4- 2 sin b sin c cos^ J A (131)<br />

The computation of these requires the use of natural cosines and<br />

numbers, the signs of which must be carefully observed.<br />

EXAMPLE.<br />

Given b = 99° 40' 48", c = 100° 49' 30", A = 65° 33' 10";<br />

find a.<br />

By (130).*<br />

IA= 32° 46' 35" log sin'-1 J. = 2 log sin J A 9-4669752<br />

6 _ c = - 1° 8'42" log si'n c 9-9922023<br />

log sin b 9-9937722<br />

log 2 0-3010300<br />

- 2 sin J sin e sin^ i J. = - 0-5675181 log 9-7539797<br />

nat cos (6 — c) = 4- 0-9998003<br />

nat cos a = 4- 0-4322822 a = 64° 23' 15"<br />

By (131).<br />

1 ^ = 32° 46' 35" log cos^ J ^ = 2 log cos i J. 9-8493748<br />

6 4- e = 200° 30' 18" log sin c 9-9922023<br />

log sin 6 9-9937722<br />

log 2 0-3010300<br />

•f 2 sin b sin c cos^ J JL = 4- 1-3689240 log 0-1363793<br />

nat cos (5 4- c) = — 0-9366416<br />

nat cos a = 4- 0-4322824 a = 64° 23' 15"<br />

76. In Art. 14, we have deduced several formulse by which J a may be computed<br />

VVe may adapt (17) and (18) for logarithmic computation, as follows :<br />

sin' 0 = sin S sin c cos' ^ A 1<br />

sin' J a = sin' J (i -f- c) — sin' 0 I (132)<br />

= sin [J (b+c) + f\ sin [J (J + c) _ 0] J<br />

sin' (6 = sin i sin c sin' ^A ]<br />

COB' J a = cos' J (4 — c) — sin' «i I (133)<br />

= cos [J (b — c)+ 0] cos [J (b — c) — 0] J<br />

of wHch (132) is to be preferred when J a < 45°, and (133) when J a > 45°.<br />

* The computation of (130) is facilitated by the use of a special table (given in<br />

many treatises on navigation), from which, with the arguments is taken the logarithm<br />

of 2 sin' J -4 = versin A. [PI. Trig. (4) and (139)].<br />

•2i P 2


186 SPHERICAL TRIGONOMETRY.<br />

Fig. 9.<br />

^B<br />

77. CASK II. Given two angles and the<br />

included side, or A, 0 and b. (Fig. 9).<br />

Pirst Solution ; when the third angle and<br />

one of the remaining sides are required.<br />

To find B. The relation between A, G, b<br />

and B, is, by (5),<br />

cos B = — cos (7 cos A -f sin Gain A cos b<br />

(M)<br />

which is adapted for logarithms by the method employed in the preceding<br />

case. Thus, let<br />

then (M) becomes<br />

A sin 9- = cos A )<br />

AcosS- = sin^ cos 5<br />

coaB = h (sin Ccos9- — cos CsinB)<br />

= h sin ((7-9-)<br />

,. . . , cos A<br />

or, eliminating h = -:—^, the formulse for finding B are<br />

(m)<br />

(m')<br />

cot 9" = tan A cos b<br />

cos<br />

B= sin {G — B-) coa A<br />

sinS-<br />

To find a. From the third equation of (10), we find,<br />

sin OcotA + cos (7 cos 5<br />

cot a =<br />

sin 6<br />

sin GcosA -{- cos Gain A cos 6<br />

sin A sin b<br />

which, by (m), becomes<br />

cot a<br />

_ ACQS ((7—9-)<br />

sin A sin b<br />

(134)<br />

(«)<br />

«^ T • J.- I sin J. COS 5<br />

or, ehmmatmg h = ^^^ ^ , we have, for finding a,<br />

cot 9- = tan A cos J<br />

cot a<br />

cos ((7— 9-) cot 5<br />

cos 9-<br />

(135)<br />

As in the preceding case, we may either take 9- always between 0<br />

and 180°, less or greater than 90° according as its tangent is posi-


SOLUTION OF SPHERICAL OBLIQUE TRIANGLES. 187<br />

tive or negative; or we may take 9- numerically less than 90° in ail<br />

cases, positive or negative, according to the sign of its tangent.<br />

(PL Trig. Art. 174.)<br />

Gheck. The quotient of (w) by (w') is<br />

which, multiplied by<br />

gives<br />

cot a _ cot ((7—9-)<br />

cos B sin A sin b<br />

sin JB sin a = sin J. sin b<br />

tan -B cos a = cot ((7 — 9-) (136)<br />

by which the values of B and a, found by (134) and (135), may be<br />

verified.<br />

78. If B and e were required, the solution would be similar, only<br />

interchanging a and e, A and G. By the fundamental formulae, we<br />

should have,<br />

cos B = — cos A cos G -\- ainA sin G cos b<br />

sin A cos G -{- cos ^4 sin (7 cos b<br />

cot c = •<br />

sin G sin b<br />

and denoting the auxiliary angle by ^, the logarithmic solution<br />

would be<br />

cot ^= tan Ccos b<br />

sin (A — ^) cos C<br />

cos B =<br />

sin^<br />

cos {A — ^) cot b<br />

cot e =<br />

cos ^<br />

Gheck. tan JB COS C = cot (J. — ^)<br />

EXAMPLES.<br />

(0)<br />

(137^<br />

1. Given A = 135° 5'28"-8, C= 50° 30'8"-4, 6 = 69° 34'55".9,<br />

find .B and a.<br />

By (134).<br />

J.= 135° 5'28"-8 log tanJ.-9-9986154<br />

b = 69° 34' 55"-9 log cos b 4- 9-5426553<br />

9- = 109° 10' 31"-0* log cot 9- -'9-5412707<br />

0 = 50° 30' 8"-4<br />

G-B- = — 58° 40'22'-6<br />

» -We may also take3- = — 70° 49' 29"-0, whence C—9- = 121° 19' 37"-l, which<br />

will evidently give the same results as those obtained in the text.


188 SPHERICAL TRIGONOMETRY.<br />

By (134). By (135). By (136).<br />

logsin(C—3-) —9 9315664 logcos(C—3-)4-9-7159386 logcot(C'—S-) —9-7843722<br />

arcologsin9--f-0-0247897 ar co log cosa—0-4835187 log tan P4-0-0787962<br />

logcosJ:—9-8501762 log cot J 4-9-5708352 log cos a—9-7055757<br />

log cos P4-9-8065323 log cot a—9-7702925* Glieck. —9-7843719<br />

P= 50°10'10"-0 a=120°30'29"-9<br />

Ans.B= 50°10'10"-0<br />

a=120°30'29"-9<br />

2. Given J. = 135° 5'28"-8, C=50°30'8"-4, 5 = 69°34'55"-9;<br />

find B and c.<br />

Ans.B= 50°10'10"-0<br />

c = 70°20'20"-0<br />

3. Given A = 65° 33' 10", C = 95° 38' 4", b = 100° 49' 30";<br />

find 5 and a.<br />

Ans.B= 97° 26'29"<br />

a = 64° 23'15"<br />

4. Given J. = 97° 26'29", C = 95°38'4", 5 = 64° 23'15";<br />

find B and a.<br />

Ans.B= 65° 33'10"<br />

a = 100° 49' 30'<br />

79. If a, e and B were all required, we might find B and c by<br />

(137), and then a by Art. 3, which gives,<br />

Bin B: sin. J. = sin 6 : sin a<br />

sin ^ sin 5<br />

^„„,<br />

sm a = ^^— (138)<br />

sm i? ^ '<br />

Of the two values of a given by this equation, the proper one is to<br />

be selected, if possible, by the principles of Art. 69.* But as cases<br />

occur in which all the conditions there given are satisfied by both<br />

values of a, it is preferable, in general, to combine (135) and (137),<br />

or to employ the following solution when the three unknown parts<br />

are all to be found.<br />

-*• By Art. 69, -VII., when A differs more from 90° than G, a must be taken in the<br />

same quadrant with A, and all ambiguity is removed. If, then, by A we always<br />

denote that angle which differs more from 90° than the other given angle, we may<br />

always solve this case by means of (187) and (138), without meeting with any difficulty<br />

in determining the quadrant in which c is to be taken.


SOLUTION OF SPHERICAL OBLIQUE TRIANGLES. 189<br />

80. CASE II. Given J,, C and 5. Seco7id Solution ; when the two<br />

remaining sides, or when the three unknown parts are all required.<br />

We have, by Napier's Analogies, (40) and (41),<br />

sin 1 (J. 4- C): sin i (.4 - C) = tan i 5 : tan i {a - c)<br />

.cos 1 (J. 4- C): cos J (J. - C) = tan J 5 : tan J (a 4- c)<br />

whence<br />

tani(a-e) = ^!^iC^^)<br />

sin 1{A+ G) tan I b<br />

1 / , cos i (J. — C)<br />

tani(a4-e)= cos 1 (J. 4- C) tani5<br />

>(139)<br />

which determine J (a — e) and J (a -f c); then the half difi'erence<br />

added to the half sum gives the greater side, and the half difi'erence<br />

subtracted from the half sum gives the less side. If C> -4, we may<br />

-write C — A, e — a in the place of ^ — G, a — e.<br />

We may now find B by either of Napier's Analogies, (42) and<br />

(43), which give*<br />

sin i (a 4- c) , , .<br />

coti-B= • . ^ ^ ^ . tan 1 (^ - C) (140)<br />

sin J (a — e)<br />

cot J ^ = — ^('*'^'')tani(-^+C) -<br />

(141)<br />

cos J (a — c)<br />

EXAMPLES.<br />

1. Given A = : 135° 5'28"-6, C= 50° 30' 8"-6, b •• 69° 34' 56"-2;<br />

find a, G and B.<br />

We have 1{A+ C) = 92°17'48"-6<br />

i(^-C) = 42°17'40"-0<br />

Then, by (139),<br />

i5 = 34°47'28"-l<br />

ar colog sinl (J.+ C)4- 0-0005176 ar co log cosi(J.4- C)-l-3116286<br />

log sinl (J.- (7)4- 9-8279768 log cosi(^- C)4- 9-8690535<br />

log tan I b 4-9-8418527 log tan i b -f 9-8418527<br />

iogtanJ(a — e) 4-9-6703471<br />

i(a-c)= 25° 5' 5"-0<br />

a = 120°30'30"-0<br />

logtani(a 4-e)-1-0225348<br />

J(a4-e) = 95°25'25"-0<br />

e = 70°20'20"-0<br />

* We may also find P by any one of Gauss's Equations, (44), interchanging B<br />

iTiiI C. // and c.


190 SPHERICAL TRIGONOMETRY.<br />

By (140). By (141).<br />

arCO logsini(a -e) 4- 0-3726772 arcologcosi( a - c) 4- 0-0430243<br />

logsini(a 4-c) 4-9-9980523 log cosi( a 4-c)-8-9755171<br />

logtani(J.-C)4- 9-9589234 logtani(^4-C) -1-3111110<br />

logcotJ5 4-0-32C6529 *log cot i .B 4-0-3296524<br />

i.B = 25°,V5"-0<br />

Ans. a = 120° 30' 30"<br />

e= 70° 20'20"<br />

B = 50° 10' 10"<br />

2. Given A = 95° 38'4", C = 97° 26' 29", b = 64° 23' 15";<br />

find a, G and B.<br />

Ans. a= 99° 40'48"<br />

e = 100° 49' 30"<br />

B= 65° 33'10"<br />

81. CASE II. Given A, G and b. Third Solution. When the<br />

third angle B is zlone reqwi'^ed, the computation by (134) is'in most<br />

cases as convenient as «ny -"ther, but there are other methods (corresponding<br />

to those given ifi Art. 75 for finding a) which may occasionally<br />

be serviceable. By jl4) and (15) we have<br />

coaB = — cos (A 4- C) - 2 sin A sin Csin^ J b (142)<br />

cos .B = - cos (J. - ,C) 4-' 2 sin A sin C cos^ i b (143)<br />

the computation of which is similar to that of (130) and (131).<br />

EXAMPLE.<br />

Given A = 95° 38' 4", C = 97° 26' 29", b = 64° 23' 15";<br />

find-B.<br />

By (142).<br />

ij= 32°ir37"-5 log sin^ 1 5 = 2 log sin J 5 9-4531022<br />

A4-C = 193° 4'33" log sin J. 9-9978967<br />

log sin C 9-9963268<br />

log 2 0-3010300<br />

- 2 sin J. sin Csin^ J 5 = - 0-5602162 log 9-7483557<br />

— nat cos (J. 4- C) = 4- 0-9740715<br />

nat cos B = 4- 0-4138553 B = 65° 33' 9"-9<br />

-* For the reasons given in Art. 74, (141) is, in this example, not so accurate<br />

fls (140).


SOLUTION OF SPHERICAL OBLIQUE TRIANGLES. 191<br />

82 In Art. 14, several formulse are given, by which J P may be computed By<br />

(21) and (22) we have<br />

sin' J P = cos' i (A — G) — sin A sin G cos' J b<br />

cos' J P = sin' J {A + G) — sin A sin G sin' J b<br />

which may be adapted for logarithms, thus:<br />

sm"<br />

sin' i^ P : cos' i (A— C) — sin'


192 SPHERICAL TRIGONOMETRY.<br />

The auxiliary cp will be fully determined by (m), being taken between<br />

0 and 180°, and always positive (PI. Trig. Art. 174); but, as<br />

the cosine of an angle is also the cosine of the negative of that<br />

angle [PL Trig. (56)], we may take 0' in (m') either with the positive<br />

or the negative sign, so that c = cp ± cp'- There will thus be<br />

two values of e answering to the same data, both of which will be admissible,<br />

except when


SOLUTION OF SPHERICAL OBLIQUE TRIANGLES. 193<br />

cot 9- = tan A cos b \<br />

cos 9-' = cos 9- tan 6 cot a |- (14T)<br />

C = 9- ± 9-'<br />

J<br />

To find B. We have several methods : Ist, directly by (3),<br />

_ sin A sin b<br />

sm B = -. (118)<br />

sin a<br />

^<br />

which gives two values of B, supplements of each other, corresponding<br />

respectively to the two values of c and C. We shall presently<br />

see how to determine which are the corresponding values of e,<br />

C and B.<br />

2d. In (123), cp has the same value as in (146), and therefore putting<br />

in (123), e — cp = cp', we have<br />

.,^ sin Cp' cot^<br />

cot B = r— (149)<br />

sm (p ^ '<br />

which gives two values of B by the positive and negative values<br />

of cp'.<br />

3d. By (124),<br />

cos B = tan


194 SPHERICAL TRIGONOMETRY.<br />

nse of (149), (150), (151) and (152), it is only necessary carefully to<br />

observe the signs of the several terms.<br />

Checks. Of the various formulae above given for finding B, one or<br />

more may be employed for the purpose of verification. When c and 0<br />

have been found, the most simple check is the following, from (3),<br />

^ = ^ (153)<br />

sm c sm-a ^ -'<br />

which, indeed, might have been employed to find C, after e was<br />

found, and reciprocally, but for the ambiguity attaching to the sines.<br />

85. According to Art. 69, VI., if b differs more from 90° than a,<br />

B must be in the same quadrant as b, and, since but one of the twc<br />

values of B can satisfy this condition, there will be but one solution.<br />

In that case c and C will each be found to have but one admissible<br />

value.<br />

86. The problem will be altogether impossible, when a differs more<br />

from 90° than b, and is yet not in the same quadrant with A. In<br />

such case, we should find that cp 4-cp'> 180°, and $ —cp' 1.<br />

EXAMPLES.<br />

1. Given a =40° 16', J = 47° 44', J. = 52° 30'; find .B.<br />

By (148).<br />

a= 40° 16' arCO log sin a 0-1895350<br />

b = 47° 44' log sin b 9-8692449<br />

4= 52° 30' log sin A 9-8994667<br />

B= 65° 16'35" log sin .B 9-9582466<br />

or B = 114° 43' 25"<br />

2. With the same data, find c and B.<br />

By (146.)<br />

a= 40° 16' logcosa4-9.8825499<br />

}== 470 44' ]og tan b 4-0-0414996 ar co log cos 64-0-1722647<br />

4== ,52° 30' log cos J.4-9-7844471<br />

(p= 33°48'51"-4 logtancp4-9-8259467 log cos cp4-9-9195201<br />

(p'=±19° 30' 29"-0<br />

e,= 53°19'20"-4<br />

".= 14°18'22"-4<br />

log cos (p'4-9'9743250


SOLUTION OF SPHERICAL OBLIQUE TRIANGLES., 195<br />

By (149). Gheck. (150).<br />


19« SPHERICAL TRIGONOMETRY.<br />

5. Given a = 99° 40' 48", b = 64° 23' 15", A = 95^ 38' i"; find<br />

e, C and B.<br />

Ans. c = 100° 49' 30"<br />

C= 97° 26'29"<br />

B= 65° 33'10"<br />

6. Given a = 40° 5' 25"-6, b = 118° 22' 7"-3, A = 29° 42' 33"-8 ;<br />

find e, C and B.<br />

Ans, e=153°38'42"-4 ) T c = 90° 5'41"-0-<br />

C = 160° r24"-4 > a C= 50°18'55"-2<br />

B= 42°37'17"-5 ) .B = 137°22'42"-5<br />

7. Given a = 69° 34' 56", b = 120° 30' 30", A = 50° 10' 10";<br />

find c and C.<br />

Ans. c = 70° 20' 20'<br />

(7= 50° 30' 8"-4<br />

8. Given a = 120° 30' 30", b = 69° 34' 56", A = 50° 10' 10";<br />

find c and C.<br />

^ws. Impossible.<br />

9. Given a = 40°, b = 60°, ^ = 50° ; solve the triangle.<br />

Ans. Impossible.<br />

87. CASE III. Given a, b and A. Second Solution. We find<br />

B by the formula<br />

sin A sin b<br />

ainB =. sm a<br />

and then by Napier's Analogies, (41) and (43),<br />

or by (40) and (42),<br />

^ cos i (^ 4- -B) ^ 1/ , .A<br />

^'^^^^ =cosH^--g) ^"^^''"^^^<br />

cot : *''-Sll^'""i(^ cos J (a — J) + ^)<br />

. J sin J (J. 4- -B)<br />

tant e = -:;—f4--j j^; tan^ (a — b)<br />

^ sml{A — B) ^ ^ ''<br />

(154]<br />

• (155)<br />

m which we employ successively the two values of B, and obtain<br />

two solutions, except when for one of these values the second members<br />

become negative, for J e and J C being less than 90°, their<br />

tangents must be positive.


SOLUTION OF SPHERICAL OBLIQUE TRIANGLES. 197<br />

We leave it to the student to a,pply these formulse to the preceding<br />

examples.<br />

88. To determine by inspection of the data a, b and A, whether there are two solutions,<br />

or but one.<br />

1st. It has already been seen. Art. 85, that when b differs more from 90° than a,<br />

B must be in the same quadrant as b, and there can be but one solution. It remains<br />

to show,<br />

2d. That when a differs more from 90° than b, there will necessarily be two solu<br />

tions. We have, by the first of (4),<br />

cos a — cos b cos c<br />

Bin c = :—<br />

sin b cos A<br />

Two solutions exist so long as both values of c are positive, and less than 180°, that<br />

is, 80 long as sin c is positive. Now when a differs more from 90° than b, we have,<br />

(neglecting the signs for a moment),<br />

cos a > cos b > cos b cos c<br />

therefore the numerator of the above value of sin c has the sign of cos a. But by<br />

Art. 69, \1., a and A are in the same quadrant, and cos a and cos A have the same<br />

sign; consequently also, the numerator and denominator have the same sign, and<br />

the value of the fraction, or of sin c, is positive, as was to be proved.-^<br />

Hence, there is but one solution when the side opposite the given angle differs less from 90°<br />

than the other given side, and two solutions when the side opposite the given angle differs<br />

more from 90° than the other given side.<br />

89. CASE IV. Given two angles and a side opposite one of them,<br />

or A, B and b. (Fig. 9).<br />

First Solution, in which each required Hg.g. a<br />

part is deduced directly from the fundamental<br />

formulse.<br />

To find c. We have, by (10),<br />

sin e cot b — cos e cos ^ = sin ^ cot B<br />

or multiplying by sin b,<br />

sin e cos 6 — cos e sin 5 cos J. = sin A cot B sin b (M)<br />

to solve which we take<br />

fc sin cp = sin b cos A I , s<br />

r ^^<br />

k cos cp = cos 0<br />

I<br />

* The same proposition may be otherwise proved thus. By the equations (m)<br />

»nd (m') Art. 84, we have<br />

cos b COS a , sin b .<br />

Cos^ = - ^ COS0'=-^ A = -^cos^<br />

from the third of which we see that k has the sign of cos ^ ; if then a differs more<br />

from 90° than b, that is, if cos a and cos A have the same sign, cos (p' is positive,<br />

and 0' < 90°. Also since, (neglecting signs), cos a > cos b, we have cos '/ > cos f,<br />

or 'p' differs more from 90° than and 0<br />

and ip+ Ip' < 180°, or both values oi c are between 0 and 180°.<br />

ii2


198 SPHERICAL TRIGONOMETRY,<br />

then, putting e —


SOLUTION OF SPHERICAL OBLIQUE TRIANGLES. 199<br />

To find a. We have several methods : 1st, .directly by (3),<br />

which gives<br />

sin b sin A<br />

2d. By (146), where cp and ip' have the same values as in this case,<br />

3d. By (150),<br />

cos


200 SPHERICAL TRIGONOMETRY,<br />

sin C<br />

sin c<br />

sin B<br />

sin b<br />

(163)<br />

which might have been employed for finding C after c was found, or<br />

reciprocally, but for the ambiguity attaching to the sines.<br />

90. According to Art. 69, VII., if A differs more from 90° than<br />

B, a must be in the same quadrant with A. But since the two<br />

values of a are supplements of each other, only one of them can<br />

satisfy this condition, and there will then be but one solution. In<br />

such case c and C will each be found to have but one admissiblo<br />

value.<br />

91. The problem will be impossible when B differs more from 90°<br />

than A, and yet is not in the same quadrant with b. In such case<br />

we should find both values of c (and both values of C) to be greater<br />

than 180°, or both negative.<br />

The problem will also be impossible when sin 6 sin ^ > sin B,<br />

since by (158) we shall then have sin a > 1.<br />

EXAMPLES.<br />

1. Given A = 132° 16', B = 139° 44', b = 127° 30'; find a.<br />

• By (158).<br />

B = 139° 44' 0" ar co log sin B 0-1895350<br />

A = 132° 16' 0" log sin A 9-8692449<br />

b = 127° 30' 0" log sin b 9-8994667<br />

a = 65° 16' 35"-l log sin a 9-9582466<br />

or a = 114° 43' 24"-9<br />

2. With the same data, find C and a.<br />

By (157).<br />

J5= 139° 44' 0" log cos £-9.8825499<br />

A= 132° 16' 0"logtanJ.-0-0414996arcologcosJ.-0-1722.547<br />

J= 127°30• 0" logcos 5-9-7844471<br />

9-= 56° 11' 8"-6 log cot9-4-9-8259467 log sin 9-4-9-9195201<br />

9-'^=4- 70°29'31"0 1<br />

'<br />

&''=4-109°30'29"-0 I ' • log sin 9-'4-9.9743250<br />

C^= 126°40'39"-6<br />

t7= 165°41'37"-6


SOLUTION OF SPHERICAL OBLIQUE TRIANGLES. 201<br />

By (161). Cheek. (162).<br />

9- = 56° 11' 8"-6 arCO log cos 9-4-0-2545328 log cot 5-0-0720848<br />

tCim° f o m ] ^°S cos&'±9-5236676 log cot y±9-5493427<br />

6 ^=127° 30' 0" log cot 5-9-8849805 rp9.6214275<br />

a' =^65° 16'34"!9 }<br />

^°^ °°* «4=9-6631809 log cos «.F9-6214275<br />

^ws. C=126°40'39"-6 "I T C=165°41'-37"-6<br />

a = 114°43'25"-l j "'^ I a= 65°16'34"-9<br />

3. Given A = 110°, B = 60°, 6 = 50° ; find c and a.<br />

By (156).<br />

B =<br />

A =<br />

b =<br />

60°<br />

110°<br />

50°<br />

logcosJ--9.5340517<br />

logtan6 4-0-0761865<br />

log cot5 4-9-9614394<br />

logtan.4-0-4389341<br />

cp = 157°49'26".4 logtancp-9-6102382 log sin cp 4-9-5768627<br />


202 SPHERICAL TRIGONOMETRY.<br />

J. Given A = 70°, B = 120°, 5 = 80° ; solve the triangle.<br />

Ans. Impossible.<br />

8. Given A = 60°, B = 40°, b=50°; solve the triangle.<br />

Ans. Impossible.<br />

92. CASE IV. Given A, B and b. Second Solution. We find a<br />

by the formula<br />

sin b sin A<br />

sm a ••<br />

sinB<br />

and then by Napier's Analogies we find e and C, precisely as in<br />

Case III., Art. 87, employing successively, in (154) or (155), the<br />

two values of a given by the preceding equation. There will be but<br />

one solution, if one of these values renders the second members of<br />

(154) or (155) negative.<br />

The student should apply this method to the preceding examples.<br />

93. To determine by inspection of the data A, B and b, whether there are two solutiont<br />

or but one.<br />

1st. It has already been seen. Art. 90, that when A differs more from 90° than P,<br />

a must be in the same quadrant with A, and there can be but one solution. It<br />

remains to show that,<br />

2d. When P differs more from 90° than A, there will necessarily be two solutions<br />

We have, by (5),<br />

. „ cos P 4-cos J4 COS C<br />

sm C = +-: J<br />

sm A COS 0<br />

Two solutions exist so long as both values of G are less than 180°, and both positive,<br />

that is,BO long as sin G is positive. Now when P differs more from 90° than A, we<br />

have, (neglecting signs for a moment),<br />

COB P > COB A > COB A COB O<br />

therefore the numerator of the value of sin G has the sign of cos P. But by<br />

Art. 69, -YIL, P and b are in the same quadrant, consequently the numerator and<br />

denominator have the same sign, and the value of the fraction, or of sin C is always<br />

positive, as was to be proved.*<br />

Hence, there is but one solution when the angle opposite the given aide differs less from<br />

Q0° than the other given angle; and two solutions when the angle opposite the given aide<br />

differs more from 90° than the other given angle.<br />

94. CASE IV. might have been reduced to Case III. by means of the polar triangle<br />

of Art. 8. For there will be known in the polar triangle two sides and an angle<br />

opposite one of them, being the supplements of the given angles and side of the<br />

proposed triangle. The polar triangle being solved, therefore, by Case III., and its<br />

twc remaining angles and third side found, the supplements of these parts will be<br />

the required sides and third angle of the proposed triangle.<br />

-*• It may be shown that both values of C will be admissible, by a process of reasoning<br />

similar to that employed in the note on page 197, applied to the equations<br />

of Art 8'.>.


SOLUTION OF SPHE.IICAL OBLIQUE TRIANGLES.<br />

95. CASE V. Given the three sides, or a, b<br />

and e. (Fig. 9.) We have three methods<br />

for computing the half angles :<br />

ist. By the sines, from (31), remembering<br />

that<br />

s = 1 (a 4- 6 4- e)<br />

sin IA=\ ('^^^i^3^{LriA\<br />

^ \ sin 6 sm e /<br />

sin J 5 •<br />

sin i C<br />

2d. By the cosines, from (33),<br />

sin (s — c) sin (s — a)^<br />

sin e sin a /<br />

sin (s - a) sin (s — 5)^<br />

sin a sin b<br />

Mg. 9.<br />

203<br />

(164)<br />

cosi^= //Z^^^^^MfJ^N<br />

^ >/ \ sin 6 sin e /<br />

sin (s — 5)<br />

cos J 5 >J V SI<br />

sm e sm a<br />

cos J'^-JC<br />

sin s sin (s — e)<br />

sin a sin 6<br />

3d. By the tangents, from (34),<br />

tan IA _<br />

/ , sin (s — b) sin {s — c)\<br />

*>* V sin s sin {s-a) '<br />

tan»5=/(^-^f-^)f^(\-^))<br />

b)<br />

tan|C=/C(f~^)^;°(^7Jl)<br />

^ V sin sm s .s sin am (« ix — c) el<br />

'<br />

(165)<br />

(166)<br />

When only one of the angles is required, the simplest method will<br />

be by (165), but if the required angle is less than 90°, it will be<br />

found more accurately by (164), for then ^ A


204 SPHERICAL TRIGONOMETRY.<br />

No ambiguity can arise in these solutions, since the half angles<br />

must be less than 90°; they require therefore no attention to tho<br />

algebraic signs.<br />

EXAMPLES.<br />

1. Given a = 100°, b = 50°, e = 60° ; find A.<br />

a =100°<br />

b= 50°<br />

e= 60°<br />

2 8 = 210°<br />

s = 105°<br />

8 — a= 5°<br />

IA= 69°<br />

A = 138°<br />

7'52' -7<br />

15'45" •4<br />

2. With the same data, find all the angles.<br />

By (166).<br />

log cosec 0-1157460<br />

log cosec 0-0624694<br />

log sin 9-9849438<br />

log sin 8-9402960<br />

2)9-1034552<br />

logcos 9-5517276<br />

8=105° 1. cosec 0-0150562 L cosec 0-0150562 h cosec 0-0150562<br />

s-a= 5° h cosec 1-0597040 1. sin 8-9402960 h sin 8-9402960<br />

8-5=55° 1. sin 9-9133645 1. cosec 0-0866355 h sin 9-9133645<br />

s-c^ 45° 1. sin 9-8494850 1. sin 9-8494850 L cosec 0-15051-50<br />

IA=<br />

2)0-8376097 2)8-8914727 2)9-0192317<br />

1. tan 0-4188049 1. tan 9-4457364 \. tan 9-5096159<br />

69° 7'52"-7 i5=15°35'37"-0 JC= 17°54'59"-1<br />

Ans. J. = 138°15'45"-4 .B=31°ll'14".0<br />

C= 35° 49'58"-2<br />

8. Given a = 10°, J = 7°, e = 4°; find the angles.<br />

Ans.A = 12ii°4:4:'i5"-l<br />

B = 33° 11' 12"-0<br />

C= 18°15'31"-1<br />

96. The method by (166), may be put under the following convenient form. Let<br />

I/Bin (s — a) sin (s — J) sin (s — c)\<br />

N \ sin« /<br />

then . (167)<br />

P P P<br />

ia.niA = -^— r, tanJP= -. — y-, tanJC = -;—-^ ^<br />

sm (a — a) •* sin (a — b) ^ sin (s — c)<br />

which are similar to the formulae of PI. Trig. Art. 146, and are computed in the same<br />

manner.


SOLUTION OF SPHERICAL OBLIQUE TRIANGLES. 205<br />

97. CASK V. Given a, b and c. Second Solution. If the whole angle is re.juired<br />

directly,* we have<br />

cos a '— cos b cos c<br />

coa A =<br />

sin b sin c<br />

which may be adapted for logarithms by an auxiliary thus:<br />

Or thus,<br />

cos


2Q6<br />

SPHERICAL TRIGONOMETRY.<br />

SOLUTION OF OBLIQUE SPHEKIOAL TEIANQLES BY MEANS OF A PERPENDICCHB.<br />

101. All the cases of oblique spherical triangles may be solved by dividing tho<br />

triangle into two right triangles by a perpendicular from one of the vertices to the<br />

"riposite side, and solving these partial triangles by the methods of the preceding<br />

chapter. Bowditch has given two rules, based upon Napier's Rules, (Art. 40), by<br />

which the application of this method is facilitated.<br />

102. Bowditch's Rules for Oblique Triangles. "If in a<br />

Hg. 10. Q spherical triangle, (Fig- 10), two right triangles aro<br />

formed by a perpendicular let fall from one of its vertices<br />

upon the opposite side ; and if, in the two right<br />

triangles, the middle parts are so taken that the perpen-<br />

\j^ dicular is an adjacent part in both of them ; then<br />

The sines of the middle parts in the two triangles are proportional<br />

to the tangents of the adjacent parts.<br />

But if the perpendicular is an opposite part in both the triangles, then<br />

The sines' of the middle parts are proportional to thecosines of the opposite parts.<br />

To prove which rules, let M denote the middle part in one of the right triangles,<br />

A an adjacent part, and 0 an opposite part. Also, let m denote the middle part in<br />

the other triangle, a an adjacent part, and o an opposite part; and \ot p denote the<br />

perpendicular.<br />

First. If the perpendicular is an adjacent part in both triangles, we have, by<br />

Napier's Rules, (Art. 46,)<br />

whence<br />

or<br />

sin M = tan yt tan ^<br />

sin m = tan a tan p<br />

sin if tan A tan p tan A<br />

sin m tan a tan p tan a<br />

sin Hf: sin m = tan A : tan a<br />

Secondly. If the perpendicular is an opposite part in both triangles, we have, bj<br />

Napier's Rules<br />

sin M = cos 0 cos p<br />

whence<br />

sm m = cos o coap<br />

Bin M cos 0 cosjo cos 0<br />

sin m cos o coap cos o<br />

or sin M: sin m = cos 0 : cos o" *<br />

We proceed to solve the six cases of spherical triangles with the aid of a perpen-<br />

•iicular. It will be seen, however, that Bowditch's Rules are applicable but in the<br />

first four cases.<br />

•* Peirce's Spherical Trigonometry, Art. 44.


SOLUTION OF SPHERICAL OBLIQUE TRIANGLES.<br />

'•ZOI<br />

i03. CASE I. Given b, c and A. Let the perpendicular G P, Fig. 10, be drawn<br />

from C, (that is, in such a manner as to put two given parts in one of the right<br />

triangles). Then the right triangle AGP gives, by Napier's Rules, if we put<br />

AP = Ip,<br />

tan *= tan 6 cos .4 (112)<br />

then talking co. b and co. a as middle parts in the two triangles, A P =i p and<br />

B P= c —-1^ * are the opposite parts, whence, by Bowditch's Rules,<br />

whence<br />

cos Ip : cos (c —ip) =<br />

cos b : cos a<br />

cos (c — *) cos 5<br />

cos a = !^ Li (173)<br />

cos 0 ^ '<br />

Again, taking A P and PB as middle parts, co. A and co. P are adjacent parts,<br />

whence, by Bowditch's Rules,<br />

whence<br />

.sin 0 : sin (c — ip) = cot ^ : cot P<br />

, „ sin (c — (P) cot A<br />

cot P = !^ r^ (174)<br />

sm 0 * '<br />

and the formulae (172), (173), (174), agree entirely with (122) and (123).<br />

The triangle P GP gives as a check<br />

tan a cos P = tan (c —


208 SPHERICAL TRIGONOMETRY.<br />

Again, taking co.^CPandco.PCPas middle parts, and therefore co. 6 and co. •<br />

us adjacent parts, Bowditch's Rules give<br />

whence<br />

cos S : cos (G — S) = cot b : cot a<br />

cos (C — S) cot t<br />

cot a ^ (178)<br />

and (176), (177), (178), agree entirely with (134) .and (185).<br />

The triangle B G P gives<br />

tan P cos a = cot (C— 3)<br />

(179)<br />

which agrees with (136).<br />

By drawing the perpendicalar from A, we may in the same manner obtain the<br />

formuloB (137).<br />

The side c may be found from the proportion<br />

sin A : sin G = sin a : sin o<br />

and Art. 69 ; or c being found by means of a perpendicular from A, we may find a<br />

by a similar proportion.<br />

105. CASE III. Given a, b and A. Let the perpendicular<br />

be drawn from G, Fig 10, as in the preced­<br />

Fig.10.<br />

ing cases, and let A P = ip, B P=


SOLUTION OF SPHERICAL OBLIQUE TRIANGLES. 2011<br />

and by Bowditch's Rules,<br />

whence<br />

and since in Fig. 11,<br />

cot b : cot a = cos ^ ; cos 3'<br />

cos 3' 7= COS 3 tan 4 cot a (184)<br />

C = ^CP = ^CP 4- BCP = 5 + y, oT G = ACB' = AGP — B'CP = 3 — 3',<br />

we have<br />

C = 3 ± .y (185)<br />

and the formulse (180), (181), (182), (183), (184), (185), agree entirely with (146)<br />

and (147).<br />

After c was found, we might have found Cfrom the proportion<br />

and B is found from the proportion<br />

sin a : sin c = sin 4 : sin G (186;<br />

sin a : sin J = sin .A : sin P (187)<br />

The two values of P determined by (187), are both admissible when c has two values<br />

as above. It is also evident, from Fig. 11, that the two values of P are supplemental.<br />

To determine the corresponding values of c and P, we observe that, by Art. 49, the<br />

perpendicular CP is in the same quadrant with A and with CBP and OB'P, and<br />

therefore GB'A is in a different quadrant from A. Hence, that value of P lohich is in<br />

the same quadrant as A corresponds to the value of c = ip + ip', and that value of B<br />

which is in a different quadrant from A corresponds to the value of c = ip —^ 0'; which<br />

agrees with what is shown in Art. 84.<br />

In computing (186), the two values of c must be employed successively, and the<br />

formula computed twice. At each computation we shall have two values of G found<br />

from the sine, one of which must be selected by Art. 69. But as the application of<br />

the principles of Art. 69 is tedious and embarrassing, it is better to find C by (184)<br />

and (185).<br />

The formulije (149), (150), (151), (152), for finding P, may easily be deluced by<br />

Napier's and Bowditch's Rules.<br />

106. CASE IV. Given A, B and b. Let the perpendicular be dra-wn as before,<br />

Fig. 10, and let AP = ip, BP = ip', then as before,<br />

and by Bowditch's Rules,<br />

whence<br />

tan 0 = tan b cos A (188)<br />

cot ^ : cot P = sin 'P : sin /<br />

sin 0' = sin 0 tan .4 cot P 1 ,.„„,<br />

c = 0 -t- 0' / ^ ^<br />

which agree with (156). But 0' having two supplemental values determined by the<br />

sine, e has two values, as already explained in Art.<br />

To show the same geometrically, let BP,<br />

Fig. 12, be the acute value of ip', and about<br />

(7 as a pole, let a small circle be described<br />

passing through P, and intersecting the ^"^<br />

great circle AB again in B". Let P"Cbe<br />

drawn, and produced to meet AB again in<br />

B', foi:ming the lune B"B'. Then we have<br />

P' = P" =<br />

27 s 2<br />

OBA


210 SPHERICAL TRIGONOMETRY.<br />

Kg. 12.<br />

In the triangle AGB'vc have<br />

C =<br />

and in the triangle A CB' we have<br />

so that in both triangles, -4e'P and ACB,<br />

the value of the angle opposite the side i<br />

.jji is the same, that is, both triiingles contain<br />

the same data. A, B and b. Now<br />

180° = B"B' = B"P + B'P =zBP\- B'P,<br />

BO that BP and B'P are supplements of<br />

each other.<br />

lp+i<br />

= ^P 4- PP<br />

c = 0 4- 0' = ^P 4- B'P<br />

and hence the two values of c are found by giving 0' its acute and obtuse values<br />

successively, as already shown analytically.<br />

By Art. 49, CP must be in the same quadrant with A; hence, if P is in the same<br />

quadrant with A, P falls between A and B, as in the figure, and for the same reason,<br />

between A and P'. But if A and P were in different quadrants, both points,<br />

P and P', might fall between A and P. The two values of c would then be found<br />

by the formula<br />

c =


SOLUTION OF SPHERICAL OBLIQUE TRIANGLES. 211<br />

107. CASE-v. Given a, i and c. The perpendicular Kg. 13.<br />

cannot be drawn, in this case, so that two of the given<br />

parts shallbe in one triangle; nevertheless the case can<br />

be solved by means of a perpendicular. Let the perp.<br />

be drawn from any angle, as G, Fig. 13, and as before,<br />

put^P = 0, BP = 0'; then by Bowditch's Rules,<br />

cos 0 : cos 0' = cos b : cos a<br />

, cos 0' — cos I<br />

Whence cos 0' -j- cos 0 cos a + cos b<br />

or, by PI. Trig. (110),<br />

whence, since 0 -|- 0' = c,<br />

tan \(


212 SPHERICAL TRIGONOMETRY.<br />

Downey's Table XXII., with the argument log 2:, the difference of tiis given logarithms,<br />

gives log (1 4" 2;), wliich being added to log q, the less logarithm, gives the<br />

required log. sum, or log (p + q). Table XXIII., with the argument log x, gives<br />

log M -|<br />

) which, being added to log^, the greater logarithm, gives the required<br />

log. sum. Either table may, in general, be employed, but one or the other may be<br />

found more convenient in a particular application, and therefore both are given<br />

Again, we have<br />

p-q=p{l-l)<br />

p<br />

BO that, putting, as before, z = —, we have<br />

log z = log jci — log q<br />

log (p — q) = logp + log {l ——)<br />

Downes's Table XXIV., with the argument, log z, gives log (1<br />

j which, being<br />

added to the greater logarithm, gives the required log. difference, or log (p — q).<br />

With these tables, then, we may readily compute any of the preceding formulso<br />

which contain two terms in the second member, without the aid of auxiliary angles<br />

EXAMPLES.<br />

1. Given b = 120° 30' 30", c = 70° 20' 20", A = 50° 10' 10" ; find a. (Sams<br />

as Ex. 1. p. 182).<br />

The formula is<br />

cos a = cos b cos c 4- sin 5 sin e cos A<br />

which will be thus computed :<br />

log COB b — 9-70557<br />

log cos c + 9-52693<br />

log q — 9-28250<br />

log sin b + 9-93529<br />

log sin c + 9-97891<br />

logcos^ 4- 9-80054<br />

log^ 4- 9-71574<br />

log.? — log 2 = log X — 0-48324<br />

The terms p and q have opposite signs, and although, by the formula, they are to be<br />

added (algebraically), an arithmetical difference is required. By marking the signs<br />

of all the quantities, as above, we shall always know whether a sum or difference<br />

is required by the sign before log z. In this case this sign being negative, we are<br />

to find a difference, and therefore, by Table XXIV., we take<br />

log^l ^ 9-82694<br />

log;; 4- 9-71574<br />

log cos a + 9-54268<br />

a = 69° 34' 52"


SOLUTION OP SPHERICAL OBLIQUE TRIANGLES. 213<br />

2. With the same data, find P. The formula is<br />

which mast here be put under tlie form<br />

and is thus computed:<br />

or, by Table XXIII., we have<br />

. „ sin c cot b — cos c cos A<br />

•cot P =: :—;<br />

sm A<br />

^ „ sin c cot S<br />

cot B = —T—3<br />

sva. A<br />

(which is to be added to the greater log.)<br />

cos c cot A<br />

log sin c + 9-97391<br />

log cot b — 9-77029<br />

arCO log sin^ -j- 0-11467<br />

log p — 9-85887<br />

log (— cos c) — 9-52693<br />

log cot ^4- 9-92121<br />

log q — 9-44814<br />

logp — log g- = log z + 0-41073<br />

where the sign before log z being positive, the tables for log. sum must be used. By<br />

Table XXII., we have -<br />

log (1 4- z) 0-55325<br />

(wliich is to be added to the less log.)<br />

logq — 9-44814<br />

log cot P — 0-00139<br />

0-14252<br />

logp -- 9-85887<br />

log cot P — 0-00139<br />

P = 135° 5'81"<br />

In these isolated examples, the labor of computation is very little less than with<br />

the use of an auxiliary angle, as on p. 182 ; but the Gaussian Table has greatly the<br />

advantage when the same formula is to be repeatedly computed with successive<br />

values of one of the data while the others remain constant. Thus, in the first of<br />

the preceding examples, if successive values of a are to be found corresponding to<br />

successive values of A, while b and c are constant, log q will be constant, and log x<br />

•will take successive values, corresponding to those of log cos A, so that .after the<br />

first value of a is found the succeeding ones are rapidly obtained. On the other hand,<br />

as the auxiliary 0 in the formulse (122), depends upon A, the whole process would<br />

have to be repeated in finding each value of a.<br />

For other forms of the Gaussian Table, see the original table, (to five places of<br />

decimals), by Gauss, published in Zaoh's Mona.tUche Gorrespondenz, Nov. 1812; Matthiessen's,<br />

(to seven places), Altona, 1817 ; in Ye'ga,'a Sammlung maihematischer Tajkln,<br />

(five places), Leipzig, 1840; Zech, (seven places), Leipzig, 1849; Shortrede's<br />

C.'llection of Tables, (seven places), Edinburgh, 1849 ; Gray's Tables for the Computation<br />

of Life Contingencies, (six places), London, 1849; Schumacher's Hiilfstafeln,<br />

new ed. (four places).


214 SPHERICAL TRIGONOMETRY.<br />

CHAPTER IV.<br />

SOLUTION OF THE GENERAL SPHERICAL TRIANGLE.<br />

109. WE have thus far, following the usual coui-se, considered those spherical<br />

tiiangles only whose sides and angles are less than 180°. In the applications of this<br />

subject in astronomy, however, it is often necessary to consider triangles whose<br />

sides or angles exceed 180°. (For example, the right ascension of a heavenly body,<br />

admitting of all values from 0° to 360°, may be one part of such c triangle). Vfc<br />

may, it is true, in such cases, always substitute another triangle whose parts are the<br />

supplements to 180° or 360° of those of the proposed triangle; but this modcr<br />

although very generally regarded as the simplest, is not really so in the cases<br />

alluded to. The construction of figures for discovering the supplemental triangles<br />

is often embarrassing and liable to mistake, while the solutions, when obtained, are<br />

mostly deficient in generality, and can only be regarded as solutions of the particular<br />

cases of a general problem. But if we proceed by a method that is as applicable<br />

when the parts of the triangle exceed as when they are less than 180°, we may investigate<br />

a problem under the simplest supposition of the values of these parts, and<br />

rely upon the generality of the method to cover all the particular cases.<br />

110. We shall first endeavor, in an elementary manner, to give the student a conception<br />

of the nature of the general spherical triangle.<br />

ng. 14.<br />

Let^PC7, Fig. 14, be any spherical triangle whose partt<br />

are all less than 180°; then the remainder or complemen*<br />

of the sphere is also a spherical triangle whose sides are<br />

II, b und c, iind whose angles are 860° — A, 860° — B<br />

and 860° — C. We shall distinguish these triangles from<br />

each other by means of accents, writing the letters wiilm<br />

the triangle to which they respectively belong, as ip<br />

Fig. 14. The sides are common, but when referred to ne<br />

sides of A'B'G', they will be denoted by a', b' and c'.<br />

Again, one of the sides may exceed 180°, as the side a of the triangle ABG,<br />

Fig. 15. In this triangle, it is evident that we must have A > 180°, so long as P,<br />

C, b and c are each < 180°. In the triangle A'B'G' we have A' < 180°, whil(<br />

B' > 180°, C" > 180°.<br />

Kg. 16<br />

If we next suppose two of the sides to exceed 180°, as a and b. Fig 16, these sides<br />

intersecting in two points whose distance is 15^0°, the figure ceases to present the


SOLUTION OF THE GENERAL SPHERICAL TRIANGLE. 215<br />

triangle as an enclosed surface, but it will presently appear that such triangles are<br />

solved by tho same general methods that apply in other cases. To form a just conception<br />

of the triangle in this case, we may conceive Fig. 16 to be obtained from<br />

Fig. 15 by carrying the point A along the arc CA produced until it crosses the side a<br />

the points ^1 and P may then be joined either by an arc less than 180°, as in Fig. 16,<br />

or by its supplement to 360°, as in Fig. 17, in Fig. 17<br />

which last case every side exceeds 180°. In •^'•'fc -•^'<br />

these figures, to avoid confusion, the point A p, /^^\ IA\<br />

is not placed in its true position according to ,d / \ /<br />

perspective. I \ \ /<br />

In each figure we have two triangles, whose N^ »'-'<br />

sides are common, and whose angles are sup- \ ^ —-w^<br />

plements to 360°. It will be easy to trace the \ \ /<br />

two triangles signified by ^PC and ^'P'C, by \^ ^^^y y '<br />

remarking that the letters in each case are all ^ ^^--^.^^^-^<br />

on the same side of the perimeter of the triangle.<br />

We may go farther, ana suppose the arc joining A and P to be a circumference<br />

+ the arc ^4P, or any number of circumferences 4- -^B; and similarly the angles<br />

may be supposed to be altogether unlimited; but since the relative positions of any<br />

three points of the sphere must be fully determined by arcs and angles less than<br />

300°, nothing is gained by passing beyond this limit.<br />

111. All the formulce of Chapter I. are applicable to the general spherical triangle.<br />

This proposition might be considered as established by the principle of PI. Trig.<br />

Art. 49, but it is also very easily established by a continuation of the process of<br />

Spher. Trig. Art. 6, where tho fundamental equation was shown to apply to all<br />

triangles whose parts are less than 180°.<br />

It was proved in Art. 29, that all the equations of Chap. I. may be deduced from<br />

the fundamental one,<br />

cos a = cos b cos c + sm b sin c cos A<br />

We have then only to prove the generality of this single equation.<br />

Ist. Let all the sides be < 180, but A' >180°, Fig. 14. The formula being true for<br />

the triangle ABC, we have<br />

cos a = cos 5 cos c + sinb sin c cos (860° — A')<br />

or in the triangle A'B'G', by PI. Trig. (76),<br />

cos a' ^<br />

cos V COB c' + sin b' sin c' cos A'<br />

2d. Let «> 180°, Fig. 15, and produce a to complete the great circle. The triangles<br />

ABC and A'B'G' are respectively the difference and sum of " hemisphere and the<br />

triangle A'ik, all of whose parts are < 180°. In the triangle A'ik we have, in terms<br />

of the parts of ABG,<br />

cos (360° — a) = cos i cos c 4" sin b sin c cos (360° — A)<br />

and in terms of the parts of A'B'G',<br />

cos (360° — a') = cos V cos c' 4- sin b' sin d cos A'<br />

both of which reduce to the form (M). But it is here necessary to show that the<br />

formula may also be applied to each of the other angles: thus the triangle A'ik<br />

COB b = cos (360° — a) cos c + sin (360" — a) sin c cos (180° — P)<br />

cos b'= cos (860° —a') cos c'+ sin (860° — a') sin c'cos (P' —180°)<br />

both of which reduce to the form (M).<br />

(M)


216 SPHERICAL TRIGONOMETRY.<br />

3d Let -I > 180°, i>180°. Fig. 18: those arcs intersect at (, and the triangle<br />

A' B' I gives<br />

Fig. 18.<br />

cos (a — 180°) = cos (b — 180°) cos c + sin (5 — 180°) sin c cos (300° — A)<br />

cos (a'— 180°) = cos (V — 180°) cos c' + sin (b' — 180°) sin c' cos A'<br />

which reduce to the form (M) ; and in the same way the formula applies to the angle<br />

P. We have also<br />

cos c = cos (a -^ 180°) cos (b —180°) 4- sin (a —180°) sin (5 — 180°) cos i<br />

and since cos i = cos C = cos (860° — C") = cos G', this also reduces to the form<br />

(M) for both ABC and A' B' C.<br />

4th. Let a > 180°, b > 180°, c > 180°, Fig. 19; the side e being produced to<br />

complete the circle, the triangle ikl gives<br />

cos (a —180°) = cos (b —180°) cos (860°—c) + sin (b — 180°) sin (360° — c) cos I<br />

and since cos I = cos (180° — A) = — cos A = cos (A' —180°) = — cos A', this<br />

reduces to the form (M) for both ABG and A'B' G'; and in the same way the formula<br />

applies to the angle P. We have also<br />

cos (360° — c) = cos (a — 180°) cos (b —180°) 4- sin (a — 180°) sin (b —180°) cos i,<br />

and since cos i = cos G = cos C", this reduces to the form (M) for both ABO<br />

and A' B' C.<br />

The cases in which the angles or sides exceed 360° are included in the preceding,<br />

in consequence of PI. Trig. Art. 45.<br />

112. The preceding demonstration, though tedious, has the advantage of giving a<br />

definite conception of the figures which our formulse represent. But perhaps the<br />

most satisfactory (as it is the most elegant) method, is to rest the demonstration<br />

of our fundamental equations themselves upon the principles- of analytical<br />

geometry, and, for the sake of those who are acquainted with that subject, we add<br />

the following investigation:<br />

Any point of the sphere may be referred by rectangular co-ordinates to three<br />

planes passing through the centre of the sphere at right angles to each other. Let<br />

O be the centre of the sphere. Fig. 20, and ^ P C a spherical triangle upon its<br />

surface. Let one of the co-ordinate planes, as JT Y, coincide with the gr«at cir-<br />

•i\e A P. and let the axis of JTpass through P. If CP be drawn perpendicular


SOLUTION OP THE GENERAL SPHERICAL TRIANGLE.<br />

to the plane XT, and GP' and PP' to the axis OX, the oo-ordinat-s of the<br />

pomt G are<br />

X = OP', y = PP', z=GP<br />

rig. 20. I'ig. 21.<br />

the values of which (0 C being taken = 1) are<br />

X = cos a<br />

y = sin a cos P<br />

2 = sin a sin P<br />

If now the axis of X be made to pass through A, Fig. 21, without changing<br />

the position of the plane X F we shall have for z', y', z', the co-ordinates of Creferred<br />

to the new axes,<br />

x' = cos b<br />

y' = — sia b cos A<br />

z' = sin b sin A<br />

The axis of z being unchanged, the relations between z', y', and x, y, are expressed<br />

simply by the formulae for the transformation of co-ordinates in a plane; the inclination<br />

of the new axes to the first is here expressed by c, and the formulse of<br />

transformation are therefore<br />

X = z' cos c — y' sin c<br />

y = x' sin c + y' cos c<br />

substituting the values of the co-ordinates, we have at once the three following<br />

' indamental equations :<br />

cos a = cos c cos b + a\n c sin b cos A<br />

sin a cos P = sin c cos b — cos c sin b cos A<br />

(N)<br />

sin a sin P ^<br />

sin b sin A<br />

which are identical with (4), (6), and (3).<br />

113. Having established the complete generality of our fundamental equations,<br />

we may now employ for the solution of the general triangle any of those deduced<br />

from them in Chap. I.<br />

As a single trigonometric function is not sufEcient to determine an unlimited<br />

angle or arc, (PI. Trig. Art. 53), it becomes necessary in most oases to deduce expressions<br />

for both the sine and cosine of the required part.<br />

It will be found that all the six cases of the general triangle admit of two solutions,<br />

but that they all become determinate, when, in addition to the other data, the sign of<br />

the sine cr msine of one df the required parts is given. In the practical applications in<br />

Bstronomj', 't mostly happens that the conditions of the problem supply this sign.<br />

28 T


218 SPHERICAL TRIGONOMETRY.<br />

114. CASE I. Given b, c and A. First Solution; when one of the remaining angles,<br />

as B, and the third side a are required. The relations between the given and<br />

required parts are<br />

cos a = cos c cos b + sin c sin b cos A 1<br />

sin a cos P = sin c cos b — cos c sin b cos A<br />

Y (l^*')<br />

sin a sin B =i sin i sin jl J<br />

The signs of the second members will be known from their computed numerical<br />

values; the sign of cos a is therefore known. If the sign of sin a ia also<br />

given, the quadrant in which a must be taken will be known ; the second and third<br />

equations will determine the sign of the sine and cosine of B, and therefore the<br />

quadrant in which P is to be taken.<br />

In like manner, if the sign of either cos P or sin P is given, that of sin a becomes<br />

known, and the problem is determinate. If no conditions are atttached to the required<br />

parts, there must be two solutions.<br />

The numerical solution will be conducted as follows: The values of the second<br />

members (or simply their logarithms) are to te separately computed, and their<br />

signs carefully noted; then the quotient of the 3d by the 2d (or the difference<br />

of their logs.) will give tan P, and hence P, which will be taken in the quadrant<br />

indicated by the signs of the sine and cosine. Then the 3d divided by sin ,P,<br />

or the 2d by cos P, will give sin a, which, agreeing with tho value from the<br />

1st equation, will serve to verify the correctness of the whole process.<br />

This solution may be adapted for logarithms by the methods employed in the preceding<br />

chapter.<br />

1st. Let k and 0 be determined by the equations<br />

k sin 0 :^ sin 6 cos A<br />

k cos 0 = cos b<br />

k being a positive number (PI. Trig. Art. 174); then<br />

cos a ^ k cos (c — 0)<br />

sin a cos B = k sin (c — 0)<br />

sin a sin P = sin S sin A<br />

[ (197)<br />

2d. Eliminating k, and taking 0 < 180°, (PI. Trig. Art. 174),<br />

tan 0 = tan S cos A (ip < 180°)<br />

cos b<br />

cos a = cos (c — 0)<br />

„ cosb . ,<br />

Bin a cos P = sm (c — 0)<br />

OOS0 ^ '<br />

sin a sin P = sin b sin A<br />

\ (198)<br />

3d If tli quadrant in which a, is to be taken is given, we may give the preceding<br />

equations the following form:<br />

tan 0 = tan b cos A (0 < 180°)<br />

tan a cos P = tan (c — 0)<br />

sin 0 tan A<br />

tan a sin P =: cos ( C 0)<br />

(19i>;


SOLUTION OF THE GENERAL SPHERICAL TRIANGLE. 219<br />

4th. If both a and b are less than 180°, as not unfrequentlj happens in tho applications<br />

of this problem, let<br />

k<br />

sin b<br />

sm a<br />

tht-u jrj and n are both positive numbers [k being positive) and (197) gives<br />

Check. We find<br />

m sin 0 = cos A<br />

m. cos 0 = cot b<br />

n sin P = sin 0 tan A<br />

n cos P = sin (e — 0)<br />

cot a = cot (c — 0) cos P<br />

(200)<br />

Bin (c — 0)<br />

ain 0<br />

cos (c — 0)<br />

cos 0<br />

sin a cos P<br />

sin b cos >4<br />

cos a<br />

cos b<br />

tan ^<br />

tan B<br />

(201)<br />

besides which we may employ, in connection with (200), the equation sin a sin P<br />

= sin 4 sin ^ ; or in connection with (197) or (198) the equation tan a cos P =<br />

tan (c — ip). Or when (197) and (198) are employed, we may find a both by its<br />

sine and its cosine.<br />

115. The angle C may be found in the same manner as P, interchanging P and G,<br />

b and c, in the preceding formulie. But when P and 0 are both required, the Second<br />

Solution to be given presently is preferable.<br />

EXAMPLE.<br />

Given A = 261° 16' b = 45° 54', c = 138° 32', and a < 180°; to find a and P.<br />

We shall first employ (197). The first column of the following computation, con<br />

taining the symbols expressing the operations to be performed, should be prepared<br />

before opening the tables:<br />

A 261° 16'<br />

b 45° 54'<br />

c 138° 32'<br />

log sin .4 —9-9949352<br />

logcos^ —9-181.3744<br />

• log sin b + 9-8562008<br />

log cos J = log/c cos 0 + 9-8425548<br />

log sin i cos ^ = log/c sin ip — 9-0375752<br />

log tan 0 — 9-1950204<br />

log cos 0 4- 9-9947336<br />

log* 4- 9-8478212<br />

0 351° 5'42"-6<br />

*c —0 147°26'17"-4<br />

' As 0 > c, we take c == 138° 32' 4- 360°, so that c — 0 may be a.positive angle;<br />

but it would be equally convenient to take c — 0 = — 212° 33' 42"-6


220 SPHERICAL TRIGONOMETRY.<br />

log sin (c — 0) 4- 9-7309514<br />

log cos (c — 0) — 9-9257303<br />

log A cos (c-r-


SOLUTION OF THE GENERAL SPHERICAL TRIANGLE.<br />

22 i<br />

Same as in Art. 115. a < 180°.<br />

EXAMPLE.<br />

A<br />

b<br />

e<br />

J (* -


222 SPHERICAL TRIGONOMETRY.<br />

Adapting (208) for logarithms, we find<br />

Ist.<br />

*k sin 9 z= cos A<br />

k cos 9 = sin A cos b<br />

cosP= ksin (C —9)<br />

sin P cos a z= A; cos (0 — 9)<br />

sin P sin a = sin A sin b<br />

2d.<br />

cot 9 = tan A cos b<br />

cosP =<br />

sin P cos a -.<br />

cos A<br />

sin (C —3)<br />

sin 3<br />

cos A<br />

cos (C —3)<br />

sin 3<br />

sin P sin a = sin A sin d<br />

3d. When th quadrant in which B is to be taken is given<br />

cot 3 = tan A cos b<br />

tan P cos a = cot (C — 3)<br />

tan 6 COB 9<br />

tan P sm a = -. ~ -= -<br />

sm (C — 3)<br />

(k positive)<br />

(3 < 180°)<br />

(3 < 180°)<br />

(204)<br />

y (205)<br />

(206)<br />

4th. When A and B are both less than 180°, let<br />

k<br />

P = sin A<br />

sin P<br />

I<br />

thenj7 and q are positive numbers, and we have from (204),<br />

Checks. We have<br />

cos<br />

/) sin 3 =: cot A<br />

p cos 9 =: cos b<br />

q sin a = tan b cos 3<br />

q cos a = cos (G — 3)<br />

cotP= tan (0—3) cos a<br />

(Ccos<br />

3<br />

sin (C-<br />

s)<br />

sin 3<br />

s)<br />

sin P COB a<br />

sin .4 cos b<br />

COS P<br />

COS A<br />

tan b<br />

tan a<br />

^•(207)<br />

L (208)<br />

'* The same factor k is used here and in (197), although the auxiliaries 0 and 9 are<br />

different. To show that k has the same value in (197) and (204), let the squares of<br />

the equations<br />

A sin 0 = sin b cos A<br />

k cos 0 = cos b<br />

be added ; we find<br />

F (sin" 0 -f- cos' 0) = F = cos' b + sin' b cos' 4 = 1 — sin' 6 sin' .4<br />

and in the same manner, from the equations<br />

k sin 9 = cos A<br />

k cos 9 = sin 4 cos b<br />

we find<br />

^ = 1 — sin' b sin' 4<br />

and therefore, in both cases, J = v' (1 — sin' b sin' 4)


SOLUTION OF THE GENERAL SPHERICAL TRIANGLE. 223<br />

besides which we may employ, with (207), the equation sin P sin a = sin A sin b;<br />

or with (204) and (205), the equation tan P cos a = cot (C-3)! Also, wlieu<br />

(204) or (205) is employed, we m.ay find P both by its sine and its cosine.<br />

These formulse are computed in the same manner as thosfe of preceding case.<br />

119. CASE II. Given 4, C and b. Second Solution; when the two sides, a and c,<br />

or all the remaining parts, are required. We employ Gauss's Equations in the following<br />

form:<br />

sin i P sin J (a 4- c) = cos ^ (A — C) sin J b<br />

sin J P cos J (a -j- c) z= cos ^ (A + G) cos J b<br />

cos J P sin J (a — c) = sin J (4 — C) sin J b<br />

cos J P cos J (a — c) = sin i (A + G) cos J 6<br />

(209)<br />

whicii are solved in the same manner as (202).<br />

ExAItlPLB.<br />

G./^ 4 = 121° 30' 19"-8, G = 42° 15' 13"-7, 5 = 40° 0' 10", and P > 180°.<br />

By (209).<br />

b<br />

A<br />

G<br />

iiA-c)<br />

i{^+0)<br />

40° 0'10"-0<br />

121°36'19"-8<br />

42° 15' 13"-7<br />

39° 40' 33"-0<br />

81° -55' 46"-7<br />

20° 0' 5"-0<br />

d = log cos i (A — C) + 9-8863038<br />

log e = log sin J (4 — (7) 4- 9-8051224<br />

log/ = log cos I (4 -f c] 4- 9-1473326<br />

logg = log sin J (4 -f C) -j- 9-9956775<br />

log sin J 4 4- 9-5340806<br />

log cos J 6 4- 9-9729820<br />

log dsin^b = log sin J P sin i (a+ c) + 9-4203844<br />

log/ cos I 6 = log sin J P cos J (a -[- c) + 9-1203146<br />

log tan I (ffl 4- c) -i- 0-3000698<br />

i (a+ c) 63° 23' 8"-8<br />

log sin ^ (a 4- c) -f 9-9518526<br />

log sin J P 4- 9-4690318<br />

JP<br />

162°52'28"-6<br />

'log (— e sin J 6) = log (— cos J P) sin ^ (a — c) — 9-3392080<br />

*log (—ff cos J 5) = log (— cos I P) cos i(a — c) — 9-9686595<br />

log tan J (a — c) + 9-3705435<br />

i(a — e) 193°12'32"-9<br />

r a = 256° 35'36"-2<br />

fAns. J (; = 280°10'30"-4<br />

[ P= 325°44'57"-2<br />

* The sign of eacn of these factors is changed because P > 180°, and cos J P is<br />

negative.<br />

f It was necessary to increase J (a 4- o) by 360°, to obtain c. The corresponding<br />

value of b would be 016° 3-5' 30 "-2. See note at the end of this chapter, p..227


224 SPHERICAL TRIGONOMETRY.<br />

120. When P only is required, we may employ the methods of Arts. 81 and 8i,,<br />

which are determinate when the sign of sin P is given; or when that of either sin a<br />

or sin c is given, since we may then find that of sin P by inspecting the equations<br />

sin 4 sin b sin C sin 4<br />

sin P ;<br />

sm a sm c<br />

121. CASE III. Given a, b and 4. First Solution; when the three remaining parts<br />

P, C, and c are all required.<br />

We find P by the equation<br />

sin P = sin 4 sin b sm a<br />

(210)<br />

which is determinate when the sign of cos P is given.<br />

Then, to find C, we have<br />

— cos 0 cos 4 + sin C sin 4 cos b = cos P<br />

sin Coos4 -j- cos C sin A cos 6 = sin P cos a<br />

which have already been employed and adapted for logarithms in Art. 118. If we<br />

denote the auxiliary by 9, and put C — 3 = 3', we find, from (204),<br />

A sin 9 = cos A (k positive)<br />

k cos 9 ^ sin A cos b<br />

k sin 3' = cosP \ ;211)<br />

k cos 9' = sin P cos a<br />

C = 9 -f 9'<br />

To find c, we have<br />

cos c cos b + sin c sin b cos 4 = cos a<br />

sin c cos b — cos c sin b cos A = sin a cos P<br />

which have already been employed and adapted for logarithms in Art. 113.<br />

denote the auxiliary by 0, and put c — 0 = 0', we find, from (197),<br />

Checks. We have<br />

sin a<br />

sin ¥'<br />

sin Ip<br />

k sin 0 = sin b cos 4<br />

k cos 0 = cos b<br />

k sin 0' = sin a cos P<br />

A OOS0' = cos a<br />

c =0 4-*'<br />

cos 4<br />

cosP<br />

tanP<br />

tan 4<br />

COB 9<br />

cos 9'<br />

cos 0<br />

COS0'<br />

(k positive)<br />

tan a<br />

tan b<br />

cos i<br />

If we<br />

(212<br />

\ (213)<br />

One of which maybe used as a check when either C or c has been alone computed.^<br />

When both C and c have been found, the obvious check is<br />

sin_^ _ sin4.<br />

sin c sm a ^ '<br />

* The following relations deserve a passing notice:<br />

sin 0 cos 3'<br />

cos 0 sm 9<br />

sin b sin P<br />

tan 0 tan 0'<br />

tan 9 tan 3'<br />

sin 0' cos 3<br />

= sin a sin .4<br />

cos 0 sin 3'<br />

sin' a sin' P = sin' 4 sin'4<br />

sin 2 0 sin 2 3'<br />

sin 2 9 sin 2 0'<br />

sin' b<br />

sin'a


SOLUTION OF THE GENERAL SPHERICAL TRIANGLE. 225<br />

EXAMPLE.<br />

Given a = 126° 25' 6"-6. b = 138° 82' 0', 4 = 26r 16' 0", and cos P negative<br />

a 126° 25' 6"-6<br />

b 138° 32' 0"-0<br />

4 261° 16' 0"-0<br />

^y (210)> log sin a + 9-9056351<br />

log sin b -f 9-8209788<br />

log sin 4 —9-9949352<br />

log sinP — 9-9102789<br />

P 234°2.5'29"-3<br />

^y (^ll)> log cos 6 — 9-8746795<br />

log sin Aoosb = log A cos 3 + 9-8696147<br />

log cos4 = log 4 sin 3 —9-1813744<br />

log tan 3 — 9-3117597<br />

9 848° 24' 58"-0<br />

log cos a — 9-7735515<br />

log sin P COB a = log k cos 3' + 9-6838304<br />

log cos P = log i sin 3' — 9-7647520<br />

log tan 9' — 0-0809216<br />

9' 809° 41' 83"-7<br />

9-f. 9' = C 298° 6'26"-7<br />

By (212), log sin 4 cos 4 = log A sin 0 — 9-0028532<br />

log cos 4 =: log A cos 0 — 9-8746795<br />

log tan 0 4- 9-1276737<br />

0 187°38'81"-3<br />

log sin a cos P = log A sin 0' — 9-6703871<br />

log cos a = log k cos 0' — 9-7785515<br />

log tan 0' 4- 9-8968356<br />

0' 218°15'28"-6<br />

0 4-0' =


226 SPHERICAL TRIGONOMETRY.<br />

i sin 3 = cos 4<br />

(k positive)<br />

k cos 3 = sin 4 cos 4<br />

cos 3' = cos 3 cot a tan 4<br />

(3' < 180° with the sign of cos P)<br />

(7 = 9 -f 3'<br />

(215)<br />

To find c, we observe that sin 0' has the sign of sin a cos P, so that we have the<br />

foUo-wing formulse:<br />

/c sin 0 = sin 4 cos A (k positive)<br />

k cos 0 = cos 4<br />

cos 0 cos a ,„,„,<br />

(0' < 180° with the sign of sin a cos P)<br />

c = 0 4- 0'<br />

Gheck. The equation (214).<br />

128. CASE IV. Given 4, P and 4. First Solution,- when the three remaining parts<br />

a, c and 0 are all required.<br />

We find a by the equation<br />

Bin A sin 4<br />

sin P<br />

which is determinate when the sign of cos a is given.<br />

is by (211) and (212).<br />

(2171<br />

The remainder of the solution<br />

124. CASE IV. Given 4, P and 4. Second Solution; when 0 and G are required,<br />

without finding a.<br />

We easily find, from (211),<br />

A sin 3 = cos A<br />

(k positive)<br />

k cos 3 = sin A cos 4<br />

And from (212),<br />

CWeA The equation (214)<br />

sin 9' , sin 9 cos P<br />

cos 4 - (218)<br />

(COB 3' and sin P cos a to have the same sign)<br />

C =<br />

9-}-y<br />

A sin 0 = sin 4 cos 4<br />

k cos 0 == cos 4<br />

sin 0' = sin 0 tan 4 cot P<br />

(cos 0' and cos a to have the same sign)<br />

c = 0 4- $'<br />

125. CASE V. Given a, 4 and c. The formula<br />

cos a — cos 4 cos c<br />

cos 4 =<br />

sin 4 sin c<br />

(A positive)<br />

(219)<br />

(220)<br />

determines A when the sign of sin 4 is known. If the sign of sin P or of sin 0 if<br />

gi fsn, that of sin A becomes known by the equation<br />

sin 4<br />

sin a<br />

sin P<br />

sin 4<br />

sin 0<br />

sin c


SOLUTION OF THE GENERAL SPHERICAL TRIANGLE. 227<br />

The formulse (31), (33), (34), may be used, each of which will become determinate<br />

when the sign of either sin 4, sin P, or sin 0 is kno-wn.<br />

126. CASE VI. Given 4, P and C. The formula<br />

cos A + coa B cos Q<br />

cos ffl = . ' .—^= (221)<br />

sm P sm C '• ^<br />

determines a when the sign of sin a is given. If the sign of sin 4 or of sin c i?<br />

given, that of sin a becomes known by the equation<br />

sin 4 sm c<br />

sin 4 sin P sin C<br />

The formulse (36), (38), (39), may be used, each of which will be determinate<br />

when the sign of either sin a, sin b, or sin c is known.<br />

NOTE UPON GAUSS'S EQUATIONS.<br />

In the unlimited spherical triangle, we may consider any part, as a, to have an<br />

infinite number of values, viz. a, a 4- 360°, a + 720°, &c., expressed generally by<br />

the formula a + 2 n jr, n being any whole number or zero ; and since<br />

sin a = sin (a -f 2 re rr)<br />

cos a = coa (a + 2 nn-)<br />

all those equations of Chap. I. that involve only sin a and cos a will not be changed<br />

by the substitution of a -|- 2 n sr for a. A similar substitution may be made for<br />

each of the parts, or for all of them, at the same time, so that there is an infinite<br />

series of triangles to which these equations are applicable.<br />

But the substitution of a -f- 360° for a, in Gauss's Equations, (202), will change<br />

the sign of all of them, since<br />

sin J (a 4- 360°) = — sin J a<br />

cos ^ (a + 860°) = — cos J a<br />

while the substitution oi a + 720° for a will not change their sign, since<br />

sin J (a 4- 720°) = sin J a<br />

cos J (a'4- 720°) = cos J a<br />

In general, their sign is changed by the substitution of a + (A n + 2) TT for a, and<br />

it is not changed by the substitution ot a + i n rr. The same results follow like<br />

substitutions for each of the parts. It follows that these equations taken only with<br />

the positive sign, do not include all the triangles of the infinite series above spoken<br />

of, and that they are complete only when taken with the double sign, and expressed<br />

in two distinct groups, as (44) and (45) of Art. 27.<br />

In practice, however, we may take them with tlie positive sign only; for they will then<br />

give at least one of the triangles of the series, from which all the others, (and particularly<br />

that whose parts are less than 360°), may be directly deduced by the appli<br />

cation of 360°.-*<br />

This will be illustrated by the example of Art. 119, p. 223 ; we there find<br />

J (ffl -f c) = 63° 28' 3"-3<br />

J (a — c) = 193° 12' 32"-9<br />

or rather, since J (a -j- c) should be greater than J (a — c),<br />

J (a 4- c) = 428° 23' 3"-3<br />

J (a — c) = 193° 12' 32"-9<br />

•* Gauss (Tiieoria Motus Corp. Cml. Art. 54) recommends the use of the positive<br />

sign only, observing that any side or angle may be diminished or increased by 360°,<br />

as the case may require, but confines himself to the statement of this practical presept,<br />

without explaini-ig the grounds upon which it rests.


228 SPHERICAL TRIGONOMETRY.<br />

whence<br />

a = bl6° 35' 36"-2<br />

c = 280° 10' 30"-4<br />

which is the proper solution of the equations taken with the positive sign.<br />

we deduct 360° from a, and take, as on p. 223,<br />

a = 256° 35' 86"-6<br />

e = 230° 10' 30"-4<br />

If now<br />

we have the solution that would have been obtained by taking the negative sign in<br />

all the equations ; for we now have<br />

J (a 4- c) = 248° 23' 3"-3<br />

J (a — c) = 13° 12' 32"-9<br />

which, differing from the former values by 180°, must change the sign of all the<br />

equations.<br />

I have given some further particulars respecting unlimited spherical triangles,<br />

and a fuller discussion of Gauss's Equations, in an essay which the reader will find<br />

in the Astronomical Journal, Vol. I., published at Cambridge, Mass.


AREA OP A SPHERICAL TRIANGLE. 229<br />

CHAPTER V.<br />

AREA OF A SPHERICAL TRIANGLE.<br />

127. Given the three angles of a spherical triangle, to compute<br />

the area.<br />

This problem is solved in geometry, -where it is proved that the<br />

surface of a spherical triangle is measured by the excess of the sum of<br />

its three angles over t-wo right angles, by which is meant, that the area<br />

is as many times the area of the tri-rectangular triangle as there are<br />

right angles in the excess of the sum of the angles over two right angles.<br />

To express this analytically, let<br />

r = radius of the sphere<br />

T = surface of the tri-rectangular triangle<br />

= \ surface of a sphere = J TT /"^<br />

2S= A-\-B-¥ G<br />

K = area of the triangle ABG.<br />

Also, let the angles A, B and C be expressed in the unit of Art. ] 1,<br />

that is, let A, B, C denote the arcs -which measure the angles in a<br />

circle -whose radius is unity. The right angle expressed in the same<br />

unit is -^, therefore the number of right angles in2 S is<br />

2S '^ ^'^<br />

•2 TT<br />

and we have, according to the above theorem of geometry,<br />

,4 ^ ^x 2T<br />

K = 2'xf-^-2) = --(2^-.)<br />

or ir = r'(2/S'-3-) (222)<br />

and if the radius of the sphere is taken = 1<br />

K=2S-% :223)<br />

128. In a plane triangle the sum of the angles is equal to %,<br />

and in a spherical triangle the sum exceeds -K by K; hence this<br />

quantity, K, is commonly called the spherical excess.


23(? SPHERICAL TRIGONOMETRY<br />

129 Given the three sides, to find the area.<br />

By (223), we have<br />

in ^ K =s sin (S — -^) = — cos<br />

cos 1.2"= cos (s' —y) = sinS<br />

tan i K = — cot S<br />

> (2245<br />

in which we havo only to substitute the values of cos S, sin S, and cot S, given in<br />

Art. 34, to obtain the required solution. We find, [s = J (a -f- 4 -f- c)].<br />

. , „ v" [sin s sin (s — a) sin (s — 4) sin (s — c)]<br />

2 cos J a COS J 4 cos J o<br />

(225)<br />

cos J .£•=<br />

cos a 4- cos 4 -^ cos c 4- 1<br />

4 cos J a cos J 4 cos Jc<br />

cos' J ffl-f- cos' J 4 -f- cos' J c — 1<br />

2 cos J a cos J b cos Jc<br />

(226)<br />

The numerator of (225) being denoted by n, we find,<br />

^ , „ 1 4- cos a -i- cos 6 -I- cos c<br />

cot J .2^ = —'• '-^ C<br />

which is known as De Gua's formula. 2 n<br />

Again, from the formulse of Art. 35, since 1 — sin /S = 2 sin 'Jj^, 1 4- sin iS:<br />

2 cos "iK, we find<br />

[<br />

sin I s sin J (s — a) sin J (s — 4) sin ^ (s — c)~\<br />

cos J a cos J 4 cos j c<br />

J<br />

(227)<br />

cos i.K'<br />

tan ^ K^-^<br />

rcosJ«cosJ(s — a) cos|(s — 4) cosJ(s — c]<br />

^ L cos J a cos J 6 cos J c ]<br />

[tan Js tan } (s — a) tan J (a — b) tan J (s —


AREA OF A SPHERICAL TRLANGLE. 281<br />

131. it' we admit more than three parts of the triangle into the expression of K,<br />

we have, by (56) and (67),<br />

the quotient of which gives (229).<br />

sin J a Bin J 4 .<br />

. * A = —;—=— sm G<br />

cos J 0<br />

COB i K = cos J ffl COS J 6-f-siu Ja sin J 4 cos 0 cos J 0<br />

> (230)<br />

132. Since there are always two triangles upon the surface of the sphere which<br />

have the same three sides, (Art. 110), the angles not being limited to values less than<br />

180°, the formulse (225), (226)_, (227) should give the areas of both of them, and<br />

their sum should be equal to the surface of the sphere = 4 jr. In fact, by (225),<br />

Bin J .ff- may be either positive or negative, while by (226) the cosine is fully determined,<br />

so that these formulse give two values ot^K whose sum is 2 a-, and therefore<br />

two values of K, whose sum is 4^.<br />

It lollows that (225) alone is not sufSciently determinate when the triangle is unlimited,<br />

since it gives four solutions. The most convenient formula is therefore<br />

(228), for we must always have J .ff < a-, and the double sign of the radical give«<br />

the two values of J K, one lesB and the other greater than -3-.


232 SPHERICAL TRIGONOMETRY.<br />

CHAPTER VI.<br />

LIFFERENCES AND DIFFERENTIALS OF SPHERICAL TRLVNGLES.<br />

133. Two parts of a spherical triangle being constant, and a third<br />

receiving an increment, it is required to deduce the corresponding<br />

increments of the remaining three parts. As in plane triangles,<br />

(PL Trig. Chap. XII.), this will be effected by a comparison of two<br />

triangles having two parts in common. The triangle formed from<br />

the given one by applying the increments to the variable parts will<br />

be distinguished as the derived triangle.<br />

We shall first consider the increments as finite differences, and<br />

give them the positive sign, (PI. Trig. Art. 187).<br />

134. CASE I. A and c constant. The parts of<br />

ABG, Fig. 22, being A, c, B, 0, a, b, those of the<br />

derived triangle ABG' are J., c, i? 4- h.B, C -\- AC,<br />

a -f A«, 5 -f A5; and the parts of the differential<br />

triangle BCQ' are a,a-\- [^a,^b, 180° — G, G + LG<br />

and A5. We have, then, in BOG', by (3),<br />

sin A6 sin {a + i^d) sin a<br />

sin Ai? ~ sin C sin(C4-AC^<br />

Also, in BCG', by (40), we have<br />

>vhence<br />

sin i (180° - C 4- C 4- AC) _<br />

tan|A6<br />

sinJ(180°-C-C-AC) ~ tan J (« 4- Aa - a)<br />

By (41) we<br />

By (42),<br />

tanjAa<br />

taniA5<br />

cos(C-fiAC)<br />

COSJAC<br />

find in a similar manner,<br />

tan J A5<br />

sin J AC<br />

tan (a 4- J Aa)<br />

sin (C 4-J AC)<br />

(231)<br />

(232)<br />

(233)<br />

sin J Aa sin (a -f |- Aa)<br />

(234)<br />

taniA.B tan(C4-iAC) ^ '


By (43),<br />

DIFFERENCES OP SPHERICAL TRIANGLES, 233<br />

tan|AC_ __ cos(a4-|Aa)<br />

tanjA.B cos ^ La ^<br />

By combining (232) and (233),<br />

tan I Aa ^ _ tan {a + j Aa)<br />

.^„„.<br />

tan 1 AC tan (C 4-i AC) • ^<br />

As these formute involve the increments in the second members,<br />

they are to be computed by successive approximations. (See<br />

PI. Trig. Art. 201).<br />

135. CASE II. A and a constant. The given<br />

triangle being ABC, Fig. 23, the parts of the<br />

derived triangle A'BG are ^, a, ^ 4- A -S, 5 4- Ab,<br />

C-f AC, C -f Ac. Although .the figure appears to'^'V<br />

show that the angle B is diminished, it is still proper -^<br />

to represent the angle A'B G hj B -h AS, to preserve uniformity<br />

in the algebraic signs of the increments; the essential signs being<br />

given by the equations of diiferences themselves. Hence we put<br />

the angle AB A'= AB G - A'B G= B - {B-{• AB) = - A B.<br />

Joining A A' we have in ^ J. J.' and CA A', by (43),<br />

cos (c 4- i Ac) : cos 1 Ac = — cot i AB : tan i {A'AB 4- A A'B)<br />

cos (5 4- i Ab) : cos J A6 = cot J AC : tan J {A'AC-hA A'G)<br />

but since A is constant, or B A G = B A' G, Vfc find that the fourth<br />

terms of these proportions are equal; whence<br />

tan ^ AB _ cos {b -\- ^ A6) cos | Ac ^o-a"\<br />

tan J AC COS (c 4- i Ac) cos J A6 \ ' )<br />

In the polar triangle oi AB G, the constants are still an angle<br />

and its opposite side, and the preceding equation applied to this<br />

polar triangle (by Art. 8) gives<br />

tan J Ab<br />

tan J Ac<br />

cos (S 4- i AB) COS I AC , ,<br />

cos(C4-jAC)co3iA^ ^ '<br />

In J. .B C and A'B C we have<br />

sin tl sin S = sin A sin b<br />

sin a sin {B f AB) = sin A sin {b -i- A?^<br />

30 u2


£34 SPHERICAL TRIGONOMETRY,<br />

the difference and sum of which give<br />

sin a cos (S 4- J AB) sin J AB = sin A cos (5 -4- i AJ) sin J Ai<br />

sin a sm (-B 4- J AB) cos J Ai? = sin A sin (S 4- J A5) cos J A5<br />

from which, by division, we find<br />

tan J A5 _ tan (S 4- J A5)<br />

tan i AB ~ tan (54-1 AB)<br />

(2391<br />

and in the same manner<br />

tan J Ac _ tan (c 4- ^ Ae)<br />

tan J AC ~ tan (C4- i AC)<br />

(240)<br />

The product of (237) and (239) gives<br />

whence also*<br />

sin I Ab ^ _<br />

tan J AC<br />

sin {b -\- ^ Ab) cos | Ac<br />

cos (e 4- J Ac) tan (-B 4- J AB)<br />

(241)<br />

sin J Ac<br />

tan ^AB<br />

_ sin (c -f I Ac) COS | .A5<br />

cos(6 4-J A6)tan(C4-JAC)<br />

(242)<br />

Fig. 24.<br />

a 136. CASE III. b and c constant. The given<br />

c-,triangle being ABG, Fig. 24, the parts of the<br />

derived triangle ABC are J, c, a -f Aa, S -f AB,<br />

C4- AC, A 4- AJ.. Joining C G' we have in B C C,<br />

ty (42),<br />

sin (a 4- J Aa): sin J Aa = cot | AB : tan ^{BCG'—B G'G)<br />

But observing that J. C = J. C, J. CC = J. C'C, we have<br />

BCG' = AGG' - G<br />

BC'C = AC'C + O-if AC<br />

i {B OC -BG'0) = -{Q+l AG)<br />

and the above proportion gives, therefore.<br />

sin I Aa _<br />

tan J AB<br />

sin (a -f J Aa)<br />

cot (C 4-i AC)<br />

(243)<br />

* The equations (239), (240), (241), and (242), contain each two factors less thin<br />

the corresponding equations given by Cagnoli.


DIFFERENCES OF SPHERICAL TRIANGLES. 235<br />

In the same manner we should find<br />

sin I Aa ^ _ sin (a 4- j Aa)<br />

tan J AC cot(B4-jAB) ^''*'"<br />

The quotient of (243) and (244) gives<br />

tan I AB _ tan (S 4- | AB)<br />

tan J A"C ~ tan (C 4" J AC)<br />

(245)<br />

In J -B C and J. S C, by (4), we have<br />

cos a = cos 6 cos e -f sin 5 sin e cos A<br />

cos (a -f Aa) = cos 6 cos e 4- sin b sin c cos (-4. -f AA)<br />

the difference of which gives<br />

sin J Aa _ sin b sin o sin (A -j- ^<br />

sin -J A J. sin (a -f J Aa)<br />

The quotient of .(243) divided by (246) gives<br />

AA)<br />

(246)<br />

sin i AA _ _ sin^ (a 4- | Aa) tan (C 4- j AC)<br />

tan J AB sin b sin c sin (J -f J AA)<br />

(247)<br />

and from (244) and (246), in the same manner,<br />

sin I AA sin^ {a + j Aa) tan (B + | AB)<br />

tan J AC ~ sin b sin e sin (A -{- ^ AA)<br />

(248)<br />

137. CASE IV. B and C constant. The equations of the preceding<br />

case (243 to 248), applied to the polar triangle, give<br />

sin ^ AA _ sin (J 4- j- A.4)<br />

tan § Ab cot (c 4- J Ac)<br />

sin I- A J _ sin (A 4- i A^)<br />

tan ^ Ac cot (5 4- i A6)<br />

tan J A5 _ tan (5 4- | A5)<br />

tan J Ac tan (c -f |- Ac)<br />

sin J A J. _ sin B sin C sin (a 4- | Aa)<br />

sin J Aa sin (J 4- i A^)<br />

(249)<br />

(250)<br />

(251)<br />

(252)


286 SPHERICAL TRIGONOMETRY.<br />

sin i Aa _ sin' (-A 4- | AA) tan (c 4- | Ac)<br />

tan J Ab ~ sin B sin C sin (a 4- J Aa)<br />

sin i AA ^ sin^ (-A 4- | A.A) tan {b + j Ab)<br />

tan J Ac sin B sin C sin (a -f- J Aa)<br />

(253)<br />

(254)<br />

FINITE DIFFERENCES OF SPHERICAL RIGHT TRIANGLES.<br />

138. All the preceding equations are, of course, applicable to<br />

right triangles, or to quadrantal triangles, and in some cases they<br />

assume simpler forms. Thus in Case I., if the variable C = 90°<br />

(231) and (232) become<br />

sin Ab = sin (a 4- Aa) sin<br />

AB<br />

tan J Aa = — tan J Ab tan J AC<br />

and similar modifications take place in other cases.<br />

139. When one of the constants is 90°, the preceding equations<br />

do not generally assume any simpler forms, but they may be transformed<br />

so as to involve the same variables in both members, which is<br />

generally desirable in their practical applications.*<br />

The method that we shall follow is so simple that it will be unnecessary<br />

to repeat it in every case. A single example will suffice<br />

to explain it.<br />

Let C (= 90°) and b be the constants; to find the relation of As<br />

and A B,^ we have between the two variables and the constant b, the<br />

equations<br />

sin B = sin 5 cosec c<br />

sin (B -f AB) = sin b cosec (e 4- Ac)<br />

the difference and sum of which, by PI. Trig. (105), (106), (131),<br />

and (132), are<br />

o rr, , -, •r.N . , r, 2 sin J cos (e -f i Ac) sin i Ac<br />

2 cos (B 4- i AB) sm A AB = — -.—\ / ' ^—<br />

sm c sm (c 4- A c)<br />

n • rr, , -, -r-v , ' -r. 2 sin 5 sin (c 4-J Ac) cos i Ac<br />

2sin(B4-*AB)cosiA.B=<br />

-.—K—r^—^—r-^—<br />

sm c sin (c -f Ac)<br />

» Cagnoli gives these equations reduced so as to involve the same variables in<br />

Doth members ; but in almost every instance his formulse involve two factors more<br />

than are necessary, and are far less simple and convenient than those here given.


DIFFERENCES OF SPHERICAL TRIANGLES.<br />

and the quotient of these is<br />

23;<br />

tan i AB<br />

tan {B 4-1 AB)<br />

tan J Ac<br />

tan (c 4- J Ac)<br />

which gives the first equation of the following article. This process<br />

always eliminates the constant, and is applicable in every case.<br />

When the equation to be differenced involves cosines, we employ<br />

PI. Trig. (107) and (108); if tangents, (115) and (116); if cotangents,<br />

(122); if secants, (129) and (130). The results are as<br />

follows:<br />

140. CASE I. C = 90° and b constant.<br />

tanjAe<br />

tanjA^<br />

tan(e-|-jAc<br />

tan(B4-iAB<br />

tanjA c<br />

taniAa<br />

cot( e 4-JA c<br />

cot(a -f jAa<br />

(255)<br />

sin A a<br />

sin A A ~<br />

sin(2 a-f A a<br />

sin(2J.4-AJ.<br />

tanJ^A a<br />

sinAB^<br />

tan(a -\-^Aa<br />

'ain{2B+AB<br />

(256)<br />

sin Ac<br />

tanjAJ ^<br />

sin(2 e -f A c<br />

cotp~+|AZ<br />

tan^A^<br />

tan|AB<br />

ta,n{A 4-J A J.<br />

• cot (B4-iAB;<br />

(257)<br />

141. CASE II. C = 90<br />

sin(2JL-fAJ-<br />

'sin{2B+AB<br />

smA J _<br />

sinAi?<br />

tan JA a<br />

tanjAj4<br />

sm A a<br />

tan|AB""<br />

tan(a -f jAa<br />

tan(J.4-iAJ<br />

sin (2 a 4-A a<br />

cot(B4-iA5<br />

142. CASE III. C = 901° and A constant,<br />

tan|A e<br />

tan|Aa ~<br />

tanjA e<br />

sinAB<br />

tan( c -FJA c<br />

tan(a-i-2-Aa<br />

cot(c 4-jAc<br />

sin(2B4-AB<br />

sinAe sin (2 c -t-Ac<br />

sin A 5 sin (2 5-f A 5<br />

and c constant.<br />

tan JA a<br />

tan JA b<br />

tanJA b<br />

tanjAB<br />

sin A 5<br />

taniA4<br />

sin A a<br />

tanjA b<br />

tanJA b<br />

tanjAB<br />

tanJA a •<br />

tanjAB<br />

cot (a 4-JAa<br />

"cot(64-jA5<br />

tan( b+^Ab<br />

tan(B4-iAB<br />

sin(2 5 4-A5<br />

'cot(J.4-iAJ.<br />

sin (2 a-f A a<br />

tan(5 4-jA6<br />

cot{b-hiAb<br />

cot(B4-iAB<br />

cot(a4-jAa<br />

tan(B4-jAB<br />

(258)<br />

(259)<br />

(260)<br />

(261).<br />

(262)<br />

(263)<br />

143. If a constant side is 90°, the equations of finite differences<br />

for the triangle may be obtained by applying the preceding eq-iations<br />

to the jiolar triangle.


238 SPHERICAL TRIGONOMETRY.<br />

DIFFERENTIAL YARIATIONS OF SPHERICAL OBLIQUE TRIANGLES.<br />

144. To obtain the differential variations, we have only to maka<br />

the increments infinitely small in the equations of finite differences,<br />

observing the principles of PL Trig. Art. 192. Or we may differentiate<br />

the equations of spherical triangles directly, employing the<br />

differentials of the trigonometric functions given in PI. Trig. Art. 192<br />

For example, A and c being constant, to find the relation of c^ a<br />

and dB, we have<br />

the differential of which is<br />

da<br />

5C<br />

sin Aaxnc = sin a sin C<br />

0 = sin a


DIFFERENTIAL VARIATH-lNS OF SPHERICAL TRIANGLES. 23!)<br />

146. CASE II. A and a constant.<br />

dB<br />

d<br />

d b<br />

dB~<br />

d b<br />

d G<br />

G~'<br />

'<br />

cos b<br />

cos e<br />

tan b<br />

tanB<br />

sin b<br />

cos c tan B<br />

147. CASE III. b and c constant.<br />

d b<br />

d G ~<br />

d c<br />

d c<br />

da . ^ da<br />

dB~' — sin a tan C<br />

dG~<br />

dB<br />

d G~<br />

dA<br />

dB~<br />

tanB<br />

tan C<br />

sin A<br />

sin B cos C<br />

d a<br />

d~A~<br />

dA<br />

d C~<br />

cosB<br />

cos C<br />

tan G<br />

tan C<br />

sine<br />

cos b tan C<br />

• sin a tan B<br />

sin b sin C<br />

sin J.<br />

sin C cos B<br />

(267)<br />

(268)<br />

(269)<br />

(270)<br />

(271)<br />

(272)<br />

1. CASE IV.<br />

dA<br />

d b~<br />

d b<br />

d G<br />

d a<br />

d b~<br />

B and C constant<br />

sin A tan c<br />

tan 6<br />

tan c<br />

sina<br />

sin 6 cos e<br />

dA<br />

d c<br />

dA_<br />

d a<br />

d a<br />

d G~<br />

sin ^ tan b<br />

ain B sin 'e<br />

sin a<br />

sin c cos 6<br />

(273)<br />

(274)<br />

(275)<br />

DIFFERENTIAL VARIATIONS OF SPHERICAL RIGHT TRIANGLES.<br />

The preceding may also be used for right triangles; but it<br />

may be desirable to have the same variables in both members, as in<br />

the following formulae derived from those of Arts. 140, 141, and<br />

142:<br />

149. CASE I. C = 90° and b constant.<br />

d 0 tan c d c<br />

d B ~ ~ tan B d a ~<br />

d a sin 2 a da<br />

JA ^ sin 2 A CTB "<br />

cot c<br />

cot a<br />

2 tan a<br />

sin 2 5<br />

do sin 2 c d^ _ tan A<br />

d^ '^ TcotA ~d~B "" ~ coT^<br />

{2m<br />

(277)<br />

^278^


240 SPHERICAL TRIGONOMETRY,<br />

150. CASE II. C = 90° and c constant.<br />

d A sin 2 A da<br />

d~B^~ sin 2B ~dT<br />

d a tan a d b<br />

(TA. ~ tan A d B<br />

da sin 2 a d b<br />

d B 2 cot 5 (^ ^<br />

cot a<br />

cot b<br />

tan b<br />

tan B<br />

sin 2 b<br />

2 cot J.<br />

151. CASE III. C = 90° and A constant.<br />

d c tun c djz_ _ sin 2 a<br />

Ta ^ tana a<br />


APPROXIMATE SOLUTION OP SPHERICAL TRIANGLES. 241<br />

CHAPTER VII.<br />

APPROXIMATE SOLUTION OF SPHERICAL TRIANGLES IN CERTAIN<br />

CASES.<br />

154. WHEN some of the parts of the triangle are small, or nearly 90°, or nearly<br />

180°, approximate solutions may be employed with advantage. These are generally<br />

found by means of series.<br />

155. In a spherical right triangle (the right angle being C), given A and c, to find b<br />

We have<br />

tan J = cos A tan c (288)<br />

which is of the form in PL Trig. (493), and may therefore be developed by (495)<br />

and (496) by putting x = b, y = c, p = cos A, whence<br />

p — 1 1 — cos A , ,<br />

g = I T = — rn i = — tan' J A<br />

p + \ 1 -4- cos ^1 2<br />

and (495) and (496) become, [taking n = 0 in (495), and n = 1 in (496)],<br />

i = c — tan' \Aa\n2c+\ tan' J ^ sin 4 c — &c. (289)<br />

b = TT—c 4- cot' J 4 sin 2 0 — J oof .} A sin 4 o 4- &c. (290)<br />

If .4 is small, cos A is nearly equal to unity, and b exceeds c by a small quantity<br />

which is approximately found by one or more terms of the series (289).<br />

If A is nearly 180°, or cos^ nearly = — 1, S exceeds^-—c by a small quantity,<br />

which is found by (290).<br />

For examples of the mode of computation, see PI. Trig. Art. 255.<br />

156. Althougli these solutions are termed approximate, it must not be inferred that<br />

they are less accurate in practice tlian the direct solution of (288) by the tables ; for<br />

the logarithmic tables are themselves only approximate, and the neglect of the<br />

higher powers in such series as (289) and (290) may involve a less theoretical error<br />

than the similar neglect of the higher powers in the series by which the tables are<br />

computed. In the examples of PI. Trig. Art. 255, the thousandths of a second were<br />

found with accuracy, which could not have been effected by a direct solution with<br />

less than eight decimal places in the logarithms.<br />

These considerations lead to the frequent employment of approximate solutions<br />

in astronomy.<br />

157. If A and b are given, to find c, we have<br />

tan c = sec J. tan 5<br />

whicli is reduced to PI. Trig. (493), by putting z •== c, y = b, p = sec A,<br />

and the series -will be<br />

sec.4 — 1 1 — 003^ , „, ,<br />

secA + \ l-|-cos4. ^<br />

c = b + tan' J 4 sin 2 i 4- J tan' J A sin 4 6 -f &c. (291)<br />

c =!r—i — cot' J 4 sin 2 5 — J cot* I A ainib — &o. (292).<br />

31 V


242 SPHERICAL TRIGONOMETRY.<br />

158. Similar solutions apply to the equations of right triangles,<br />

the last being solved under the form<br />

tan o = sin 6 tan A<br />

cotB = cos c tan A<br />

tan (90° — B) = cos c tan A<br />

We may also compute, in the same manner, the auxiliaries ip and S in (122) and<br />

(134), so frequently employed in the solutions of oblique triangles.<br />

159. In a right spherical triangle, given c and A, to find a, when A is nearly 90°.<br />

We have<br />

sin a = sin A sin c (293)<br />

from which we deduce<br />

tan J (c — a) = tan' (45° — ^ A) tan ^ (c + a) (294)<br />

From this we may find c —a, which is supposed very small, by successive approximations.<br />

For a first approximation, let a = ein the second member, and find thence<br />

the value of c — a and of a; for a second approximation substitute in the second<br />

member the value of a just found; and so on until two successive values agree as<br />

nearly as may be desired.<br />

EXAMPLE.<br />

Given A = 89°, c = 87° ; find a.<br />

Here 45° — J ^ = 0° 30', and for the first approximation i (o + a) = 87°.<br />

log tan J (c 4-ffl) 1-28060<br />

log tan' (45° — ^ A)<br />

arCO log sin 1"<br />

J (e _ ffl) == 299"-74 log i(c — a)<br />

a = 87° — 9' 59"-48 = 86° 50' 0"-52<br />

5-88172<br />

5-31443<br />

2-47675<br />

2D APPKOX.<br />

3D APPEOX.<br />

4TH APPEOX.<br />

i(c+a)<br />

log tan ^ (c + a)<br />

, tan' (45° — i A)<br />

log ^ . .„ —-<br />

Bin 1<br />

log J (c — ffl)<br />

i(c — a)<br />

c<br />

ffl<br />

a<br />

86° 55' 0"<br />

1-26868<br />

1-19615<br />

2-46488<br />

29r'-63<br />

9' 43"-26<br />

86° 50' 16"-74<br />

86° 55' 8"<br />

1-26899<br />

1-19615<br />

2-46514<br />

291"-83<br />

9' 43"-66<br />

86° 50' 16"-34<br />

86° 55' 8"-17<br />

1-26900<br />

1-19815<br />

2-46515<br />

291"-84<br />

9' 43"-68<br />

86° 50' 16"-32<br />

The direct solution of (293) gives a = 86° 50' 16", but cannot give the fractions<br />

of a second without tables of more than seven figure logs. We have given this problem,<br />

however, not so much on account of its particular utility, as for the purpose<br />

of introducing the method of approximation to which it leads, and which is often<br />

employed.<br />

The process here explained may obviously be applied to any equation if the form<br />

sm X = m sm y<br />

when m is nearly equal to unity.


APPROXIMATE SOLUTION OF SPHERICAL TRIANGLES. 243<br />

160. In ffl spherical oblique triangle, given two sides and the included angle, to find the<br />

other angles and side by series.<br />

If a, b and C are the data, to find c, we have<br />

Substituting half arcs,<br />

cos c = cos fflcos i 4- sin a sin b cos G<br />

sin' J c = sin' J a cos' J S 4- cos' J a sin' J b<br />

— 2 sin J fflcos J 6 cos J ffl sin J S cos C<br />

which is of the form Pi; Trig. (507), and may be developed by (508) by substituting<br />

Bin I c for c, sin J fflcos J 6 for a, and cos J a sin J 6 for b ; so that (508) becomes<br />

lo,Binic = logcosiaainib-M[^^cosC + { ^ J ^ ^ + .c] (295)<br />

To find A and B, we have,<br />

t^ni(A + B) = ''°'i(;~''KotiG<br />

' cos J (ffl 4- 4) '<br />

whence<br />

t^nl(A-B) = '2^^^^lcotlG<br />

^ ' sm J (a 4" o)<br />

tan lln-l(A + B)-\ = ^-"^ f " "^^^"f f) tan J C<br />

^ ^ Vcot Jo 4-tan Ji/ '•<br />

. ri 1 , A T,^^ /tan J ffl4-tan iJ\ , „<br />

*^°^^"-H^-^)] = Unjffl.-tanjJ^^°*^<br />

Comparing these equations with PL Trig. (493), and developing by (495), 7» = 0,<br />

irr-UA^B) = ,C-^^J^C+ (SH-^y ^ ^ - . c . (290)<br />

i.-H.-.)=i. + £^.n.4-(Sf^y^%.c. (297)<br />

If we develop by (496), we find<br />

T 1 , , . ^ . ^ . /cot i ffl \ . „ /cot i a\ sin 2 C , „ ,„„„,<br />

J._J(^ + ^ ) = _ j a 4 - ( ^ ) s m ( 7 - ( ^ - ^ J _ _ + &c. (298)<br />

1 1 / .1 T,N 1^ /tanlfflX . ^ /tan Jffl \' sin2 C „ ,„„„^<br />

from which a selection will be made in any particular case, according to the convergenoy<br />

of the series. The terms of the series are in arc, and must be reduced to<br />

seconds, by dividing by sin 1"-<br />

This solution may be applied to the case where two angles and the included side<br />

are the data, by means of the polar triangle.<br />

161. To express the area of a spherical triangle in series.<br />

Companug (229) with PI. Trig. (500), and developing by (502), we find<br />

J £"= tan J ffl tan J 6 sin C — J tan' J a tan' J 6 sin 2 C 4- &c.<br />

(3i)Ci)


244 SPHERICAL TRIGONOMETR":^.<br />

162 LEGENDRE'S THEOBEM. If the sides of a spherical triangle are very smuil co'ipur'd<br />

with the radius of the sphere, and a plane triangle be formed wliose sides are eqi'-al<br />

to those of tlie spherical triangle, then each angle of the plane triangle is equal to the eorrisponduig<br />

angle of the spherical triangle minus one-third of the spherical excess.<br />

Let (.-, b and c be the sides of the spherical triangle expressed in .arc, the radius<br />

of the sphere being unity; and let A', B' and C" be the angles of the plane triangle<br />

whose sides are a, b and c. Then we have, in the spherical triangle,<br />

. cos a — cos b cos c<br />

cos ..4 =<br />

sin b sin c<br />

Substitute in the second member of this, the values of cos a, &c., in series, by<br />

PI. Trig. (405) and (406), neglecting only powers above the fourth, viz.<br />

we find<br />

cos a = 1 — ^ a''+ 2j ffl'<br />

cos 6 = 1 — J S' 4- jL i* sin 5 = * — -J 6'<br />

cos c = 1 — J c' 4- j)^ c' sin c = c — J c'<br />

cos A<br />

io[l-i(6'4-c')]<br />

• 6 J' c')<br />

Multiplying the numerator and denominator hj 1 + i (b'' + c'), and neglecting<br />

terms of a higher order than the fourth, as before, we have<br />

cos 4 _ i'4- c' —"'.I ffl'4-i'4-(;' —2ffl'&' —2ffl'c' —2i'c'<br />

2 o b ;, c , "T" ' 24 6 c<br />

which, by PI. Trig. (225) and (239), becomes<br />

cos A = cos A' — i be sin' A'<br />

Let A = A' + z, then since x is small, we may put cos x = 1, so that, by<br />

PI. Trig. (38),<br />

cos A = cos A' — X sin A'<br />

whence<br />

a: = J J c sin A<br />

But i be sin A — area of the plane triangle = very nearly area of the spherical<br />

triangle = K, whence<br />

=' = iS: A' = A — iK<br />

The same reasoning applies to each of the other angles, so that<br />

S'^B-iK<br />

G'=G-iK<br />

which proves the theorem.<br />

^ 103. This theorem is applied in geodetical surveying, and is found to be sufficiently<br />

accurate for triangles whose sides are considerably greater than 1°. It is to<br />

be remembered that the sides are to be expressed in arc; and if they are given in<br />

feet (for example), they must be reduced to arc by dividing by the radius in feet,<br />

or, which is equivalent, the area must be divided by the square of this radius. If<br />

then 7-= radius of the earth in units of any kind, a, b and e the sides of the triangle<br />

in units of the same kind, and k the area of the plane triangle, we shall have<br />

K in seconds, by the equation<br />

K=-<br />

k<br />

r' sin 1"<br />

EXAMPLE.<br />

In a triangle upon the earth's surface, given b = 183496-2 feet, c — 1561221 feet<br />

and A = 48° 4' 32"-35 ; to find the remaining parts.


APPROXIMATE SOLUTION OF SPHERICAL TRIANGLES. 245<br />

We have k = }, b c sin ..l.and the mean value of ?- = 20888780 feet. Hence<br />

log b 5-26363<br />

log c 5-19346<br />

log sin ^ 9-87159<br />

arCO log 2 r' sin 1" 0-37356<br />

K = 5"-04 log K 0-70224<br />

It is evident that great accuracy in the value of r and of the other data is not<br />

required in computing K. We now have ^ K = l"-68. A' = 48° 4' 30'--67, and by<br />

solving the plane triangle with the data A', b and c, we iind<br />

ffl = 140580-0 feet B' = 76° 12' 22"-19 C" = 55° 43' 7"-13<br />

Adding ^ K to each of these angles, the angles of the spherical triangle are<br />

B = 76° 12' 23"-87 C = 55° 43' 8"-81.<br />

For further details respecting geodetical triangles, and for the methods of solving<br />

spheroidal triangles, special works upon geodesy must be consulted, such as<br />

Legendre's Analyse des Triangles traces sur la surface d'une sphSroide; Puissant's Traile<br />

de Geodesic; Vnisssmt's Nouvel essai de trigonometric spheroidique; Wisohex's Lehrbuch<br />

der hoheren Geoddsie; various papers by Gauss, Bessel, &c.<br />

164. To solve a spherical triangle when two of its sides are nearly 90°.<br />

If a and b are nearly 90°, c and C are nearly equal, and it will be expedient to<br />

compute the small quantity O— c by an approximate method. We have, by (25),<br />

sin' J c = sin' ^ (a + b) sin' J C -f sin' J (ffl — 5) cos' J C<br />

and by PI. Trig.<br />

sin' J C= [sin' ^ (a + b) + cos' J (ffl 4- *)] sin' } G<br />

the difference of which equations is<br />

sin J (C 4- c) sin ^(0 — c) = cos' i (a + b) sin' J C — sin' } (a — 5) cos'* 0<br />

Let<br />

ffl' = 90° — ffl i' = 90° — b<br />

a' and b' being very small: also, since 0 and c are nearly equal, put<br />

i(C+c)=.C<br />

then the above equation becomes<br />

sin C sin J (C—c) = sin' J (ffl' 4- b') sin' J C — sin' | (a' — b') cos' J C<br />

Dividing by sin C = 2 sin J C cos J C, and substituting the arcs J {C — c),<br />

J (ffl' -|- b'), J (ffl' — b'), for their sines, we find<br />

C-c = Bin 1" [ ( ^ y tan J C _ ( ^ 7 cot J c] (301)<br />

which is the required approximate formula for the case when ffl', b' and G are given<br />

to find e.<br />

If a', 4' and c are given, to find C, we may exchange G for c in the second member,<br />

whence<br />

G-c = sin 1" [ ( ^ ' ) ' tan J c - ( ^ ) ' cot J .] (802)<br />

V2


246 SPHERICAL TRIGONOMETRY.<br />

CHAPTER VIII.<br />

MISCELLANEOUS PROBLEMS OF SPHERICAL<br />

TRIGONOMETRY.<br />

165. In a given spherical triangle, to find the perpendicular from one of the angles upon<br />

the opposite side.<br />

Kg. 25.<br />

Denoting the perpendicular upon the side e (Fig. 25)<br />

by p, we have<br />

sin^ = sin b sin A (303;<br />

If the three sides or the three angles are given, we<br />

find by (48), or (51), and<br />

2n 2N<br />

sm p 5= sin sm -: c =<br />

sin (7 n ^ '<br />

in which n and iVare given by (47) and (50).<br />

If we admit more than three parts of the triangle into the expression of p, we<br />

have, by (55), (56), and (303),<br />

„•„.„ 2 sin J ^ sin J B . 2 cos} ffl cos ii<br />

smp = cos -,„ J 1 C n sm s = r? , _ ^ cos S (805)<br />

sin J c<br />

166. To find the radius of the circle described about a given spherical triangle.<br />

Kg. 26. The radius here understood is the aro £)^ = 0 B r=<br />

0 C, Fig. 26, drawn from the pole of the small circle<br />

AB G to either of the angles. Let<br />

then<br />

0,4.8= OBA = x<br />

G=OGA+OCB=OAG+OBG<br />

= A — X + B — z<br />

putting S=l(A + B+0).<br />

z = i(A + B—G) = S—G<br />

The triangle AOB being isosceles, the perpeu-licular<br />

0 P bisects the side c, therefore if 0.4 =: iS, we have<br />

or, by (70),<br />

, _ tan 1 e tan i c<br />

tan R = i— =<br />

cosz cos (^S — 0)<br />

_ 2 sin J ffl sin J b sin J c<br />

tan li ^= ~—<br />

(806)<br />

(3071<br />

By applying the principles of Art. 37, this will give the corresponding formulse of<br />

PI. Trig. (285).<br />

167. From (69) and (70) we find<br />

cos (S— 0) = — cos 5cot J ffl cot J b<br />

by which (306) is reduced to<br />

. „ tan J a tan J b tan J c<br />

•— cos JS<br />

'?Off.


MISCELLANEOUS PROBLEMS.<br />

Also, by the last equation of (56), (306) becomes<br />

or, by (50),<br />

tan R =<br />

J. r> sin J c<br />

tan .S = ±<br />

cos J fflcos ,} b sin G<br />

168. Substituting in (306) for tan J c by (39),<br />

— cos S<br />

tan R — ^\oo6(S (S—A) — cos (-S— B) cos (Scos<br />

S<br />

If<br />

169. Let the sides of the triangle ABC, Fig. 27, be produced<br />

to meet in A', B', and C" ; and denote the radii of<br />

the circles circumscribed about 4'7? C, B'A 0, O'AB by<br />

R', R", R'" respectively. Then if 2 S' denote the sum of<br />

the angles of A' B 0, (A, B and C being the angles of<br />

ABC),<br />

j<br />

2S'= 2^ — B— C+A<br />

S' — A' = w — i (A + B + C) z= TT — S<br />

so that (306) applied to A' B G gives<br />

and in like maimer<br />

tan R' =<br />

tan M" =<br />

tan J ffl<br />

cos (-S" — A')<br />

tan ^ 5<br />

— cos S<br />

tan i c<br />

tan R'" =<br />

— cos S<br />

• cos S<br />

Substituting for tan § ffl, &c., by (39), or for cos S by (69),<br />

ro)<br />

Fig. 27<br />

247<br />

(809,<br />

(310)<br />

(311)<br />

cos (S — A) 2 sin J a cos J b cos J c<br />

tan B':<br />

iV<br />

COS (S — B) 2 cos I ffl sin | & cos } e<br />

tan iJ" =<br />

I (312)<br />

N<br />

cos (5 — G) 2 cos ^ fflcos } 5 sin } c<br />

tan R'" =<br />

N<br />

n<br />

170. Combining (310) with (312), we find the relation<br />

cot R cot R' cot R" cot B'" = iV"'<br />

(313)<br />

If this be multiplied successively by the squares of (310) and (312), we obtain<br />

tan R cot .8' cot B" cot R'" = cos' 5<br />

cot B tan B' cot R" cot J!'" = cos' (S-A)<br />

(314)<br />

cot iZ cot R' tan iJ" cot iS'" = cos' (S — B)<br />

cot R cot ii' cot R" tan i?'" = cos' (S — C)


248 SPHERICAL TRIGONOMETRY.<br />

171. Again, from (310) and (312) we find<br />

— tan iJ 4- tan R' + tan R" + tan R'"<br />

_ cos S + cos (S — A) + cos (S — B)+ cos (S •<br />

= ~ N<br />

_ 2 cos ^ A cos i (B + C) + 2 cos ^ A cos i (B<br />

~ N<br />

whence<br />

4 cos J A cos J B cos } G<br />

— tan iJ 4- tan R' + tan R" + tan R'" =<br />

(315)<br />

We shall find in a similar manner<br />

4 cos } ^ sin } 5 sin J C<br />

tan R — tan R' + tan R" + tan R'"<br />

If<br />

4 sin J J4 COS ^ B sin^ G<br />

tan B + tan i2' — tan R" + tan R'" =<br />

(316i<br />

If<br />

4 sin J ^ sin J B cos J 0<br />

tan .K 4- tan R' + tan i2" — tan R'" =<br />

N<br />

It is also easily shown that<br />

tan' R + tan' R' + tan' R" + tan' i2"' = 2 4- 2 cos J1 COS B COS (7 (317)<br />

172. To find the radius of the circle inscribed in a given spherical triangle.<br />

Kg. 28. ^ In Pig, 28, 0 being the pole of the required circle,<br />

draw OP', OP" and OP'" to the points of contact, and join<br />

OA, OB. We have OP" = OP'" and the triangles<br />

ip, A OP" and A OP'" right-angled at P" and P'"; hence<br />

Bin OAP" =<br />

sin OP" sin OP'"<br />

sin ^ 0 sin .4 0 = sin0.4P"'<br />

therefore 0 A P" = 0 A P'", (for we cannot have<br />

0A F" := TT — OA P'"), and the pole of the inscribed circle is consequently found<br />

by the same construction as inplano, namely, by bisecting the angles of the triangle.<br />

If then we put s z=^ ^ (a + b + c), and r = radius of the inscribed circle, we<br />

have<br />

AP'" + BP' + GP' = AP'"+ a = a, AP'" = s — a<br />

and the right triangle A 0 P'" gives<br />

tan r = sin (a — ffl) tan } A (818)<br />

corresponding with the formula of PI. Trig. (288).<br />

Substituting, in (318), the value of tan } A,<br />

^sin (s — a) sin (s — 6) sin (s — c)\<br />

sin s /<br />

tan r -.<br />

'J(c<br />

tan r = •<br />

Substituting, in (318), the value of sin (s — a) given by (58),<br />

^^^ ^<br />

^<br />

2 cos J A cos J B cos J G<br />

Also, by (51), we have iV = } sin B sin G sin a, which reduces (320) to<br />

Bin i B sin i C .<br />

tan r = =—=~-r^— sm a<br />

'.OS ^ A<br />

(319)<br />

(320)<br />

(321)


MISCELLANEOUS PROBLEMS. 249<br />

173- Let the radii of the circles inscribed in the three triangles A'BG, B'AG,<br />

CAB of Fig. 27, be r', r" and r'". Then if V denote the half sum of the sides of<br />

A'BG, we have<br />

2s' = 27r — b — c+a<br />

s' — a =z 7r — ^ (a + b + c) =z TT — a<br />

BO that (318) applied to the throe triangles, gives<br />

tan r" = sin a tan J B (322)<br />

Substitutiug in these the values of tan \ A, &c., or of sin s,<br />

Also, by (321),<br />

tan /<br />

tan r"<br />

tan r''<br />

n<br />

sin (s — a)<br />

n<br />

sin (s — b)<br />

n<br />

sin. (s — c)<br />

N<br />

2 cos J .4 sin f iS sin J G<br />

N<br />

2 sin \ A cos \ B s-i.n\ G<br />

iv;<br />

2 sin J J. sin J iJ cos J C<br />

cos If B cos if G .<br />

tan r = = -,—i-^— sm a<br />

cos J A<br />

cos J C cos \ A . ,<br />

tan r" = -— T—n — ^ *<br />

cos f B<br />

cos J -4 cos i B .<br />

tan r'" = ^ r-PT— ^m c<br />

cos J 0<br />

174. The product of (319) and (323) gives<br />

tan r tan / tan r" tan /" = -^-^ = «*<br />

whence, as in Art. 170,<br />

cot r tan r' tan /' tan r'" := sin' a<br />

tan ?- cot / tan r" tan /" = sin' (s — a)<br />

tan r tan r' cot r" tan /" = sin' (s — 6)<br />

tan r tan r' tan r" cot »-'" = sin' (s — c)<br />

- (323)<br />

. (324)<br />

(325)<br />

(326)<br />

175. We find from (319) and (323), as in Art. 171,<br />

4 sin J a sm J J sin J c<br />

— cotr + cot r' -f cot r" + cot /" =<br />

cot r — cot / + cot J-'-f cot r"' =<br />

4 sin J fflcos J 6 cos } c<br />

4 cos J a sin J S cos J e<br />

cot r -4- cot r' — cot r" 4- cot r'" =<br />

(32V)<br />

4 cos } a cos } S sin J e<br />

cot r 4- cot r' 4- cot r" — cot r"' =<br />

oof r 4- cot V 4- cot' r" + cot' j'" = 2 ' — 2 cos fflcos J cos c<br />

82


250 SPHERICAL TRIGONOMETRY.<br />

176. From (309) and (321), we find<br />

tan r<br />

tan R<br />

From (307) and the first of (327),<br />

From (315) and (320),<br />

4 Bin J J1 sin J .S sin J C cos J a cos J h cos J o<br />

— cot r + cot r' 4- cot r" + cot ?-'" :<br />

— tan iB -J- tan R' + tan R" + tan .B"<br />

: 2 tan iJ<br />

: 2 cot »-<br />

(328)<br />

(329)<br />

(330)<br />

and other similar relations are found by comparing (312) with (327), and (31G)<br />

with (323).<br />

177. The following relations are also worth remarldng.<br />

If f is the perpendicular from G upon c.<br />

tan R a\np =<br />

cot r sin^ =:<br />

2 sin J a sin \ b<br />

cos J c<br />

2 cos ^A cosJJS<br />

sin J 0<br />

y (331)<br />

178. TAe ^o?c of the circle inscribed in a spherical triangle is also the pole of the circle<br />

circumscribed about the polar triangle ; and the radii of these circles are complements of each<br />

other.<br />

The arcs bisecting the angles of a given triangle will evidently bisect the sides<br />

of the polar triangle, and wUl be perpendicular to those sides respectively; the<br />

common intersection of these arcs is therefore at once the pole of the circle inscribed<br />

in the first and circumscribed about the second.<br />

Again, if we join the angular points of the polar triangle with this common pole,<br />

the arcs thus drawn, being produced to meet the sides of the first triangle, are<br />

perpendicular to those sides, and therefore pass through the points of contact of<br />

the inscribed circle. Each of these arcs = 90°, and is at the same time the sum<br />

of the two radii of the circles in question.<br />

This latter property is also obvious from the analytical expressions of the two<br />

radii. By means of it, we might have deduced all the formulse for the inscribed<br />

from those for the circumscribed circle, or vice versa.<br />

179. To find the are joining the poles of the circles inscribed in, and cirmmseribed about<br />

a qiven spherical triangle.*<br />

Let O be the pole of the circumscribed circle,<br />

Fig. 29, and 0' that of the inscribed circle. Put<br />

00' = D; then<br />

coaD = cos J. 0 cos .40'4" sin ^0 sin ^0'cos OAO'<br />

By Art. 166, we have 0.4.B = S — G, whence<br />

OAO' = S— G—iA = i(B—G)<br />

We have also<br />

cos .40' = cos O'P cos AP = cos r cos (s — a)<br />

sin O'P<br />

Bin AC : sin O'AP<br />

sin r<br />

sin ^ A<br />

* Hymer's Spherical Trigonometry


Ther'fore,<br />

cos D<br />

cos R sin T<br />

Substituting by (819), (307), and (44),<br />

MISCELLANEOUS PROBLEMS. 251<br />

1. , % I J. r. COS i (i? — C,<br />

:= cot r COS (s — a) + tan .K -.—,—^—'<br />

^ ' sm^ A<br />

COS D sin s cos (-s — a) -|- 2 sin J i sin J c sin \(b + c)<br />

I as R sin r<br />

whence, jy (53),<br />

sin a 4- sin b + sin c<br />

2 n<br />

' cosi? V - 1 4-sin a sin 6 4-sin a sin c 4-! sin b sin c — cos a cos 6 cos e<br />

\cos R sin r)<br />

2 n'<br />

by PI. Trig. (17<br />

sin s 4- 2 sin J ffl sin J 5 sin J c<br />

(<br />

)•<br />

= (ootr 4-tani?)'<br />

cos' D = cos' {R — r) + cos' iJ sin' r<br />

sin' 7) = sin' (ie — ?•) — cos' iJ sin' r (832)<br />

If the inscribed circle is inscribed in A'BG, Fig. 27, and its radius = /, w«<br />

have, by a similar process,<br />

sin' D' = sin' (R + r') — cos' R sin' /<br />

(0-U)<br />

180. To find the equilateral spherical triangle inscribed in a given circle.<br />

U R = radius of the given circle, and A = one of the angles of the equilateral<br />

triangle, we have, by (310), and PI. Trig. Art. 76,<br />

• cos f ^<br />

tan' R = •<br />

cos" J A<br />

3 COS J -4 — 4 cos' J A<br />

cos' J A<br />

whence<br />

0Si-4=J(; 4 4- tan' B<br />

(334)<br />

181. To find the equilateral spherical triangle circumscribed about a given circle.<br />

If 7- = radius of the given circle, and a = one of the sides of the triangle, we find<br />

sm J • = J( 4 4- cot' r<br />

(335)<br />

182. Given the base and area of a -tphirical triangle, to find the locus of the vertex.<br />

Kg. ao. Let a s= the given base, and K = area of ABC, Fig. 30.<br />

Produce AB and ^O to meet in A'. Let 0 be the pole<br />

of the circle described about A'BG. The radius of this<br />

circle is given by the first equation of (311), which, by<br />

(224) becomes<br />

tan R'<br />

tan J ffl<br />

sin J K<br />

(336)<br />

The second member of this equation, being constant for<br />

all the triangles of the same base a, and the same area A",<br />

shows that R'ia also constant, and consequently, that the


252 SPHERICAL TRIGONOMETRY.<br />

point A is always found upon the circumference of the same small circle A'BC.<br />

But ^4 and A' being the extremities of the same diameter of the sphere, A is als»<br />

found upon a small circle, equal and parallel to the circle A'BC.<br />

The perpendicular distance (p') of 0 from the base BO, is found by the equation<br />

cos R'<br />

coap = r—<br />

cos J ffl<br />

and the pole of the locus of ^ is in the same perpendicular, at a distance from<br />

BG = B- —p' = p, whence<br />

cosig'<br />

COS P = 1— (°°'J<br />

•^ COS J ffl ^ '<br />

The equations (336) and (337) determine the radius and position of the pole of the<br />

required locus, wliich may therefore be constructed.<br />

This elegant proposition is due to Lexell.<br />

183. To find the angle between the chords of two sides of a spherical triangle.<br />

Kg. 31.<br />

In Fig. 31, 0 being the center of the circumscribed circle, the angle between tho<br />

chords of the sides AG and BG ia half the spherical angle AOB. If, then,<br />

we have<br />

0, == angle between the chords of a and b<br />

cos 0, = cos A OP = sin OAF cos AP<br />

or, by Art. 166, cos 0, = sin (S — C) cos J c (338)<br />

By (72) this becomes<br />

cos 0, = sin J ffl sin J 6 -|- cos J ffl cos J 4 cos 0 (339)<br />

184. The preceding problem is employed for geodetical triangles, in which C,<br />

differs very little from C, in which case it is expedient to compute the small difference<br />

G — 01 = 2;. We easily reduce (339) to the following:<br />

.SOS 0, := COS J (ffl — b) cos' JO— COS J (ffl 4- i) sin' J C<br />

= COB' J O —2 sin' i(a — b) cos'J 0—-sin' J 04-2 sin' i(a+b) sin' J 0<br />

Subtracting cos 0 = cos' JO — sin' J 0, we have,<br />

Bin J (04- 0.) sinj (0—0.) = sin'i(ffl4-J) sin'J 0—sin'^ (ffl—J)cc8'JO<br />

or approximately, taking<br />

sin J (O4- 0,) = sin 0 = 2 sin J 0 cos J O<br />

and sin J(0—0.) = J3;sinl"<br />

X being expressed in seconds,<br />

1 1<br />

" ^ alTF ^'"'^ (" -t- i) tan J O— ^j^-p sin' J (a — i) cot } O '840)


MISCELLANEOUS PROBLEMS. 253<br />

18!^ TJ a great circle (DE, Pig. 32) bisect the ba.^e of - spherical triangle at rigln<br />

angles, any great circle (FG), perpendicular to it, dividc.-< the sides [AC, BC) into segments<br />

whose sines are proportional; that is,<br />

sin FA : sin FC = am GB : sin GC<br />

Jjet P be the pole of ED, (DP = 90°), and<br />

PGF any great circle drawn through P, and<br />

therefore perpendicular to DE. Then, since<br />

we have, by (3),<br />

PB + PA = 2 PD = 180°<br />

(341)<br />

sin F sin FA = sin P sin PA = sin P sin PB<br />

= sin G sin GB.<br />

sin .f sin FC = sin O sin GO<br />

whence, by division, the theorem (341). The arc FG is analogous to the parallel to<br />

the base in plane triangles.<br />

186. If two arcs of great circles, (AB, CD, Fig. 33), terminated by any circle, intersect,<br />

the products of the tangents of the semi-segments are equal to one another; that is,<br />

tan J AE tan ^ EB = tan J CE tan J ED<br />

(342)<br />

Let P be the pole of the ctrcle DACB. Join PE ^'S- ^•<br />

and draw the perpendiculars PF, PG, bisecting the<br />

arcs AB and CD. Then we have<br />

cos FE cos PE cos PE cos GE<br />

cosi^i? COS PB cos PD coa GD<br />

cos FE — cos FB<br />

cos FE + cos FB<br />

cos GE — cos GD<br />

cos GE + cos GD<br />

which, by PI. Trig. (110), gives (342).<br />

187. If three arcs be drawn from the angles of ffl spherical triangle through the same<br />

point, to meet the opposite sides, the products of the ainea of the alternate segments of the sides<br />

will be equal.<br />

Thus, in Fig. 34, we shall have<br />

sin AB' sin 0.4' sin .BO' = sin GB' sin BA' sin A G' (343)<br />

For we easily find<br />

sin AB' _ sin AP _ sin APB'<br />

sin GB' ~ sin GP ' sin CPB'<br />

sin 0.4' sin GP sin 0P.4'<br />

sin BA! sin BP sin BPA'<br />

ain BC<br />

'a\n AG'<br />

sin BP<br />

sin BPG'<br />

smAPO'<br />

Multiplying these equations together, the product of the second members is unity,<br />

whence (343).<br />

Tho same property is easily extended to the segments of the angles.<br />

188. It follows, that when three arcs .are drawn from the three angles, so as to<br />

satisfy the condition (343), they must intersect in the same point. This occurs in<br />

the same cases as in plane triangles, that is, when the angles are bisected - when the<br />

sides are bisected ; when the three arcs are drawn from the angles to the points of<br />

W


254<br />

SPHERICAL TRIGONOMETRY.<br />

contact of the inscribed circle ; and when the three arcs are the three perpendiculars<br />

upon the sides.<br />

The first three of these cases are obvious. To prove the last, if A', B' and C,<br />

Fig. 34, are right angles, we have<br />

cos .45' cos 0.4' cos BO' cos AB cos CA cos BC = 1<br />

cos CB' cos BA! " sin P"<br />

Bin a = .—p.— sin P = "--.<br />

sm y • sin J.


whence<br />

and similarly<br />

.MISCELLANEOUS PROBLEMS.<br />

255<br />

sin it =<br />

sin ^<br />

sin y<br />

sin /S'<br />

sin a.' =<br />

sin y'<br />

, sin y" sin P"<br />

sin Q<br />

sin )." sin P"<br />

sin g<br />

sin /3" _ sin y" sin P"<br />

sin at!' =<br />

sin y" sm Q<br />

which, substituted above, give<br />

^-^ • , , m , sin^' • I , » , sin R" .<br />

-: sm (a' — a") -i !_ sm (a" — a) 4- jv^-^ sin (a _ a') = 0 (345<br />

smy ^ ' smy<br />

Again, if we express (344) in the notation of this article, it becomes<br />

cos (/S4- y) sin (a'—a")4-cos (/S'-l- y) sin (a"—a) 4- cos (fS'+ y") sin (a—a')=0 (346)<br />

which, added to (<br />

45), gives<br />

-^)sin(a'-a")4-"'^^^'+>'^sin(a"-a)-F^^f "+„>")<br />

t&ny ' tany ^ ' ' tany<br />

191. If P is the pole of ^ Q, we have<br />

and (345) and (347) both give<br />

^ 4- J. = /S' 4- y = ^"4- y = 90°<br />

sin(a—a')=0 (347)<br />

tan /3 sin (a! — a") + tan /S' sin (a" — a) -f- tan /S" sin (a — a') =5 0 (348)<br />

192. To find the inclination of two adjacent faces of a regular polyhedron, and the radii<br />

of the inscribed and circumscribed spheres.<br />

Let 0 and E, Fig. 36, be the centres of two adjacent faces whose common edge is<br />

AB; 0 the centre of the inscribed and circumscribed spheres. Draw 0 D bisecting<br />

AB at right angles; draw CD, ED, which wiU also evidently be perpendicular<br />

to J1 P ; and put<br />

/ = inclination of the faces = GD E<br />

R = radius of the circumscribed sphere = 0A = OB<br />

r •= radius of the inscribed sphere =00=: 0 E<br />

a = one of the edges = .4 P<br />

m = number of faces that form a solid angle<br />

n = number of sides of a face<br />

oi<br />

Suppose a sphere to be described about the centre 0<br />

with any radius, and cad the triangle formed upon its surface<br />

by the planes GOD, COA, AO D; this triangle is<br />

right-angled at d and gives<br />

But cos c d ^ cos GOD, and<br />

cos cad<br />

cos ed =s sin acd<br />

O 01? = 90° — GDO =<br />

-^-\I<br />

cad •= \ angle of the planes 0 AG and 0 A!E<br />

2 TT<br />

m<br />

tcd=<br />

^AOB-


256<br />

SPHERICAL TRIGONOMETRY.<br />

therefore<br />

sin J i' ^<br />

(3-J'B)<br />

Then from the triangles 0 0 D, AG D, kc,<br />

we find<br />

!- = — tan \ I cot •<br />

2 ^<br />

(350)<br />

J- = P cos ac = R cot ac(icotca(Z=: P cot — cot —<br />

n m<br />

P = — tan i / tan —<br />

2 ^ m<br />

(-351)<br />

193. Jlj .^n(Z the surface and volume of ffl regular polyhedron.<br />

Let / = number of faces of the polyhedron ; 5 = the surface, and V= the<br />

volume; then the area of each face (the notation of the preceding article being<br />

continued) is equal to<br />

whence<br />

A AB X OD xn = a'- — cot —<br />

i n<br />

S = ffl' - -f cot —<br />

4 n<br />

(352)<br />

and since V=:: S i

Hooray! Your file is uploaded and ready to be published.

Saved successfully!

Ooh no, something went wrong!